You are on page 1of 403

9709

Paper-1
(Topical Past Paper with Mark Scheme)
(2002-2022)

CONTENTS
Quadratics ---------------------------------------------------------------------------------------------- 2
UNIT 1
Answer Section---------------------------------------------------------------------------------------- 7

UNIT 2 Function and Transformation Graph

Function ------------------------------------------------------------------------------------------------- 13
2.1
Answer Section---------------------------------------------------------------------------------------- 29
Transformation---------------------------------------------------------------------------------------- 46
2.2
Answer Section---------------------------------------------------------------------------------------- 50
Coordinate Geometry --------------------------------------------------------------------------------- 52
UNIT 3
Answer Section---------------------------------------------------------------------------------------- 64
Circular Measure --------------------------------------------------------------------------------------- 90
UNIT 4
Answer Section---------------------------------------------------------------------------------------- 114

Trigonometry ------------------------------------------------------------------------------------------- 131


UNIT 5
Answer Section---------------------------------------------------------------------------------------- 147

Binomial Expansion----------------------------------------------------------------------------------- 168


UNIT 6.1
Answer Section---------------------------------------------------------------------------------------- 175

Arithmetic and geometric progressions------------------------------------------------------- 185


UNIT 6.2
Answer Section---------------------------------------------------------------------------------------- 196

Differentiation-------------------------------------------------------------------------------------------- 234
UNIT 7
Answer Section---------------------------------------------------------------------------------------- 249

Integration ---------------------------------------------------------------------------------------------- 289


UNIT 8
Answer Section---------------------------------------------------------------------------------------- 326

001
UNIT 1 Quadratics

1 M/J 22/P12/Q5
The equation of a curve is y = 4x2 − kx + 12 k2 and the equation of a line is y = x − a, where k and a are
constants.
(a) Given that the curve and the line intersect at the points with x-coordinates 0 and 34 , find the values
of k and a. [4]
(b) Given instead that a = − 72 , find the values of k for which the line is a tangent to the curve. [5]

2 M/J 22/P11/Q2
(a) Express x2 − 8x + 11 in the form x + p2 + q where p and q are constants. [2]
(b) Hence find the exact solutions of the equation x2 − 8x + 11 = 1. [2]

3 M/J 22/P11/Q9 b
2 2
The equation of a circle is x + y + 6x − 2y − 26 = 0.
(b) Find the set of values of the constant k for which the line with equation y = kx − 5 intersects the
circle at two distinct points. [6]
4 M/J 22/P13/Q11
The point P lies on the line with equation y = mx + c, where m and c are positive constants. A curve
m
has equation y = − . There is a single point P on the curve such that the straight line is a tangent to
x
the curve at P.
(a) Find the coordinates of P, giving the y-coordinate in terms of m. [6]
The normal to the curve at P intersects the curve again at the point Q.
(b) Find the coordinates of Q in terms of m. [4]

5 O/N 21/P12/Q2
A curve has equation y = kx2 + 2x − k and a line has equation y = kx − 2, where k is a constant.
Find the set of values of k for which the curve and line do not intersect. [5]

6 M/J 21/P12/Q1
(a) Express 16x2 − 24x + 10 in the form 4x + a2 + b. [2]
2
(b) It is given that the equation 16x − 24x + 10 = k, where k is a constant, has exactly one root.
Find the value of this root. [2]
7 M/J 21/P11/Q6
The equation of a curve is y = 2k − 3x2 − kx − k − 2, where k is a constant. The line y = 3x − 4 is a
tangent to the curve.
Find the value of k. [5]

8 M/J 21/P13/Q3

A line with equation y = mx − 6 is a tangent to the curve with equation y = x2 − 4x + 3.


Find the possible values of the constant m, and the corresponding coordinates of the points at which
the line touches the curve. [6]

9 O/N 20/P12/Q3

The equation of a curve is y = 2x2 + m 2x + 1, where m is a constant, and the equation of a line is
y = 6x + 4.
Show that, for all values of m, the line intersects the curve at two distinct points. [5]

002
Unit 1: Quadratics
UNIT 1 Quadratics
10 O/N 20/P11/Q1
Find the set of values of m for which the line with equation y = mx − 3 and the curve with equation
y = 2x2 + 5 do not meet. [3]

11 O/N 20/P13/Q4
A curve has equation y = 3x2 − 4x + 4 and a straight line has equation y = mx + m − 1, where m is a
constant.
Find the set of values of m for which the curve and the line have two distinct points of intersection.

12 M/J 20/P12/Q6 [5]


The equation of a curve is y = 2x2 + kx + k − 1, where k is a constant.
(a) Given that the line y = 2x + 3 is a tangent to the curve, find the value of k. [3]
It is now given that k = 2.
(b) Express the equation of the curve in the form y = 2 x + a2 + b, where a and b are constants, and
hence state the coordinates of the vertex of the curve. [3]

13 M/J 20/P11/Q5
The equation of a line is y = mx + c, where m and c are constants, and the equation of a curve is
xy = 16.
(a) Given that the line is a tangent to the curve, express m in terms of c. [3]

(b) Given instead that m = −4, find the set of values of c for which the line intersects the curve at
two distinct points. [3]
14 M/J 20/P13/Q1
Find the set of values of m for which the line with equation y = mx + 1 and the curve with equation
y = 3x2 + 2x + 4 intersect at two distinct points. [4]
15 O/N 19/P11/Q6

A straight line has gradient m and passes through the point 0, −2. Find the two values of m for
which the line is a tangent to the curve y = x2 − 2x + 7 and, for each value of m, find the coordinates
of the point where the line touches the curve. [7]

16 O/N 19/P13/Q6
A line has equation y = 3kx − 2k and a curve has equation y = x2 − kx + 2, where k is a constant.
(i) Find the set of values of k for which the line and curve meet at two distinct points. [4]

(ii) For each of two particular values of k, the line is a tangent to the curve. Show that these two
tangents meet on the x-axis. [3]
17 M/J 19/P11/Q2
The line 4y = x + c, where c is a constant, is a tangent to the curve y2 = x + 3 at the point P on the
curve.
(i) Find the value of c. [3]
(ii) Find the coordinates of P. [2]
18 O/N 18/P12/Q10
The equation of a curve is y = 2x + and the equation of a line is y + x = k, where k is a constant.
12
x
(i) Find the set of values of k for which the line does not meet the curve. [3]
In the case where k = 15, the curve intersects the line at points A and B.
(ii) Find the coordinates of A and B. [3]
(iii) Find the equation of the perpendicular bisector of the line joining A and B. [3]

003
UNIT 1 Quadratics

19 O/N 20/P11/Q1 1
Showing all necessary working, solve the equation 4x − 11x 2 + 6 = 0. [3]

20 O/N 20/P11/Q2
A line has equation y = x + 1 and a curve has equation y = x2 + bx + 5. Find the set of values of the
constant b for which the line meets the curve. [4]

21

[2]
[3]
22
[3]

23

[4]
[4]
24

[4]
25
[4]
2
[6]
2
[2]
[2]
2

[3]
[3]
2
[3]
3 M/J 16/P12/Q11
The function f is defined by f : x → 6x − x2 − 5 for x ∈ >.
[3]
(i) Find the set of values of x for which f x ≤ 3.
(ii) Given that the line y = mx + c is a tangent to the curve y = f x, show that 4c = m2 − 12m + 16.
[3]
31

[3]
32.

[3]
33.
[3]

004
34

35

36.

37.

38.

39. O/N 20/P11/Q1


A curve has equation y = 2x2 − 3x.
(i) Find the set of values of x for which y > 9.
(ii) Express 2x2 − 3x in the form a x + b2 + c, where a, b and c are constants, and state the coordinates
of the vertex of the curve.
The functions f and g are defined for all real values of x by f x = 2x2 − 3x and g x = 3x + k,
where k is a constant.
(iii) Find the value of k for which the equation gf x = 0 has equal roots.

40.

41

42

43 O/N 12/P12/Q4
x
The line y = + k, where k is a constant, is a tangent to the curve 4y = x2 at the point P. Find
k
(i) the value of k,
(ii) the coordinates of P.

005
44 O/N 12/P13/Q10

45

46

47

48 O/N /P13/Q7
(i) A straight line passes through the point (2, 0) and has gradient m. Write down the equation of
the line. [1]

(ii) Find the two values of m for which the line is a tangent to the curve y = x2 − 4x + 5. For each
value of m, find the coordinates of the point where the line touches the curve. [6]

(iii) Express x2 − 4x + 5 in the form (x + a)2 + b and hence, or otherwise, write down the coordinates
of the minimum point on the curve. [2]

49 M/J /P11/Q3
The equation x2 + px + q = 0, where p and q are constants, has roots −3 and 5.
(i) Find the values of p and q. [2]

(ii) Using these values of p and q, find the value of the constant r for which the equation
x2 + px + q + r = 0 has equal roots. [3]

50 M/J 11 /P11/Q10
2 2
(i) Express 2x − 4x + 1 in the form a(x + b) + c and hence state the coordinates of the minimum
2
point, A, on the curve y = 2x − 4x + 1. [4]

The line x − y + 4 = 0 intersects the curve y = 2x2 − 4x + 1 at points P and Q. It is given that the
coordinates of P are (3, 7).

(ii) Find the coordinates of Q. [3]

(iii) Find the equation of the line joining Q to the mid-point of AP. [3]

006
1 (a) M/J 22/P12/Q5
M/J 22/P13/Q11
1
4  02  0  k 2  0  a m
2 4 a) mx  c    mx 2  cx  m  0
2 x
3 3 1 3
4   k  k2   a b 2  4ac  0  c 2  4m 2  0
4 4 2 4
c     2m
k = 2, a   2
mx 2 [  ] 2mx  m  0  x 2    2 x  1  0
(b) 1 7 1 7
4 x 2  kx  k 2  x   4 x 2  kx  x  k 2    0  x  1
2
 0  x  1 only
2 2 2 2
y  m only or (‒1, m) only
 k  12  4  4 
1 2 7
k  
2 2 4 b) 1
Equation of normal is y  m   x  1
 7 k 2  2k  57 m
x 1 m
 m
m m x
  
x 2  x 1  m 2  m 2   0
 k  3 7 k  19  or other valid method
19  x  1  x  m 2    0  x  m2
k = 3, k = 
7 m 1
M/J 22/P12/Q5 y 
m2 m
2(a) x2 – 8x + 11 = (x – 4)2 … or p = − 4 5 O/N 21/P11/Q2
… 5 or q = − 5 kx 2 + 2 x − k = kx − 2 leading to kx 2 + ( − k + 2 ) x − k + 2 [ = 0]

2(b)
2
(x – 4) − 5 = 1 so (x – 4) = 6 so 2 x  4   6 ( − k + 2 ) 2 − 4k ( − k + 2 )
5k 2 − 12k + 4 or ( − k + 2 )( −k + 2 − 4k )
8  24 ( −k + 2 )( −5k + 2 )
x  4  6 or
2
2
M/J 22/P11/Q9 b <k<2
5
Intersects when x 2   kx  5  6 x  2  kx  5  26  0
2
3 O/N 21/P12/Q1
or  x  3   kx  5  1  36
2 2

2 2 2
6(a) (4x – 3)2 or (4x +(– 3))2 or a = −3
x  k x  10kx  25  6 x  2kx  10  26  0
+ 1 or b = 1
or x 2  6 x  9  k 2 x 2  12kx  36  36
leading to k 2 x 2  x 2  6 x  12kx  9   0 6(b) [For one root] k = 1 or ‘their b’
3
 
or k 2  1 x 2   6  12k  x  9   0 [ Root or x =] or 0.75
4
 6 12k 
2
 
 4 k 2  1  9 [ 0]
O/N 21/P11/Q6

 leading to 144k  144k  36  36k 2  36  0 


2
7 ( 2k − 3) x 2 − kx − ( k − 2 ) = 3x − 4
 
4 ( 2k − 3) x 2 − ( k + 3) x − ( k − 6 ) [ = 0]
[108k2 – 144k = 0 leading to] k = 0 or k =
3
4
k  0, k  ( k + 3)2 + 4 ( 2k − 3 )( k − 6 ) [ = 0]
3

9k 2 − 54k + 81[ = 0] [leading to k 2 − 6k + 9 = 0 ]

k=3

007
M/J 20/P11/Q5

8 M/J 22/P13/Q11
14 M/J 20/P13/Q1
x 2 − 4 x + 3 = mx − 6 leading to x 2− x (4 + m ) + 9
2 3x 2 + 2 x + 4 = mx + 1 → 3 x 2 + x ( 2 − m ) + 3 ( = 0 )
b 2 − 4ac leading to ( 4 + m) − 4 × 9
( 2 − m )2 − 36 SOI
4 + m = ±6 or ( m − 2 ) (m + 10 ) = 0 leading to m = 2 or −10
Substitute both their m values into their equation in line 1 m + 4)(m ‒ 8) (>/= 0) or 2 ‒ m >/= 6 and 2 ‒ m </= ‒6 OE
m = 2 leading to x =3 ; m = ‒10 leading to x = ‒3 m < ‒ 4, m > 8 WWW
(3, 0), ( ‒ 3, 24)
O/N 19/P11/Q6
9 O/N 20/P12/Q3 15 Equation of line is y = mx − 2
x 2 − 2 x + 7 = mx − 2 → x 2 − x ( 2 + m ) + 9 = 0
2 x 2 + m ( 2 x + 1) − 6 x − 4 ( = 0 )
Apply b 2 − 4ac ( = 0 ) → ( 2 + m ) − 4 × 9 ( = 0 )
2
Using b ² − 4ac on 2 x 2 + x ( 2m − 6 ) + m − 4 ( = 0)
m = 4 or −8
4m2 − 32m + 68 or 2m2 − 16m + 34 or m2 − 8m + 17
m = 4→ x2 ‒ 6x + 9 = 0 → x = 3
10 O/N 20/P11/Q1 m = ‒8→ x2 + 6x + 9 = 0 → x = ‒3
2 x 2 + 5 = mx − 3 → 2 x 2 − mx + 8 ( = 0 ) (3, 10), (‒3, 22)
m 2 − 64
−8 < m < 8 O/N 19/P13/Q6
O/N 20/P13/Q4
16 (i) 3kx − 2k = x 2 − kx + 2 → x 2 − 4kx + 2k + 2 ( = 0 )
11 3 x 2  4 x  4  mx  m  1 
 3x 2 
Attempt to find b 2 − 4ac
(4  m) x  (5  m)( 0)
1 and − 12
b 2 − 4ac = ( 4 + m ) − 4 × 3 × ( 5 − m )
2 k > 1, k < − 12
3
m 2 + 20m − 44 (ii) y = 3x − 2, y = − x + 1
3 2
( m + 22)(m ‒ 2) 3x − 2 = − x + 1 OR y + 2 = 2 − 2 y
2
m > 2 , m < −22 2
x = , → y = 0 in one or both lines
3
12 M/J 20/P12/Q6 17 M/J 19/P11/Q2

(a) 2 x + kx + k − 1 = 2 x + 3 → 2 x 2 + ( k − 2 ) x + k − 4 = 0 2(i) Eliminates x or y → y ² − 4 y + c − 3 = 0


Use of b² − 4ac = 0 → (k – 2)² = 8(k – 4) or x² + ( 2c − 16) x + c² − 48 = 0
k=6 Uses b ² = 4 ac → 4c – 28 = 0
12(b) 2x² + 2x + 1 = 2  x +  ² + 1 −
1 1 c=7
 2 2 (ii) Uses c = 7, y² − 4y + 4 = 0 (1, 2)
1 1
a= ,b=
2 2 18 O/N 18/P12/Q10
 1 1 12 12
vertex  − ,  (i) 2x + = k − x or y = 2 ( k − y ) + → 3 term quadratic.
 2 2 x k−y
(FT on a and b values) Use of b² − 4ac → k² − 144 < 0

13 M/J 20/P11/Q5 (ii) − 12 < k


13(a) x ( mx + c ) = 16 → mx 2 + cx − 16 = 0 Using k = 15 in their 3 term quadratic
Use of b² − 4ac = c² + 64m
−c ² x = 1,4 or y = 11, 14
Sets to 0 → m = (iii) Gradient of AB = −1 → Perpendicular gradient = +1
64
13(b) x ( −4 x + c ) = 16 Finding their midpoint using their (1, 14) and (4, 11)
Use of b² − 4ac → c² − 256 Equation: y – 12½ = (x – 2½) [y = x + 10]
c > 16 and c < −16

008
19 O/N 18/P11/Q1 20 O/N 18/P11/Q1
 12   12  x + bx + 5 = x + 1 → x 2 + x ( b − 1) + 4 ( = 0 )
2

 4 x  3   x  2  oe soi (b 2 − 4ac =) ( b − 1) − 16
2
  
b associated with ‒3 & +5 or b −
Alt : 4 x  6  11 x  16 x 2  73x  36 1 associated with ±4
x½ = 3 / 4 or 2 b ⩾ 5, b ⩽ –3
(16 x − 9 )( x − 4 )
x = 9 / 16 oe or 4
21 26

27

28
22

23

24
29

25
30 M/J 16/P12/Q11/i,ii
f : x ⟼ 6x – x² − 5
(i) 6x – x² − 5 ⩽ 3
→ x² − 6x + 8 ⩾ 0
→ x = 2, x = 4
x ⩽ 2, x ⩾ 4
condone < and/or >

009
11

37
30 M/J 16/P12/Q11/i,ii
(ii) Equate mx + c and 6x – x² − 5
Use of “ b ² − 4ac "
4c = m ² − 12m + 16. AG
OR
dy 6−m
= 6 − 2x = m → x =  
dx  2 
2
6−m 6−m 6−m
m  + c = 6 −  −5
 2   2   2 
4c = m ² − 12m + 16. AG
38

33 39 O/N 13/P12/Q10
f : x a 2 x 2 − 3 x , g : x a 3x + k ,
(i) 2 x 2 − 3x − 9 > 0
34 → x =3 or −1½
Set of x x > 3 , or x < −1½
3 9
(ii) 2x² − 3x = 2( x − ) 2 −
4 8
3 9
Vertex ( ,− )
4 8
(iii) gf(x) = 6x² − 9x + k = 0
27
Use of b² − 4ac → k = oe.
8
40

35
41

36

010
12

42

45

43

46

47
44

011
13

48

49

50

012
UNIT 2 Function and Transformation Graph

2.1: Functions

1 M
M/J 22/P12/Q10/a,b,c,d
Functions f and g are defined as follows: 2x + 1
f x = for x ≠ 12 ,
2x − 1
g x = x2 + 4 for x ∈ >.

y
(a)
4

x
−6 −4 −2 0 2 4 6

−2

The diagram shows part of the graph of y = f x.


State the domain of f −1 . [1]
(b) Find an expression for f −1 x. [3]
(c) Find gf −1 3. [2]
(d) Explain why g−1 x cannot be found. [1]

2 M/J 22/P11/Q6
The function f is defined as follows: x2 − 4
f x = for x > 2.
x2 + 4

(a) Find an expression for f −1 x. [3]


8 x2 − 4
(b) Show that 1 − 2 can be expressed as 2 and hence state the range of f. [4]
x +4 x +4
(c) Explain why the composite function ff cannot be formed. [1]

3 M/J 22/P13/Q6
The function f is defined by f x = 2x2 − 16x + 23 for x < 3.
(a) Express f x in the form 2 x + a2 + b. [2]

(b) Find the range of f. [1]

(c) Find an expression for f −1 x. [3]

The function g is defined by g x = 2x + 4 for x < −1.

(d) Find and simplify an expression for fg x. [2]

013
Unit 2: Function & Transformation Graph

4 O/N 21/P12/Q3

The function f is defined as follows: x+3


f x = for x > 1.
x−1
(a) Find the value of ff 5. [2]
(b) Find an expression for f −1 x. [3]

5 O/N 21/P12/Q3

(a) Express −3x + 12x + 2 in the form −3 x − a + b, where a and b are constants.
2 2
[2]

The one-one function f is defined by f : x → −3x2 + 12x + 2 for x ≤ k.


(b) State the largest possible value of the constant k. [1]
It is now given that k = −1.

(c) State the range of f. [1]

(d) Find an expression for f −1 x. [3]


−3
A @
The result of translating the graph of y = f x by is the graph of y = g x.
1
(e) Express g x in the form px2 + qx + r , where p, q and r are constants. [3]

y
6 O/N 21/P13/Q6

x
O

y = f x

The diagram shows the graph of y = f x.


(a) On this diagram sketch the graph of y = f −1 x. [1]
x
It is now given that f x = −  where −2 < x < 2.
4 − x2

(b) Find an expression for f −1 x. [4]


The function g is defined by g x = 2x for −a < x < a, where a is a constant.
(c) State the maximum possible value of a for which fg can be formed. [1]

(d) Assuming that fg can be formed, find and simplify an expression for fg x. [2]

014
Unit 2: Function & Transformation Graph

7 O/N 21/P13/Q6
(a) Express 5y2 − 30y + 50 in the form 5 y + a2 + b, where a and b are constants. [2]
(b) The function f is defined by f x = x5 − 10x3 + 50x for x ∈ >.
Determine whether f is an increasing function, a decreasing function or neither. [3]

8 M/J 21/P12/Q5
The function f is defined by f x = 2x2 + 3 for x ≥ 0.

(a) Find and simplify an expression for ff x. [2]

(b) Solve the equation ff x = 34x2 + 19. [4]

9 M/J 21/P11/Q9
Functions f and g are defined as follows:
f x = x − 22 − 4 for x ≥ 2,
g x = ax + 2 for x ∈ >,
where a is a constant.
(a) State the range of f. [1]
(b) Find f −1 x. [2]
(c) Given that a = − 53 , solve the equation f x = g x. [3]
(d) Given instead that ggf −1 12 = 62, find the possible values of a [5]

10 M/J 21/P13/Q6/a
Functions f and g are both defined for x ∈ > and are given by

f x = x2 − 2x + 5,
g x = x2 + 4x + 13.

(a) By first expressing each of f x and g x in completed square form, express g x in the form
f x + p + q, where p and q are constants. [4]

11 M/J 21/P13/Q8
Functions f and g are defined as follows:
f : x → x2 − 1 for x < 0,
1
g:x→ for x < − 12 .
2x + 1
(a) Solve the equation fg x = 3. [4]
(b) Find an expression for fg−1 x. [3]

12 O/N 20/P12/Q5
Functions f and g are defined by
f x = 4x − 2, for x ∈ >,
4
g x = , for x ∈ >, x ≠ −1.
x+1
(a) Find the value of fg 7. [1]
(b) Find the values of x for which f −1 x = g−1 x. [5]

O/N 20/P11/Q9
13 The functions f and g are defined by f x = x2 + 3 for x > 0,
g x = 2x + 1 for x > − 12 .

(a) Find an expression for fg x. [1]

015
Unit 2: Function & Transformation Graph

(b) Find an expression for fg−1 x and state the domain of fg−1 [4]
(c) Solve the equation fg x − 3 = gf x. [4]

O/N 20/P13/Q6
14 The function f is defined by f x = 2x
for x > 13 .
3x − 1
(a) Find an expression for f −1 x. [3]

+
2 2 2x
3 3 3x − 1 3x − 1
(b) Show that can be expressed as [2]

(c) State the range of f. [1]

M/J 20/P12/Q5
15 The function f is defined for x ∈ > by

where a is a constant. f : x → a − 2x,

(a) Express ff x and f −1 x in terms of a and x. [4]

(b) Given that ff x = f −1 x, find x in terms of a. [2]

M/J 20/P11/Q6
16 Functions f and g are defined for x ∈ > by
f : x → 12 x − a,
g : x → 3x + b,
where a and b are constants.
(a) Given that gg 2 = 10 and f −1 2 = 14, find the values of a and b. [4]
(b) Using these values of a and b, find an expression for gf x in the form cx + d , where c and d are
constants. [2]

M/J 20/P13/Q9
17 The functions f and g are defined by

f x = x2 − 4x + 3 for x > c, where c is a constant,


1
g x = for x > −1.
x+1
(a) Express f x in the form x − a2 + b. [2]
It is given that f is a one-one function.
(b) State the smallest possible value of c. [1]
It is now given that c = 5.
(c) Find an expression for f −1 x and state the domain of f −1 . [3]
(d) Find an expression for gf x and state the range of gf. [3]

Mar 20/P12/Q9
18 (a) Express 2x2 + 12x + 11 in the form 2 x + a2 + b, where a and b are constants. [2]
2
The function f is defined by f x = 2x + 12x + 11 for x ≤ −4.
(b) Find an expression for f −1 x and state the domain of f −1 . [3]
The function g is defined by g x = 2x − 3 for x ≤ k.
(c) For the case where k = −1, solve the equation fg x = 193. [2]

016
Unit 2: Function & Transformation Graph

O/N 19/P12/Q9
19 Functions f and g are defined by f x = 2x2 + 8x + 1 for x ∈ >,
g x = 2x − k for x ∈ >,
where k is a constant.
(i) Find the value of k for which the line y = g x is a tangent to the curve y = f x. [3]
(ii) In the case where k = −9, find the set of values of x for which f x < g x. [3]
−1 −1
(iii) In the case where k = −1, find g f x and solve the equation g f x = 0. [3]
(iv) Express f x in the form 2 x + a2 + b, where a and b are constants, and hence state the least
value of f x. [3]

O/N 19/P11/Q7
3
20 Functions f and g are defined by f:x→ for x > 0,
2x + 1
1
g : x → + 2 for x > 0.
x
(i) Find the range of f and the range of g. [3]
ax
(ii) Find an expression for fg x, giving your answer in the form , where a, b and c are integers.
bx + c
[2]
(iii) Find an expression for fg−1 x, giving your answer in the same form as for part (ii). [3]

O/N 19/P13/Q2
21 The function g is defined by g x = x2 − 6x + 7 for x > 4. By first completing the square, find an
expression for g−1 x and state the domain of g−1 . [5]

M/J 19/P12/Q7
22 Functions f and g are defined by f : x → 3x − 2, x ∈ >,
2x + 3
g:x→ , x ∈ >, x ≠ 1.
x−1
(i) Obtain expressions for f −1 x and g−1 x, stating the value of x for which g−1 x is not defined.
[4]
(ii) Solve the equation fg x = 73 . [3]

M/J 19/P11/Q5
23 The function f is defined by f x = −2x2 + 12x − 3 for x ∈ >.
(i) Express −2x2 + 12x − 3 in the form −2 x + a2 + b, where a and b are constants. [2]
(ii) State the greatest value of f x. [1]
The function g is defined by g x = 2x + 5 for x ∈ >.
(iii) Find the values of x for which gf x + 1 = 0. [3]

M/J 19/P13/Q1
24 The function f is defined by f x = x2 − 4x + 8 for x ∈ >.
(i) Express x2 − 4x + 8 in the form x − a2 + b. [2]
(ii) Hence find the set of values of x for which f x < 9, giving your answer in exact form. [3]

017
Unit 2: Function & Transformation Graph

M/J 19/P13/Q4
48
25 The function f is defined by f x = for 3 ≤ x ≤ 7. The function g is defined by g x = 2x − 4 for
x−1
a ≤ x ≤ b, where a and b are constants.
(i) Find the greatest value of a and the least value of b which will permit the formation of the
composite function gf. [2]
It is now given that the conditions for the formation of gf are satisfied.
(ii) Find an expression for gf x. [1]
(iii) Find an expression for gf−1 x. [2]
March 19/P12/Q8
26 (i) Express x2 − 4x + 7 in the form x + a2 + b. [2]
2
The function f is defined by f x = x − 4x + 7 for x < k, where k is a constant.
(ii) State the largest value of k for which f is a decreasing function. [1]
The value of k is now given to be 1.
(iii) Find an expression for f −1 x and state the domain of f −1 . [3]

(iv) The function g is defined by g x = for x > 1. Find an expression for gf x and state the
x−1
range of gf. [4]

O/N 18/P12/Q4
27 Functions f and g are defined by f : x → 2 − 3 cos x for 0 ≤ x ≤ 20,

g : x → 12 x for 0 ≤ x ≤ 20.
(i) Solve the equation fg x = 1.
[3]
(ii) Sketch the graph of y = f x. [3]

O/N 18/P12/Q9
28 The function f is defined by f : x → 2x2 − 12x + 7 for x ∈ >.

(i) Express 2x2 − 12x + 7 in the form 2 x + a2 + b, where a and b are constants. [2]
(ii) State the range of f. [1]
The function g is defined by g : x → 2x − 12x + 7 for x ≤ k.
2

(iii) State the largest value of k for which g has an inverse. [1]
(iv) Given that g has an inverse, find an expression for g−1 x. [3]

29 O/N 18/P11/Q11
(a) The one-one function f is defined by f x = x − 32 − 1 for x < a, where a is a constant.
(i) State the greatest possible value of a. [1]
(ii) It is given that a takes this greatest possible value. State the range of f and find an expression
for f −1 x. [3]
(b) The function g is defined by g x = x − 3 2for x ≥ 0.
(i) Show that gg 2x can be expressed in the form 2x − 34 + b 2x − 32 + c, where b and c are
constants to be found. [2]
(ii) Hence expand gg 2x completely, simplifying your answer. [4]
30 O/N 18/P13/Q11
(i) Express 2x 2 − 12x + 11 in the form a x + b 2 + c, where a, b and c are constants. [3]
The function f is defined by f x = 2x2 − 12x + 11 for x ≤ k.
[1]
(ii) State the largest value of the constant k for which f is a one-one function.

018
Unit 2: Function & Transformation Graph

(iii) For this value of k find an expression for f −1 x and state the domain of f −1 . [4]
The function g is defined by g x = x + 3 for x ≤ p.
(iv) With k now taking the value 1, find the largest value of the constant p which allows the composite
function fg to be formed, and find an expression for fg x whenever this composite function
exists. [3]
31 M/J 18/P12/Q7
The function f is defined by f : x → 7 − 2x2 − 12x for x ∈ >.
(i) Express 7 − 2x2 − 12x in the form a − 2 x + b2 , where a and b are constants. [2]
(ii) State the coordinates of the stationary point on the curve y = f x. [1]
The function g is defined by g : x → 7 − 2x2 − 12x for x ≥ k.
(iii) State the smallest value of k for which g has an inverse. [1]
−1
(iv) For this value of k, find g x. [3]

M/J 18/P11/Q9
32 Functions f and g are defined for x ∈ > by f : x → 12 x − 2,

g : x → 4 + x − 12 x2 .
(i) Find the points of intersection of the graphs of y = f x and y = g x. [3]
(ii) Find the set of values of x for which f x > g x. [2]
(iii) Find an expression for fg x and deduce the range of fg. [4=]

The function h is defined by h : x → 4 + x − 12 x2 for x ≥ k.


(iv) Find the smallest value of k for which h has an inverse. [2]

33 M/J 18/P13/Q10
The one-one function f is defined by f x = x − 22 + 2 for x ≥ c, where c is a constant.
(i) State the smallest possible value of c. [1]
In parts (ii) and (iii) the value of c is 4.
(ii) Find an expression for f −1 x and state the domain of f −1 . [3]
(iii) Solve the equation ff x = 51, giving your answer in the form a + ïb. [5]

34 O/N 17/P12/Q2
A function f is defined by f : x → 4 − 5x for x ∈ >.
(i) Find an expression for f −1 x and find the point of intersection of the graphs of y = f x and
y = f −1 x. [3]
−1
(ii) Sketch, on the same diagram, the graphs of y = f x and y = f x, making clear the relationship
between the graphs. [3]

35 O/N 17/P11/Q9
Functions f and g are defined for x > 3 by 1
f:x→ 2
,
x −9
g : x → 2x − 3.
(i) Find and simplify an expression for gg x. [2]
(ii) Find an expression for f −1 x and state the domain of f −1 . [4]

(iii) Solve the equation fg x = 17 . [4]

019
Unit 2: Function & Transformation Graph

O/N 17/P13/Q6
36 The functions f and g are defined by 2
f x = 2
for x < −1,
x −1
g x = x2 + 1 for x > 0.
(i) Find an expression for f −1 x. [3]
(ii) Solve the equation gf x = 5. [4]

M/J 17/P11/Q9
2
37 The function f is defined by f : x → for x ∈ >, x ≠ 32 .
3 − 2x
(i) Find an expression for f −1 x. [3]
The function g is defined by g : x → 4x + a for x ∈ >, where a is a constant.
(ii) Find the value of a for which gf −1 = 3. [3]
−1 −1
(iii) Find the possible values of a given that the equation f x = g x has two equal roots. [4]

38 M/J 17/P13/Q9
(i) Express 9x2 − 6x + 6 in the form ax + b2 + c, where a, b and c are constants. [3]
2
The function f is defined by f x = 9x − 6x + 6 for x ≥ p, where p is a constant.
(ii) State the smallest value of p for which f is a one-one function. [1]
(iii) For this value of p, obtain an expression for f −1 x, and state the domain of f −1 . [4]
(iv) State the set of values of q for which the equation f x = q has no solution. [1]

39 O/N 16/P12/Q10
A function f is defined by f : x → 5 − 2 sin 2x for 0 ≤ x ≤ 0.
(i) Find the range of f. [2]
(ii) Sketch the graph of y = f x. [2]
(iii) Solve the equation f x = 6, giving answers in terms of 0. [3]
The function g is defined by g : x → 5 − 2 sin 2x for 0 ≤ x ≤ k, where k is a constant.
(iv) State the largest value of k for which g has an inverse. [1]
(v) For this value of k, find an expression for g−1 x. [3]

40 O/N 16/P11/Q8
The functions f and g are defined by 4
f x = − 2 for x > 0,
x
4
g x = for x ≥ 0.
5x + 2
(i) Find and simplify an expression for fg x and state the range of fg. [3]
−1 −1
(ii) Find an expression for g x and find the domain of g . [5]

41 O/N 16/P13/Q8
(i) Express 4x2 + 12x + 10 in the form ax + b2 + c, where a, b and c are constants. [3]
(ii) Functions f and g are both defined for x > 0. It is given that f x = x2 + 1 and fg x = 4x2 + 12x + 10.
Find g x. [1]
(iii) Find fg−1 x and give the domain of fg−1 . [4]

020
Unit 2: Function & Transformation Graph
M/J 16/P12/Q1
42 Functions f and g are defined by
f : x → 10 − 3x, x ∈ >,
10
g:x→ , x ∈ >, x ≠ 32 .
3 − 2x
Solve the equation ff x = gf 2. [3]

M/J 16/P12/Q11 iii,iv,v


43 The function g is defined by g : x → 6x − x2 − 5 for x ≥ k, where k is a constant.

(iii) Express 6x − x2 − 5 in the form a − x − b2 , where a and b are constants. [2]

(iv) State the smallest value of k for which g has an inverse. [1]

(v) For this value of k, find an expression for g−1 x. [2]

M/J 16/P11/Q6/b
44 (b) The function f is defined for x ∈ > by f x = x2 + ax + b, where a and b are constants. The
solutions of the equation f x = 0 are x = 1 and x = 9. Find
(i) the values of a and b, [2]
(ii) the coordinates of the vertex of the curve y = f x. [2]

M/J 16/P11/Q11
45 The function f is defined by f : x → 4 sin x − 1 for − 12 0 ≤ x ≤ 12 0.
(i) State the range of f. [2]
(ii) Find the coordinates of the points at which the curve y = f x intersects the coordinate axes. [3]
(iii) Sketch the graph of y = f x. [2]
(iv) Obtain an expression for f −1 x, stating both the domain and range of f −1 . [4]

M/J 16/P13/Q10
46 The function f is such that f x = 2x + 3 for x ≥ 0. The function g is such that g x = ax2 + b for x ≤ q,
where a, b and q are constants. The function fg is such that fg x = 6x2 − 21 for x ≤ q.
(i) Find the values of a and b. [3]
(ii) Find the greatest possible value of q. [2]
It is now given that q = −3.
(iii) Find the range of fg. [1]
(iv) Find an expression for fg−1 x and state the domain of fg−1 . [3]

O/N 15/P12/Q1
47 Functions f and g are defined by
f : x → 3x + 2, x ∈ >,
g : x → 4x − 12, x ∈ >.

Solve the equation f −1 x = gf x. [4]

021
Unit 2: Function & Transformation Graph

48 O/N 15/P12/Q8

The function f is defined, for x ∈ >, by f : x → x2 + ax + b, where a and b are constants.


(i) In the case where a = 6 and b = −8, find the range of f. [3]
(ii) In the case where a = 5, the roots of the equation f x = 0 are k and −2k, where k is a constant.
Find the values of b and k. [3]
(iii) Show that if the equation f x + a = a has no real roots, then a2 < 4 b − a. [3]
49 O/N 15/P11/Q9
(i) Express −x2 + 6x − 5 in the form a x + b2 + c, where a, b and c are constants. [3]
2
The function f : x → −x + 6x − 5 is defined for x ≥ m, where m is a constant.
(ii) State the smallest value of m for which f is one-one. [1]
(iii) For the case where m = 5, find an expression for f −1 x and state the domain of f −1 . [4]

50 O/N 15/P13/Q8
The function f is defined by f x = 3x + 1 for x ≤ a, where a is a constant. The function g is defined
by g x = −1 − x2 for x ≤ −1.
(i) Find the largest value of a for which the composite function gf can be formed. [2]
For the case where a = −1,
(ii) solve the equation fg x + 14 = 0, [3]
(iii) find the set of values of x which satisfy the inequality gf x ≤ −50. [4]

51 M/J 15/P12/ Q11


The function f is defined by f : x → 2x2 − 6x + 5 for x ∈ >.

(i) Find the set of values of p for which the equation f x = p has no real roots. [3]

The function g is defined by g : x → 2x2 − 6x + 5 for 0 ≤ x ≤ 4.

(ii) Express g x in the form a x + b2 + c, where a, b and c are constants. [3]

(iii) Find the range of g. [2]

The function h is defined by h : x → 2x2 − 6x + 5 for k ≤ x ≤ 4, where k is a constant.

(iv) State the smallest value of k for which h has an inverse. [1]

(v) For this value of k, find an expression for h−1 x. [3]

022
Unit 2: Function & Transformation Graph

52 M/J 15/P13/ Q6
y

x
O

1 − 5x
y=
2x

1 − 5x
The diagram shows the graph of y = f −1 x, where f −1 is defined by f −1 x = for 0 < x ≤ 2.
2x

(i) Find an expression for f x and state the domain of f. [5]

1
(ii) The function g is defined by g x = for x ≥ 1. Find an expression for f −1 g x, giving your
x
answer in the form ax + b, where a and b are constants to be found. [2]

53 O/N 14/P11/ Q10


(i) Express x2 − 2x − 15 in the form x + a2 + b. [2]

The function f is defined for p ≤ x ≤ q, where p and q are positive constants, by

f : x → x2 − 2x − 15.
The range of f is given by c ≤ fx ≤ d, where c and d are constants.

(ii) State the smallest possible value of c. [1]

For the case where c = 9 and d = 65,


(iii) find p and q, [4]
(iv) find an expression for f −1 x. [3]

O/N 14/P13/ Q10/a


54 (i) The functions f and g are defined for x ≥ 0 by
1
f : x → ax + b 3 , where a and b are positive constants,
g : x → x2 .

Given that fg1 = 2 and gf9 = 16,


(i) calculate the values of a and b, [4]
(ii) obtain an expression for f −1x and state the domain of f −1 . [4]

023
Unit 2: Function & Transformation Graph

55 M/J 14/P12/ Q10


Functions f and g are defined by
f : x → 2x − 3, x ∈ >,
g : x → x + 4x,
2
x ∈ >.

(i) Solve the equation ff x = 11. [2]

(ii) Find the range of g. [2]

(iii) Find the set of values of x for which g x > 12. [3]

(iv) Find the value of the constant p for which the equation gf x = p has two equal roots. [3]

Function h is defined by h : x → x2 + 4x for x ≥ k, and it is given that h has an inverse.

(v) State the smallest possible value of k. [1]

(vi) Find an expression for h−1 x. [4]

y
56 M/J 14/P11/ Q10

x
O

The diagram shows the function f defined for −1 ≤ x ≤ 4, where


t
3x − 2 for −1 ≤ x ≤ 1,
f x = 4
for 1 < x ≤ 4.
5−x

(i) State the range of f. [1]

(ii) Copy the diagram and on your copy sketch the graph of y = f −1 x. [2]

(iii) Obtain expressions to define the function f −1 , giving also the set of values for which each
expression is valid. [6]

024
Unit 2: Function & Transformation Graph

57 O/N 13/P11/ Q5
The function f is defined by
f : x → x2 + 1 for x ≥ 0.

(i) Define in a similar way the inverse function f −1 . [3]

(ii) Solve the equation ff x = 185


16
. [3]

58 O/N 13/P13/ Q10


The function f is defined by f : x → x2 + 4x for x ≥ c, where c is a constant. It is given that f is a
one-one function.

(i) State the range of f in terms of c and find the smallest possible value of c. [3]

The function g is defined by g : x → ax + b for x ≥ 0, where a and b are positive constants. It is given
that, when c = 0, gf 1 = 11 and fg 1 = 21.

(ii) Write down two equations in a and b and solve them to find the values of a and b. [6]

59 M/J 13/P12/ Q8
(i) Express 2x2 − 12x + 13 in the form a x + b2 + c, where a, b and c are constants. [3]
(ii) The function f is defined by f x = 2x2 − 12x + 13 for x ≥ k, where k is a constant. It is given
that f is a one-one function. State the smallest possible value of k. [1]
The value of k is now given to be 7.
(iii) Find the range of f. [1]
−1 −1
(iv) Find an expression for f x and state the domain of f . [5]

60 M/J 13/P13/ Q10


The function f is defined by f : x → 2x + k, x ∈ >, where k is a constant.
(i) In the case where k = 3, solve the equation ff x = 25. [2]
2
The function g is defined by g : x → x − 6x + 8, x ∈ >.
(ii) Find the set of values of k for which the equation f x = g x has no real solutions. [3]
The function h is defined by h : x → x2 − 6x + 8, x > 3.
(iii) Find an expression for h−1 x. [4]

61 O/N 12/P12/ Q2 r
  + 1, for x ≥ −3. Find
x+3
A function f is such that f (x) =
2
(i) f −1 (x) in the form ax2 + bx + c, where a, b and c are constants, [3]
(ii) the domain of f −1 . [1]

025
Unit 2: Function & Transformation Graph
62 O/N 12/P11/ Q10
2
The function f is defined by f (x) = 4x − 24x + 11, for x ∈ >.
(i) Express f (x) in the form a(x − b)2 + c and hence state the coordinates of the vertex of the graph
of y = f (x). [4]
The function g is defined by g(x) = 4x2 − 24x + 11, for x ≤ 1.
(ii) State the range of g. [2]
(iii) Find an expression for g−1 (x) and state the domain of g−1 . [4]

63 O/N 12/P11/ Q10


y

y = 11 – x 2

A (p, q)

x
O

y=5–x

(i) The diagram shows part of the curve y = 11 − x2 and part of the straight line y = 5 − x meeting at
the point A (p, q), where p and q are positive constants. Find the values of p and q. [3]

(ii) The function f is defined for the domain x ≥ 0 by

f (x) = 
11 − x2 for 0 ≤ x ≤ p,
5−x for x > p.
Express f −1 (x) in a similar way. [5]

64 M/J 12/P12/ Q10


Functions f and g are defined by
f : x → 2x + 5 for x ∈ >,
8
g:x→ for x ∈ >, x ≠ 3.
x−3

(i) Obtain expressions, in terms of x, for f −1 (x) and g−1 (x), stating the value of x for which g−1 (x)
is not defined. [4]

(ii) Sketch the graphs of y = f (x) and y = f −1 (x) on the same diagram, making clear the relationship
between the two graphs. [3]

(iii) Given that the equation fg(x) = 5 − kx, where k is a constant, has no solutions, find the set of
possible values of k. [5]

026
Unit 2: Function & Transformation Graph

65 M/J 12/P11/ Q8
The function f : x → x2 − 4x + k is defined for the domain x ≥ p, where k and p are constants.

(i) Express f (x) in the form (x + a)2 + b + k, where a and b are constants. [2]

(ii) State the range of f in terms of k. [1]

(iii) State the smallest value of p for which f is one-one. [1]

(iv) For the value of p found in part (iii), find an expression for f −1 (x) and state the domain of f −1 ,
giving your answers in terms of k. [4]

66 M/J 12/P13/ Q11


The function f is such that f (x) = 8 − (x − 2)2 , for x ∈ >.

(i) Find the coordinates and the nature of the stationary point on the curve y = f (x). [3]

The function g is such that g(x) = 8 − (x − 2)2 , for k ≤ x ≤ 4, where k is a constant.

(ii) State the smallest value of k for which g has an inverse. [1]

For this value of k,


(iii) find an expression for g−1 (x), [3]
(iv) sketch, on the same diagram, the graphs of y = g(x) and y = g−1 (x). [3]

67 O/N 11/P12/ Q2
The functions f and g are defined for x ∈ > by f : x → 3x + a,
g : x → b − 2x,
where a and b are constants. Given that ff (2) = 10 and g−1 (2) = 3, find
(i) the values of a and b, [4]
(ii) an expression for fg(x). [2]

68 O/N 11/P12/ Q2
Functions f and g are defined by

f : x → 2x2 − 8x + 10 for 0 ≤ x ≤ 2,
g:x→x for 0 ≤ x ≤ 10.

(i) Express f (x) in the form a(x + b)2 + c, where a, b and c are constants. [3]

(ii) State the range of f. [1]

(iii) State the domain of f −1 . [1]

(iv) Sketch on the same diagram the graphs of y = f (x), y = g(x) and y = f −1 (x), making clear the
relationship between the graphs. [4]

(v) Find an expression for f −1 (x). [3]

027
Unit 2: Function & Transformation Graph
69 O/N 11/P13/ Q9
Functions f and g are defined by f : x → 2x + 3 for x ≤ 0,
2
g : x → x − 6x for x ≤ 3.

(i) Express f −1 (x) in terms of x and solve the equation f (x) = f −1 (x). [3]
(ii) On the same diagram sketch the graphs of y = f (x) and y = f −1 (x), showing the coordinates of
their point of intersection and the relationship between the graphs. [3]
(iii) Find the set of values of x which satisfy gf (x) ≤ 16. [5]

70 M/J 11/P12/ Q6
x+3
The function f is defined by f : x → , x ∈ >, x ≠ 12 .
2x − 1
(i) Show that ff (x) = x. [3]
(ii) Hence, or otherwise, obtain an expression for f −1 (x) [2]

71 M/J 11/P11/ Q11


Functions f and g are defined for x ∈ > by f : x → 2x + 1,
g : x → x2 − 2.
(i) Find and simplify expressions for fg(x) and gf (x). [2]
(ii) Hence find the value of a for which fg(a) = gf (a). [3]
(iii) Find the value of b (b ≠ a) for which g(b) = b. [2]
(iv) Find and simplify an expression for f −1 g(x). [2]
The function h is defined by
h : x → x2 − 2, for x ≤ 0.
(v) Find an expression for h−1 (x). [2]

72 M/J 11/P13/ Q2
Find the set of values of m for which the line y = mx + 4 intersects the curve y = 3x2 − 4x + 7 at two
distinct points. [5]

73 M/J 11/P13/ Q10

Functions f and g are defined by


f : x → 3x − 4, x ∈ >,
g : x → 2(x − 1) + 8,
3
x > 1.

(i) Evaluate fg(2). [2]

(ii) Sketch in a single diagram the graphs of y = f (x) and y = f −1 (x), making clear the relationship
between the graphs. [3]

(iii) Obtain an expression for g ′(x) and use your answer to explain why g has an inverse. [3]

(iv) Express each of f −1 (x) and g−1 (x) in terms of x. [4]

028
Unit 2.1: Answer Section
1 M
M/J 22/P12/Q10/a,b,c,d
(a) x  1 or x  1, x  1 or  ,1 , 1,    x
   1
(b) 2x  1
y leading to  2 x  1 y  2 x  1 leading to 2 xy  y  2 x  1 1
2x 1
2 xy  2 x  y  1 leading to 2 x  y  1  y  1
y 1
leading to x 
2  y  1
x 1 x 1 1 1 1
[f 1  x ]  ,  or 
2  x  1 x  1 2 x 1 2
 their f  3   
(c) 2
1
leading to their f 1  3  4  f 1  3  1,1  4 
  1
5
1
(d) Sight of ‘not one to one’ or ‘many to one’ or ‘one to many’

2 M/J 22/P11/Q6
x2  4
(a) y
x2  4
leading to x 2
 
 4 y  x2  4  leading to x 2 y  4 y  x 2  4 1
1
x 2 y  x 2  4 y  4 leading to x 2 1  y   4 y  4 leading to x 2  ...

4y  4 4y  4 4x  4
x2  leading to x  leading to [f 1  x ] 
1 y 1 y 1 x
1
8 x 4 8  x2  4  8  x2  4
2
(b) 1 2  2  2  2  1
x 4 x 4 x 4 x  4  x2  4
1
0  f  x  1
(c) Because the range of f does not include the whole of the domain of f (or any
1
of it)

3 M/J 22/P13/Q6
(a)
 2  x  4 
2
9
(b) y  7
y9
(c)  x  4 2 
2
y9
x  4 
2
x9
[f 1  x  ] 4 
2
fg(x) = f(2x + 4) = 2  2 x  4  4   9
2
(d)

8 x 2  9 only

029
Unit 2.1: Answer Section
4 O/N 21/P12/Q3 (b)
(a) x+3 y+3
= y  x + 3 = xy − y OR = x  y + 3 = xy − x
 2 + 3 x −1 y −1
f ( 5 ) = [ 2] and f ( their 2 ) = [5] OR ff ( 5 ) =  
 2 −1 
x+3 y+3  x + 3
+3 xy − x = y + 3  x = OE OR y + 3 = xy − x  y =   OE
y −1  x −1 
OR x − 1 and an attempt to substitute x =5.
x+3
−1
x −1 x+3
[f −1 ( x ) or y ] =
5 x −1

5 O/N 21/P12/Q3
(b) [ k =] 2
(a)
{−3 ( x − 2 ) }
2
{+14}
14 − y
(d) y = −3 ( x − 2 ) + 14 leading to ( x − 2 )2 =
2
(c) [Range is] [y] ⩽ –13 3
14 − y
x = 2(±)
3
14 − x
[f −1 ( x )] = 2 −
3

6 O/N 21/P13/Q6 (b) −x


y=
2
(
leading to x 2 = y 2 4 − x 2 )
(a) 4− x

( )
x2 1 + y 2 = 4 y 2
2y
x = (±)
1 + y2
−2 x
f −1 ( x ) =
1 + x2
(d) −2 x
[ fg ( x ) = f ( 2 x ) =]
(c) 1 or a = 1 4 − 4 x2

−x −x x
fg ( x ) = or 2
1 − x 2 or 2 1 − x2
1 − x2 1− x x −1

7 O/N 21/P13/Q6 8 O/N 21/P13/Q6

(a) {5 ( y − 3) } 2
{+5} (a) ff( x) = 2(2x2 + 3)2 + 3
8x4 + 24x2 + 21
(b) f ′ ( x ) = 5 x − 30 x + 50
4 2
(b) 8x4 + 24x2 + 21 = 34x2 + 19 ⇒
8x4 + 24x2 – 34x2 + 21 – 19 [= 0]
( )
2
5 x 2 − 3 + 5 or b 2 < 4ac 8x4 – 10x2 + 2[= 0]
and at least one value of f’(x) > 0 [2](x2 – 1)(4x2 – 1)
 2 1  1
> 0 and increasing  x = 1 or 4 leading to  x = 1 or x = 2
 

030
Unit 2.1: Answer Section
9 M/J 21/P12/Q5 (b) 2
y = (x – 2) – 4 ⇒ (x – 2) = y + 4 ⇒2

(a) Range of f is f(x) ⩾ −4 x –2 = + ( y + 4) or ± ( y + 4)


5  f −1 ( x )  = ( x + 4) + 2
(c) (x – 2)2 – 4 = – x+2⇒  
3
5 7 (d) f -1 (12) = 6
x2 – 4x + 4 – 4 = – x + 2 [⇒ x 2 − x − 2 = 0]
3 3
g(f–1 (12)) = 6a + 2
7 ± 7 2 − 4 ( 3)( −6 )
( 3x + 2 )( x − 3) [ = 0] or OE
g(g(f–1 (12))) = a(6a + 2) + 2 = 62
6

x = 3 only 6a2 + 2a – 60 [= 0]

10
a= − or 3
10 M/J 21/P13/Q6 a 3

(a) f ( x ) = ( x − 1) + 4
2

g ( x ) = ( x + 2) + 9
2

g ( x ) = f ( x + 3) + 5

11 M/J 21/P13/Q8
(a)
 fg ( x ) = 1 / ( 2 x + 1) − 1
2

1 / ( 2 x + 1) − 1 = 3 leading to 4 ( 2 x + 1) = 1
2 2

1
or = [ ± ] 2 or 16 x 2 + 16 x + 3 = 0
( 2 x + 1)
2 x + 1 = ±½ or 2 x + 1 = −½ or ( 4 x + 1)( 4 x + 3) [ = 0]
3
x=− only
4

(b) 1 1 1
( 2 x + 1) leading to 2 x + 1 = [ ± ]
2
y= − 1 leading to =
( 2 x + 1) 2
y +1 y +1

1 1
x = [±] −
2 y +1 2

1 1
− −
2 x +1 2

12 O/N 20/P12/Q5
5(a) 0

5(b) (f−1(x)) = x + 2 , (g−1(x)) = 4 − x or 4 − 1


4 x x
x² + 6x – 16 (= 0)
( x + 8 ) and ( x − 2 )
x = ( 2 or -8)

031
13 O/N 20/P11/Q9
Unit 2.1: Answer Section
4−2 1
(a) mAB = =−
−1 − 3 2
Equation of tangent is y − 2 = 2 ( x − 3)

(b) AB 2 = 42 + 2 2 = 20 or r 2 = 20 or r = 20 or AB = 20
Equation of circle centre B is ( x − 3) + ( y − 2 ) = 20
2 2

(c) ( x − 3)2 + ( 2 x − 6 )2 = their 20

2
5 x 2 − 30 x + 25 = 0 or 5 ( x − 3) = 20

( 5)( x − 5)( x − 1) or x − 3 = ±2  x = 5, 1

14 O/N 20/P13/Q6
2x
(a) y= → 3xy − y = 2 x → 3xy − 2 x = y (or − y = 2 x − 3xy )
3x − 1
y −y
x (3 y − 2) = y → x = (or x = )
3y − 2 2 − 3y
x
( )
f −1 ( x ) =
3x − 2
 2 ( 3 x − 1) + 2   6x 2x 
(b)   =  = 
 3 ( 3 x − 1)   3 ( 3 x − 1) 3 x − 1 

(c) 2
(f(x)) >
3
15 M/J 20/P12/Q5
a−x
(a) ff(x) = a − 2 ( a − 2 x ) (b) 4x − a = → 9 x = 3a
2
ff(x) = 4 x − a a
x=
a−x 3
f−1(x) =
2

16 M/J 20/P11/Q6
1
(a) 3( 3x + b ) + b = 9 x + 4b → 10 = 18 + 4b (b) gf(x) = 3  x − 5  − 2
2 
b = −2 3
Either f(14) = 2 or f−1(x) = 2(x + a) etc. gf(x) = x − 17
2
a=5

032
Unit 2.1: Answer Section
17 M/J 20/P13/Q9

(a) ( x − 2 )2  [ −1] (d)


  1 1
gf ( x ) = = OE
(b) Smallest c = 2 ( x − 2) 2
−1 +1 ( x − 2 )2
(FT on their part (a))
(c) y = ( x − 2) − 1 → ( x − 2) = y + 1
2 2
1
Range of gf is 0 < gf(x) <
9
x = 2(±) y +1

(f −1 ( x )) = 2 + x + 1 for x > 8

18 Mar 20/P12/Q9

(a)  2 ( x + 3 ) 2  [ −7 ]
 
y+7
y = 2 ( x + 3) − 7 → 2 ( x + 3) = y + 7 → ( x + 3 ) =
2 2 2
(b) 1
2 (d) (Largest k is) −
2
y+7 y+7 x+7
x + 3 = (±) → x = (±) − 3 → f −1 ( x ) = − −3
2 2 2
Domain: x ⩾ ‒5 or ⩾ -5 or [–5, ∞)
(c) fg(x) = 8 x 2 − 7
8 x 2 − 7 = 193 → x 2 = 25 → x = −5 only
g(x) = f −1 (193) → 2 x − 3 = − 100 − 3
x = ‒5 only

19 O/N 19/P12/Q9

(i) f(x) = g(x) → 2x2 + 6x + 1 + k (= 0)


(ii) 2 x² + 6x – 8 (< 0)
Use of b2 = 4ac – 4 and 1
(k =) 3½ −4<x<1
Alternative method for question 9(i) −1 x −1
(iii) (g (x)) =
4 x + 8 = 2 (→ x = –1½) 2
Substitutes their x value into either 2 2 x² + 8x + 1 − 1
(g−1f(x)) =
2 x² + 6x + 1 + k = 0 OR into the 2
 −13  = 0 → (2x² + 8x = 0) → x =
curve to find y  =  then both values into the line.
 2  0, –4
(k =) 3½
(iv) 2( x + 2)2 − 7

(Least value of f(x) or y =) −7 or ⩾ −7

033
20 O/N 19/P11/Q7 Unit 2.1: Answer Section

(i) Range of f is 0 < f( x) < 3 3 3x


(ii) ( fg ( x ) = ) 2 =
Range of g is g(x) > 2 ( 1
x
+ 2) + 1 2 + 5x
3x
(iii) y = → 2 y + 5 xy = 3 x → 3 x − 5 xy = 2 y
2 + 5x
2y
x (3 − 5 y ) = 2 y → x =
3 − 5y
(( fg ) ( x ) =
−1
)
2x
3 − 5x

21 O/N 19/P13/Q2

( y = ) ( x − 3)2  [ −2]


x − 3 = ( ± ) y + 2 or y − 3 = ( ± ) x + 2

(g −1
( x )) = 3 + x+2
Domain (of g −1 ) is (x) > ‒1

22 M/J 19/P12/Q7

(i) (f−1(x)) = x + 2 oe (ii)


3 3 ( 2 x + 3) 7
2x + 3 ( fg ( x ) = ) x −1
– 2 (= )
y= → (x – 1)y = 2x + 3 → x ( y − 2) = y + 3 3
x −1 18 x + 27 = 13x – 13 or 3(4x + 11) = 7(x – 1)
x+3  5 
(g−1(x) or y) = oe  eg + 1 (5x = – 40)
x−2  x−2 
x ≠ 2 only Alternative method for question 7(ii)
7 13
(f−1( )) =
3 9
2 x + 3 13
= → 9(2x + 3)= 13(x – 1) (→ 5x = – 40)
x −1 9
x = −8

23 M/J 19/P11/Q5

(i) −2 ( x − 3) + 15 (a = − 3, b = 15)
2
(ii) (f(x) ⩽) 15
(iii) gf(x) = 2( −2 x ² + 12 x − 3 ) + 5 = −4x² + 24x – 6 + 5
gf(x) + 1 = 0 → −4x² + 24x = 0
x = 0 or 6

24 M/J 19/P13/Q1
(i)  ( x − 2 ) 2  [ +4 ]
 
(ii) ( x − 2)
2
< 5 → − 5 < x − 2 and/or x − 2 < 5

2− 5 < x<2+ 5

034
Unit 2.1: Answer Section
25 M/J 19/P13/Q4 96 96 96
(iii) y= − 4→ y + 4 = → x −1 =
(i) Max (a) is 8 x −1 x −1 y+4
96
Min(b) is 24 ( gf ) ( x ) =
−1
+1
x+4
(ii) 96 100 − 4 x
gf(x) = − 4 or gf(x) =
x −1 x −1

26 March 19/P12/Q8

(i) 2 2
( x − 2 )2  + [3] (iv) gf(x) = =
  x − 4 x + 7 − 1 ( x − 2 )2 + 2
2

(ii) Largest k is 2 Accept k - 2 Since f ( x ) > 4 ⇒ gf ( x ) < 2 / 3 (or since x < 1etc )
(iii) y = ( x − 2) + 3 ⇒ x − 2 = ( ± ) y − 3
2
range of gf(x) is 0 < gf(x)( < 2/3)

⇒ f −1 ( x ) = 2 − x − 3 for x > 4

27 O/N 18/P12/Q4
(ii)
1
(i) fg(x) = 2 – 3cos( x )
2
1 1 1
2 – 3cos( x ) = 1 → cos( x ) = →
2 2 3
1  −1  1 
 x  = cos  their 
2   3
4.7 π
x = 2.46 awrt or ( 0.784πawrt )
6

y = 2 ( x − 3) − 11 → y + 11 = 2(x – 3)²
28 O/N 18/P12/Q9 2
(iv)
9(i) 2x² − 12x + 7 = 2 ( x − 3) − 11
2 y + 11
= ( x − 3) ²
9(ii) Range (of f or y) ⩾ ‘their – 11’ 2

9(iii) (k =) –“their a” also allow x or k ⩽ 3  y + 11   y + 11 


x = 3+   or 3 −  
 2   2 

 x + 11 
(g−1(x) or y) = 3 −  
 2 

29 O/N 18/P11/Q11

(a)(i) [Greatest value of a is] 3


(a)(ii) Range is y > −1

y = ( x − 3) − 1→ ( x − 3) = 1 + y → x = 3 ( ± ) 1 + y
2 2

f −1 ( x ) = 3 − 1 + x cao
(b)(i) 2
gg(2x) = ( 2 x − 3) − 3
2
 

( 2 x − 3 )4 − 6 ( 2 x − 3)2 + 9
(b)(ii)  4 3 2
  2
( 
16 x − 96 x + 216 x − 216 x + 81 +  −24 x + 72 x − 54 + 9  )
4 3 2
16 x − 96 x + 192 x − 144 x + 36

035
30 O/N 18/P13/Q11 Unit 2.1: Answer Section

[ 2] ( x − 3)2  [ −7] (iv) x + 3 - 1 . Allow x + 3 = 1


(i)
(ii) Largest value of k is 3. Allow (k = ) 3. largest p is − 2 . Allow (p =) ‒2

(iii) y = 2 ( x − 3) − 7 → ( x − 3) = ½ ( y + 7 )
2 2
fg ( x ) = f ( x + 3) = 2 x 2 − 7 cao
or with x/y transposed
x = 3 ± ½ ( y + 7 ) Allow 3 + √ or 3 ‒√
or with x/y transposed
f −1 ( x ) = 3 − ½ ( x + 7 )
(Domain is x) . their − 7

31 M/J 18/P12/Q7
(iv) EITHER
(i) 25 – 2(x + 3)² y = 25 – 2(x + 3)² → 2(x + 3)² = 25 – y
(ii) (–3, 25)
x + 3 = ( ± ) ½ ( 25 – y )
OR
y = 7 – 2x2 – 12x → 2x2 + 12x + y – 7 (= 0)
(iii) (k) = –3 also allow x or k ⩾ – 3
−12 ± 122 − 8 ( y − 7 )
x=
4
 25 − x 
g─1(x) =   – 3 oe
 2 
isw if substituting x = – 3

32 M/J 18/P11/Q9
x x2
f:x↦ − 2, g:x↦4+x–
2 2

(i) x2 x
4+x– = − 2 → x ² − x − 12 = 0
2 2
→ (4, 0) and (−3, −3.5)
Trial and improvement, B3 all correct or B0

(ii) f( x) > g(x) for x > 4, x < −3


(iii) x x2 x x2
fg(x) = 2 + − – 2 (= − )
2 4 2 4
1 dy 1 2 x
i.e. − ((x − 1)² − 1) or = − =0 → x=1
4 dx 2 4

1 1
→y= → Range of fg ⩽ ,
4 4

(iv) Calculus or completing square on ‘h’ → x = 1

k = 1 (accept k . 1 )

036
33 M/J 18/P13/Q10 Unit 2.1: Answer Section

(i) Smallest value of c is 2. Accept 2, c = 2 , c . 2 . Not in terms of x


(ii) y = ( x − 2) + 2 → x − 2 = ( ± ) y − 2 → x = ( ± ) y − 2 + 2
2

f −1 ( x ) = x − 2 + 2
Domain of f −1 is x . 6 . Allow . 6.
(iii) 2
( x − 2 )2 + 2 − 2  + 2 = 51 SOI Allow 1 term missing for M mark
 
( ) ( )
2
Or x 2 − 4 x + 6 − 4 x 2 − 4 x + 6 + 6 = 51

( x − 2)
4
= 49 or ( x 2 − 4 x + 4) 2 = 49
OR x 4 − 8 x 3 + 24 x 2 − 32 x − 33 = 0 often implied by next line

( x − 2) = ( ± ) 7 OR x 2 − 4 x − 3 = 0 . Ignore x 2 − 4 x + 11 = 0
2

x = 2 + √ 7 only CAO x = 2 + 4 49 scores 3/5

34 O/N 17/P12/Q2

(i) 4− x 2(ii)
5
Equate a valid attempt at f-1 with f, or with x, or f with x
2 2
→  ,  or (0.667, 0.667)
3 3

35 O/N 17/P11/Q9

(i) gg( x) = g(2x ‒ 3) = 2(2x ‒ 3) ‒ 3 = 4x ‒ 9

1 1 (iii) EITHER:
(ii) y= 2
→ x 2 = + 9 OE 1 1
x −9 y =
( 2 x − 3) 2
−9 7
1
f −1
( x) = + 9 ( 2 x − 3)2 = 16 or 4 x 2 − 12 x − 7 = 0
x
1 x = 7/2 or ‒1/2
Attempt soln of + 9 > 3 or attempt to find range of f.
x x = 7/2 only
( y > 0)
OR:
Domain is x > 0 CAO 1
g(x) = f −1  
7

g(x) = 4

2x ‒ 3 = 4

x = 7/2

037
36 O/N 17/P13/Q6 Unit 2.1: Answer Section

2
2 2  2 
y= ⇒ x2 = +1 OE
(i) 2
x −1 y (ii)  2  +1 = 5
 x −1

2 2
x = (±) +1 OE 2
= (±)2 OE OR x 4 − 2 x 2 = 0 OE
y x −1
x 2 − 1 = ( ± )1 ⇒ x 2 = 2 ( or 0 )
2
f −1 ( x ) = − +1 OE x=− 2 or −1.41 only
x

37 M/J 17/P11/Q9
2
(ii) gf(−1) = 3 f(−1) =
2 5
f:x⟼ g : x ⟼ 4x + a,
3 − 2x 8 7
+a =3 → a =
2 2 5 5
(i) y= → y (3 − 2x ) = 2 → 3 − 2x = x − a
3 − 2x y (iii) g−1(x) = = f−1(x)
4
2 3 1 → x2 − x ( a + 6) + 4 ( = 0)
→ 2x = 3 − → f−1(x) = −
Solving ( a + 6 ) = 16 or a 2 + 12a + 20 ( = 0 )
2
y 2 x
→ a = −2 or −10
38 M/J 17/P13/Q9
(i) ( 3x − 1)
2
+5 (iii) For start ( − ) + or ( − )
(ii) Smallest value of p is 1/3 seen. (Independent of (i)) + (a≠ 0) ft for their a, b, c
(iii) y = ( 3x − 1)2 + 5 ⇒ 3x − 1 = ( ± ) y − 5 For start ( − ) + ft but award only
B1 for 3 correct operations
x = ( ± ) ⅓ y − 5 + ⅓ OE For start ( − ) + ft but award B1
f −1 ( x ) = ⅓ x − 5 + ⅓ OE domain is x ≥ their 5 for first2 operations correct and B1
for the next 3 operations correct
(iv) q < 5 CAO

39 O/N 16/P12/Q10
(i) 3 ⩽ f(x) ⩽ 7 (iii) 5-2sin2x = 6 → sin2x = −½
7π 11π
(ii) → 2x = or
6 6
7π 11π
→ x= or
12 12
0.583π or 0.917π
π + 0.524 2π − 0.524
or
2 2
1.83c or 2.88c
π
(iv) k=
4
(v) 2sin2x = 5 – y → sin2x = ½(5 – y)

(g−1(x)) = ½ sin−1
(5 − x )
40 O/N 16/P11/Q8
2
(ii) y = 4 / (5x + 2) ⇒ x = ( 4 − 2 y ) / 5 y oe
(i) fg(x) = 5x
Range of fg is y . 0 oe g −1 ( x ) = ( 4 − 2 x ) / 5 x oe
0, 2 with no incorrect inequality
0 < x - 2 oe, c.a.o.

038
Unit 2.1: Answer Section
41 O/N 16/P13/Q8
y = ( 2 x + 3) + 1 ⇒ 2 x + 3 = ( ± ) y − 1 or ft from (i)
2
(iii)
( 2 x + 3)
2
(i) +1 1 3
x = (±) y − 1 − or ft from (i)
Cannot score retrospectively in (iii) 2 2
1 3
(ii) g ( x ) = 2 x + 3 cao ( fg ) ( x ) = x − 1 − cao Note alt. method g−1f −1
−1

2 2
Domain is ( x ) > 10
ALT. method for first 3 marks:
Trying to obtain g −1  f −1 ( x ) 
g −1 = ½ ( x − 3 ) , f −1 = x − 1
1 3
A1 for x −1 −
2 2
42 M/J 16/P12/Q1
10
f : x ⟼10 – 3x, g : x ⟼ ,
3 − 2x
ff(x) = 10 − 3 (10 − 3x )
10
gf(2) = (= −2)
3 − 2 (10 − 3 ( 2 ) )
x=2
43 M/J 16/P12/Q11 iii,iv,v
(iv) k = 3.
(iii) 6x – x² − 5 = 4 − (x – 3)²
(v) g−1(x) = 4− x + 3

44 M/J 16/P11/Q6/b

(b) (i) f(x) = x² + ax + b


Eqn of form ( x − 1)( x − 9 )
→ a = −10, b = 9
(or using 2 sim eqns M1 A1)
1
(ii) Calculus or x = (1 + 9) by symmetry
2
→ (5, −16)

45 M/J 16/P11/Q11
π π (iii)
(i) f : x → 4sinx – 1 for − ⩽x⩽
2 2
Range −5 ⩽ f(x) ⩽ 3
1
(ii) 4s – 1 = 0 → s = → x = 0.253
4
x = 0 → y = −1 1 1
(iv) range − π ⩽ f−1(x) ⩽ π
2 2
domain −5 ⩽ x ⩽ 3
 x +1
Inverse f−1(x) = sin−1  
 4 
46 M/J 16/P13/Q10

(i) ( )
2 ax 2 + b + 3 = 6 x 2 − 21
(ii) 3x 2 − 12  0 or 6 x 2 − 21  3
a = 3, b = −12
x  − 2 i.e. (max) q = −2
(iii) y  6( −3) 2 − 21⇒ range is ( y )  33

039
Unit 2.1: Answer Section
46 M/J 16/P13/Q10
y + 21
(iv) y = 6 x 2 − 21 ⇒ x = ( ± )
6
x + 21
( fg )−1 ( x ) = −
6
Domain is x  33

47 O/N 15/P12/Q1
f : x ⟼3x + 2, g : x ⟼4x – 12
x−2
f−1(x) =
3
gf(x) = 4(3x + 2) – 12
2
Equate → x =
7
48 O/N 15/P12/Q8

f : x → x 2 + ax + b ,
(ii) (x – k)(x + 2k) = 0
2 2
(i) x + 6 x − 8 = ( x + 3) − 17 ≡ x 2 + 5x + b = 0
or 2x + 6 = 0 → x = −3 → y = −17 → k=5
→ Range f(x) ⩾ − 17 → b = −2k² = − 50

(iii) ( x + a) 2 + a( x + a) + b = a
Uses b² − 4ac → 9a² − 4(2a² + b − a)
→ a² < 4(b – a)

49 O/N 15/P11/Q9
(iii) ( x − 3) 2 = 4 − y
(i) − (1)( x − 3) + 42
Correct order of operations
(ii) Smallest (m) is 3
f -1 ( x) = 3 + 4 − x cao
Domain is x ⩽ 0
50 O/N 15/P13/Q8
(iii) gf(x) = −1 − (3 x + 1) 2 oe
(i) 3x + 1 ⩽–1 (Accept 3 x + 1 = −1,3a + 1 = −1)
x⩽ − 2 / 3 ⇒ largest value of a is – 2/3 ( in terms gf(x) ⩽ − 50 ⇒ (3 x + 1) 2 ⩾49 (Allow ⩽ or =
of a ) 3x + 1 ⩾7 or 3x + 1 ≤ −7 (one sufficient)
(ii) fg(x) = 3(−1 − x 2 ) + 1 x⩽ − 8 / 3
fg(x) + 14 = 0 ⇒ 3x 2 = 12 oe (2 terms)
x = –2 only

51 M/J 15/P12/ Q11

f : x a 2x2 − 6x + 5 (iv) Smallest k = 3


2
(i) 2 x 2 − 6 x + 5 − p = 0 has no real roots
(v) h(x) = 2(x − 32 ) + 12
2
2
Uses b − 4ac → 36 − 8(5 − p )
3
1 Order of operations ± 12 , ÷2, √, ± 2
Sets to 0 → p < 2
→ Inverse = 3
+ ( 2x − 14 )
2 x − 6 x + 5 = 2(x − )
3 2
2 1 2
(ii) 2
+ 2

(iii) Range of g 1
2 < g(x) < 13
2
h : x a 2 x − 6 x + 5 for k < x < 4

040
52 M/J 15/P13/ Q6 Unit 2.1: Answer Section
−1 −1
(i) Attempt to find (f )
1 5 1− 5
2 xy = 1 − 5 x or
1
2x
= y+
2
Allow 1 sign error (ii)
f −1
()
1 =
x 2
x

x= oe Allow 1 sign error (total) x


2y + 5 x−5 1 5
1 9 or x−
(f ( x)) = for x [ − 2 2 2
2x + 5 4
9
(Allow − Y x Y ∞ )
4

53 O/N 14/P11/ Q10

(iii) 9 Y (x – 1)2 – 16 Y=65 OR x 2 − 2 x − 15 = 9 → 6,−4


2
(i) ( x − 1) − 16 25 Y=(x – 1)2 Y=81 x 2 − 2 x − 15 = 65 → 10,−8
(ii) –16 5 Y=x – 1 Y=9 p=6
6 Y=x Y=10 q = 10
(iv) x = ( y − 1) 2 − 16 [interchange x/y]
y − 1 = (±) x + 16
f −1 ( x) = 1 + x + 16 1
(ii) x = (7 y + 1) 3 (x/y interchange as first or last
54 O/N 14/P13/ Q10/a step)
1 2 x 3 = 7 y + 1 or y 3 = 7 x + 1
(a) (i) (a + b) 3 = 2, ( 9a + b ) 3 = 16
a + b = 8, 9a + b = 64
1 3
f −1 (x ) =
7
x − 1 cao ( )
a = 7, b = 1 Domain of f −1 is x > 1 cao

55 M/J 14/P12/ Q10


(iv) gf(x) = (2x − 3)² + 4(2x − 3) = p
f : x a 2x − 3, x ∈ , → 4x² − 4x − 3 − p = 0
g : x a x² + 4x, x ∈ . Uses "b² − 4ac" 16 = 16(−3 −p)
→ p=−4
(i) ff = 2(2x − 3) −3
Solves = 11 → x = 5 (v) − 2
(or 2x−3 =11, x = 7. 2x−3=7 → x = 5) y = (x + 2) − 4
2
(vi)
(ii) min at x = − 2
y+4 = x+2
→ Range [ −4
−1
(iii) x ² + 4x − 12 ( > 0) h (x) = x+4 −2
→ x = 2 or − 6
→ x < − 6 , x > 2.

56 M/J 14/P11/ Q10


1
(i) –5 Y f(x) Y 4 For f(x) allow x or y; (iii) LINE: f–1(x) = (x + 2)
3
allow <, [–5, 4], (–5,4)
for –5 Y x Y 1
(ii) f–1(x) approximately correct (independent of f) 4 4
CURVE: 5 − y = OR x = 5−
Closed region between (1, 1) and (4, 4); line x y
reaches x-axis 4
f −1 (x ) = 5 − oe
x
for 1 < x Y 4

041
Unit 2.1: Answer Section
57 O/N 13/P11/ Q5
(ii) ( )2
ff (x ) = x 2 + 1 + 1
(i) x = (± ) y − 1
x 2 + 1 = (± )13 / 4
−1
f : x a x − 1 for x > 1 x = 3/2
Alt. (ii) f (x ) = f −1 (185 / 16 ) = 13 / 4
x = f −1 (13 / 4 )
x = 3/ 2

58 O/N 13/P13/ Q10

10 (i) Range is (y) ≥ c2 + 4c (ii) 5a + b = 11


x2 + 4x = (x + 2)2 – 4 (a + b)2 + 4 (a + b) = 21
(Smallest value of c is) ‒2 (11 – 5a + a)2 + 4 (11 – 5a + a) = 21
Alt. (ii) Last 5 marks (8) (2a2 – 13a + 18) = (8) (2a – 9) (a – 2)
=0
f–1 (x) = x + 4 − 2 9  23 
g (1) = f–1 = (21) used a = , 2 OR b =  −  , 1
2  2 
a + b = 25 – 2 = 3
Solve a + b = 3, 5a + b = 11 M1
a = 2, b = 1
59 M/J 13/P12/ Q8

250 dS 500π
8 (i) πr 2 h = 250π → h = (ii) = 4πr − 2
r2 dr r
→ S = 2πrh + 2πr 2 = 0 when r³ = 125 → r = 5
500π → S= 150π
→ S = 2πr 2 +
r d2S 1000π
(iii) = 4π +
dr 2
r3
This is positive → Minimum
60 M/J 13/P13/ Q10
2
f : x a 2x + k , g : x a x − 6x + 8 , (ii) x 2 − 6 x + 8 = 2x + k
x 2 − 8x + 8 − k = 0
(i) 2(2x + 3) + 3 = 25 Uses b² − 4ac < 0
→x=4 →k<−8
or {f(11) = 25, f(4) = 11}
(iii) x 2 − 6 x + 8 = (x − 3)² − 1
y = (x − 3)² − 1
Makes x the subject → ±√(x + 1) + 3
61 O/N 12/P12/ Q2 Needs specifically to lose the “−”.

x+3 (ii) domain of f –1 is ≥ 1.


(i) f(x) = + 1 , for x ≥ −3
2
Make x the subject or interchanges x,y
→ 2( x − 1) 2 − 3
→ 2x 2 − 4x − 1

62 O/N 12/P11/ Q10



(iii)   3   25
 
(i) 4  3  25 
Vertex is (3, –25) 3 B C  25

  
g 3  √  25

(ii) range is ( g(x) ) ≥ –9 Allow > Domain is x ≥ –9

042
63 O/N 12/P13/ Q7
Unit 2.1: Answer Section
(i) 11 – x = 5 – x ⇒ x – x – 6(= 0)
2 2 11 – x for 2 ≤ x ≤ 11
(ii) f –1 x= 
(x + 2)(x – 3)
p = 3; q = 2 5 – x for x < 2

64 M/J 12/P12/ Q10


8
10 f : x a 2 x + 5 g : x a 16
x−3 (iii) fg(x) = +5
(i) f –1
= ½(x − 5) x−3
8 16
g –1 = + 3 , x ≠ 0 Forms eqn + 5 = 5 − kx
x x−3
kx 2 − 3kx + 16 = 0
(ii) 64
y = f(x) y = x Uses b² = 4ac → k = or 0
9
y y = f–1(x) 64
Set of values 0 < k <
9

65 M/J 12/P11/ Q8

(i) (x − 2)2 − 4 + k (iv) x − 2 = (± ) y + 4 − k


x = 2+ y+4−k
(ii) f(x) > k – 4 or [k – 4, ∞ ] or (k – 4, ∞ ) oe f −1 ( x ) = 2 + x + 4 − k
Domain is x > k – 4 or [k – 4, ∞ ]
or (k – 4, ∞ ) oe
(iii) smallest value of p = 2

66 M/J 12/P13/ Q11


(iii) y = 8 − ( x − 2) 2
2
f(x) = 8 − ( x − 2) , → ( x − 2) 2 + y = 8
(i) Stationary point at x = 2 → (x − 2) = ± 8 − y
y – coordinate = 8
Nature Maximum → g –1 = 2 + 8 − x
(iv)
(or y y = − x 2 + 4 x + 4
−2x + 4 = 0 → (2, 8) Max)
(ii) k = 2

67 O/N 11/P12/ Q2

f : x  3x + a , g : x  b − 2 x
(ii) fg(x) = 3(b − 2 x) + a
(i) f2(x) = 3(3x + a) + a
f²(2) = 18 + 4a = 10 → a = −2 = 22 − 6x
b−x b−2
g–1(x) = → =3 b=8
2 2
or g(3) = 2 → b − 6 = 2 b=8

043
68 O/N 11/P11/ Q11 Unit 2.1: Answer Section

(iv) f ( x ) :≈ half parabola from (0,10) to (2,2 )


(i) 2( x − 2 ) + 2
2

g ( x ) : line through 0 at ≈ 45°


(ii) 2 ≤ f (x ) ≤ 10
f −1
(x ) : reflection of their f (x ) in g (x )
(iii) 2 ≤ x ≤ 10 Everything totally correct

69 O/N 11/P13/ Q9

−1 1 3 (iii) gf(x) = (2x + 3)2 – 6(2x + 3)


(i) f ( x) = x − 4x2 – 9
2 2
1 3 25
2 x + 3 = x − ⇒ x = −3 4x2 – 9 ≤ 16 ⇒ x2 ≤
2 2 4
5
(ii) 2 lines approximately correct, − ≤ x≤0
reflected in y=x & meeting at (‒3, ‒3) 2

70 M/J 11/P12/ Q6 x+3


(ii) y =
x+3 2x − 1
(i) f(x) = → 2 xy − y = x + 3
2x − 1
x+3 → x(2 y − 1) = y + 3
+3 x+3
7x
ff(x) = 2 x − 1 = =x → f –1(x) =
2( x + 3) 7 2x − 1
−1 or since ff(x) = x,
2x − 1
x+3
f –1(x) = f(x) = (M1, A1)
2x − 1

71 M/J 11/P11/ Q11


(iii) b2 – b – 2 = 0 → (b + 1)(b – 2) = 0
2 2
(i) fg(x) = 2x – 3, gf(x) = 4x + 4x – 1 b=2 Allow b = –1 in addition
1
(ii) 2a 2 − 3 = 4a 2 + 4a − 1 ⇒ 2a 2 + 4a + 2 = 0 (iv) f –1(x) = ( x − 1)
2
(a + 1)2 = 0 1 2
–1
a = –1 f g(x) = 2 ( x − 3)

(v) x = (±) y + 2
–1
h (x) = − x + 2
72 M/J 11/P13/ Q2

y = mx + 4 y = 3x2 – 4x + 7
Equate → 3x2 – (4 + m)x + 3 = 0
Uses b2 – 4ac → (4 + m)2 – 36
Solution of quadratic m = 2 or –10
Set of values m > 2 or m < –10

044
Unit 2.1: Answer Section
73 M/J 11/P13/ Q10

g : x a 2(x − 1) + 8
3
f : x a 3x − 4

(i) fg(2) = f(10) = 26


f –1(x)

(ii)
f(x)

(iii) g′(x) = 6(x – 1)2


g′(x) > → no turning points
→ g is 1 : 1, g has an inverse.

x+4
(iv) f −1 (x ) =
3
Attempt at making x
Order correct. – 8, ÷ 2, 3 ,+1
x −8
g −1 (x ) = 3 +1
2

045
UNIT 2 Function and Transformation Graph Unit 2.2: Transformation

2.2: Transformation
1 M
M/J 22/P11/Q8/a
 
(a) The curve y = sin x is transformed to the curve y = 4 sin 1 x − 30Å .
2
Describe fully a sequence of transformations that have been combined, making clear the order
in which the transformations are applied. [5]

2 M/J 22/P13/Q4
@ A
−1
(a) The curve with equation y = x2 + 2x − 5 is translated by .
3
Find the equation of the translated curve, giving your answer in the form y = ax2 + bx + c. [3]
(b) The curve with equation y = x + 2x − 5 is transformed to a curve with equation y = 4x + 4x − 5.
2 2

Describe fully the single transformation that has been applied. [2]

3 O/N 21/P12/Q2
The graph of y = f x is transformed to the graph of y = f 2x − 3.
(a) Describe fully the two single transformations that have been combined to give the resulting
transformation. [3]
The point P 5, 6 lies on the transformed curve y = f 2x − 3.
(b) State the coordinates of the corresponding point on the original curve y = f x [2]

4 O/N 21/P13/Q1
The graph of y = f x is transformed to the graph of y = 3 − f x.

Describe fully, in the correct order, the two transformations that have been combined. [4]

5 M/J 21/P12/Q2
(a) The graph of y = f x is transformed to the graph of y = 2f x − 1.
Describe fully the two single transformations which have been combined to give the resulting
transformation. [3]
(b) The curve y = sin 2x − 5x is reflected in the y-axis and then stretched by scale factor 13 in the
x-direction.
Write down the equation of the transformed curve. [2]

6 M/J 21/P11/Q4
y

x
− 14 π 0 1

1

3

π 5

3

7

−2

−4

−6

The diagram shows part of the graph of y = a tan x − b + c.

Given that 0 < b < π, state the values of the constants a, b and c. [3]

046
7 M/J 21/P13/Q6/b Unit 2.2: Transformation
Functions f and g are both defined for x ∈ > and are given by
f x = x2 − 2x + 5,
g x = x2 + 4x + 13.
(a) Describe fully the transformation which transforms the graph of y = f x to the graph of y = g x.
[2]
8 O/N 20/P12/Q11/d,e
Functions f, g and h are defined for x ∈ > by
f x = 3 cos 2x + 2,
g x = f 2x + 4,
 
h x = 2f x + 12 π .
(a) Describe fully a sequence of transformations that maps the graph of y = f x on to y = g x. [2]
(b) Describe fully a sequence of transformations that maps the graph of y = f x on to y = h x. [2]

9 O/N 20/P11/Q4
y

1 y = cos 1
1
0 π 2π 3π 4π
−1

In the diagram, the lower curve has equation y = cos 1. The upper curve shows the result of applying
a combination of transformations to y = cos 1.
Find, in terms of a cosine function, the equation of the upper curve. [3]

10 O/N 20/P13/Q1
(a) Express x2 + 6x + 5 in the form x + a2 + b, where a and b are constants. [2]
(b) The curve with equation y = x2 is transformed to the curve with equation y = x2 + 6x + 5.

Describe fully the transformation(s) involved. [2]

11 M/J 20/P12/Q9
Functions f and g are such that
f x = 2 − 3 sin 2x for 0 ≤ x ≤ π,
g x = −2f x for 0 ≤ x ≤ π.

(a) State the ranges of f and g. [3]


The diagram below shows the graph of y = f x.

047
Unit 2.2: Transformation

y = f x

x
O π

(b) Sketch, on this diagram, the graph of y = g x. [2]


The function h is such that
h x = g x + π for − π ≤ x ≤ 0.

(c) Describe fully a sequence of transformations that maps the curve y = f x on to y = h x. [3]

12 M/J 20/P11/Q4
y

y = f x

x
O π

The diagram shows the graph of y = f x, where f x = 32 cos 2x + 12 for 0 ≤ x ≤ π.

(a) State the range of f. [2]


A function g is such that g x = f x + k, where k is a positive constant. The x-axis is a tangent to the
curve y = g x.
(b) State the value of k and hence describe fully the transformation that maps the curve y = f x on
to y = g x. [2]

(c) State the equation of the curve which is the reflection of y = f x in the x-axis. Give your answer
in the form y = a cos 2x + b, where a and b are constants. [1]

048
Unit 2.2: Transformation
13 M/J 20/P13/Q3
In each of parts (a), (b) and (c), the graph shown with solid lines has equation y = f x. The graph
shown with broken lines is a transformation of y = f x.
(a)
y

2
y = f x
1

x
−3 −2 −1 0 1 2 3

State, in terms of f, the equation of the graph shown with broken lines. [1]

(b)
y

2
y = f x
1

x
0 1 2 3

State, in terms of f, the equation of the graph shown with broken lines. [1]
y
(c)
3

2
y = f x
1

x
−3 −2 −1 0 1 2 3
−1

−2

State, in terms of f, the equation of the graph shown with broken lines. [2]

14 MAR/ 20/P12/Q2
 
The graph of y = f x is transformed to the graph of y = 1 + f 12 x .

Describe fully the two single transformations which have been combined to give the resulting
transformation. [4]

049
UNIT 2 Function and Transformation Graph Unit 2.2: Answer Section

2.2: Transformation Answer Section


1 M
M/J 22/P11/Q8/a
 30   60 
(a) EITHER (1){Translation}   OR (2){Translation}  
 0   0 
(3){Stretch} {factor 2} {in x-direction}
(4) Stretch factor 4 in y-direction and correct order

2 M/J 22/P13/Q4
(a)  x  1 2

 2  x  1  5  3 , or  x  1  1   6  3
2

 y  x2  4 x  1
(b) {Stretch}{ x direction or horizontally or y-axis invariant}{ factor ½}

3 O/N 21/P12/Q2
1
(a) Stretch with [scale factor] either ± 2 or ±
1 2
Scale factor in the x-direction
2
 0
Translation   or translation of 3 units in negative y-direction
 −3 
(b) ( 10 , 9)

4 O/N 21/P13/Q1
{Reflection} {[in the] x-axis}
or
{Stretch of scale factor -1} {parallel to y-axis}

 0  
Then {Translation}   
 3  
5 M/J 21/P12/Q2
(a) 1
Translation  
Stretch 0
Factor 2 in y -direction
(b) [ −sin 6x][ + 15x] or [sin(−6x)][ + 15x] OE

6 M/J 21/P11/Q4
π
a=2 b= c=1
4
7 M/J 21/P13/Q6/b
(b) Translation or Shif t
 −3 
  or acceptable explanation
5

8 O/N 20/P12/Q11/d,e
1 (b)  π
(a) Stretch by (scale factor) −
2 Translation of  2 
 
, parallel to x-axis or in x direction (or  0 
horizontally) Stretch by (scale factor) 2 parallel to
 0 y-axis (or vertically).
Translation of  
 4

050
9 O/N 20/P11/Q4 Unit 2.2: Answer Section

( y = ) [3] + [ 2] cos θ 
1
 2 

10 O/N 20/P13/Q1

(a) ( x + 3)2  [ −4]


 
(b)  −3 
[Translation or shift]  
 −4 
11 M/J 20/P12/Q9

(a) f( x) from −1 to 5

g(x) from −10 to 2


(FT from part (a))

(b)

(c) Reflect in x-axis


Stretch by factor 2 in the y direction
 0 
Translation by –π in the x direction OR translation by   .
 −π 
12 M/J 20/P11/Q4

(a) −1 ⩽ f(x) ⩽ 2

(b) k=1 Translation by 1 unit upwards parallel to the y-axis

3 1
(c) y=− cos2 x −
2 2
13 M/J 20/P13/Q3

(a) ( y ) = f ( −x)
(b) ( y ) = 2f ( x )
(c) ( y ) = f ( x + 4) − 3

14 MAR/ 20/P12/Q2

[Stretch] [factor 2, x direction (or y-axis invariant)]


[Translation or Shift] [1 unit in y direction] or
 0  
[Translation/Shift]   
 1  
051
UNIT 3 Coordinate Geometry Unit 3: Coordinate Geometry

1 M
M/J 22/P12/Q8
The equation of a circle is x2 + y2 + ax + by − 12 = 0. The points A 1, 1 and B 2, −6 lie on the
circle.

(a) Find the values of a and b and hence find the coordinates of the centre of the circle. [4]
(b) Find the equation of the tangent to the circle at the point A, giving your answer in the form
px + qy = k, where p, q and k are integers. [4]
M
2 M/J 22/P11/Q9/a
The equation of a circle is x2 + y2 + 6x − 2y − 26 = 0.
(a) Find the coordinates of the centre of the circle and the radius. Hence find the coordinates of the
lowest point on the circle. [4]
3 M/J 22/P13/Q7 y

B 0, 2

O P
x

x − 22 + y + 42 = 20

The diagram shows the circle with equation x − 22 + y + 42 = 20 and with centre C. The point B
has coordinates 0, 2 and the line segment BC intersects the circle at P.

(a) Find the equation of BC. [2]


(b) Hence find the coordinates of P, giving your answer in exact form. [5]

4 O/N 21/P12/Q12 y

B
P

A
x
O

052
4 O/N 21/P12/Q12 Unit 3: Coordinate Geometry

The diagram shows the circle with equation x2 + y2 − 6x + 4y − 27 = 0 and the tangent to the circle at
the point P 5, 4.

(a) The tangent to the circle at P meets the x-axis at A and the y-axis at B.

Find the area of triangle OAB, where O is the origin. [5]


(b) Points Q and R also lie on the circle, such that PQR is an equilateral triangle.
Find the exact area of triangle PQR. [3]

5 O/N 21/P11/Q7
A circle with centre 5, 2 passes through the point 7, 5.
(a) Find an equation of the circle. [2]
The line y = 5x − 10 intersects the circle at A and B.
(b) Find the exact length of the chord AB. [7]

6 O/N 21/P13/Q9
The line y = 2x + 5 intersects the circle with equation x2 + y2 = 20 at A and B.
(a) Find the coordinates of A and B in surd form and hence find the exact length of the chord AB.
[7]
A straight line through the point 10, 0 with gradient m is a tangent to the circle.
(b) Find the two possible values of m. [5]

7 M/J 21/P12/Q3

The equation of a curve is y = x − 3 x + 1 + 3. The following points lie on the curve. Non-exact
values are rounded to 4 decimal places.
A 2, k B 2.9, 2.8025 C 2.99, 2.9800 D 2.999, 2.9980 E 3, 3
(a) Find k, giving your answer correct to 4 decimal places. [1]
(b) Find the gradient of AE, giving your answer correct to 4 decimal places. [1]
The gradients of BE, CE and DE , rounded to 4 decimal places, are 1.9748, 1.9975 and 1.9997
respectively.
(c) State, giving a reason for your answer, what the values of the four gradients suggest about the
gradient of the curve at the point E. [2]

8 M/J 21/P12/Q6

Points A and B have coordinates 8, 3 and p, q respectively. The equation of the perpendicular
bisector of AB is y = −2x + 4.
Find the values of p and q.
[4]
9 M/J 21/P12/Q7
The point A has coordinates 1, 5 and the line l has gradient − 23 and passes through A. A circle has

centre 5, 11 and radius 52.
(a) Show that l is the tangent to the circle at A. [2]

(b) Find the equation of the other circle of radius 52 for which l is also the tangent at A. [3]

10 M/J 21/P11/Q10
The equation of a circle is x2 + y2 − 4x + 6y − 77 = 0.
(a) Find the x-coordinates of the points A and B where the circle intersects the x-axis. [2]
(b) Find the point of intersection of the tangents to the circle at A and B. [6]

053
11 M/J 21/P13/Q10 Unit 3: Coordinate Geometry
Points A −2, 3, B 3, 0 and C 6, 5 lie on the circumference of a circle with centre D.
(a) Show that angle ABC = 90Å. [2]
(b) Hence state the coordinates of D. [1]
(c) Find an equation of the circle. [2]
The point E lies on the circumference of the circle such that BE is a diameter.
(d) Find an equation of the tangent to the circle at E. [5]

12 O/N 20/P12/Q9
A circle has centre at the point B 5, 1. The point A −1, −2 lies on the circle.
(a) Find the equation of the circle. [3]
Point C is such that AC is a diameter of the circle. Point D has coordinates 5, 16.
(b) Show that DC is a tangent to the circle. [4]
The other tangent from D to the circle touches the circle at E.
(c) Find the coordinates of E. [2]

13 O/N 20/P13/Q11
A circle with centre C has equation x − 82 + y − 42 = 100.
(a) Show that the point T −6, 6 is outside the circle. [3]
Two tangents from T to the circle are drawn.
(b) Show that the angle between one of the tangents and CT is exactly 45Å . [2]
The two tangents touch the circle at A and B.

(c) Find the equation of the line AB, giving your answer in the form y = mx + c. [4]
(d) Find the x-coordinates of A and B. [3]

14 M/J 20/P12/Q11

The equation of a circle with centre C is x2 + y2 − 8x + 4y − 5 = 0.

(a) Find the radius of the circle and the coordinates of C. [3]
The point P 1, 2 lies on the circle.

(b) Show that the equation of the tangent to the circle at P is 4y = 3x + 5. [3]

The point Q also lies on the circle and PQ is parallel to the x-axis.

(c) Write down the coordinates of Q. [2]

The tangents to the circle at P and Q meet at T .

(d) Find the coordinates of T . [3]

15 M/J 20/P11/Q10
The coordinates of the points A and B are −1, −2 and 7, 4 respectively.

(a) Find the equation of the circle, C, for which AB is a diameter. [4]
(b) Find the equation of the tangent, T , to circle C at the point B. [4]
(c) Find the equation of the circle which is the reflection of circle C in the line T . [3]

054
16 M/J 20/P13/Q10 Unit 3: Coordinate Geometry

(a) The coordinates of two points A and B are −7, 3 and 5, 11 respectively.
Show that the equation of the perpendicular bisector of AB is 3x + 2y = 11. [4]
(b) A circle passes through A and B and its centre lies on the line 12x − 5y = 70.
Find an equation of the circle. [5]

17 MAR 20/P12/Q12
A diameter of a circle C1 has end-points at −3, −5 and 7, 3.
y
(a) Find an equation of the circle C1 . [3]

C2
R
C1

x
O

@ A
8
The circle C1 is translated by to give circle C2 , as shown in the diagram.
4

(b) Find an equation of the circle C2 . [2]


The two circles intersect at points R and S.
(c) Show that the equation of the line RS is y = −2x + 13. [4]

(d) Hence show that the x-coordinates of R and S satisfy the equation 5x2 − 60x + 159 = 0. [2]

18 O/N 19/P12/Q2
The point M is the mid-point of the line joining the points 3, 7 and −1, 1. Find the equation of the
x y
line through M which is parallel to the line + = 1. [4]
3 2
19 M/J 19/P12/Q2

Two points A and B have coordinates 1, 3 and 9, −1 respectively. The perpendicular bisector of
AB intersects the y-axis at the point C. Find the coordinates of C. [5]
20 M/J 19/P11/Q3

A sector of a circle of radius r cm has an area of A cm2. Express the perimeter of the sector in terms
of r and A. [4]

055
21 M/J 19/P11/Q4 Unit 3: Coordinate Geometry

y
C h, 3h D

B
0, 2

x
O A 4, 0

The diagram shows a trapezium ABCD in which the coordinates of A, B and C are 4, 0, 0, 2 and
h, 3h respectively. The lines BC and AD are parallel, angle ABC = 90Å and CD is parallel to the
x-axis.
(i) Find, by calculation, the value of h. [3]
(ii) Hence find the coordinates of D. [3]
22 M/J 19/P13/Q7
The coordinates of two points A and B are 1, 3 and 9, −1 respectively and D is the mid-point of
AB. A point C has coordinates x, y, where x and y are variables.
(i) State the coordinates of D. [1]
(ii) It is given that CD2 = 20. Write down an equation relating x and y. [1]
(iii) It is given that AC and BC are equal in length. Find an equation relating x and y and show that
it can be simplified to y = 2x − 9. [3]
(iv) Using the results from parts (ii) and (iii), and showing all necessary working, find the possible
coordinates of C. [4]
23 O/N 18/P11/Q3
Two points A and B have coordinates 3a, −a and −a, 2a respectively, where a is a positive
constant.
(i) Find the equation of the line through the origin parallel to AB. [2]
1
(ii) The length of the line AB is 3 3 units. Find the value of a. [3]

24 O/N 18/P13/Q4
Two points A and B have coordinates −1, 1 and 3, 4 respectively. The line BC is perpendicular to
AB and intersects the x-axis at C.

(i) Find the equation of BC and the x-coordinate of C. [4]


(ii) Find the distance AC, giving your answer correct to 3 decimal places. [2]
25 M/J 18/P12/Q8
Points A and B have coordinates h, h and 4h + 6, 5h respectively. The equation of the perpendicular
bisector of AB is 3x + 2y = k. Find the values of the constants h and k. [7]

056
Unit 3: Coordinate Geometry

26 M/J 18/P11/Q5
y
B

A
0, 4

x
O C 8, 0

The diagram shows a kite OABC in which AC is the line of symmetry. The coordinates of A and C
are 0, 4 and 8, 0 respectively and O is the origin.

(i) Find the equations of AC and OB. [4]


(ii) Find, by calculation, the coordinates of B. [3]

27 M/J 18/P13/Q6

The coordinates of points A and B are −3k − 1, k + 3 and k + 3, 3k + 5 respectively, where k is a


constant (k ≠ −1).

(i) Find and simplify the gradient of AB, showing that it is independent of k. [2]
(ii) Find and simplify the equation of the perpendicular bisector of AB. [5]

28 O/N 17/P11/Q6
The points A 1, 1 and B 5, 9 lie on the curve 6y = 5x2 − 18x + 19.
(i) Show that the equation of the perpendicular bisector of AB is 2y = 13 − x. [4]
The perpendicular bisector of AB meets the curve at C and D. O@ A
p
(ii) Find, by calculation, the distance CD, giving your answer in the form , where p and q are
q
integers. [5]

29 O/N 17/P13/Q11
y

1
y = x − 1 2
B 5, 2

x
O A 1, 0
1
The diagram shows the curve y = x − 1 2 and points A 1, 0 and B 5, 2 lying on the curve.

(i) Find the equation of the line AB, giving your answer in the form y = mx + c. [2]
(ii) Find, showing all necessary working, the equation of the tangent to the curve which is parallel to
AB. [5]
(iii) Find the perpendicular distance between the line AB and the tangent parallel to AB. Give your
answer correct to 2 decimal places. [3]

057
Unit 3: Coordinate Geometry
30 M/J 17/P12/Q2
The point A has coordinates −2, 6. The equation of the perpendicular bisector of the line AB is
2y = 3x + 5.
(i) Find the equation of AB. [3]
(ii) Find the coordinates of B. [3]
31 M/J 17/P13/Q8
A −1, 1 and P a, b are two points, where a and b are constants. The gradient of AP is 2.
(i) Find an expression for b in terms of a. [2]
(ii) B 10, −1 is a third point such that AP = AB. Calculate the coordinates of the possible positions
of P. [6]
32 O/N 16/P12/Q5
x y
The line + = 1, where a and b are positive constants, intersects the x- and y-axes at the points A
a b
and B respectively. The mid-point of AB lies on the line 2x + y = 10 and the distance AB = 10. Find
the values of a and b. [6]

33 O/N 16/P11/Q4
C is the mid-point of the line joining A 14, −7 to B −6, 3. The line through C perpendicular to AB
crosses the y-axis at D.
(i) Find the equation of the line CD, giving your answer in the form y = mx + c. [4]
(ii) Find the distance AD. [2]

34 O/N 16/P13/Q6
Three points, A, B and C, are such that B is the mid-point of AC. The coordinates of A are 2, m and
the coordinates of B are n, −6, where m and n are constants.
(i) Find the coordinates of C in terms of m and n. [2]
The line y = x + 1 passes through C and is perpendicular to AB.
(ii) Find the values of m and n. [5]

35 M/J 16/P12/Q8
Three points have coordinates A 0, 7, B 8, 3 and C 3k, k. Find the value of the constant k for
which
(i) C lies on the line that passes through A and B, [4]
(ii) C lies on the perpendicular bisector of AB. [4]

36 M/J 16/P11/Q8
4
A curve has equation y = 3x − and passes through the points A 1, −1 and B 4, 11. At each of
x
the points C and D on the curve, the tangent is parallel to AB. Find the equation of the perpendicular
bisector of CD. [7]

37 M/J 16/P13/Q11
Triangle ABC has vertices at A −2, −1, B 4, 6 and C 6, −3.

(i) Show that triangle ABC is isosceles and find the exact area of this triangle. [6]

(ii) The point D is the point on AB such that CD is perpendicular to AB. Calculate the x-coordinate
of D. [6]

058
Unit 3: Coordinate Geometry
38 O/N 15/P12/Q6
Points A, B and C have coordinates A −3, 7, B 5, 1 and C −1, k, where k is a constant.
(i) Given that AB = BC, calculate the possible values of k. [3]
The perpendicular bisector of AB intersects the x-axis at D.
(ii) Calculate the coordinates of D. [5]

39 M/J 15/P12/Q7
The point C lies on the perpendicular bisector of the line joining the points A 4, 6 and B 10, 2.
C also lies on the line parallel to AB through 3, 11.
(i) Find the equation of the perpendicular bisector of AB. [4]
(ii) Calculate the coordinates of C. [3]

40 M/J 15/P11/Q6

The line with gradient −2 passing through the point P 3t, 2t intersects the x-axis at A and the y-axis
at B.
(i) Find the area of triangle AOB in terms of t. [3]
The line through P perpendicular to AB intersects the x-axis at C.
(ii) Show that the mid-point of PC lies on the line y = x. [4]

41 M/J 15/P13/Q7

The point A has coordinates p, 1 and the point B has coordinates 9, 3p + 1, where p is a constant.
(i) For the case where the distance AB is 13 units, find the possible values of p. [3]
(ii) For the case in which the line with equation 2x + 3y = 9 is perpendicular to AB, find the value
of p. [4]
y
42 O/N 14/P12/Q9
D

A
2, 6

C 8, 3

x
O

B 5, −3

The diagram shows a trapezium ABCD in which AB is parallel to DC and angle BAD is 90. The
coordinates of A, B and C are 2, 6, 5, −3 and 8, 3 respectively.
(i) Find the equation of AD. [3]
(ii) Find, by calculation, the coordinates of D. [3]
The point E is such that ABCE is a parallelogram.
(iii) Find the length of BE. [2]

059
Unit 3: Coordinate Geometry
43 O/N 14/P11/Q4
The line 4x + ky = 20 passes through the points A 8, −4 and B b, 2b, where k and b are constants.
(i) Find the values of k and b. [4]
(ii) Find the coordinates of the mid-point of AB. [1]

44 O/N 14/P13/Q6
A is the point a, 2a − 1 and B is the point 2a + 4, 3a + 9, where a is a constant.
(i) Find, in terms of a, the gradient of a line perpendicular to AB. [3]

(ii) Given that the distance AB is 260, find the possible values of a. [4]

45 M/J 14/P12/Q1
Find the coordinates of the point at which the perpendicular bisector of the line joining 2, 7 to
10, 3 meets the x-axis. [5]

46 M/J 14/P11/Q7
The coordinates of points A and B are a, 2 and 3, b respectively, where a and b are constants. The

distance AB is 125 units and the gradient of the line AB is 2. Find the possible values of a and
of b. [6]
y
47 M/J 14/P13/Q11
C

y = 3x
E 6 21 , 8 21 

4y = x + 11
A

x
O
The diagram shows a parallelogram ABCD, in which the equation of AB is y = 3x and the equation
of AD is 4y = x + 11. The diagonals AC and BD meet at the point E 6 12 , 8 12 . Find, by calculation,
the coordinates of A, B, C and D. [9]

060
48 O/N 13/P12/Q5 Unit 3: Coordinate Geometry
y
D

A
8y + x = 6
(0, 8) 4
C

x
O B (4, 0)

The diagram shows a rectangle ABCD in which point A is 0, 8 and point B is 4, 0. The diagonal
AC has equation 8y + x = 64. Find, by calculation, the coordinates of C and D. [7]
49 O/N 13/P11/Q7
The point A has coordinates −1, 6 and the point B has coordinates 7, 2.

(i) Find the equation of the perpendicular bisector of AB, giving your answer in the form y = mx + c.
[4]
(ii) A point C on the perpendicular bisector has coordinates p, q. The distance OC is 2 units, where
O is the origin. Write down two equations involving p and q and hence find the coordinates of
the possible positions of C. [5]
50 O/N 13/P13/Q3
The point A has coordinates 3, 1 and the point B has coordinates −21, 11. The point C is the
mid-point of AB.

(i) Find the equation of the line through A that is perpendicular to y = 2x − 7. [2]

(ii) Find the distance AC. [3]


51 M/J 13/P12/Q7
The point R is the reflection of the point −1, 3 in the line 3y + 2x = 33. Find by calculation the
coordinates of R. [7]

52 M/J 13/P13/Q7 y
A (2, 14)

X
B (14, 6)

C (7, 2)
x
O

The diagram shows three points A 2, 14, B 14, 6 and C 7, 2. The point X lies on AB, and CX is
perpendicular to AB. Find, by calculation,
(i) the coordinates of X , [6]
(ii) the ratio AX : XB. [2]

061
Unit 3: Coordinate Geometry
53 O/N 12/P12/Q5 y
B (5, 11)

X (4, 4)
A (1, 3)

x
O

The diagram shows a triangle ABC in which A has coordinates (1, 3), B has coordinates (5, 11) and
angle ABC is 90◦ . The point X (4, 4) lies on AC . Find
(i) the equation of BC, [3]
(ii) the coordinates of C. [3]

54 M/J 12/P12/Q4
The point A has coordinates (−1, −5) and the point B has coordinates (7, 1). The perpendicular
bisector of AB meets the x-axis at C and the y-axis at D. Calculate the length of CD. [6]

55 M/J 12/P11/Q5
y
y = 6x + k

y = 7 Öx
B

x
O


The diagram shows the curve y = 7 x and the line y = 6x + k, where k is a constant. The curve and
the line intersect at the points A and B.
(i) For the case where k = 2, find the x-coordinates of A and B. [4]

(ii) Find the value of k for which y = 6x + k is a tangent to the curve y = 7 x. [2]

56 M/J 12/P11/Q9
The coordinates of A are (−3, 2) and the coordinates of C are (5, 6). The mid-point of AC is M and
the perpendicular bisector of AC cuts the x-axis at B.
(i) Find the equation of MB and the coordinates of B. [5]
(ii) Show that AB is perpendicular to BC . [2]
(iii) Given that ABCD is a square, find the coordinates of D and the length of AD. [2]

062
57 M/J 11/P12/Q7 Unit 3: Coordinate Geometry

The line L1 passes through the points A (2, 5) and B (10, 9). The line L2 is parallel to L1 and passes
through the origin. The point C lies on L2 such that AC is perpendicular to L2 . Find
(i) the coordinates of C, [5]
(ii) the distance AC. [2]

58 M/J 11/P13/Q3
x y
The line + = 1, where a and b are positive constants, meets the x-axis at P and the y-axis at Q.
a b √
Given that PQ = (45) and that the gradient of the line PQ is − 12 , find the values of a and b. [5]

063
9709/12 M/J 22/P12/Q8 Unit 3: Answer Section May/June 2022

Question Answer Marks Guidance

1(a) 1  1  a  b  12  0  a  b  10 B1 B1 B1 for each equation. Allow unsimplified. Can be implied by


correct values for a and b.
4  36  2a  6b  12  0  2a  6b  28

a  4, b  6 B1

 their a their b  B1 FT Or x  2, y  3


Centre is   ,   2, 3
 2 2 

4
1(b) 1  their y 1  3 1  3 4 *M1 Using their centre correctly.
Gradient of AC is [   ]
1  their x 1  2 1  2 3

1  3 A1 FT Use of m1m2  1 to obtain the gradient of the tangent.


Gradient of tangent is    
4 4
their 
3

3 3 7 DM1 Using 1,1 with their gradient of the tangent at A.


Equation: y  1  ‘their  ’  x  1 or y   x 
4 4 4

3 x  4 y  7 or 4 y  3 x  7 . or integer multiples of these A1

1(b) 3 3 7 DM1 Using 1,1 with their gradient of the tangent at A.


Equation: y  1  ‘their  ’  x  1 or y   x 
4 4 4

3 x  4 y  7 or 4 y  3 x  7 . or integer multiples of these A1

064
9709/11 M/J 22/P11/Q9/a Unit 3: Answer Section May/June 2022

Question Answer Marks Guidance

2(a)
Express as  x 32   y 12  26  9 1  36 M1 Completing the square on x and y or using the form
x2  y2 2gx 2fy c 0, centre  g,f  and radius

g2  f 2  c .
SOI by correct answer.

Centre (−3, 1) B1

Radius 6 B1

So lowest point is (−3, −5) A1 FT FT on their centre and their radius.

2(b) Intersects when x 2   kx  5  6 x  2  kx  5  26  0


2 *M1 Substituting y  kx  5 into their circle equation or
rearranging and equating y.
or  x  3   kx  5  1  36
2 2

x 2  k 2 x 2  10kx  25  6 x  2kx  10  26  0 DM1 Rearranging to 3-term quadratic (terms grouped, all on


one side). Allow 1 error.
or x 2  6 x  9  k 2 x 2  12kx  36  36
A1 Correct quadratic (need to see 9 as constant term).
 
leading to k 2 x 2  x 2  6 x  12kx  9   0 or k 2  1 x 2   6  12k  x  9   0

 6 12k 
2
 
 4 k 2  1  9 [ 0] DM1 Using discriminant b 2  4ac   0 with their values.
Allow if in square root.
 leading to 144k 2  144k  36  36k 2  36  0 
 

4 A1 Need not see method for solving.


[108k2 – 144k = 0 leading to] k = 0 or k =
3

4 A1 4
k  0, k  Do not accept  k  0.
3 3

065
9709/13 Unit 3: Answer Section May/June 2022

Question Answer Marks Guidance

3(a) Equation of BC is  y 23 x B2, 1, 0 OE forms y  4  3  x  2  or y  2  3  x  0  .

3(b)  x  2   2  3x  4   20 *M1 OE
2 2

Sub line equation into equation of circle to eliminate y.

10(x – 2)2 = 20 or [10](x2 – 4x + 2)[= 0] A1 OE Accept (10x2 – 40x + 20).

4    16  8 DM1 Correctly solving their quadratic.


x  2     2 or x 
2

x 2 2 A1 OE only solution. Answer only SC B1 If DM1 not scored.

y 3 2 4 A1 OE only solution. Answer only SC B1 If DM1 not scored.

9709/12 O/N 21/P12/Q12 October/November 2021

Question Answer Marks Guidance

4(a) Centre is (3, – 2) B1

( their −2 ) − 4 [= *M1 Finding gradient using their centre (not (0, 0)) and P (5,4).
Gradient of radius = 3]
( their 3) − 5
1 DM1 Using P and the negative reciprocal of their gradient to find
Equation of tangent y − 4 = − ( x − 5) the equation of AB.
3

17 A1
Sight of [x =]17 and [y =]
3

 1 17  289 A1 1
Or 48 or AWRT 48.2.
 Area = 2 × 3 × 17 =  6 6
 

066
Unit 3: Answer Section
9709/12 Cambridge International AS & A Level – Mark Scheme October/November 2021

Question Answer Marks Guidance

4(a) Alternative method for question 12(a)


cont’d
 1
 dy 1 B1 OE. Correct differentiation of rearranged equation.


(
 y = −2 ± 40 − ( x − 3)
2
) 2
OE leading to 
 dx
(
= ( 3 − x ) 31 + 6 x − x 2 )

2

1 *M1 Find the gradient using x = 5 in their differential (with clear


dy
( )  dy 1

= ( 3 − 5 ) 31 + 6 ( 5 ) − ( 5 )
2
 dx = − 3 
2
use of chain rule).
dx  

1 DM1 Using P and their value for the gradient to find the equation
Equation of tangent y − 4 = − ( x − 5) of AB.
3

17 A1
Sight of [x =]17 and [y =]
3

 1 17  289 A1 1
Or 48 or AWRT 48.2.
 Area = 2 × 3 × 17 =  6 6
 

067
9709/12 Cambridge International AS & A Level – Mark Scheme Unit 3: Answer Section October/November 2021

Question Answer Marks Guidance

4(b) Radius of circle = 40 , B1 Or 2 10 or 6.32 AWRT or r 2 = 40 .

1  1 3 M1 1 2
Area of ∆CRQ = × (their r ) 2 sin120  = × 40 ×  Using r sinθ with their r and 120 or 60 [ ×3 ]
2 2
 2 2 
OR 1
1  1  Using ×base ×height in a correct right-angled triangle
Area of ∆CQX = × 40cos30 × 40cos60 OE  = × 30 × 10  2
2  2  [ ×6 ].
OR
π
Area of circle ‒ 3× Area of segment = 40π ‒ 3 × (40 ‒ 10 3)
3
OR
1
QR = 120 or 2 30 and area = QR 2 sin60 Use of cosine rule and area of large triangle
2

30 3 A1 AWRT 52[.0] implies B1M1A0.

3 See diagram for points stated in ‘Answer’ column.

068
9709/11 O/N 21/P11/Q7 Cambridge International AS & A Level – Mark Scheme Unit 3: Answer Section October/November 2021

Question Answer Marks Guidance

5(a) r 2  = ( 5 − 2 ) + ( 7 − 5)  = 13
2 2 B1 r 2 = 13 or r = 13
 

Equation of circle is ( x − 5) + ( y − 2 ) = 13
2 2 B1 FT 2 OE. FT on their 13 but LHS must be correct.

5(b) ( x − 5)2 + ( 5x − 10 − 2 )2 = 13 M1 Substitute y = 5 x − 10 into their equation.

26 x 2 − 130 x + 156 [ = 0] A1 FT OE 3-term quadratic with all terms on one side.


FT on their circle equation.

[ 26] ( x − 2)( x − 3) [ = 0] M1 Solve 3-term quadratic in x by factorising, using formula or


completing the square. Factors must expand to give their
coefficient of x2.
(2, 0), (3, 5) A1 A1 Coordinates must be clearly paired; A1 for each correct
point. A1 A0 available if two x or y values only.
If M0 for solving quadratic, SC B2 can be awarded for
correct coordinates, SC B1 if two x or y values only.

( AB )2 = ( 3 − 2)2 + ( 5 − 0 )2 M1 SOI. Using their points to find length of AB.

AB = 26 A1 ISW. Dependent on final M1 only.

9709/13 Cambridge International AS & A Level – Mark Scheme October/November 2021


Question O/N 21/P13/Q9 Answer Marks Guidance

9(a) x 2 + ( 2 x + 5) = 20
2
leading to x 2 + 4 x 2 + 20 x + 25 = 20 M1 Substitute y = 2 x + 5 and expand bracket.

( 5 ) ( x 2 + 4 x + 1) [ = 0] A1 3-term quadratic.

−4 ± 16 − 4 M1 OE. Apply formula or complete the square.


x=
2

(
A = −2 + 3,1 + 2 3 ) A1 Or 2 correct x values.

(
B = −2 − 3,1 − 2 3 ) A1 Or all values correct.
SC B1 all 4 values correct in surd form without working.
SC B1 all 4 values correct in decimal form from correct
formula or completion of the square
069
9709/13 Cambridge International Unit 3: Answer Section October/November 2021

Question O/N 21/P13/Q9 Answer Marks Guidance

6(a) AB 2 = their ( x2 − x1 ) + their ( y2 − y1 )


2 2 M1 Using their coordinates in a correct distance formula.
Condone one sign error in x2 − x1 or y2 − y1

 AB 2 = 48 + 12 leading to  AB = 60 A1 OE. CAO. Do not accept decimal answer. Answer must


  come from use of surd form in distance formula.

6(b) x 2 + m2 ( x − 10 ) = 20
2 *M1 Finding equation of tangent and substituting into circle
equation.

( ) (
x 2 m 2 + 1 − 20m 2 x + 20 5m 2 − 1 ) [ = 0] DM1 OE. Brackets expanded and all terms collected on one side
of the equation.

( )(
[b 2 − 4ac =]400m 4 − 80 m 2 + 1 5m 2 − 1 ) M1 Using correct coefficients from their quadratic equation.

( )
400m 4 − 80 5m 4 + 4m 2 − 1 = 0 → ( −80 ) 4m 2 − 1 = 0 ( ) A1 OE. Must have ‘=0’ for A1.

1 A1
m=±
2

Alternative method for question 9(b)

Length, l of tangent, is given by l 2 = 10 2 − 20 M1

l = 80 A1

20 1 M1 A1 Where α is the angle between the tangent and the x-axis.


tan α = =
80 2

1 A1
m=±
2

070
9709/12 Cambridge International Unit 3: Answer Section May/June 2021

Question M/J 21/P12/Q3 Answer Marks Guidance

7(a) 1.2679 1 B1 AWRT. ISW if correct answer seen. 3 – 3 scores B0

7(b) 1.7321 1 B1 AWRT. ISW if correct answer seen.

7(c) Sight of 2 or 2.0000 or two in reference to the gradient *B1

This is because the gradient at E is the limit of the gradients of the DB1 Allow it gets nearer/approaches/tends/almost/approximately 2
chords as the x-value tends to 3 or ꝺx tends to 0. 2
9709/12 M/J 21/P12/Q6 Cambridge International AS & A Level – Mark Scheme May/June 2021
Question Answer Marks Guidance
8 1 B1 SOI
Gradient AB =
2
1 *M1 Equating given perpendicular bisector with the line through
Lines meet when −2 x + 4 = ( x − 8) + 3 (8, 3) using their gradient of AB (but not -2) and solving.
2
Solving as far as x = Expect x = 2, y = 0.

Using mid-point to get as far as p = or q = DM1 8+ p 3+ q


Expect = 2 or =0
2 2
p = −4, q = −3 A1 Allow coordinates of B are (−4, −3).

9709/12 M/J 21/P12/Q7 Cambridge International AS & A Level – Mark Scheme May/June 2021
Question Answer Marks Guidance

9(a) 11 − 5 M1 For substituting (1,5) into circle equation or showing


(5 – 1)2 + (11 – 5)2 = 52 or
5 −1 3
gradient = .
2
For both circle equation and gradient, and proving line is A1 Clear reasoning.
perpendicular and stating that A lies on the circle

9(b) Centre is (−3, −1) B1 B1 B1 for each correct co-ordinate.

Equation is (x + 3)2 + (y + 1)2 = 52 B1 FT FT their centre, but not if either (1, 5) or (5, 11). Do not accept
3 522 .

071
Unit 3: Answer Section

Question M/J 21/P11/Q10 Answer Marks Guidance

10(a) When y = 0 x2 − 4 x − 77 = 0 [⇒ ( x + 7 )( x − 11) = 0 or ( x − 2 ) = 81 ]


2 M1 Substituting y = 0

So x-coordinates are −7 and 11 A1 2

10(b) Centre of circle C is (2, −3) B1

1 1 M1 For either gradient (M1 sign error,


Gradient of AC is − or Gradient of BC is M0 if x-coordinate(s) in numerator)
3 3

Gradient of tangent at A is 3 or Gradient of tangent at B is −3 M1 For either perpendicular gradient

Equations of tangents are y = 3x + 21, y = −3x + 33 A1 For either equation

Meet when 3x + 21 = −3x + 33 M1 OR: (centre of circle has x coordinate 2) so x


coordinate of point of intersection is 2

Coordinates of point of intersection (2, 27) A1

9709/13 Cambridge International AS & A Level – Mark Scheme May/June 2021

Question M/J 21/P13/Q10 Answer Marks Guidance

11(a) 3 5 M1 Attempting to find required gradients, sides or


Gradient of AB = − , gradient of BC = or lengths of all 3 sides or vectors vectors
5 3

mab mbc = −1 or Pythagoras or AB.BC =0 or cos ABC = 0 from cosine rule A1 2 WWW

11(b) Centre = mid-point of AC = (2,4) B1 1

11(c) ( x − their x c )
2
+ ( y − their yc )  = r 2  or ( their xc − x ) + ( their yc − y ) =  r 2 
2 2 2 M1 Use of circle equation with their centre

( x − 2 )2 + ( y − 4 )2 = 17 A1 Accept x 2 − 4 x + y 2 − 8 y + 3 = 0 OE

072
Unit 3: Answer Section
9709/13 Cambridge International AS & A Level – Mark Scheme May/June 2021

Question M/J 21/P13/Q10 Answer Marks Guidance

11(d)  x+3 y+0  −1  M1 Use of mid-point formula, vectors, steps on a


 ,  = ( 2, 4 ) or BE = 2BD = 2  4  diagram
 2 2   
Or Equation of BE is y = −4 ( x − 3) or y − 4 = −4 ( x − 2 ) leading to y = −4 x + 12 May be seen to find x coordinate at E
Substitute equation of BE into circle and form a 3-term quadratic.

 3   −2   1  A1 E = (1, 8)
( x, y ) = (1,8) or OE =   +   =   Accept without working for both marks SC B2
 0  8  8

1 B1 Or gradient of BE = -4
Gradient of BD, m, = ‒4 or gradient AC = = gradient of tangent
4

Equation of tangent is y − 8 = ¼ ( x − 1) OE M1 A1 For M1, equation through their E or (1, 8) (not,


−1
A, B or C) and with gradient
5 their − 4
9709/12 O/N 20/P12/Q9 Cambridge International AS & A Level – Mark Scheme October/November 2020

Question Answer Marks Guidance

12(a)
r= (6 2
)
+ 32 or r² = 45
B1 Sight of r = 6.7 implies B1

(x − 5)² + (y − 1)² = r² or x 2 − 10 x + y 2 − 2 y = r 2 − 26 M1 Using centre given and their radius or r in correct formula

(x − 5)² + (y − 1)² = 45 or x 2 − 10 x + y 2 − 2 y = 19 A1
( )
2
Do not allow 45 for r 2

13(b) C has coordinates (11, 4) B1

0.5 B1 OE, Gradient of AB, BC or AC.

Grad of CD = −2 M1 Calculation of gradient needs to be shown for this M1.

1 A1 Clear reasoning needed.


( × −2 = −1 ) then states + perpendicular → hence shown or tangent
2

073
9709/13 O/N 20/P13/Q11 Cambridge International AS & A Level – Mark Scheme Unit 3: Answer Section October/November 2020

Question Answer Marks Guidance

13(a) ( −6 − 8)
2
+ ( 6 − 4)
2 M1 OE

= 200 A1

200 > 10, hence outside circle A1 AG (‘Shown’ not sufficient). Accept equivalents of 200 > 10

13(b)  their 10  M1 Allow decimals for 10√2 at this stage. If cosine used, angle ACT
angle = sin −1   or BCT must be identified, or implied by use of 90°‒ 45°.
 their 10 2 

1 2 10 10 A1 AG Do not allow decimals


angle = sin −1 ( or or or ) = 45º
2 2 10 2 200

13(c) 1 B1 OE
Gradient, m, of CT = −
7

Attempt to find mid-point (M) of CT *M1 Expect (1, 5)

Equation of AB is y − 5 = 7 ( x − 1) DM1 1
Through their (1, 5) with gradient −
m

y = 7x − 2 A1

13(d) ( x − 8)
2
+ ( 7 x − 2 − 4 ) = 100 or equivalent in terms of y
2 M1 Substitute their equation of AB into equation of circle.

50 x 2 − 100 x ( = 0 ) A1

x = 0 and 2 A1 WWW

074
9709/12 M/J 20/P12/Q11 Cambridge International AS & A Level – Mark Scheme Unit 3: Answer Section May/June 2020

Question Answer Marks

14(a) Express as ( x − 4 ) ² + ( y + 2 ) = 16 + 4 + 5
2 M1

Centre C(4, −2) A1

Radius = 25 = 5 A1 3

14(b) 4 (FT on coordinates of C) B1FT


P(1,2) to C(4, − 2) has gradient −
3
3 M1
Tangent at P has gradient =
4
3 A1
Equation is y − 2 = ( x − 1) or 4y = 3x + 5 3
4
14(c) Q has the same coordinate as P y = 2 B1

Q is as far to the right of C as P x = 3 + 3 + 1 = 7 Q (7, 2) B1 2

14(d) 3 B1FT
Gradient of tangent at Q = − by symmetry
4
(FT from part (b))

3 M1
Eqn of tangent at Q is y − 2 = − ( x − 7 ) or 4y + 3x = 29
4

17 A1
T (4, )
4

075
9709/11 Cambridge International AS & A Level – Mark Scheme Unit 3: Answer Section May/June 2020

Question M/J 20/P11/Q10 Answer Marks

15(a) Centre is (3, 1) B1

Radius = 5 (Pythagoras) B1

Equation of C is ( x − 3) + ( y − 1) = 25
2 2 (FT on their centre) M1 A1FT
4
15(b) Gradient from (3, 1) to (7, 4) = ¾ (this is the normal) B1
4 M1
Gradient of tangent = −
3
4 M1A1
Equation is y − 4 = − ( x − 7 ) or 3 y + 4 x = 40 4
3
15(c) B is centre of line joining centres → (11, 7) B1

Radius = 5 M1
New equation is ( x − 11) + ( y − 7 ) = 25
2 2
(FT on coordinates of B) 3 A1FT

9709/13 M/J 20/P13/Q10 Cambridge International AS & A Level – Mark Scheme May/June 2020

Question Answer Marks

16(a) Mid-point is (‒1, 7) B1

Gradient, m, of AB is 8/12 OE B1
12 M1
y−7 =− ( x + 1)
8
3 x + 2 y = 11 AG A1 4

16(b) Solve simultaneously 12 x − 5 y = 70 and their 3 x + 2 y = 11 M1

x = 5, y = −2 A1

Attempt to find distance between their (5, ‒2) and either (‒7,3) or (5, 11) M1

A1
(r ) = 122 + 52 or 132 + 0 = 13

Equation of circle is ( x − 5)2 + ( y + 2 )2 = 169 A1 5


076
Unit 3: Answer Section
9709/12 MAR 20/P12/Q12 Cambridge International AS & A Level – Mark Scheme March 2020

Question Answer Marks Guidance

17(a) Centre = (2, ‒1) B1

2 2 M1 1
r 2 =  2 − ( −3)  + −
 1 − ( −5)  [ 2 − 7] + [ −1 − 3]
2 2
or OE OR

( −3 − 7 )2 + ( −5 − 3)2  OE
2

( x − 2)
2
+ ( y + 1) = 41
2 A1 3 Must not involve surd form
SCB3 ( x + 3)( x − 7 ) + ( y + 5 )( y − 3) = 0

17(b) 8 B1FT SOI


Centre = their (2, ‒1) +   = (10, 3) FT on their (2, ‒1)
 4

( x − 10)
2
+ ( y − 3) = their 41
2 B1FT FT on their 41 even if in surd form
2 SCB2 ( x − 5 )( x − 15 ) + ( y + 1)( y − 7 ) = 0
17(c) 4 B1
Gradient m of line joining centres = OE
8
Attempt to find mid-point of line. M1 Expect (6, 1)

Equation of RS is y − 1 = −2 ( x − 6 ) M1 −1
Through their (6, 1) with gradient
m

y = −2 x + 13 A1 AG

17(d) ( x − 10)
2
+ ( −2 x + 13 − 3) = 41
2 M1 Or eliminate y between C1 and C2

x 2 − 20 x + 100 + 4 x 2 − 40 x + 100 = 41 → 5 x 2 − 60 x + 159 = 0 A1 2 AG


9709/12 O/N 19/P12/Q2 Cambridge International AS/A Level – Mark Scheme October/November 2019

Question Answer Marks Guidance

18 Attempt to find the midpointM M1

(1, 4) A1

Use a gradient of ±⅔ and their M to find the equation of the line. M1

Equation is y – 4 = −⅔(x – 1) A1 AEF


077
9709/12 M/J 19/P12/Q2 Cambridge International AS/A Level – Mark Scheme Unit 3: Answer Section May/June 2019

Question Answer Marks Guidance

19 Midpoint ofAB is (5, 1) B1  10 2 


Can be seen in working, accept  ,  .
 2 2
mAB = −½ oe B1

C to (5, 1) has gradient 2 *M1 Use of m1× m2 = −1.

Forming equation of line (y = 2x − 9) DM1 Using their perpendicular gradient and their
midpoint to form the equation.

C (0, −9) or y = −9 A1 5
9709/11 M/J 19/P11/Q3 Cambridge International AS/A Level – Mark Scheme May/June 2019

Question Answer Marks Guidance

20 Uses A = ½r²θ M1 Uses area formula.


2A A1
θ=

P = r + r + rθ B1
2A A1 4 Correct simplified expression for P.
P = 2r +
r
9709/11 M/J 19/P11/Q4 Cambridge International AS/A Level – Mark Scheme May/June 2019
Question Answer Marks Guidance

21(i) Gradient ofAB = −½ → Gradient of BC = 2 M1 Use of m1.m2 = −1 for correct lines

3h − 2 M1 Uses normal line equation or gradients for h.


Forms equation in h =2
h
h=2 A1

21(ii) y coordinate of D is 6, (3 × ‘their’ h) B1 FT


6−0
= 2 → x = 7 → D (7, 6)
x−4
Vectors: AD.AB=0 M1 A1 Must use y = 6
Realises the y values of C and D are equal.
3
Uses gradient or line equation to find x
078
Unit 3: Answer Section
9709/13 M/J 19/P13/Q7 Cambridge International AS/A Level – Mark Scheme May/June 2019

Question Answer Marks Guidance


22(i) D = (5, 1) B1

22(ii) ( x − 5) + ( y − 1) = 20 oe B1 FT on their D.
2 2

Apply ISW, oe but not to contain square roots

22(iii) ( x − 1) + ( y − 3) = ( 9 − x ) + ( y + 1) soi M1 Allow 1 sign slip


2 2 2 2

For M1 allow with √ signs round both sides but sides must be
equated

x 2 − 2 x + 1 + y 2 − 6 y + 9 = x 2 − 18 x + 81 + y 2 + 2 y + 1 A1

y = 2 x − 9 www AG A1

22(iv) Eliminatey (or x) using equations in (ii) and (iii) *M1 To give an (unsimplified) quadratic equation

5x2 ‒50x + 105 (= 0) or 5(x‒5)2 = 20 or 5y2‒10y‒75 (= 0) or DM1 Simplify to one of the forms shown on the right (allow arithmetic
5(y‒1)2 = 80 slips)

x = 3 and 7, or y = ‒3 and 5 A1

(3, ‒3), (7, 5) A1 Both pairs of x & y correct implies A1A1.


SC B2 for no working

O/N 18/P11/Q3

079
Unit 3: Answer Section
9709/11 O/N 18/P11/Q3 Cambridge International AS/A Level – Mark Scheme October/November 2018

Question Answer Marks Guidance

23(i) Gradient ofAB = ‒3/4 B1 Accept ‒3a/4a

3 B1FT Answer must not include a. Ft on their numerical gradient


y = − x oe
4
2
23(ii) ( 4a )2 + ( 3a )2 = (10 / 3)2 soi M1 May be unsimplified

25a 2 = 100 / 9 oe A1

a = 2/3 A1 3

9709/13 O/N 18/P13/Q4 Cambridge International AS/A Level – Mark Scheme October/November 2018

Question Answer Marks Guidance

24(i) Gradient,m, of AB = 3/4 B1


−4 M1A1 −1
Equation of BC is y − 4 = ( x − 3) Line through (3, 4) with gradient (M1). (Expect
3 m
−4
y= x +8)
3

x=6 A1 4 Ignore any y coordinate given.

24(ii) ( AC ) M1A1
2
= 7 2 + 12 → AC = 7.071 2 M mark for ( their 6 + / −1)2 + 1 .
9709/12 M/J 18/P12/Q8 Cambridge International AS/A Level – Mark Scheme May/June 2018
Question Answer Marks Guidance

8 3 B1
Gradient of bisector = –
2
*
5h − h M1 y − step
gradient AB = Attempt at
4h + 6 − h x − step
*
5h − h 2 4h + 6 − h 3 M1 Using m1m2 = – 1 appropriately to form an equation.
Either = or – = –
4h + 6 − h 3 5h − h 2

OR
080
Unit 3: Answer Section

9709/12 Cambridge International AS/A Level – Mark Scheme May/June 2018


Question M/J 18/P12/Q8 Answer Marks Guidance
3 B1
25 Gradient of bisector = –
2
*
2 h M1 Obtain equation of AB using gradient from m1m2 = – 1 and a point.
Using gradient of AB and A, B or midpoint → x+ = y oe
3 3
*
Substitute co-ordinates of one of the other points M1 Arrive at an equation in h.

h=2 A1

 5h + 6  B1FT Algebraic expression or FT for numerical answer from ‘their h’


Midpoint is  , 3h  or (8, 6)
 2 
3
Uses midpoint and ‘their h’ with 3x + 2y = k DM1 Substitutes ‘their midpoint’ into 3x + 2y = k. If y = − x + c is used
2
(expect c = 18) the method mark should be withheld until they ×2.
→ k = 36 soi A1 7

081
Unit 3: Answer Section

26 M/J 18/P11/Q5
1
(i) Eqn of AC y = − x + 4 (gradient must be ∆y / ∆x )
2
Gradient of OB = 2 → y = 2x (If y missing only penalise once)
(ii) Simultaneous equations → ((1.6, 3.2))
This is mid-point of OB. → B (3.2, 6.4)
or
Let coordinates of B (h, k)
OA = AB → h² = 8k − k²
OC =BC → k² = 16h – h² → (3.2, 6.4)
or k −4 k
gradients ( × = −1 )
h h −8
or
Pythagoras: h 2 + ( k − 4 ) + ( h − 8 ) + k 2 = 42 + 82
2 2

27 M/J 18/P13/Q6

(i) 3k + 5 − ( k + 3)  2k + 2  1
Gradient, m, of AB = OE  =  =
k + 3 − ( −3k − 1)  4k + 4  2
(ii) 1 1
Mid-pt = [ (‒3k ‒ 1 + k + 3), (3k + 5 + k + 3)] =
2 2
 −2k + 2 4k + 8 
 ,  SOI
 2 2 
−1
Gradient of perpendicular bisector is SOI Expect ‒2
their m
Equation: y − ( 2k + 4 ) = −2  x − ( −k + 1)  OE

y + 2x = 6

28 O/N 17/P11/Q6
(i) Mid-point of AB = (3, 5)
Gradient of AB = 2
Eqn of perp. bisector is y − 5 = −½ ( x − 3) → 2 y = 13 − x
(ii)
( )
−3x + 39 = 5 x 2 − 18 x + 19 → ( 5) x 2 − 3x − 4 ( = 0 )
x = 4 or −1
y = 4½ or 7
125
CD 2 = 52 + 2½ 2 → CD =
4
082
29 O/N 17/P13/Q11 Unit 3: Answer Section

1
(i) Gradient of AB =
2
1 1
Equation of AB is y = x–
2 2
dy 1
= ½ ( x − 1) 2

(ii)
dx
1
dy
½ ( x − 1)

2 = ½ . Equate their to their ½
dx
x = 2, y = 1
y ‒ 1 = ½(x ‒ 2) (thro' their(2,1) & their ½) → y = ½ x

(iii) EITHER:
d
sin θ = → d = sin θ
1
gradient of AB = ½ ⇒ tan θ = ½ ⇒ θ = 26.5 ( 7 ) °
1
d = sin 26.5 ( 7 ) ° = 0.45 (or )
5
OR1:
Perpendicular through O has equation y = −2 x
 1 −2 
Intersection with AB: −2 x = ½ x − ½ →  , 
5 5 
2 2
1 2 1
d =   +   = 0.45 (or )
5  5 5
OR2:
Perpendicular through (2, 1) has equation y = −2 x + 5
 11 3 
Intersection with AB: −2 x + 5 = ½ x − ½ →  , 
 5 5
2 2
1 2
d =   +  = 0.45 (or 1/√5)
5  5

(iii) OR3:
1 1
∆OAC has area [where C = (0, − )]
4 2
1 5 1 1
× ×d= →d=
2 2 4 5

30 M/J 17/P12/Q2

(i) Gradient = 1.5 Gradient of perpendicular = −⅔


Equation of AB is y − 6 = −⅔( x + 2 )
Or 3 y + 2 x = 14 oe
(ii) Simultaneous equations → Midpoint (1, 4)

Use of midpoint or vectors → B (4, 2)

083
31 M/J 17/P13/Q8 Unit 3: Answer Section

(i) ( b − 1) / ( a + 1) = 2
b = 2a + 3 CAO

(ii) AB 2 = 112 + 2 2 = 125 oe

( a + 1) + ( b − 1) = 125
2 2

( a + 1) + ( 2a + 2 ) = 125
2 2

( 5 ) ( a 2 + 2a − 24 ) = 0 → eg ( a − 4 )( a + 6 ) = 0
a = 4 or −6
b = 11 or −9

32 O/N 16/P12/Q5
A(a, 0) and B (0, b)
a 2 + b 2 = 100
a b
M has coordinates  , 
 2 2
M lies on 2 x + y = 10
b
→ a + = 10
2
Sub → a 2 + ( 20 − 2a ) = 100
2

2
 b
or  10 −  + b 2 = 100
 2
→ a = 6, b = 8.

33 O/N 16/P11/Q4
(i) C = (4, −2)
m AB = −1 / 2 → mCD = 2 AD 2 = (14 − 0 ) + ( −7 − ( −10 ) )
2 2
(ii)
Equation of CD is y + 2 = 2 ( x − 4 ) oe AD = 14.3 or √205
y = 2 x − 10

34 O/N 16/P13/Q6

2+ x (ii) Sub their x, y into y = x + 1 → −12 − m = 2n − 2 + 1


(i) = n ⇒ x = 2n − 2
2 m+6
m+ y = −1 oe Not nested in an equation
= −6 ⇒ y = −12 − m 2−n
2 Eliminate a variable
m = −9, n = −1
35 M/J 16/P12/Q8
A (0, 7), B (8, 3) and C (3k, k) OR
7−k 3− k
(i) m of AB is −½ oe. =
0 − 3k 8 − 3k
Eqn of AB is y = −½x + 7
Let x = 3k, y = k → 20k = 56 → k = 2.8
k = 2.8 oe OR
7−k 7−3
=
0 − 3k 0 − 8
→ 20k = 56 → k = 2.8

084
35 M/J 16/P12/Q8 Unit 3: Answer Section
(ii) M(4, 5)
Perpendicular gradient = 2.
Perp bisector has eqn y − 5 = 2 ( x − 4 )
Let x = 3k, y = k
3
k = oe
5
OR
(0 – 3k)2 + (7 – k)2 = (8 – 3k)2 + (3 – k)2

–14k +49 = 73 – 54k → 40k = 24 → k = 0.6

36 M/J 16/P11/Q8
4
y = 3x −
x
dy 4
= 3+
dx x²
m of AB = 4
Equate → x = ±2
→ C (2, 4) and D (−2, −4)

→ M (0, 0) or stating M is the origin


m of CD = 2
1
Perpendicular gradient (= − )
2
1
→y=− x
2
37 M/J 16/P13/Q11

(i) AB 2 = 62 + 72 = 85, BC 2 = 22 + 92 = 85 (ii) Gradient of AB = 7 / 6


7
(→ isosceles) Equation of AB is y + 1 = ( x + 2)
6
AC 2 = 82 + 22 = 68 Gradient of CD = −6 / 7
−6
2
M = (2, −2) or BM 2 = ( 85) − (½ 68)
2
Equation of CD is y + 3 =( x − 6)
7
BM = 22 + 82 = 68 or 85 − 17 = 68 −6 36 7 14
Sim Eqns 2 = x+ − x−
1 7 7 6 6
Area ∆ABC = 68 68 = 34 34 2
2 x= = oe
85 5

38 O/N 15/P12/Q6

4
A(−3, 7), B(5, 1) and C(−1, k) (ii) m of AB = −¾ m perp =
3
M = (1, 4)
(i) AB = 10
4
6² + (k – 1)² = 10² Eqn y − 4 = ( x − 1)
k = −7 and 9 3
Set y to 0, → x = –2

39 M/J 15/P12/Q7

A(4, 6), B (10, 2).


(i) M = (7, 4) (ii) Eqn of line parallel to AB through (3, 11)
m of AB = − 23 → y − 11 = − 23 ( x − 3)
3
m of perpendicular = 2 Sim eqns → C (9, 7)
→ y − 4 = 32 ( x − 7)
085
40 M/J 15/P11/Q6 Unit 3: Answer Section

1
(i) y − 2t = −2( x − 3t )( y + 2 x = 8t ) (ii) m=
2
Set x to 0 → B(0, 8t)
1
Set y to 0 → A(4t, 0) → y − 2t = ( x − 3t )(2 y = x + t )
→ Area = 16t² 2
Set y to 0 → C (−t, 0)
Midpoint of CP is (t, t)
This lies on the line y = x.
41 M/J 15/P13/Q7
2
(ii) Gradient of given line = −
(i) 2 2
(9 − p ) + (3 p ) = 169 3
3
10 p 2 − 18 p − 88 ( = 0) oe Hence gradient of AB =
2
p = 4 or − 11 / 5 oe
3 3p  − 2   3p 
= oe eg     = 1
2 9− p  3 9− p
(includes previous M1)
p=3
42 O/N 14/P12/Q9
−9
(i) mAB = −3 or
3
1 (iii) Use of vectors or mid-point
mAD =
3 → E (5, 12) or mid-point (5,4.5)
1 Length of BE = 15
Eqn AD y – 6 = (x – 2) or 3y = x + 16
3
(ii) Eqn CD y – 3 = −3(x – 8) or y = −3x + 27
Sim Eqns
→ D (6½, 7½)

43 O/N 14/P11/Q4
(i) 32 − 4k = 20 ⇒ k = 3
4b + 3 × 2b = 20
b=2
(ii) Mid-point = (5, 0)

44 O/N 14/P13/Q6

3a + 9 − (2a − 1) a + 10 − a − 10 (ii) (√)[(a + 4)2 + (a + 10)2] = (√)260


(i) m= = oe e.g.
2a + 4 − a a+4 −a−4
− (a + 4) (√)[(a + 4)2 + (a + 10)2] cao
Gradient of perpendicular =
a + 10
oe but
( )
(2) a 2 + 14a − 72 (= 0)
−1 a = 4 or − 18 cao
not
 a + 10 
 
 a+4 

45 M/J 14/P12/Q1

(2, 7) to (10, 3)
Mid-point (6, 5)
Gradient = −½
Perp gradient = 2 (5)(
Eqn y − 5 = 2(x − 6 )
Sets y to 0, → (3½, 0)

086
46 M/J 14/P11/Q7 Unit 3: Answer Section

(a − 3)2 + (2 − b)2 = 125 oe


2−b
=2 oe
a −3
(a − 3)2 + (2a − 6)2 = 125
(sub for a or b)
a + 2)(a – 8) (= 0) Attempt factorise/solve
a = –2 or 8, b = 12 or –8

47 M/J 14/P13/Q11
Sim eqns → A (1, 3)
Vectors or mid-point → C (12, 14)
Eqn of BC 4y = x + 44 or CD y = 3x − 22
Sim eqns → B(4, 12) or D (9, 5)
Vectors or mid-point → B(4, 12) or D (9, 5)

48 O/N 13/P12/Q5
A (0, 8) B (4, 0) 8y + x= 33
m of AB = −2
m of BC = ½
Eqn BC → y − 0 = ½(x − 4)
Sim eqns → C (16, 6)
Vector step method → D (12, 14)
(or AD y = ½x +8, CD y = −2x + 38)
(or M = (8, 7) → D = (12, 14) )

49 O/N 13/P11/Q7
(i) mid-point = (3, 4)
Grad. AB = –½ → grad. of perp., = 2
y − 4 = 2( x − 3 )
y = 2x − 2

(ii) q = 2p − 2 p 2 + q 2 = 4 oe
p 2 + (2 p − 2 ) = 4 → 5 p 2 − 8 p = 0
2

{OR ¼ (q + 2 ) + q 2 = 4 → 5q 2 + 4q − 12 = 0 }
2

8 6
(0,−2) and  , 
5 5
50 O/N 13/P13/Q3
(i) gradient of perpendicular = ‒½ soi
y – 1 = – ½ (x – 3)
(ii) C = (‒9, 6)
AC2 = [3 – (–9)]2 + [1 – 6]2 (ft on their C)
AC = 13
51 M/J 13/P12/Q7
3 y + 2 x = 33.
Gradient of line = −⅔
Gradient of perpendicular = 3/2
Eqn of perp y − 3 = 32 ( x + 1)
Sim Eqns → (3, 9)
(−1, 3) → (3, 9) → (7, 15)

087
52 M/J 13/P13/Q7 Unit 3: Answer Section

A (2, 14), B (14, 6) and C (7, 2).


(i) m of AB = −⅔
3
m of perpendicular = 2

eqn of AB y − 14 = − 23 ( x − 2)
eqn of CX y − 2 = 32 ( x − 7)
Sim Eqns → X (11, 8)
(ii) AX : XB = 14−8 : 8− 6 = 3 : 1
Or √(9²+6²) : √(3²+2²) = 3: 1
53 O/N 12/P12/Q5
A (1, 3), B (5, 11) , X (4, 4)

(i) Gradient of AB = 2
Gradient of BC = −½
→ Eqn of BC is y − 11 = − 12 ( x − 5)

(ii) gradient of AC (or AX) is ⅓


→ eqn of AC is y − 3 = 13 ( x − 1)
or y − 4 = 13 ( x − 4)
Sim equations → C (13,7)
54 M/J 12/P12/Q4
A (−1, −5), B (7, 1).
M (3, −2)
Gradient = 34
Perpendicular gradient = − 4
3
Eqn y + 2 = − 43 ( x − 3)
Sets x and y to 0 C( 32 , 0) D(0, 2)

→ Pythagoras → CD = 2.5
55 M/J 12/P11/Q5
(i) ( ) 2
6x + 2 = 7 x ⇒ 6 x − 7 x + 2 = 0
(3 x − 2)(2 x − 1) = 0
2 1
x= or
3 2
4 1
x = or (or 0.444, 0.25)
9 4
OR (6 x + 2 ) = 49 x → 36 x 2 − 25 x + 4 = 0
2

(9 x − 4)(4 x − 1) = 0
4 1
x = or (or 0.444, 0.25) oe
9 4
(ii) 7 2 − 4 × 6 × k (= 0 )
49
k= or 2.04
24
OR
d
dx
( )
1 d
dx
7 −1
7 x 2 = (6 x + k ) → x 2 = 6
2
49 49 49
x= , y= →k = or 2.04
144 12 24

088
Unit 3: Answer Section
56 M/J 12/P11/Q5
1
(i) M = (1, 4) gradient = soi
2
grad of MB = –2 soi
Equation MB : y – 4 = –2(x – 1)
When y = 0, x = 3 or B = (3, 0)
2 6
(ii) grad of AB = − ; grad of BC = oe
6 2
m1m2 = −1(⇒ AB ⊥ AC )

(iii) D = (–1, 8)
AD = 40 or 6.32

57 M/J 11/P12/Q7
(i) (2, 5) to (10, 9) gradient = ½
Equation of L2 y = 12 x .
Gradient of perpendicular = − 2
Eqn of Perp y − 5 = −2( x − 2)
Sim Eqns → C(3.6, 1.8)

(ii) d² = 1.6² + 3.2² → d = 3.58

58 M/J 11/P13/Q3
x y
+ =1
a b
P (a, 0) and Q(0, b)
Distance → (a 2
)
+ b 2 = 45
− a −1
Gradients → =
b 2
Solution of sim eqns → a = 6, b = 3

089
UNIT 4 Circular Measure Unit 4: Circular Measure

B
1 M/J 22/P12/Q7 2 cm
A

10 cm

1

O P C
The diagram shows a sector OBAC of a circle with centre O and radius 10 cm. The point P lies on
OC and BP is perpendicular to OC. Angle AOC = 16 π and the length of the arc AB is 2 cm.

(a) Find the angle BOC. [2]


(b) Hence find the area of the shaded region BPC giving your answer correct to 3 significant figures.
[4]
C
D
2 M/J 22/P11/Q5

A 1

B
The diagram shows a sector ABC of a circle with centre A and radius r. The line BD is perpendicular
to AC. Angle CAB is 1 radians.

(a) Given that 1 = 16 π, find the exact area of BCD in terms of r. [3]
3
(b) Given instead that the length of BD is r, find the exact perimeter of BCD in terms of r. [4]
2

3 M/J 22/P13/Q9
D

1.8 rad
6 cm
6 cm

C A
The diagram shows triangle ABC with AB = BC = 6 cm and angle ABC = 1.8 radians. The arc CD is
part of a circle with centre A and ABD is a straight line.

(a) Find the perimeter of the shaded region. [5]


(b) Find the area of the shaded region. [3]

090
Unit 4: Circular Measure
4 O/N 21/P12/Q7
15 c m P B
Q
A

9 cm

15
cm

In the diagram the lengths of AB and AC are both 15 cm. The point P is the foot of the perpendicular
from C to AB. The length CP = 9 cm. An arc of a circle with centre B passes through C and meets
AB at Q.
(a) Show that angle ABC = 1.25 radians, correct to 3 significant figures. [2]
(b) Calculate the area of the shaded region which is bounded by the arc CQ and the lines CP and
PQ. [4]
C
5 O/N 21/P11/Q6

A 6 cm B

The diagram shows a metal plate ABC in which the sides are the straight line AB and the arcs AC
and BC. The line AB has length 6 cm. The arc AC is part of a circle with centre B and radius 6 cm,
and the arc BC is part of a circle with centre A and radius 6 cm.

(a) Find the perimeter of the plate, giving your answer in terms of π. [3]
(b) Find the area of the plate, giving your answer in terms of π and 3.

[4]

B
6 O/N 21/P13/Q5
Y

9 cm

11 cm
X C

In the diagram, X and Y are points on the line AB such that BX = 9 cm and AY = 11 cm. Arc BC is
part of a circle with centre X and radius 9 cm, where CX is perpendicular to AB. Arc AC is part of a
circle with centre Y and radius 11 cm.

(a) Show that angle XYC = 0.9582 radians, correct to 4 significant figures. [1]
(b) Find the perimeter of ABC. [6]
091
7 M/J 21/P12/Q12 Q Unit 4: Circular Measure
P
A B

F C

E D

The diagram shows a cross-section of seven cylindrical pipes, each of radius 20 cm, held together by a
thin rope which is wrapped tightly around the pipes. The centres of the six outer pipes are A, B, C, D,
E and F. Points P and Q are situated where straight sections of the rope meet the pipe with centre A.

(a) Show that angle PAQ = 13 π radians. [2]



(c) Find the area of the hexagon ABCDEF, giving your answer in terms of 3. [2]
(d) Find the area of the complete region enclosed by the rope. [3]

8 M/J 21/P11/Q8
P Q

S R

The diagram shows a symmetrical metal plate. The plate is made by removing two identical pieces
from a circular disc with centre C. The boundary of the plate consists of two arcs PS and QR of the
original circle and two semicircles with PQ and RS as diameters. The radius of the circle with centre
C is 4 cm, and PQ = RS = 4 cm also.

(a) Show that angle PCS = 23 π radians. [2]


(b) Find the exact perimeter of the plate. [3]
  
(c) Show that the area of the plate is 20
3
π + 8 3 cm2. [5]

092
9 M/J 21/P13/Q5 Unit 4: Circular Measure
A

D
4 cm

B C

The diagram shows a triangle ABC, in which angle ABC = 90Å and AB = 4 cm. The sector ABD is
part of a circle with centre A. The area of the sector is 10 cm2.

(a) Find angle BAD in radians. [2]


(b) Find the perimeter of the shaded region. [4]

10 O/N 20/P12/Q8 D
A C
1 rad

r cm r cm

B
In the diagram, ABC is an isosceles triangle with AB = BC = r cm and angle BAC = 1 radians. The
point D lies on AC and ABD is a sector of a circle with centre A.

(a) Express the area of the shaded region in terms of r and 1. [3]
(b) In the case where r = 10 and 1 = 0.6, find the perimeter of the shaded region. [4]

093
11 O/N 20/P11/Q10 C Unit 4: Circular Measure

1 rad

F E
r
O

A B
D
The diagram shows a sector CAB which is part of a circle with centre C. A circle with centre O and
radius r lies within the sector and touches it at D, E and F, where COD is a straight line and angle
ACD is 1 radians.
(a) Find CD in terms of r and sin 1. [3]
It is now given that r = 4 and 1 = 16 π.
(b) Find the perimeter of sector CAB in terms of π. [3]

(c) Find the area of the shaded region in terms of π and 3. [4]

12 O/N 20/P13/Q9 B

12 cm
8 cm

A 8 cm O C

In the diagram, arc AB is part of a circle with centre O and radius 8 cm. Arc BC is part of a circle
with centre A and radius 12 cm, where AOC is a straight line.
(a) Find angle BAO in radians. [2]
(b) Find the area of the shaded region. [4]
(c) Find the perimeter of the shaded region. [3]

094
Unit 4: Circular Measure
13 M/J 20/P12/Q7 A
r

C
r
1
6 π rad
O B
2r

In the diagram, OAB is a sector of a circle with centre O and radius 2r, and angle AOB = 16 π radians.
The point C is the midpoint of OA.
 
(a) Show that the exact length of BC is r 5 − 2 3. [2]
(b) Find the exact perimeter of the shaded region. [2]
(c) Find the exact area of the shaded region. [3]

14 M/J 20/P11/Q8 15 cm

A C
6 cm O X

In the diagram, ABC is a semicircle with diameter AC, centre O and radius 6 cm. The length of the
arc AB is 15 cm. The point X lies on AC and BX is perpendicular to AX .

Find the perimeter of the shaded region BXC. [6]

15 M/J 20/P13/Q5

O
5
A B

13

P
The diagram shows a cord going around a pulley and a pin. The pulley is modelled as a circle with
centre O and radius 5 cm. The thickness of the cord and the size of the pin P can be neglected. The
pin is situated 13 cm vertically below O. Points A and B are on the circumference of the circle such
that AP and BP are tangents to the circle. The cord passes over the major arc AB of the circle and
under the pin such that the cord is taut.

Calculate the length of the cord. [6]

095
A Unit 4: Circular Measure
16 MAR 20/P12/Q7

0.8 rad
O B
C
6 cm

The diagram shows a sector AOB which is part of a circle with centre O and radius 6 cm and with
angle AOB = 0.8 radians. The point C on OB is such that AC is perpendicular to OB. The arc CD is
part of a circle with centre O, where D lies on OA.

Find the area of the shaded region. [6]

17 O/N 19/P12/Q4
A

r cm

O 21 rad T

The diagram shows a circle with centre O and radius r cm. Points A and B lie on the circle and
angle AOB = 21 radians. The tangents to the circle at A and B meet at T .

(i) Express the perimeter of the shaded region in terms of r and 1. [3]
(ii) In the case where r = 5 and 1 = 1.2, find the area of the shaded region. [4]

C
18 O/N 19/P11/Q8

6 cm

3
8 0 rad

O A

The diagram shows a sector OAC of a circle with centre O. Tangents AB and CB to the circle meet
at B. The arc AC is of length 6 cm and angle AOC = 38 0 radians.

(i) Find the length of OA correct to 4 significant figures. [2]


(ii) Find the perimeter of the shaded region. [2]
(iii) Find the area of the shaded region. [4]

096
Unit 4: Circular Measure
19 O/N 19/P13/Q4
C

A r O r B
The diagram shows a semicircle ACB with centre O and radius r. Arc OC is part of a circle with
centre A.
(i) Express angle CAO in radians in terms of 0. [1]
(ii) Find the area of the shaded region in terms of r, 0 and ï3, simplifying your answer. [4]

20 M/J 19/P12/Q5

1 rad
A O r B
The diagram shows a semicircle with diameter AB, centre O and radius r. The point C lies on the
circumference and angle AOC = 1 radians. The perimeter of sector BOC is twice the perimeter of
sector AOC. Find the value of 1 correct to 2 significant figures. [5]

21 M/J 19/P13/Q3 A

8 cm
E

1
5 0 rad
B D C

The diagram shows triangle ABC which is right-angled at A. Angle ABC = 15 0 radians and AC = 8 cm.
The points D and E lie on BC and BA respectively. The sector ADE is part of a circle with centre A
and is such that BDC is the tangent to the arc DE at D.

(i) Find the length of AD. [3]


(ii) Find the area of the shaded region. [3]
C
22 MAR 19/P12/Q3

7 8

Y X
A B

D
In the diagram, CXD is a semicircle of radius 7 cm with centre A and diameter CD. The straight line
YABX is perpendicular to CD, and the arc CYD is part of a circle with centre B and radius 8 cm. Find
the total area of the region enclosed by the two arcs. [6]

097
Y Unit 4: Circular Measure
23 O/N 18/P12/Q8
X

8 cm
8 cm

A 12 cm C

The diagram shows an isosceles triangle ACB in which AB = BC = 8 cm and AC = 12 cm. The arc
XC is part of a circle with centre A and radius 12 cm, and the arc YC is part of a circle with centre B
and radius 8 cm. The points A, B, X and Y lie on a straight line.

(i) Show that angle CBY = 1.445 radians, correct to 4 significant figures. [3]
(ii) Find the perimeter of the shaded region. [4]
A
24 O/N 18/P11/Q9

C 5 cm

1
5 0 rad
O D B

The diagram shows a triangle OAB in which angle ABO is a right angle, angle AOB = 15 0 radians and
AB = 5 cm. The arc BC is part of a circle with centre A and meets OA at C. The arc CD is part of a
circle with centre O and meets OB at D. Find the area of the shaded region. [8]
A
25 O/N 18/P13/Q3

5 cm

C
4 cm
D B

The diagram shows an arc BC of a circle with centre A and radius 5 cm. The length of the arc BC
is 4 cm. The point D is such that the line BD is perpendicular to BA and DC is parallel to BA.
(i) Find angle BAC in radians. [1]
(ii) Find the area of the shaded region BDC. [5]

098
Unit 4: Circular Measure
26 M/J 18/P12/Q6 B

20 cm


A C
9 cm D

The diagram shows a triangle ABC in which BC = 20 cm and angle ABC = 90Å. The perpendicular
from B to AC meets AC at D and AD = 9 cm. Angle BCA = 1Å.

(i) By expressing the length of BD in terms of 1 in each of the triangles ABD and DBC, show that
20 sin2 1 = 9 cos 1. [4]
(ii) Hence, showing all necessary working, calculate 1. [3]

27 M/J 18/P11/Q6 A
T

r cm
1 rad B

The diagram shows a circle with centre O and radius r cm. The points A and B lie on the circle and
AT is a tangent to the circle. Angle AOB = 1 radians and OBT is a straight line.

(i) Express the area of the shaded region in terms of r and 1. [3]
(ii) In the case where r = 3 and 1 = 1.2, find the perimeter of the shaded region. [4]

28 M/J 18/P13/Q5
A

5 cm
6 cm

O C B

The diagram shows a triangle OAB in which angle OAB = 90Å and OA = 5 cm. The arc AC is part
of a circle with centre O. The arc has length 6 cm and it meets OB at C. Find the area of the shaded
region. [5]

099
C Unit 4: Circular Measure
29 O/N 17/P12/Q4

P Q

A B
D O 6 cm

The diagram shows a semicircle with centre O and radius 6 cm. The radius OC is perpendicular to
the diameter AB. The point D lies on AB, and DC is an arc of a circle with centre B.
(i) Calculate the length of the arc DC. [3]
(ii) Find the value of area of region P
,
area of region Q
giving your answer correct to 3 significant figures. [4]

30 O/N 17/P11/Q5
B

10 cm 10 cm

A D E C
16 cm

The diagram shows an isosceles triangle ABC in which AC = 16 cm and AB = BC = 10 cm. The
circular arcs BE and BD have centres at A and C respectively, where D and E lie on AC.
(i) Show that angle BAC = 0.6435 radians, correct to 4 decimal places. [1]
(ii) Find the area of the shaded region. [5]

31 M/J 17/P12/Q4

O
r cm
21 rad

A B
r cm

D C
The diagram shows a circle with radius r cm and centre O. Points A and B lie on the circle and ABCD
is a rectangle. Angle AOB = 21 radians and AD = r cm.

(i) Express the perimeter of the shaded region in terms of r and 1. [3]
(ii) In the case where r = 5 and 1 = 1 0, find the area of the shaded region. [4]
6

100
Unit 4: Circular Measure
32 M/J 17/P11/Q8 X

M
A B
12 cm

10 cm

In the diagram, OAXB is a sector of a circle with centre O and radius 10 cm. The length of the chord
AB is 12 cm. The line OX passes through M , the mid-point of AB, and OX is perpendicular to AB.
The shaded region is bounded by the chord AB and by the arc of a circle with centre X and radius XA.
(i) Show that angle AXB is 2.498 radians, correct to 3 decimal places. [3]
(ii) Find the perimeter of the shaded region. [3]
(iii) Find the area of the shaded region. [3]

33 M/J 17/P13/Q7 C

8 cm 10 cm

A B

D
The diagram shows two circles with centres A and B having radii 8 cm and 10 cm respectively. The
two circles intersect at C and D where CAD is a straight line and AB is perpendicular to CD.
(i) Find angle ABC in radians. [1]
(ii) Find the area of the shaded region. [6]

34 O/N 16/P12/Q6
B

cm
10

A O 1.2 rad C

The diagram shows a metal plate ABCD made from two parts. The part BCD is a semicircle. The
part DAB is a segment of a circle with centre O and radius 10 cm. Angle BOD is 1.2 radians.
(i) Show that the radius of the semicircle is 5.646 cm, correct to 3 decimal places. [2]
(ii) Find the perimeter of the metal plate. [3]
(iii) Find the area of the metal plate. [3]

101
A Unit 4: Circular Measure
35 O/N 16/P11/Q3
r
C
r
O ! rad

In the diagram OCA and ODB are radii of a circle with centre O and radius 2r cm. Angle
AOB = ! radians. CD and AB are arcs of circles with centre O and radii r cm and 2r cm respectively.
The perimeter of the shaded region ABDC is 4.4r cm.
(i) Find the value of !. [2]
(ii) It is given that the area of the shaded region is 30 cm2. Find the value of r. [3]

36 O/N 16/P13/Q5
B
D

6 cm O
1.8 rad E

C
A

The diagram shows a major arc AB of a circle with centre O and radius 6 cm. Points C and D on OA
and OB respectively are such that the line AB is a tangent at E to the arc CED of a smaller circle also
with centre O. Angle COD = 1.8 radians.
(i) Show that the radius of the arc CED is 3.73 cm, correct to 3 significant figures. [2]
(ii) Find the area of the shaded region. [4]

37 M/J 16/P12/Q6

! rad
r cm Q

T
P

The diagram shows a circle with radius r cm and centre O. The line PT is the tangent to the circle
at P and angle POT = ! radians. The line OT meets the circle at Q.

(i) Express the perimeter of the shaded region PQT in terms of r and !. [3]

(ii) In the case where ! = 13 0 and r = 10, find the area of the shaded region correct to 2 significant
figures. [3]
102
38 M/J 16/P11/Q7 B Unit 4: Circular Measure

C
D
r

1
A r O

In the diagram, AOB is a quarter circle with centre O and radius r. The point C lies on the arc AB
and the point D lies on OB. The line CD is parallel to AO and angle AOC = 1 radians.

(i) Express the perimeter of the shaded region in terms of r, 1 and 0. [4]

(ii) For the case where r = 5 cm and 1 = 0.6, find the area of the shaded region. [3]

39 M/J 16/P13/Q6

3 E 2
A B

3 2
F G
1 1
C

The diagram shows triangle ABC where AB = 5 cm, AC = 4 cm and BC = 3 cm. Three circles with
centres at A, B and C have radii 3 cm, 2 cm and 1 cm respectively. The circles touch each other at
points E, F and G, lying on AB, AC and BC respectively. Find the area of the shaded region EFG.
[7]
C
40 O/N 15/P12/Q5

0.6 rad
O 6 cm A
The diagram shows a metal plate OABC, consisting of a right-angled triangle OAB and a sector OBC
of a circle with centre O. Angle AOB = 0.6 radians, OA = 6 cm and OA is perpendicular to OC.
(i) Show that the length of OB is 7.270 cm, correct to 3 decimal places. [1]
(ii) Find the perimeter of the metal plate. [3]
(iii) Find the area of the metal plate. [3]

103
41 O/N 15/P11/Q7 C Unit 4: Circular Measure

A
B E

The diagram shows a circle with centre A and radius r. Diameters CAD and BAE are perpendicular
to each other. A larger circle has centre B and passes through C and D.
(i) Show that the radius of the larger circle is rï2. [1]
(ii) Find the area of the shaded region in terms of r. [6]

42 O/N 15/P13/Q4 C D
r r

B E
r 1 rad r

O
r r

A F
The diagram shows a metal plate OABCDEF consisting of 3 sectors, each with centre O. The radius
of sector COD is 2r and angle COD is 1 radians. The radius of each of the sectors BOA and FOE
is r, and AOED and CBOF are straight lines.
(i) Show that the area of the metal plate is r2 0 + 1. [3]
(ii) Show that the perimeter of the metal plate is independent of 1. [4]
A
43 M/J 15/P12/Q2

O 21 rad X Y

In the diagram, AYB is a semicircle with AB as diameter and OAXB is a sector of a circle with centre
O and radius r. Angle AOB = 21 radians. Find an expression, in terms of r and 1, for the area of the
shaded region. [4]

104
44 M/J 15/P11/Q5 Unit 4: Circular Measure
A piece of wire of length 24 cm is bent to form the perimeter of a sector of a circle of radius r cm.
(i) Show that the area of the sector, A cm2, is given by A = 12r − r2 . [3]

(ii) Express A in the form a − r − b2 , where a and b are constants. [2]

(iii) Given that r can vary, state the greatest value of A and find the corresponding angle of the sector.
[2]

45 M/J 15/P13/Q11 A

O ! rad

B
In the diagram, OAB is a sector of a circle with centre O and radius r. The point C on OB is such
that angle ACO is a right angle. Angle AOB is ! radians and is such that AC divides the sector into
two regions of equal area.
(i) Show that sin ! cos ! = 12 !. [4]
It is given that the solution of the equation in part (i) is ! = 0.9477, correct to 4 decimal places.
(ii) Find the ratio perimeter of region OAC : perimeter of region ACB,
giving your answer in the form k : 1, where k is given correct to 1 decimal place. [5]

(iii) Find angle AOB in degrees. [1]

46 O/N 14/P12/Q2
P B
Q
5 cm

A O
12 cm

The diagram shows a triangle AOB in which OA is 12 cm, OB is 5 cm and angle AOB is a right angle.
Point P lies on AB and OP is an arc of a circle with centre A. Point Q lies on AB and OQ is an arc of
a circle with centre B.
(i) Show that angle BAO is 0.3948 radians, correct to 4 decimal places. [1]
(ii) Calculate the area of the shaded region. [5]

105
Unit 4: Circular Measure
47 O/N 14/P11/Q8 A
C

 rad
O B
D
4 cm

In the diagram, AB is an arc of a circle with centre O and radius 4 cm. Angle AOB is  radians. The
point D on OB is such that AD is perpendicular to OB. The arc DC, with centre O, meets OA at C.
(i) Find an expression in terms of  for the perimeter of the shaded region ABDC. [4]

(ii) For the case where  = 16 , find the area of the shaded region ABDC, giving your answer in the
form k, where k is a constant to be determined. [4]
C
48 O/N 14/P13/Q2

3
1
B rad D O

3 cm

A
In the diagram, OADC is a sector of a circle with centre O and radius 3 cm. AB and CB are tangents
to the circle and angle ABC = 13  radians. Find, giving your answer in terms of 3 and ,
(i) the perimeter of the shaded region, [3]
(ii) the area of the shaded region. [3]

49 M/J 14/P12/Q4 O
1 rad r cm

A B

The diagram shows a sector of a circle with radius r cm and centre O. The chord AB divides the
sector into a triangle AOB and a segment AXB. Angle AOB is 1 radians.
(i) In the case where the areas of the triangle AOB and the segment AXB are equal, find the value
of the constant p for which 1 = p sin 1. [2]

(ii) In the case where r = 8 and 1 = 2.4, find the perimeter of the segment AXB. [3]

106
Unit 4: Circular Measure
C
50 M/J 14/P11/Q6

2 cm

! rad
A B
E
4 cm
The diagram shows triangle ABC in which AB is perpendicular to BC. The length of AB is 4 cm and
angle CAB is ! radians. The arc DE with centre A and radius 2 cm meets AC at D and AB at E. Find,
in terms of !,
(i) the area of the shaded region, [3]
(ii) the perimeter of the shaded region. [3]

51 M/J 14/P13/Q3

O
6 cm
2.2 rad

A B
The diagram shows part of a circle with centre O and radius 6 cm. The chord AB is such that
angle AOB = 2.2 radians. Calculate
(i) the perimeter of the shaded region, [3]
(ii) the ratio of the area of the shaded region to the area of the triangle AOB, giving your answer in
the form k : 1. [3]

52 O/N 13/P12/Q2

8 cm q rad

O
A 6 cm
A
Fig. 1 Fig. 2

Fig. 1 shows a hollow cone with no base, made of paper. The radius of the cone is 6 cm and the height
is 8 cm. The paper is cut from A to O and opened out to form the sector shown in Fig. 2. The circular
bottom edge of the cone in Fig. 1 becomes the arc of the sector in Fig. 2. The angle of the sector is
1 radians. Calculate
(i) the value of 1, [4]
(ii) the area of paper needed to make the cone. [2]

107
53 O/N 13/P11/Q6 B Unit 4: Circular Measure
r
A
r
O a rad

E D
C

The diagram shows a metal plate made by fixing together two pieces, OABCD (shaded) and OAED
(unshaded). The piece OABCD is a minor sector of a circle with centre O and radius 2r. The piece
OAED is a major sector of a circle with centre O and radius r. Angle AOD is ! radians. Simplifying
your answers where possible, find, in terms of !, 0 and r,
(i) the perimeter of the metal plate, [3]
(ii) the area of the metal plate. [3]

It is now given that the shaded and unshaded pieces are equal in area.

(iii) Find ! in terms of 0. [2]

54 O/N 13/P13/Q6
A B

C D
11
cm

a rad
m
5c

O
The diagram shows sector OAB with centre O and radius 11 cm. Angle AOB = ! radians. Points C
and D lie on OA and OB respectively. Arc CD has centre O and radius 5 cm.
(i) The area of the shaded region ABDC is equal to k times the area of the unshaded region OCD.
Find k. [3]
(ii) The perimeter of the shaded region ABDC is equal to twice the perimeter of the unshaded region
OCD. Find the exact value of !. [4]
X
55 M/J 13/P12/Q4
B C

10 cm

A D
O
The diagram shows a square ABCD of side 10 cm. The mid-point of AD is O and BXC is an arc of a
circle with centre O.
(i) Show that angle BOC is 0.9273 radians, correct to 4 decimal places. [2]
(ii) Find the perimeter of the shaded region. [3]
(iii) Find the area of the shaded region. [2]

108
B Unit 4: Circular Measure
56 M/J 13/P11/Q3 8 cm

O a rad A
8 cm

C
In the diagram, OAB is a sector of a circle with centre O and radius 8 cm. Angle BOA is ! radians.
OAC is a semicircle with diameter OA. The area of the semicircle OAC is twice the area of the sector
OAB.
(i) Find ! in terms of 0. [3]
(ii) Find the perimeter of the complete figure in terms of 0. [2]

R
57 M/J 13/P13/Q2

Q
C

O 3 cm 6 cm
S
P

The diagram shows a circle C with centre O and radius 3 cm. The radii OP and OQ are extended to
S and R respectively so that ORS is a sector of a circle with centre O. Given that PS = 6 cm and that
the area of the shaded region is equal to the area of circle C,
(i) show that angle POQ = 14 0 radians, [3]
(ii) find the perimeter of the shaded region. [2]
Q
58 O/N 12/P12/Q11

x cm

P R
cm

S
20

1.2 rad

The diagram shows a sector of a circle with centre O and radius 20 cm. A circle with centre C and
radius x cm lies within the sector and touches it at P, Q and R. Angle POR = 1.2 radians.

(i) Show that x = 7.218, correct to 3 decimal places. [4]

(ii) Find the total area of the three parts of the sector lying outside the circle with centre C. [2]

(iii) Find the perimeter of the region OPSR bounded by the arc PSR and the lines OP and OR. [4]

109
59 O/N 12/P11/Q6 B
Unit 4: Circular Measure

r D

q rad
O A
C

The diagram shows a sector OAB of a circle with centre O and radius r. Angle AOB is θ radians.
The point C on OA is such that BC is perpendicular to OA. The point D is on BC and the circular arc
AD has centre C .
(i) Find AC in terms of r and θ . [1]
(ii) Find the perimeter of the shaded region ABD when θ = 13 π and r = 4, giving your answer as an
exact value. [6]

A
60 O/N 12/P13/Q4

D
2 cm

B C
2 Ö3 cm
In the diagram, D lies on the side AB of
√ triangle ABC and CD is an arc of a circle with centre A and
radius 2 cm. The line BC is of length 2 3 cm and is√perpendicular to AC. Find the area of the shaded
region BDC, giving your answer in terms of π and 3. [4]
A B
61 M/J 12/P12/Q6
2.4 rad 8 cm
O

The diagram shows a metal plate made by removing a segment from a circle with centre O and radius
8 cm. The line AB is a chord of the circle and angle AOB = 2.4 radians. Find
(i) the length of AB, [2]
(ii) the perimeter of the plate, [3]
(iii) the area of the plate. [3]

110
A
62 M/J 12/P11/Q3 Unit 4: Circular Measure

2 cm 2 cm

P R
B C
Q
2 cm
In the diagram, ABC is an equilateral triangle of side 2 cm. The mid-point of BC is Q. An arc of
a circle with centre A touches BC at Q, and meets AB √ at P and AC at R. Find the total area of the
shaded regions, giving your answer in terms of π and 3. [5]
X
63 M/J 12/P13/Q8

O r B

In the diagram, AB is an arc of a circle with centre O and radius r. The line XB is a tangent to the
circle at B and A is the mid-point of OX .
(i) Show that angle AOB = 13 π radians. [2]

Express each of the following in terms of r, π and 3:
(ii) the perimeter of the shaded region, [3]
(iii) the area of the shaded region. [2]

64 O/N 11/P12/Q6

The diagram shows a circle C1 touching a circle C2 at a point X . Circle C1 has centre A and radius
6 cm, and circle C2 has centre B and radius 10 cm. Points D and E lie on C1 and C2 respectively and
DE is parallel to AB. Angle DAX = 13 π radians and angle EBX = θ radians.

(i) By considering
√ the perpendicular distances of D and E from AB, show that the exact value of θ
3 3
is sin−1  . [3]
10

(ii) Find the perimeter of the shaded region, correct to 4 significant figures. [5]

111
Unit 4: Circular Measure
65 O/N 11/P11/Q5 C B
q rad

q rad
O A
r

The diagram represents a metal plate OABC, consisting of a sector OAB of a circle with centre O and
radius r, together with a triangle OCB which is right-angled at C . Angle AOB = θ radians and OC
is perpendicular to OA
(i) Find an expression in terms of r and θ for the perimeter of the plate. [3]
(ii) For the case where r = 10 and θ = 15 π , find the area of the plate. [3]

66 O/N 11/P11/Q5 D 10 cm C

P
10 cm
Q

0.8 rad
A 10 cm B

In the diagram, ABCD is a parallelogram with AB = BD = DC = 10 cm and angle ABD = 0.8 radians.
APD and BQC are arcs of circles with centres B and D respectively.

(i) Find the area of the parallelogram ABCD. [2]

(ii) Find the area of the complete figure ABQCDP. [2]

(iii) Find the perimeter of the complete figure ABQCDP. [2]

67 M/J 11/P11/Q9
A S B

r
P T
2q

O
In the diagram, OAB is an isosceles triangle with OA = OB and angle AOB = 2θ radians. Arc PST
has centre O and radius r, and the line ASB is a tangent to the arc PST at S.

(i) Find the total area of the shaded regions in terms of r and θ . [4]

(ii) In the case where θ = 13 π and r = 6, find the total perimeter of the shaded regions, leaving your

answer in terms of 3 and π . [5]

112
68 M/J 11/P13/Q7 A
Unit 4: Circular Measure

6 cm

1p
3
O X
B

In the diagram, AB is an arc of a circle, centre O and radius 6 cm, and angle AOB = 13 π radians. The
line AX is a tangent to the circle at A, and OBX is a straight line.

(i) Show that the exact length of AX is 6 3 cm. [1]

Find, in terms of π and 3,
(ii) the area of the shaded region, [3]
(iii) the perimeter of the shaded region. [4]

113
UNIT 4 Circular Measure Unit 4: Answer Section
Answer
(b)  5π  6   5π  6 
[BP] = 10sin   and [OP ] = 10cos  
1 M/J 22/P12/Q7  30   30 
[= 6.6208…] and [= 7.494…]
(a)  ˆ  2
 AOB  10 OR
5π  6 1  5π  6   5π  3 
[B Ô C ] or π  0.2 [BP] = 10sin   and [O B̂ P] =  
30 6  30   15 
[= 6.6208…] and [= 0.84719…]
Alternative method for question 7(a)
1  5π  6   5π  6 
10π 10π Area of ∆OBP =  10sin   10cos   or
OR [Arc AC =] or  Arc BC   2 or 7.2 2  30   30 
6 6
1  5π  6    5π  3  
 10  10sin    sin   
5π  6 1 2  30    15  
[B Ô C ] or π  0.2
30 6 [=24.809]
1  5π  6 
[Sector BOC] =  102  their  
2  30 
  5π  6  
  50  30   36.1799...
   
Area of region BPC = 11.4

2 M/J 22/P11/Q5

(b)  3 
(a) 1 2  π  π 2  r   π
1 2  
Sector area = r    r
2  6   12  Angle BAC = sin   
 r   3
 
π  1  π  3   
BD = sin r   r  and AD = cos r   r 1
Length AD = cos r   r 
6  2  6  2  π 1
[so length CD = r]
3  2  2
1 π  π  1 1 3 
so triangle area =  sin r  cos r     r  r π
2 6  6  2 2 2  Length of arc BC = r 
3
1  π  π  1 3 1
or r  cos r  sin    r  r  3 1 π
2  6  6  2 2 2 Perimeter of BCD = r r r
2 2 3

1 2 3 2
Area of BCD = πr  r
12 8

3 M/J 22/P13/Q9
(a) AC
6sin 0.9  or AC 2  62  6 2  2  6  6cos1.8
2
AC = 9.40
Angle CAB = ½(π ‒ 1.8)
Arc CD = their 9.40  their 0.6708
[Perimeter = 6 + 3.40 + 6.306 =] 15.7

(b) Sector ADC ‒ ABC = ½  their 9.402  their 0.6708 – ½  62  sin 1.8
[29.64 ‒ 17.53 =] 12.1

114
4 O/N 21/P12/Q7 Unit 4: Answer Section
(a) EITHER
ˆ = 0.6435 … and ABC
By using trigonometry: BAC ˆ = π − 0.6435
2
OR
By Pythagoras: AP = 12 ⇒ BP = 3 so tan ABC ˆ = 9
3
OR
Using ∆PBC and either the sine or cosine rule
ˆ = 3 or cos ABC
sin ABC ˆ = 10
10 10

ˆ = π − 0.6435 or tan-1 9 or sin −1 3 or cos −1 10 or


ABC
2 3 10 10
1.249 ( 04…) or 71.56° = 1.25 radians (3 sf)

(b) 9
BC = ( their 3)2 + 92 or [= 90 , 3 10 or 9.48697…]
sin1.25

1
× ( their BC ) × tan −1 3 [ = 56.207 or 56.25]
2
Area of sector =
2

1
Area of triangle PBC = 13.4 to 13.6 or ×9×3
2

[Area = ( 56.207 or 56.25 ) – their 13.5 =] 42.7 or 42.8

5 O/N 21/P11/Q6
π
(a) Recognise that at least one of angles A, B, C is
3
π π
One arc 6 × their leading to two arcs 2 × 6 × their
3 3
Perimeter = 6 + 4π
Alternative method for question 6(a)

Calculate circumference of whole circle = 12π

1 1
One arc × 12π leading to two arcs 2 × × 12π
6 6

Perimeter = 6 + 4π

(b) 1 2 π
Sector = × 6 × their  
2 3
1 π 1   π 
2
( ) 3 2
( )
× 62 × their   − × 62 × sin  their    + 6π  = 6π − 9 3 +6π 
  3 
Area = 12π − 9 3
Alternative method for question 6(b)
1 2 π
Sector = × 6 × their  
2 3
1  π  1   π 
 3  2
( )
2 ×  × 62 × their    − × 62 × sin  their   
2   3 
Area = 12π − 9 3

115
O/N 21/P13/Q5 Unit 4: Answer Section
6
(a) 9
Angle XYC = sin −1   = 0.9582
 11 
9
or sin XYC = leading to XYC = 0.9582
11
(b) XY = 112 − 92 = 40 or using 0.9582 and trigonometry
AB = 9 + 11 − theirXY
Arc AC = 11 × 0.9582
π
Arc BC = 9×
2
Perimeter = [13.6(8) + 10.5(4) +14.1(4) =] 38.4

7 M/J 21/P12/Q12

(a) Alternative method for Question 12(a)


20
Sinθ = , with θ clearly identified
40
π π ˆ
ˆ and by similar triangles = PAQ
θ = , 2θ = = FAO
6 3
(b) Each straight section of rope has length 40 cm
π
Each curved section round each pipe has length rθ = 20 ×
3
Total length = 6 × ( ( their 40 ) + kπ )
240 + 40π or 366 (AWRT) (cm)
(c) 1 π 1
[Triangle area =] × 40 × 40 × sin   or × 40 × 20 3 or
2 3 2
400 3 or 693(AWRT)

[Total area of hexagon = 6 × 400 3 =] 2400 3


Alternative method for Question 12(c)
Area of triangle ABC = 400 3 or 693 (AWRT) or
4 × Area of half of triangle ABC = 4 ×200 3 or 1390 (AWRT)
or Area of rectangle ABDE = 1600 3 or 2770 (AWRT)

[Total area of hexagon = 2 × 400 3 +1600 3 =] 2400 3


Or [= 4 ×200 3 +1600 =] 2400 3

(d) Each rectangle area = 40 × 20 (= 800)


1 1 π  200π 
Each sector area = r 2θ = × 202 ×  =
2 2 3 3 
Total area = 2400 3 + 4800 + 400π or 10 200 (cm2) (AWRT)

8 M/J 21/P11/Q8

116
8 M/J 21/P11/Q8 Unit 4: Answer Section

(a) 2 π
Either Let midpoint of PQ be H: sin HCP = ⇒ Angle HCP =
4 6
4 π
Or sin PSQ = ⇒ Angle PSQ =
8 6
π
Or using cosine rule: angle PCQ =
3 π
Or by inspection: triangle PCQ or PCT is equilateral so angle PCQ =
π π 2 3
Angle PCS = π − − = π
6 6 3
2π 8π
(b) Perimeter = 2 × 4 × or 8π −
3 3
+2π × 2
28π
3
1 2 π 8π
(c) Area sector CPQ = ×4 × =
2 3 3
Area of segment of large circle beyond CPQ
8π 1 2  π  8π
= − × 4 × sin   = − 4 3
3 2 3 3
Area of small semicircle = π × 2 or area of small circle = π × 22
Area of plate = Large circle – [2 ×] small semicircle – [2 ×] segment area

 8π  20π
π × 42 − π × 22 − 2 ×  − 4 3  = +8 3
 3  3
9 M/J 21/P13/Q5
(a) ½ × 42 × angle BAD = 10

Angle BAD = 1.25

(b) Arc BD = 4 × their1.25

BC = 4 tan ( their1.25)
4
42 + ( their BC ) − 4
2
CD = − 4 or
cos ( their1.25 )
Perimeter = 5 + 12.0(4) + 8.69 = 25.7 (cm)

10 O/N 20/P12/Q8

(a) Use of correct formula for the area of triangle ABC


1
2
r 2 sin ( π − 2θ ) or 1
2
r 2 sin 2θ or 2 × 12 r × r cos θ × sin θ or
2 × 12 r cos θ × r sin θ
[Shaded area = triangle – sector] = their triangle area − 1
2
r 2θ

(b) Arc BD = rθ = 6 cm
AC = 2rcosθ = ( 2×10cos0.6 = 20cos0.6 = 16.506)
r × sin ( π – 2θ )
(
or 2r 2 − 2r 2 cos ( π − 2θ ) or )
sin θ

DC = 2rcosθ – r or ( 2r 2
)
− 2r 2 cos ( π − 2θ ) – r ( = 6.506)

(Perimeter = 10 + 6 + 6.506 =) 22.5

117
Unit 4: Answer Section
11 O/N 20/P11/Q10

(a)  r  r
 sin θ = →  OC =
 OC  sin θ
r
CD = r +
sin θ
4
(b) Radius of arc AB = 4 + = 4 + 8 = 12
π
sin
6
2π 1  π
(Arc AB =) their 12 × or  AB =  their 12 × 
6 2  6
Perimeter = 24 + 4π

1 π Alternative method for question 10(c)


(c) Area FOC = × 4 × their OC × sin
2 3
8 3 FC = ( their OC )2 − 42
1 2π 2 16π
Area sector FOE = × ×4 = 1
2 3 3 Area FOC = × 4 × 4 3 = 8 3
16π 2
Shaded area = 16 3 − 1 π 8π
3 Area of half sector FOE = × × 42 =
2 3 3
16π
Shaded area = 16 3 −
3

12 O/N 20/P13/Q9

(a) 6 82 + 122 − 82
cos BAO = or
8 2 × 8 × 12

BAO = 0.723
(b) Sector ABC = ½ × 122 × their 0.7227
Triangle AOB = ½ × 8 × 12sin ( their 0.7227 ) or ½×12×√28
Shaded area = their 52.0 − their 31.7 = 20.3
(c) Arc BC = 12 × their 0.7227

Perimeter = 8 + 4 + their 8.67 = 20.7

13 M/J 20/P12/Q7
(c) Are a = sector – triangle
π
(a) BC² = r² + 4r² − 2r.2r × cos   = 5r² − 2r²√3 1 π
6 Sector area = 4r ²
(
BC = r 5 − 2 3 ) 2 6
π
Triangle area = ½ r. 2r sin
(b) 2π r
Perimeter =
6
+r + r (5 − 2 3 ) 6
π 1
Shaded area = r ²  − 
 3 2

118
14 M/J 20/P12/Q7 Unit 4: Answer Section

Angle AOB = 15 ÷ 6 = 2.5 radians

Angle BOC = π – 2.5 (FT on angle AOB)

BC = 6(π – 2.5) (BC = 3.850)

sin(π – 2.5) = BX ÷6 (BX = 3.59)

Either OX = 6cos(π – 2.5) or Pythagoras (OX = 4.807)

XC = 6 – OX (XC = 1.193) → P = 8.63

15 M/J 20/P13/Q5
5
cos POA = → POA = 1.17 ( 6 ) Allow 67.4°
13
12 12
or sin = or tan =
13 5
Reflex AOB = 2π − 2 ×their 1.17 ( 6) OE in degrees
or minor arc AB = 5×2×their1.17(6)
Major arc = 5 × their 3.93(1)
or 2π × 5 - their 11.7(6)

AP (or BP) = 132 − 52 = 12

Cord length = 43.7

16 MAR 20/P12/Q7
OC = 6cos0.8 = 4.18(0)
1
Area sector OCD = ( their 4.18)2 × 0.8
2
1
ΔOCA = × 6 × their 4.18 × sin 0.8
2
Required area = their ΔOCA ‒ their sectorOCD

2.01

17 O/N 19/P12/Q4
(i) Arc length AB = 2rθ (ii) Area ∆AOT = ½ × 5 × 5 tan 1.2 or
AT BT Area AOBT = 2 × ½ × 5 × 5 tan 1.2
Tan θ = or → AT or BT = r tan θ
r r Sector area = ½ × 25 × 2.4 (or 1.2)
P = 2rθ + 2r tan θ Shaded area = 2 triangles – sector

Area = 34.3 (cm2)

Alternative method for question 4(ii)


Area of ∆ ABT = ½ × (5 × tan 1.2)2 × sin(π – 2.4) (= 55.86)
Segment area = ½ × 25 × (2.4 – sin 2.4) (= 21.56)
Shaded area = triangle – segment
Area = 34.3 (cm2)

18 O/N 19/P11/Q8

119
18 O/N 19/P11/Q8 Unit 4: Answer Section

(i) 3
OA × π = 6
8
16
OA = = 5.093 ( 0 )
π
(ii) 3
AB = their 5.0930 × tan π
16
Perimeter = 2 × 3.4030 + 6 = 12.8

(iii) Area OABC = ( 2 × ½ ) × their 5.0930 × their 3.4030


3
Area sector = ½ × ( their 5.0930 ) × π
2

8
Shaded area = their17.331 − their15.279 = 2.05

19 O/N 19/P13/Q4
π
(i) Angle CAO =
3
1 π
(ii) (Sector AOC) = r 2 × their
2 3
1  π 1 3 1
(∆ ABC) = ( r )( 2r ) sin  their  or ( 2r )( r ) or ( r )( r ) 3
2  3 2 2 2
1 π 1 3 1
(∆ ABC) = ( r )( 2r ) sin   or ( 2r )( r ) or ( r )( r ) 3
2 3 2 2 2
 3  1 2 π 
r 2   − r  
 2  2 3
 

20 M/J 19/P12/Q5
Perimeter of AOC = 2r + rθ

COB = π – θ
Angle
BOC = 2r + r(π – θ)
Perimeter of

r +) πr – rθ = 2((2r) + rθ)
(2 π −2
(2 + π – θ = 4 + 2θ → θ = )
3

θ = 0.38
21 M/J 19/P13/Q3

(i) Angle EAD = Angle ACD = or 54° or 0.942 soi
π π
( theirAD ) × their  − 
10 (ii) 1 2
π Area sector =
or Angle DAC = or 36° or 0.628 soi 2 2 5
5
3π π 1 π
AD = 8sin( ) or 8cos( ) Area ∆ADC = × 8 × theirAD × sin or
10 5 2 5
(AD =) 6.47 1  3π   3π 
× 8cos   × 8sin  
Alternative method for question 3(i) 2  10   10 
 3π  (Shaded area =) 35.0 or 34.9
8sin  
8  10  or 11. 01
AB = or AB = ( )
 
π π 
tan   sin  
5 5
π
AD = 11.0 (1) sin oe
5
(AD =) 6.47

120
22 MAR 19/P12/Q3 Unit 4: Answer Section
7  −17 
Angle CBA = sin −1   = 1.0654 or CBD = cos −1   = 2.13
8  32 
Sector BCYD = ½ × 82 × 2 × their1.0654 ( rad ) soi
or sector CBY = ½ × 82 × their1.0654 ( rad )

∆BCD = 7 × 82 − 7 2 or ½ × 82 × sin ( 2 × their1.0654 ) soi

Semi-circle CXD = ½π × 7 2 = 76.9 ( 7 )


Total area = their68.19 ‒ their27.11 + their76.97 = 118.0–118.1

23 O/N 18/P12/Q8

(i) −1  7
A B̂ C using cosine rule giving cos-1( ) or 2sin−1(¾) or 2 cos −1 
8  2 
 
7 7
or B Â C = cos-1(¾) or B Â C = sin-1 or B Â C = tan −1
4 3
C B̂ Y = π – A B̂ C or 2×C Â B
OR
Find CY from ∆ ACY using Pythagoras or similar ∆s
 82 + 82 − ( their CY )2 
−1
C B̂ Y = cos  
 2×8×8 
 
c
C B̂ Y = 1.445 AG

(ii) Arc CY = 8 × 1.445


B Â C = ½(π – A B̂ C) or cos−1(¾)

Arc XC = 12 × (their B Â C)
Perimeter = 11.56 + 8.673 + 4 = 24.2 cm awrt www

24 O/N 18/P11/Q9

Angle OAB = π / 2 − π / 5 = 3π / 10 soi


1  3π  2
Sector CAB = ×  their ×5
2  10 
5
OA = = 8.507
π
sin
5
1 π
Sector COD = × ( their 3.507 ) ² ×
2 5
1 3π
∆OAB = × 5 × ( their 8.507 ) sin
2 10
= 17.20 or 17.21
Shaded area 17.20 ( or 17.21) − 11.78 − 3.86 = 1.56 or 1.57

121
25 O/N 18/P13/Q3 Unit 4: Answer Section

(i) 0.8 oe
(ii) BD = 5 sin their 0.8
DC = 5 ‒ 5cos their 0.8
Sector = ½ × 52 × their0.8
OR Seg = ½ × 52 × [their 0.8 ‒ sintheir 0.8]

Trap = ½(5 + theirDC) × theirBD oe


OR ∆BDC = ½theirBD × theirCD

Shaded area = 11.69 ‒ 10 OR 2.71(9) ‒ 1.03(3) = 1.69 cao

26 M/J 18/P12/Q6
(i) 9 9
In ∆ ABD, tanθ = → BD = or 9tan(90 – θ) or 9 cotθ
BD tanθ
9sin ( 90 − θ )
or ( 20 tanθ ) − 92  (Pythag) or
2
(Sine rule)
  sinθ
BD
In ∆ DBC, sinθ= → BD = 20sinθ
20
9
20sinθ =
tanθ
→ 20sin²θ = 9cosθ AG
(ii) Uses s² + c² = 1 → 20cos²θ + 9cosθ – 20 (= 0)

→ cosθ = 0.8

→ θ = 36.9º awrt

27 M/J 18/P11/Q6
AT r
(i) (tanθ = ) → AT = r tanθ or OT = SOI
r cosθ
1 1
→ A = r²tanθ − r²θ
2 2
(ii) AT
tanθ = → AT = 7.716
3
Arc length = rθ = 3.6
3
OT by Pythagoras or cos1.2 = ( = 8.279)
OT
Perimeter = AT + arc + OT – radius = 16.6
28 M/J 18/P13/Q5
6
Angle AOC = or 1.2
5
AB = 5 × tan(their 1.2) OR by e.g. Sine Rule
1
Area ∆OAB = × 5 × their 12.86
2
1 2
Area sector × 5 × their 1.2
2
Shaded region = 32.15 ‒ 15 = 17.2

122
Unit 4: Answer Section
29 O/N 17/P12/Q4
(i) Pythagoras → r = 72 OE
6 6
or cos 45 = → r = =6 2
r cos 45
3 2
Arc DC = 72 × ¼π = π , 2.12π, 6.66
2
(ii) Area of sector BDC is ½ × 72 × ¼π (= 9π or 28.274…)

Area Q = 9π – 18 (10.274…)

Area P is (¼π6² – area Q) = 18


18  18 
Ratio is   → 1.75
9π − 18  10.274 

30 O/N 17/P11/Q5
(i) cos A = 8 / 10 → A = 0.6435
(ii) EITHER:
Area ∆ABC = ½ × 16 × 6 or ½ × 10 × 16sin 0.6435 = 48
Area 1 sector ½ × 10 2 × 0.6435
Shaded area = 2 × their sector − their ∆ABC
OR:
∆BDE = 12, ∆BDC = 30
Sector = 32.18
2×segment + ∆BDE
=16.4
31 M/J 17/P12/Q4

rsin2θ
(i) (AB) = 2rsinθ (or r 2 − 2cos 2θ or )
π 
sin  −θ 
(Arc AB) = 2rθ 2 
rsin2θ
(P =) 2r + 2rθ + 2rsinθ (or r 2 − 2cos 2θ or )
π 
sin  −θ 
2 
25π
4(ii) Area sector AOB = ( ½ r² 2θ) or13.1
6
Area triangle AOB = (½×2rsinθ×rcosθ or ½ × r2 sin2θ)
25 3
or10.8
4
Area rectangle ABCD = (r × 2rsinθ) 25
(Area =) Either 25 – (25π/6 – 25√3/4) or 22.7

123
Unit 4: Answer Section
32 M/J 17/P11/Q8
(i) Letting M be midpoint of AB
OM = 8 (Pythagoras) → XM = 2
6
tan AXM = AXB = 2tan−13 = 2.498
2 6
(Alternative 1: sin AOM = , AOM = 0.6435, AXB = π − 0.6435 )
10
(ii) AX = √(6² + 2²) = √40
Arc AYB = rθ = √40 × 2.498
Perimeter = 12 + arc = 27.8 cm
(iii) area of sector AXBY = ½ × (√40)² × 2.498

Area of triangle AXB = ½ × 12 × 2, Subtract these → 38.0 cm²

33 M/J 17/P13/Q7
(i) sin ABC = 8 / 10 → ABC = 0.927 ( 3)
(ii) AB = 6 ( Pythagoras ) → ∆BCD = 8 × 6 = 48.0
Area sector BCD = ½ × 102 × ( 2 ) × their 0.9273

Area segment = 92.7(3) – 48


Area semi-circle ‒ segment = ½ × π × 82 − their ( 92.7 − 48 )
Shaded area = 55.8 – 56.0
34 O/N 16/P12/Q6
r r (ii) Major arc = 10(θ) (= 50.832)
(i) = sin 0.6 or = cos 0.97 θ = 2π – 1.2 (= 5.083)
10 10
or C = 2π × 10, Minor arc = 1.2 × 10
or BD = 200 − 200cos1.2 ( = 11.3)
Semicircle = 5.646π (= 17.737)
Major arc + semicircle
r = 10 × 0.5646, r = 10 × sin 0.6, = 68.6
r = 10 × cos 0.971 or r = ½ BD
→ r = 5.646
(iii) Area of major sector
= ½102 (θ ) (= 254.159)
Area of triangle OBD
= ½10²sin1.2 (= 46.602)
Area = semicircle + sector + triangle
(= 50.1 + 254.2 + 46.6)
= 351
35 O/N 16/P11/Q3
(ii) ½ ( 2r ) 0.8 − ½(r 2 )0.8 = 30
2
(i) 2rα + rα + 2r = 4.4r
α = 0.8 (3 / 2)r 2 × 0.8 = 30 → r = 5

36 O/N 16/P13/Q5

(ii) Use of (2π − 1.8 ) or equivalent method


(i) cos 0.9 = OE / 6 or = sin  π − 0.9  oe
Area of large sector = ½ × 62 × ( 2π − 1.8 ) oe
2 
OE = 6 cos 0.9 = 3.73 Area of small sector ½ × 3.732 × 1.8
Total area = 80.7(0) + 12.5(2) = 93.2

124
37 M/J 16/P12/Q6 Unit 4: Answer Section

(i) PT = r tanα
(ii) Area of triangle = ½ × 10 × 10 tan π
r 3
QT = OT – OQ = −r
cosα
or r ² + r ²tan²α − r Area of sector = ½ × 10² × ⅓π

Perimeter = sum of the 3 parts including rα Shaded region has area 34 (2sf)

38 M/J 16/P11/Q7

(i) CD = rcosθ, BD = r – rsinθ oe


1 1
1 (ii)Sector = .5².( π – 0.6) (12.135)
Arc CB = r ( π – θ) oe 2 2
2
1
1 Triangle = .5cos0.6.5sin0.6 (5.825)
→ P = rcosθ + r – rsinθ + r ( π – θ) oe 2
2
→ Area = 6.31
1
(or circle − triangle – sector)
4
39 M/J 16/P13/Q6

BAC = sin −1 (3 / 5) or cos −1 (4 / 5) or tan −1 (3 / 4)


ABC = sin −1 (4 / 5) or cos −1 (3 / 5) or tan −1 (4 / 3)

ACB = π / 2 (Allow 90º)


Shaded area = ∆ABC – sectors (AEF + BEG +
CFG)
1
∆ABC = × 4 × 3 oe
2
1
Sum sectors = 32 0.6435 +
2
)
2 2
2 0.9273 + 1 1.5708]
π
OR 3236.8 ( 7 ) + 2253.1( 3) + 1290 
360  
6 – 5.536 = 0.464

40 O/N 15/P12/Q5
6
(i) Length of OB = = 7.270 (ii) AB = 6tan0.6 or 4.1
cos 0.6 Arc length = 7.27 × (½π – 0.6) = (7.06)
(iii) Area of AOB = ½ × 6 × 7.27 × sin0.6 Perimeter = 6 + 7.27 + 7.06 + 6tan0.6 = 24.4
Area of OBC = ½ × 7.27² × (½π – 0.6)
→ area = 12.31 + 25.65 = 38.0

41 O/N 15/P11/Q7

(i) BC 2 = r 2 + r 2 = 2r 2 → BC = r 2 1 2 1 2 
Shaded area πr −  πr − r 2 
1 2 2 
(ii) Area sector BCFD = π (r 2 ) 2 soi
1 4 1 1 
πr 2 −  πr 2 +  πr 2 − r 2  
Area ∆ BCAD = (2r )r or
2 2 2 
1
Area segment CFDA = πr 2 − r 2 .oe = r2
2
1
Area semi-circle CADE = πr 2
2

125
Unit 4: Answer Section
42 O/N 15/P13/Q4
1 (ii) Arc CD = 2rθ
(i) Sector OCD = ( 2r ) 2 θ ( = 2r 2θ )
2 Arc(s) AB/EF (2)r (π − θ )
1 Straight edges = 4r
Sector(s) OAB/OEF = ( 2) r 2 (π − θ )
2 Total 2πr + 4r (which is independent of θ )
Total = r 2 (π + θ )

43 M/J 15/P12/Q2
1
Radius of semicircle = 2 AB = rsinθ
1
Area of semicircle = πr²sin²θ = A1
2
Shaded area = semicircle – segment
= A1 − 12 r²2θ + 12 r²sin2θ

44 M/J 15/P11/Q5

(i) 24 = r + r + rθ
24 − 2r (ii) ( A =)36 − (r − 6) 2
→ θ=
r
1 24r (iii) Greatest value of A = 36
A= r²θ = − r 2 = 12r – r². aef, ag
2 2
(r = 6) → θ = 2
45 M/J 15/P13/Q11
(i) OC = r cos α or AC = r sin α or oe soi
(Area ∆OAC =) 12 r 2 sin α cos α
1
2
r 2 sin α cos α = 12 × 12 r 2α oe
sin α cos α = 12 α
(ii) Perimeter ∆OAC = r + r sin α + r cos α = 2.4(0)r
Perim.
ACB = rα + r sin α + r − r cos α = 2.18r or 2.17r
2.4( 0 )
Ratio = : 1 = 1.1 : 1
2.18 or 2.17
(iii) 54.3º cao
46 O/N 14/P12/Q2
5
(i) tanθ =
12
→ ( θ = 0.3948 )

(ii) Other angle in triangle = − π – 0.3948


Area of triangle AOB = ×12×5 (= 30)


Use of r²θ once

Shaded area = sector + sector – triangle
ଵ ଵ
= ×12²×0.3948 + 5²θ – 30
ଶ ଶ

= 28.43 + 14.70 – 30 = 13.1

126
47 O/N 14/P11/Q8 Unit 4: Answer Section

(i) Arc AB = 4α
Arc DC = ( 4 cos α )α
AC (or DB ) = 4 − 4 cos α
Perimeter = 4α cos α + 4α + 8 − 8 cos α

OD = 4 cos π  = 2 3 
 
(ii)
6 

( )
2
Shaded area =  1
× 42 × π6   − 12 2 3 × π6 
 
2

π
3
48 O/N 14/P13/Q2
1
(ii) Area OABC (2) × × 3 × their AB
3 π 2
(i) CB or AB = or 3 tan
π 3 9 3
tan (=9√3 or )
6 2
 2π π 1  2π π   3π 
Arc or AC = 3 ×  or  (= 2π or π ) Area OADC × 3 2 ×  or   = 3π or 
 3 3 2  2 3  2 
Shaded area 9 3 − 3π oe
Perimeter = 6 3 + 2π oe

49 M/J 14/P12/Q4
(i)
1 2
θ = 1 2 r 2θ − 1 2 r 2 sin θ
2r
(ii) Chord length = 8sin1.2 × 2 (14.9)
→ 2sin θ = θ → p = 2. (or from cosine rule)
Arc length = 2.4 × 8 (19.2)
Perimeter = sum of these = 34.1
50 M/J 14/P11/Q6

1
(i) area ∆ = × 4 × 4tanα oe soi 4
2 (ii) DC = − 2 oe soi
cos α
1 Arc DE = 2α soi anywhere provided clear
Area sector = × 2 2 α oe soi
2 4
Perimeter = + 4 tan α + 2α cao
Shaded area = 8tanα − 2α cao cos α

51 M/J 14/P13/Q3
(i) s =rθ
Angle of major arc = 2π – 2.2 = (4.083)
Perimeter = 12 + 24.5 = 36.5 or 12π − 1.2
(or full circle − minor arc B1)
1
(ii) Area of major sector = r ² θ = (73.49)
2
1
Area of triangle = . 6 ² sin 2.2 = (14.55)
2
Ratio = 5.05 : 1 (Allow 5.03 → 5.06)

52 O/N 13/P12/Q2
(i) slant length = 10 cm.
circumference of base = 12π
arc length = 10θ ( = 12π)
→ θ = 1.2π or 3.77 radians.

(ii) ½r²θ = 188.5 cm ² or 60π.

127
53 O/N 13/P11/Q6 Unit 4: Answer Section
(iii) πr 2 − 1 r 2α = 2r 2α
(i) r (2π − α ) + 2 rα + 2 r 2
2πr + rα + 2r 2
α= π
5
(ii) 1
2
(2r )2 α + πr 2 − 1 2 r 2α
3r 2α
+ πr 2
2

54 O/N 13/P13/Q6

1 2 1 2
6 (i) sector areas are 11 α , 5 α (ii) perimeter shaded region= 11α + 5α + 6 +
2 2 6 = 16α + 12
1 1 perimeter unshaded region = 5α + 5 + 5 =
× 112 α − × 52 α
k= 2 2 5α + 10
1 2 16α + 12 = 2 (5α + 10)
×5 α
2 α = 4/3 or 1.33
96
k= or 3.84
25

55 M/J 13/P12/Q4

4 (i) BOC = 2tan−1½ = 0.9273

(ii) OB = √(10² + 5²) or 11.2 = r


Arc BXC = √125 × 0.9273
→ Perimeter = 20.4 cm
(iii) Area = ½r²θ
− ½.10.10 → 7.96 cm².

56 M/J 13/P11/Q3

(i) (OAB ) = 1 × 82 α , (OAC ) = 1 × π × 4 2


2 2
π
α=
8
1
(ii) 8 + 8 × their α + × 8 × π
2
8 + 5π
57 M/J 13/P13/Q2
(i) ½.3²π = ½9²θ - ½3²θ
→ θ = 14 π
(ii) P = 6+6 +3× 14 π + 9×¼π = 21.4 cm.
or 12 + 3π
58 O/N 12/P12/Q11
(i) OQ = x + OC = 20
x x
sin 0.6 = → OC =
OC sin 0.6
x
x+ = 20 → x = 7.218
sin 0.6
(ii) Area = ½. 20² × 1.2 – π × 7.218²
= 76.3
(iii) Angle PCR = π − 1.2
Arc PR = 7.218 × (π − 1.2) = (14.01)
x
OP = OR =
tan 0.6
→ Perimeter of 35.1 cm

128
59 O/N 12/P11/Q6 Unit 4: Answer Section
(i)     cos !

(ii) arc "

 
arc #  $%&'   4 
 

4 cos  


"# 4 sin  $%&'  2√3  2


Perimeter =  2√3  2


60 O/N 12/P13/Q4
area ∆ = 2√3
2√3 π
tan A= ⇒A=
2 3
1 π 2π
Area sector = × 22 × =
2 3 3

Shaded area =2√3 –
3

61 M/J 12/P12/Q6
(i) cosine rule or 2×r × sin ½.(2.4) (iii) Area = Sector + triangle
→ 14.9 cm ½×8² (2π − 2.4) + ½×8²sin 2.4
(ii) Perimeter = (i) + rθ 124.3 + 21.6 → 146 cm².
θ = 2π − 2.4,
→ 46.0 cm

62 M/J 12/P11/Q3

AQ (or r) = 3
3
Area ∆ = 3 (or area ∆ AQC = )
2
Area sector APR =
1
2
( 3 ) × π3 = π2
2

π
Shaded region = 3− oe cao
2
63 M/J 12/P13/Q8
r
(i) OBX = 90º, cos θ =
→ θ = ⅓ π. 2r
(ii) Arc length AB = ⅓ rπ
BX = rtan(⅓π) = r 3
P = r + (⅓ rπ + r 3 )
(iii) Area = 1
2
r 2 3 − 16 r 2π

64 O/N 11/P12/Q6
(i) D to AX = 6 sin π3 = 6√3÷2
E to AX = 10sinθ
3 3
Equate these → θ = sin −1 .
10
(ii) Arc DX = 6.⅓π = 2π
Arc EX = 10×0.5464 =5.464
Horizontal steps = 6cos⅓π and 10cosθ
DE = 10 + 6 − 6cos⅓π − 10cosθ
Perimeter = arc DX + arc BX + DE
→ 16.20

129
65 O/N 11/P11/Q5 Unit 4: Answer Section
(i) Arc AB = rθ

OC = r sin θ or BC = r cos θ

r (1 + θ + cos θ + sin θ )
correctly derived
1 π
(ii) Sector OAB = × 10 2 × (= 31.42 )
2 5
1
∆OCB =
 π  π
210 cos 10 sin 
 5  5
(= 23.78)
Total area = 55.2

66 O/N 11/P11/Q5
(i) 102 sin 0.8 = 71.7

(ii) sector(s) = (2) × 1


2 × 102 × 0.8 = (2) × 40
Total area = 80

(iii) arc(s) = (2) × 10 × 0.8


16+20 = 36

67 M/J 11/P11/Q9

(i) AS = r tan θ
2
Area OAB = r 2 tan θ or (OAS ) = 1
2r tan θ
2 2
Area of sector = 1
2
r × 2θ (= r θ )
2
Shaded area = r (tan θ − θ ) OE
π 6
(ii) cos = ⇒ OA = 12
3 OA
AP = 6
π
AS = 6 tan (⇒ AB = 12 3 )
3
π
Arc (PST) = 12
3
Perimeter = 12 + 12 3 + 4π

68 M/J 11/P13/Q7
π
(i) AX = 6tan = 6√3 π
3 (iii) Arc AB = 6 × = 2π
3
(ii) Area of triangle = ½ × 6 × 6√3
π π
Area of sector = ½ 62 × OX = 6 ÷ cos = 12, BX = 6
3 3
Area shaded = 18√3 – 6π Perimeter = 6√3 + 2π + 6

130
UNIT 5 Trigonometry

1 M/J 22/P12/Q11
The function f is given by f x = 4 cos4 x + cos2 x − k for 0 ≤ x ≤ 2π, where k is a constant.
(a) Given that k = 3, find the exact solutions of the equation f x = 0. [5]
(b) Use the quadratic formula to show that, when k > 5, the equation f x = 0 has no solutions. [5]

2 M/J 22/P11/Q4
sin3 1 sin2 1
(a) Prove the identity −  − tan2 1 1 + sin2 1. [4]
sin 1 − 1 1 + sin 1
(b) Hence solve the equation
sin3 1 sin2 1
− = tan2 1 1 − sin2 1
for 0 < 1 < 2π. sin 1 − 1 1 + sin 1
3 M/J 22/P11/Q8/b [2]
1  
(a) Find the exact solutions of the equation 4 sin 2 x − 30Å = 2 2 for 0Å ≤ x ≤ 360Å. [3]
4 M/J 22/P13/Q2
y

0 1
0 π 2π 3π 4π
−1

−2

−3
y = p sin q1 + r
−4

−5

The diagram shows part of the curve with equation y = p sin q1 + r, where p, q and r are constants.

(a) State the value of p. [1]


(b) State the value of q. [1]
(c) State the value of r. [1]

5 M/J 22/P13/Q5
 2
(a) Solve the equation 6 y +  − 7 = 0. [4]
y
 2
(b) Hence solve the equation 6 tan x +  − 7 = 0 for 0Å ≤ x ≤
. 360Å [3]
tan x
6 O/N 21/P12/Q1
3
Solve the equation 2 cos 1 = 7 − for −90Å < 1 < 90Å. [4]
cos 1
7 O/N 21/P11/Q3
Solve, by factorising, the equation
6 cos 1 tan 1 − 3 cos 1 + 4 tan 1 − 2 = 0,
for 0Å ≤ 1 ≤ 180Å. [4]

131
8 O/N 21/P11/Q5 Unit 5: Trigonometry
y

12

10

8 y = a cos bx + c

x
0 π 2π
−2

−4
The diagram shows part of the graph of y = a cos bx + c.
(a) Find the values of the positive integers a, b and c. [3]
(b) For these values of a, b and c, use the given diagram to determine the number of solutions in the
interval 0 ≤ x ≤ 2π for each of the following equations.
(i) a cos bx + c = x
6
π 6
[1]
(ii) a cos bx + c = 6 − x
π
[1]

9 O/N 21/P13/Q7
tan x + cos x
(a) Show that the equation = k, where k is a constant, can be expressed as
tan x − cos x
k + 1 sin2 x + k − 1 sin x − k + 1 = 0. [4]

tan x + cos x [4]


(b) Hence solve the equation = 4 for 0Å ≤ x ≤ 360Å
tan x − cos x

10 M/J 21/P12/Q10
1 + sin x 1 − sin x 4 tan x
(a) Prove the identity −  . [4]
1 − sin x 1 + sin x cos x
1 + sin x 1 − sin x [3]
(b) Hence solve the equation − = 8 tan x for 0 ≤ x ≤ 12 π.
1 − sin x 1 + sin x
11 M/J 21/P11/Q7
1 − 2 sin2 1
(a) Prove the identity 2
 1 − tan2 1. [2]
1 − sin 1
1 − 2 sin 1 [3]
(b) Hence solve the equation = 2 tan4 1 for 0Å ≤ 1 ≤ 180Å.
1 − sin2 1
12 M/J 21/P13/Q4
(a) Show that the equation
tan x + sin x
= k,
tan x − sin x
where k is a constant, may be expressed as
1 + cos x
= k. [2]
1 − cos x
(b) Hence express cos x in terms of k. [2]
tan x + sin x
(c) Hence solve the equation = 4 for −π < x < π [2]
tan x − sin x

132
13 O/N 20/P12/Q6 Unit 5: Trigonometry

@ A@ A
1 1 1
(a) Prove the identity − tan x +1  . [4]
cos x @ sin xA @ tan Ax
1 1
(b) Hence solve the equation − tan x + 1 = 2 tan2 x for 0Å ≤ x ≤ 180Å. [2]
cos x sin x

14 O/N 20/P12/Q11/a,b,c
A curve has equation y = 3 cos 2x + 2 for 0 ≤ x ≤ π.

(a) State the greatest and least values of y. [2]


(b) Sketch the graph of y = 3 cos 2x + 2 for 0 ≤ x ≤ π. [2]
(c) By considering the straight line y = kx, where k is a constant, state the number of solutions of the
equation 3 cos 2x + 2 = kx for 0 ≤ x ≤ π in each of the following cases.
(i) k = −3 [1]
(ii) k = 1 [1]
(iii) k = 3 [1]

15 O/N 20/P11/Q7
sin 1 sin 1
(a) Show that −  2 tan2 1. [3]
1 − sin 1 1 + sin 1
sin 1 sin 1
(b) Hence solve the equation − = 8, for 0Å < 1 < 180Å. [3]
1 − sin 1 1 + sin 1

16 O/N 20/P13/Q3
Solve the equation 3 tan2 1 + 1 = for 0Å < 1 < 180Å
2 [5]
tan2 1
17 M/J 20/P12/Q2
(a) Express the equation 3 cos 1 = 8 tan 1 as a quadratic equation in sin 1 [3]
(b) Hence find the acute angle, in degrees, for which 3 cos 1 = 8 tan 1 [2]

18 M/J 20/P11/Q7
1 + sin 1 cos 1 2
(a) Prove the identity +  . [3]
cos 1 1 + sin 1 cos 1
1 + sin 1 cos 1 3 [3]
(b) Hence solve the equation + = , for 0 ≤ 1 ≤ 2π.
cos 1 1 + sin 1 sin 1

19 M/J 20/P13/Q7
tan 1 tan 1 2
(a) Show that +  . [4]
1 + cos 1 1 − cos 1 sin 1 cos 1
tan 1 tan 1 6
(b) Hence solve the equation + = for 0Å < 1 < 180Å. [4]
1 + cos 1 1 − cos 1 tan 1

20 MAR/ 20/P12/Q5
Solve the equation tan 1 + 3 sin 1 + 2
=2
tan 1 − 3 sin 1 + 1
for 0Å ≤ 1 ≤ 90Å. [5]
21 MAR/ 20/P12/Q11
2
(a) Solve the equation 3 tan x − 5 tan x − 2 = 0 for 0Å ≤ x ≤ 180Å. [4]
(b) Find the set of values of k for which the equation 3 tan2 x − 5 tan x + k = 0 has no solutions. [2]

133
Unit 5: Trigonometry
22 O/N 19/P12/Q6
(a) Given that x > 0, find the two smallest values of x, in radians, for which 3 tan 2x + 1 = 1. Show
all necessary working. [4]
(b) The function f : x → 3 cos2 x − 2 sin2 x is defined for 0 ≤ x ≤ 0.
(i) Express f x in the form a cos2 x + b, where a and b are constants. [1]
(ii) Find the range of f. [2]

23 O/N 19/P11/Q5
1
(i) Given that 4 tan x + 3 cos x + = 0, show, without using a calculator, that sin x = − 23 . [3]
cos x
(ii) Hence, showing all necessary working, solve the equation
1
4 tan 2x − 20Å + 3 cos 2x − 20Å + =0
cos 2x − 20Å
for 0Å ≤ x ≤ 180Å. [4]
24 O/N 19/P13/Q7
(i) Show that the equation 3 cos4 1 + 4 sin2 1 − 3 = 0 can be expressed as 3x2 − 4x + 1 = 0, where
x = cos2 1. [2]

(ii) Hence solve the equation 3 cos4 1 + 4 sin2 1 − 3 = 0 for 0Å ≤ 1 ≤ 180Å. [5]

25 M/J 19/P12/Q4
Angle x is such that sin x = a + b and cos x = a − b, where a and b are constants.
(i) Show that a2 + b2 has a constant value for all values of x. [3]

(ii) In the case where tan x = 2, express a in terms of b. [2]

26 M/J 19/P12/Q6
3x
The equation of a curve is y = 3 cos 2x and the equation of a line is 2y + = 5.
0
(i) State the smallest and largest values of y for both the curve and the line for 0 ≤ x ≤ 20. [3]
3x
(ii) Sketch, on the same diagram, the graphs of y = 3 cos 2x and 2y + = 5 for 0 ≤ x ≤ 20 [3]
0
3x
(iii) State the number of solutions of the equation 6 cos 2x = 5 − for 0 ≤ x ≤ 20
0
[1]

27 M/J 19/P11/Q6
@ A2
1 1 − sin x
(i) Prove the identity − tan x  . [4]
cos x 1 + sin x
@ A2
1
(ii) Hence solve the equation − tan 2x = 13 for 0 ≤ x ≤ 0. [3]
cos 2x
28 M/J 19/P11/Q9
The function f is defined by f x = 2 − 3 cos x for 0 ≤ x ≤ 20.

(i) State the range of f. [2]


(ii) Sketch the graph of y = f x. [2]
The function g is defined by g x = 2 − 3 cos x for 0 ≤ x ≤ p, where p is a constant.

(iii) State the largest value of p for which g has an inverse. [1]
−1
(iv) For this value of p, find an expression for g x. [2]

134
29 M/J 19/P13/Q9 y Unit 5: Trigonometry

y = f x

x
O 1 0
20

The function f : x → p sin2 2x + q is defined for 0 ≤ x ≤ 0, where p and q are positive constants. The
diagram shows the graph of y = f x.

(i) In terms of p and q, state the range of f. [2]


(ii) State the number of solutions of the following equations.
(a) f x = p + q [1]
(b) f x = q [1]
(c) f x = 1p + q [1]
2

30 MAR 19/P12/Q7
(a) Solve the equation 3 sin2 21 + 8 cos 21 = 0 for 0Å ≤ 1 ≤ 180Å. [5]
(b) y

y = a + tan bx
ï3

x
− 16 0 O

The diagram shows part of the graph of y = a + tan bx, where x is measured in radians and a and
 
b are constants. The curve intersects the x-axis at − 16 0 , 0 and the y-axis at 0, ï3. Find the
values of a and b. [3]

31 O/N 18/P12/Q6
B

20 cm


A C
9 cm D

The diagram shows a triangle ABC in which BC = 20 cm and angle ABC = 90Å. The perpendicular
from B to AC meets AC at D and AD = 9 cm. Angle BCA = 1Å.

(i) By expressing the length of BD in terms of 1 in each of the triangles ABD and DBC, show that
20 sin2 1 = 9 cos 1. [4]
(ii) Hence, showing all necessary working, calculate 1. [3]

135
Unit 5: Trigonometry
32 O/N 18/P11/Q5
cos 1 − 4 4 sin 1
(i) Show that the equation − =0
sin 1 5 cos 1 − 2
may be expressed as 9 cos2 1 − 22 cos 1 + 4 = 0. [3]

(ii) Hence solve the equation


cos 1 − 4 4 sin 1
− =0 [3]
sin 1 5 cos 1 − 2
for 0Å ≤ 1 ≤ 360Å.

33 O/N 18/P13/Q7
tan 1 + 1 tan 1 − 1 2 tan 1 − cos 1
(i) Show that +  . [3]
1 + cos 1 1 − cos 1 sin2 1
(ii) Hence, showing all necessary working, solve the equation
tan 1 + 1 tan 1 − 1
+ =0
1 + cos 1 1 − cos 1 [4]
for 0Å < 1 < 90Å.

34 M/J 18/P12/Q4
 
The function f is such that f x = a + b cos x for 0 ≤ x ≤ 20. It is given that f 13 0 = 5 and f 0  = 11.

(i) Find the values of the constants a and b. [3]


(ii) Find the set of values of k for which the equation f x = k has no solution. [3]

35 M/J 18/P12/Q10

(i) Solve the equation 2 cos x + 3 sin x = 0, for 0Å ≤ x ≤ 360Å. [3]


(ii) Sketch, on the same diagram, the graphs of y = 2 cos x and y = −3 sin x for 0Å ≤ x ≤ 360Å. [3]
(iii) Use your answers to parts (i) and (ii) to find the set of values of x for 0Å ≤ x ≤ 360Å for which
2 cos x + 3 sin x > 0. [2]

36 M/J 18/P11/Q4
(i) Prove the identity sin 1 + cos 1 1 − sin 1 cos 1  sin3 1 + cos3 1. [3]
(ii) Hence solve the equation sin 1 + cos 1 1 − sin 1 cos 1 = 3 cos3 1 for 0Å ≤ 1 ≤ 360Å. [3]

37 M/J 18/P13/Q7
tan2 1 − 1
(a) (i) Express in the form a sin2 1 + b, where a and b are constants to be found. [3]
tan2 1 + 1
(ii) Hence, or otherwise, and showing all necessary working, solve the equation
tan2 1 − 1 1
=
tan2 1 + 1 4
for −90Å ≤ 1 ≤ 0Å. [2]
y

A y = sin x
O x
−0 0

B
y = 2 cos x

136
Unit 5: Trigonometry

The diagram shows the graphs of y = sin x and y = 2 cos x for −0 ≤ x ≤ 0 . The graphs intersect
at the points A and B.
(i) Find the x-coordinate of A. [2]
(ii) Find the y-coordinate of B. [2]

38 O/N 17/P12/Q5
(i) Show that the equation cos 2x tan2 2x + 3 + 3 = 0 can be expressed as
2 cos2 2x + 3 cos 2x + 1 = 0. [3]
(ii) Hence solve the equation cos 2x tan2 2x + 3 + 3 = 0 for 0Å ≤ x ≤ 180Å. [4]

39 O/N 17/P12/Q6
1  1 
(a) The function f, defined by f : x → a + b sin x for x ∈ >, is such that f 6 0 = 4 and f 2 0 = 3.
(i) Find the values of the constants a and b. [3]

(b) The function g is defined by g : x → c + d sin x for x ∈ >. The range of g is given by −4 ≤ g x ≤ 10.
Find the values of the constants c and d . [3]

40 O/N 17/P11/Q7
y
(a)
1
x
O
y = a + b sin x
−2

The diagram shows part of the graph of y = a + b sin x. Find the values of the constants a and b.
(b) (i) Show that the equation [2]
sin 1 + 2 cos 1 1 + sin 1 − cos 1 = sin 1 1 + cos 1
may be expressed as 3 cos2 1 − 2 cos 1 − 1 = 0. [3]
(ii) Hence solve the equation
sin 1 + 2 cos 1 1 + sin 1 − cos 1 = sin 1 1 + cos 1
for −180Å ≤ 1 ≤ 180Å. [4]

41 O/N 17/P13/Q5
cos 1 + 4
(i) Show that the equation + 5 sin 1 − 5 = 0 may be expressed as 5 cos2 1 − cos 1 − 4 = 0.
sin 1 + 1
[3]
cos 1 + 4
(ii) Hence solve the equation + 5 sin 1 − 5 = 0 for 0Å ≤ 1 ≤ 360Å. [4]
sin 1 + 1

42 M/J 17/P12/Q3 @ A2
1 1 − sin 1
(i) Prove the identity − tan 1  . [3]
cos 1 1 + sin 1
@ A2
1
(ii) Hence solve the equation − tan 1 = 12 , for 0Å
0Å ≤ 1 ≤ 360
360ÅÅ. [3]
cos 1

137
Unit 5: Trigonometry
43 M/J 17/P12/Q10
 
The function f is defined by f x = 3 tan 12 x − 2, for − 12 0 ≤ x ≤ 12 0.

(i) Solve the equation f x + 4 = 0, giving your answer correct to 1 decimal place. [3]
(ii) Find an expression for f −1 x and find the domain of f −1 . [5]
(iii) Sketch, on the same diagram, the graphs of y = f x and y = f −1 x. [3]

44 M/J 17/P11/Q3
1 + cos 1 sin 1 2
(i) Prove the identity +  . [3]
sin 1 1 + cos 1 sin 1
1 + cos 1 sin 1 3
(ii) Hence solve the equation + = for 0Å ≤ 1 ≤ 360Å. [3]
sin 1 1 + cos 1 cos 1

45 M/J 17/P11/Q5
.
The equation of a curve is y = 2 cos x
(i) Sketch the graph of y = 2 cos x for −0 ≤ x ≤ 0, stating the coordinates of the point of intersection
with the y-axis. [2]
1
Points P and Q lie on the curve and have x-coordinates of 3 0 and 0 respectively.

(ii) Find the length of PQ correct to 1 decimal place. [2]


The line through P and Q meets the x-axis at H h, 0 and the y-axis at K 0, k.

(iii) Show that h = 59 0 and find the value of k. [3]

46 M/J 17/P13/Q5
2 sin 1 + cos 1
(i) Show that the equation = 2 tan 1 may be expressed as cos2 1 = 2 sin2 1. [3]
sin 1 + cos 1
2 sin 1 + cos 1
(ii) Hence solve the equation = 2 tan 1 for 0Å < 1 < 180Å. [3]
sin 1 + cos 1

47 O/N 16/P12/Q2
(i) Express the equation sin 2x + 3 cos 2x = 3 sin 2x − cos 2x in the form tan 2x = k, where k is a
constant. [2]

(ii) Hence solve the equation for −90Å ≤ x ≤ 90Å. [3]

48 O/N 16/P11/Q6
(i) Show that cos4 x  1 − 2 sin2 x + sin4 x. [1]

(ii) Hence, or otherwise, solve the equation 8 sin4 x + cos4 x = 2 cos2 x for 0Å ≤ x ≤ 360Å. [5]

49 O/N 16/P13/Q3
Showing all necessary working, solve the equation 6 sin2 x − 5 cos2 x = 2 sin2 x + cos2 x for
0Å ≤ x ≤ 360Å. [4]

138
Unit 5: Trigonometry
A
50 M/J 16/P12/Q5

1
30
B x x C
M
In the diagram, triangle ABC is right-angled at C and M is the mid-point of BC. It is given that
angle ABC = 13 0 radians and angle BAM = 1 radians. Denoting the lengths of BM and MC by x,

(i) find AM in terms of x, [3]


@ A
(ii) show that 1 = 16 0 − tan−1
1
. [2]
2ï3

51 M/J 16/P12/Q7
1 + cos 1 1 − cos 1 4
(i) Prove the identity −  . [4]
1 − cos 1 1 + cos 1 sin 1 tan 1

(ii) Hence solve, for 0Å < 1 < 360Å, the equation


[3]
1 + cos 1 1 − cos 1
@ A
sin 1 − = 3.
1 − cos 1 1 + cos 1
52 M/J 16/P11/Q2
Solve the equation 3 sin2 1 = 4 cos 1 − 1 for 0Å ≤ 1 ≤ 360Å. [4]

53 M/J 16/P13/Q8
(i) Show that 3 sin x tan x − cos x + 1 = 0 can be written as a quadratic equation in cos x and hence
solve the equation 3 sin x tan x − cos x + 1 = 0 for 0 ≤ x ≤ 0. [5]

(ii) Find the solutions to the equation 3 sin 2x tan 2x − cos 2x + 1 = 0 for 0 ≤ x ≤ 0. [3]
54 O/N 15/P12/Q4
@ A
1 1 2 1 − cos x
(i) Prove the identity −  . [4]
sin x tan x 1 + cos x
@ A
1 1 2 2
(ii) Hence solve the equation − = 5 for 0 ≤ x ≤ 20. [3]
sin x tan x

55 O/N 15/P11/Q3
Solve the equation sin−1 4x4 + x2  = 16 0. [4]

56 O/N 15/P11/Q4
4 cos 1
(i) Show that the equation + 15 = 0 can be expressed as
tan 1
4 sin2 1 − 15 sin 1 − 4 = 0. 3

4 cos 1
(ii) Hence solve the equation + 15 = 0 for 0Å ≤ 1 ≤ 360Å. [3]
tan 1

139
Unit 5: Trigonometry

57 O/N 15/P13/Q7
1
(a) Show that the equation + 3 sin 1 tan 1 + 4 = 0 can be expressed as
cos 1
3 cos2 1 − 4 cos 1 − 4 = 0,
1
and hence solve the equation + 3 sin 1 tan 1 + 4 = 0 for 0Å ≤ 1 ≤ 360Å. [6]
cos 1
y
(b)
D

C x
O

y = a cos x − b

The diagram shows part of the graph of y = a cos x − b, where a and b are constants. The graph
crosses the x-axis at the point C cos−1 c, 0 and the y-axis at the point D 0, d. Find c and d in
terms of a and b. [2]

58 M/J 15/P12/Q5
sin 1 − cos 1 tan 1 − 1
(i) Prove the identity  . [1]
sin 1 + cos 1 tan 1 + 1

sin 1 − cos 1 tan 1


(ii) Hence solve the equation = , for 0Å ≤ 1 ≤ 180Å. [4]
sin 1 + cos 1 6
59 M/J 15/P12/Q6
A tourist attraction in a city centre is a big vertical wheel on which passengers can ride. The wheel
turns in such a way that the height, h m, of a passenger above the ground is given by the formula
h = 60 1 − cos kt. In this formula, k is a constant, t is the time in minutes that has elapsed since the
passenger started the ride at ground level and kt is measured in radians.

(i) Find the greatest height of the passenger above the ground. [1]

One complete revolution of the wheel takes 30 minutes.

(ii) Show that k = 1


15 0 . [2]

(iii) Find the time for which the passenger is above a height of 90 m. [3]

60 M/J 15/P11/Q1
Given that 1 is an obtuse angle measured in radians and that sin 1 = k, find, in terms of k, an expression
for
(i) cos 1, [1]
(ii) tan 1, [2]
(iii) sin 1 + 0. [1]

140
Unit 5: Trigonometry

61 M/J 15/P11/Q8
 
The function f : x → 5 + 3 cos 12 x is defined for 0 ≤ x ≤ 20.
(i) Solve the equation f x = 7, giving your answer correct to 2 decimal places. [3]
(ii) Sketch the graph of y = f x. [2]
(iii) Explain why f has an inverse. [1]
(iv) Obtain an expression for f −1 x. [3]

62 M/J 15/P13/Q4
(i) Express the equation 3 sin 1 = cos 1 in the form tan 1 = k and solve the equation for 0Å < 1 < 180Å.
[2]
(ii) Solve the equation 3 sin2 2x = cos2 2x for 0Å < x < 180Å . [4]

63 O/N 14/P12/Q5

(i) Show that the equation 1 + sin x tan x = 5 cos x can be expressed as

6 cos2 x − cos x − 1 = 0. 3


(ii) Hence solve the equation 1 + sin x tan x = 5 cos x for 0 ≤ x ≤ 180 . [3]

64 O/N 14/P12/Q11
 
The function f : x → 6 − 4 cos 12 x is defined for 0 ≤ x ≤ 2.

(i) Find the exact value of x for which fx = 4. [3]

(ii) State the range of f. [2]

(iii) Sketch the graph of y = fx. [2]

(iv) Find an expression for f −1 x. [3]

65 O/N 14/P11/Q2
Find the value of x satisfying the equation sin−1x − 1 = tan−13. [3]

66 O/N 14/P11/Q3
13 sin2 
Solve the equation + cos  = 2 for 0 ≤  ≤ 180 . [4]
2 + cos 

67 O/N 14/P13/Q5
(i) Show that sin4  − cos4   2 sin2  − 1. [3]

(ii) Hence solve the equation sin4  − cos4  = 1


2
for 0 ≤  ≤ 360. [4]

68 M/J 14/P12/Q3
The reflex angle 1 is such that cos 1 = k, where 0 < k < 1.
(i) Find an expression, in terms of k, for
(a) sin 1, [2]
(b) tan 1. [1]
(ii) Explain why sin 21 is negative for 0 < k < 1. [2]

141
Unit 5: Trigonometry

69 M/J 14/P12/Q5
1 cos 1
(i) Prove the identity −  tan 1. [4]
cos 1 1 + sin 1
1 cos 1
(ii) Solve the equation − + 2 = 0 for 0Å ≤ 1 ≤ 360Å. [3]
cos 1 1 + sin 1

70 M/J 14/P11/Q1
y

3
y = a + b sin x
2

x
− 32 0 −0 − 12 0 0 1 3
20 0 20

−1

The diagram shows part of the graph of y = a + b sin x. State the values of the constants a and b. [2]

71 M/J 14/P11/Q9
sin 1 1 1
(i) Prove the identity −  . [4]
1 − cos 1 sin 1 tan 1

sin 1 1
(ii) Hence solve the equation − = 4 tan 1 for 0Å < 1 < 180Å. [3]
1 − cos 1 sin 1

72 M/J 14/P13/Q4

tan x + 1
(i) Prove the identity  sin x + cos x. [3]
sin x tan x + cos x

tan x + 1
(ii) Hence solve the equation = 3 sin x − 2 cos x for 0 ≤ x ≤ 20. [3]
sin x tan x + cos x
73 O/N 13/P12/Q1
Given that cos x = p, where x is an acute angle in degrees, find, in terms of p, [1]
(i) sin x, [1]
(ii) tan x, [1]
(iii) tan 90Å − x.

142
Unit 5: Trigonometry

74 O/N 13/P12/Q8
A function f is defined by f : x → 3 cos x − 2 for 0 ≤ x ≤ 20.

(i) Solve the equation f x = 0. [3]

(ii) Find the range of f. [2]

(iii) Sketch the graph of y = f x. [2]

A function g is defined by g : x → 3 cos x − 2 for 0 ≤ x ≤ k.

(iv) State the maximum value of k for which g has an inverse. [1]

(v) Obtain an expression for g−1 x. [2]

75 O/N 13/P11/Q4
(i) Solve the equation 4 sin2 x + 8 cos x − 7 = 0 for 0Å ≤ x ≤ 360Å. [4]
1   
(ii) Hence find the solution of the equation 4 sin2 2
1 + 8 cos 12 1 − 7 = 0 for 0Å ≤ 1 ≤ 360Å. [2]

76 O/N 13/P13/Q7

(a) Find the possible values of x for which sin−1 x2 − 1 = 13 0, giving your answers correct to
3 decimal places. [3]
(b) Solve the equation sin 21 + 13 0 = 1
2
for 0 ≤ 1 ≤ 0, giving 1 in terms of 0 in your answers. [4]

77 M/J 13/P12/Q5
It is given that a = sin 1 − 3 cos 1 and b = 3 sin 1 + cos 1, where 0Å ≤ 1 ≤ 360Å.

(i) Show that a2 + b2 has a constant value for all values of 1. [3]

(ii) Find the values of 1 for which 2a = b. [4]

78 M/J 13/P11/Q5
sin 1 cos 1 1
(i) Show that +  2
. [3]
sin 1 + cos 1 sin 1 − cos 1 sin 1 − cos2 1

sin 1 cos 1
(ii) Hence solve the equation + = 3, for 0Å ≤ 1 ≤ 360Å. [4]
sin 1 + cos 1 sin 1 − cos 1

79 M/J 13/P13/Q5
(i) Sketch, on the same diagram, the curves y = sin 2x and y = cos x − 1 for 0 ≤ x ≤ 20. [4]

(ii) Hence state the number of solutions, in the interval 0 ≤ x ≤ 20, of the equations
(a) 2 sin 2x + 1 = 0, [1]
(b) sin 2x − cos x + 1 = 0. [1]

143
Unit 5: Trigonometry
80 O/N 12/P12/Q6
(i) Show that the equation 2 cos x = 3 tan x can be written as a quadratic equation in sin x. [3]

(ii) Solve the equation 2 cos 2y = 3 tan 2y, for 0◦ ≤ y ≤ 180◦ . [4]

81 O/N 12/P11/Q7
◦ ◦
(i) Solve the equation 2 cos2 θ = 3 sin θ , for 0 ≤ θ ≤ 360 . [4]

(ii) The smallest positive solution of the equation 2 cos2 (nθ ) = 3 sin(nθ ), where n is a positive
integer, is 10◦ . State the value of n and hence find the largest solution of this equation in the
interval 0◦ ≤ θ ≤ 360◦ . [3]

82 O/N 12/P13/Q3
Solve the equation 7 cos x + 5 = 2 sin2 x, for 0◦ ≤ x ≤ 360◦ . [4]

83 O/N 12/P13/Q6
The functions f and g are defined for − 12 π ≤ x ≤ 12 π by f (x) = 12 x + 16 π ,
g(x) = cos x.
Solve the following equations for − 12 π ≤ x ≤ 12 π .

(i) gf (x) = 1, giving your answer in terms of π . [2]

(ii) fg(x) = 1, giving your answers correct to 2 decimal places. [4]

84 M/J 12/P12/Q5

1 1
(i) Prove the identity tan x + ≡ . [2]
tan x sin x cos x
2
(ii) Solve the equation = 1 + 3 tan x, for 0◦ ≤ x ≤ 180◦ . [4]
sin x cos x

85 M/J 12/P11/Q1
Solve the equation sin 2x = 2 cos 2x, for 0◦ ≤ x ≤ 180◦ . [4]

86 M/J 12/P13/Q1
(i) Prove the identity tan2 θ − sin2 θ ≡ tan2 θ sin2 θ . [3]
(ii) Use this result to explain why tan θ > sin θ for 0◦ < θ < 90◦ . [1]

87 M/J 12/P13/Q4
◦ ◦
(i) Solve the equation sin 2x + 3 cos 2x = 0 for 0 ≤ x ≤ 360 . [5]

(ii) How many solutions has the equation sin 2x + 3 cos 2x = 0 for 0◦ ≤ x ≤ 1080◦ ? [1]

88 O/N 11/P12/Q5
(i) Sketch, on the same diagram, the graphs of y = sin x and y = cos 2x for 0◦ ≤ x ≤ 180◦ . [3]

(ii) Verify that x = 30◦ is a root of the equation sin x = cos 2x, and state the other root of this equation
for which 0◦ ≤ x ≤ 180◦ . [2]

(iii) Hence state the set of values of x, for 0◦ ≤ x ≤ 180◦ , for which sin x < cos 2x. [2]

144
Unit 5: Trigonometry

89 O/N 11/P11/Q3
1
(i) Sketch, on a single diagram, the graphs of y = cos 2θ and y = 2
for 0 ≤ θ ≤ 2π . [3]

(ii) Write down the number of roots of the equation 2 cos 2θ − 1 = 0 in the interval 0 ≤ θ ≤ 2π . [1]

(iii) Deduce the number of roots of the equation 2 cos 2θ − 1 = 0 in the interval 10π ≤ θ ≤ 20π . [1]

90 O/N 11/P13/Q5
(i) Given that 3 sin2 x − 8 cos x − 7 = 0,
show that, for real values of x, cos x = − 23 . [3]
(ii) Hence solve the equation
3 sin2 (θ + 70◦ ) − 8 cos(θ + 70◦ ) − 7 = 0
for 0◦ ≤ θ ≤ 180◦ . [4]

91 M/J 11/P12/Q5

≡1+
cos θ 1
tan θ (1 − sin θ )
(i) Prove the identity . [3]
sin θ

= 4, for 0◦ ≤ θ ≤ 360◦ .
cos θ [3]
tan θ (1 − sin θ )
(ii) Hence solve the equation

92 M/J 11/P12/Q9

The function f is such that f (x) = 3 − 4 cosk x, for 0 ≤ x ≤ π , where k is a constant.

(i) In the case where k = 2,


(a) find the range of f, [2]
(b) find the exact solutions of the equation f (x) = 1. [3]

(ii) In the case where k = 1,


(a) sketch the graph of y = f (x), [2]
(b) state, with a reason, whether f has an inverse. [1]

93 M/J 11/P11/Q5
(i) Show that the equation 2 tan2 θ sin2 θ = 1 can be written in the form

2 sin4 θ + sin2 θ − 1 = 0. [2]


(ii) Hence solve the equation 2 tan2 θ sin2 θ = 1 for 0◦ ≤ θ ≤ 360◦ [4]

94 M/J 11/P13/Q8

1 1 2 1 − cos θ
(i) Prove the identity  −  ≡ . [3]
sin θ tan θ 1 + cos θ
1 1 2 2
(ii) Hence solve the equation  −  = 5 , for 0◦ ≤ θ ≤ 360◦ . [4]
sin θ tan θ

145
Unit 5: Trigonometry

146
Unit 5: Answer Section

1 M/J 22/P12/Q11
4 2 2
 2
(a) 4cos x  cos x  3  0  4 cos x  3 cos x  1  0  
3
 cos2 x    cos 2 x  1
4
3  3
 cosx     their OE    
4  2 
π 5π 7π 11π
[x =] , , ,
6 6 6 6

(b) 1  1  16k
cos2x = < 0 [⸫ no solutions].
8
1  1  16k
[cos2x] =
8
Substituting k  5 and obtain 1 from the formula
cos2 x = 1 or cos2 x > or ⩾ 1
Concluding statement having considered both ± cases. ⸫ no
solutions
Alternative method for question 11(b)
1  1  16k
cos2x = < 0 [⸫ no solutions].
8
1  1  16k
[cos2x] =
8
1  1  16k
*1 ⇒ 1  1  16k * 8 ⇒ 1  16 k * 81
8
k *5
Concluding statement having considered both ± cases. ⸫ no
solutions
2 M/J 22/P11/Q4

(a) sin 3 sin 2 sin 3 1  sin   sin 2  sin   1


  
sin   1 1  sin   sin   1)(1  sin    sin   1)(1  sin  
 sin 3 1  sin    sin 2  sin   1 
 
  sin   1)(1  sin   
sin 2  sin 4

1  sin 2



sin 2 1  sin 2 
cos 2


 tan 2  1  sin 2  
Alternative method for Q4(a)

 tan 2 (1  sin 2 )  

sin 2 1  sin 2 
1  sin  2

sin 2  sin 4
1  sin  )(1  sin  

sin 2  sin 3  sin 3  sin 4 sin  1  sin    sin   sin   1


3 2

 sin   1)(1  sin    sin   1)(1  sin  

147
Unit 5: Answer Section

(b)   
 tan 2  1  sin 2   tan 2  1  sin 2   leading to  2 tan 2   0
tan   0 leading to   π
3 M/J 22/P11/Q8/b
(a) 1   2
4sin  x  30   2 2  sin 1     45
2   2 
1
x  30  45 or 135  x  2  45  30  or x  2 135  30 
2
x = 150°, x = 330°
4 M/J 22/P13/Q2
(a) [ p =] 3
(b) 1
[q =]
2
(c) [ r =] ‒2
5 M/J 22/P13/Q5
(a) 6 y  2  7 y1/2 [= 0]
 1  1 
 2 y 2  1 3 y 2  2  [= 0] or e.g.  2u  1 3u  2  [= 0]
 
  
1 2
[ y1/2 ] ,
2 3
1 4
 y  ,
4 9
(b) Use of tan x = their y values
x  14[.0], 24[.0],
x  194[.0], 204[.0]
6 O/N 21/P12/Q1
2cos 2θ − 7 cos θ + 3[ = 0]
( 2cosθ − 1)( cosθ − 3) = 0
1
[cos θ = or cos θ = 3 leading to] θ = −60° or θ = 60°
2
θ = −60° and θ = 60°
7 O/N 21/P11/Q3
3cosθ (2 tan θ − 1) + 2(2 tan θ − 1) [ = 0]
(2 tan θ − 1) (3cosθ + 2) [ = 0]
1 2
[leading to tanθ = , cosθ = − ]
2 3
26.6º, 131.8º

8 O/N 21/P11/Q5
(a) a=5 5(b)(i) 3 5(b)(ii) 2

b=2

c=3

148
Unit 5: Answer Section

9 O/N 21/P13/Q7
(a) tan x + cos x = k(tan x ‒cos x) leading to sin x + cos 2 x = k (sin x − cos 2 x )
sinx + 1 − sin 2 x = ksinx − k + ksin 2 x
ksin 2 x + sin 2 x + ksinx − sinx − k − 1 = 0
( k + 1) sin 2 x + ( k − 1) sin x − ( k + 1) = 0
(b) 5sin 2 x + 3sin x − 5 = 0

−3 ± 9 + 100
sin x =
10
x = 48.1°, 131.9°

10 M/J 21/P12/Q10
1 + sinx 1 − sinx (1 + sinx ) − (1 − sinx )
2 2
(a) − ≡
1 − sinx 1 + sinx (1 − sinx )(1 + sinx )


(
1 + 2sinx + sin 2 x − 1 − 2sinx + sin 2 x )
(1 − sinx )(1 + sinx )
4sinx 4sinx
≡ ≡
1 − sin x cos 2 x
2

4sinx 4tanx
≡ ≡
cosxcosx cosx
(b) 1
cosx =
2
π
x=
3
x = 0 from tanx = 0 or sinx = 0

11 M/J 21/P11/Q7

(a) cos 2θ − sin 2θ 1 − sin 2θ sin 2θ sin 2θ + cos 2θ


Reach 2
or 2
− 2
or 2
− 2tan 2θ
cos θ 1 − sin θ cos θ cos θ
2
2sin θ cos2θ
or sec2 θ − 2
or 2 − sec2 θ or
cos θ cos 2θ
1 − tan 2θ
(b) 1 − tan 2θ = 2tan 4θ ⇒ 2tan 4θ + tan 2θ −1 [= 0]

tan 2θ = 0.5 or −1 leading to tanθ = [ ± ] 0.5

θ = 35.3° and 144.7° (AWRT)


12 M/J 21/P13/Q4
(a) tanx + sinx sinx + sinxcosx
[ = k ] leading to [= k ]
tanx − sinx sinx − sinxcosx
1
+1 tanx + tanxcosx
or cos x [=k] or [=k]
1 tanx − tanxcosx
−1
cos x
1
sinx (1 + cosx ) cos x + 1 cos x tanx (1 + cosx ) 1 + cosx
or . or leading to [= k ]
sinx (1 − cosx ) 1
− 1 cos x tanx (1 − cosx ) 1 − cosx
cos x

149
12 M/J 21/P13/Q4 Unit 5: Answer Section
(b) k − k cos x = 1 + cos x leading to k − 1 = k cos x + cos x
k −1
k − 1 = ( k + 1) cos x leading to cos x =
k +1
(c) Obtaining cos x from their (b) or (a)
±0.927 (only solutions in the given range)

13 O/N 20/P12/Q6

(a) 
1 sinx  1 
−  + 1
 cosx cosx  sinx 
 1 − sinx  1 + sinx   1 − sin² x 
   or  
 cosx  sinx   cosxsinx 
 cos² x 
 
 cosxsinx 
 cos² x   cosx  1
  =  =
 cosxsinx   sinx  tanx
1 1
(b) Uses (a) → = 2tan² x tan³x =
tanx 2
(x =) 38.4°

14 O/N 20/P12/Q11/a,b,c

(a) 5, –1

(b)

(c)(i) 0 solution
(c)(ii) 2 solutions
(c)(iii) 1 solution

15 O/N 20/P11/Q7
(a)  sin θ sin θ  sin θ (1 + sin θ ) − sin θ (1 − sin θ )
 − =
 1 − sin θ 1 + sin θ  1 − sin 2θ
2sin 2θ
cos 2θ
2tan 2θ

(b) 2 tan 2 θ = 8 → tanθ = ( ± ) 2

(θ = ) 63.4°, 116.6°

150
16 O/N 20/P13/Q3 Unit 5: Answer Section

3tan 4θ + tan 2θ − 2 ( = 0 )

( 3tan θ − 2 )( tan θ + 1) ( = 0 )
2 2

2
tan θ = ( ± ) or ( ± ) 0.816 or ( ± ) 0.817
3
39.2°, 140.8°
17 M/J 20/P12/Q2
8sin θ
(a) 3cos θ = 8 tan θ → 3cosθ =
cos θ
3(1 − sin 2 θ ) = 8 sin θ
3 sin² θ + 8 sin θ – 3 = 0
(b) (3 sin θ – 1)( sin θ + 3) = 0 → sin θ = ⅓
θ = 19.5°
18 M/J 20/P11/Q7

(a) (1 + sin θ )2 + cos 2 θ


cos θ (1 + sin θ ) 2 3
(b) = → tanθ = 1.5
2 + 2sin θ 2 cosθ sinθ
Use of sin 2 θ + cos 2 θ = 1 → → .
cos θ (1 + sin θ ) cosθ
θ = 0.983 or 4.12
(FT on second value for 1st value + π)
19 M/J 20/P13/Q7
(a) tan θ tan θ tan θ (1 − cos θ ) + tan θ (1 + cos θ ) (b) 2 6cos θ
+ = =
1 + cos θ 1 − cos θ 1 − cos 2θ sin θ cosθ sin θ
2 tan θ
=
sin 2θ 1
cos 2θ = → cos θ = ( ± ) 0.5774
2sin θ 3
=
cos θ sin 2θ
54.7º, 125.3º
2 (FT for 180º ‒ 1st solution)
= AG
sin θ cosθ

20 MAR/ 20/P12/Q5

2 tan θ − 6sin θ + 2 = tan θ + 3sin θ + 2 → tan θ − 9sin θ ( = 0 )


sin θ − 9sin θ cos θ ( = 0 )
1
sin θ (1 − 9cosθ ) ( = 0 ) → sin θ = 0, cosθ =
9
θ = 0 or 83.6° (only answers in the given range)

21 MAR/ 20/P12/Q11

(a) (tan x − 2)(3tan x + 1) ( = 0 ) . or formula or completing square


1
tan x = 2 or -
3
x = 63.4° ( only value in range ) or161.6°(only value in range )
(b) Apply b 2 − 4ac < 0
25
k>
12

151
22 O/N 19/P12/Q6 Unit 5: Answer Section

(a) (2x + 1) = tan−1(⅓) (= 0.322 or 18.4 OR −0.339 rad or 8.7°)


Either their 0.322 + π or 2π
π
Or their −0.339 + or π
2
x = 1.23 or x = 2.80
(b)(i) 5 cos2 x – 2
(b)(ii) −2
3
23 O/N 19/P11/Q5
1
(i) 4 tan x + 3cos x + = 0 → 4sin x + 3cos 2 x + 1 = 0
cos x
( )
4sin x + 3 1 − sin x + 1 = 0 → 3sin 2 x − 4sin x − 4 = 0
2

2
sin x = −
3
(ii) 2 x ‒20º = 221.8º, 318.2º
x = 120.9º, 169.1º
24 O/N 19/P13/Q7
(i) ( )
3cos 4θ + 4 1 − cos 2θ − 3 ( = 0 )
3 x + 4 (1 − x ) − 3 ( = 0 ) → 3 x 2 − 4 x + 1( = 0 )
2

(ii) Attempt to solve for x


cos θ = ( ± )1, ( ± ) 0.5774
(θ = ) 0º, 180º, 54.7º, 125.3º
25 M/J 19/P12/Q4
(i) a 2 + 2 ab + b 2 , a 2 − 2 ab + b 2
sin²x + cos²x = 1 used → ( a + b ) + ( a − b ) = 1
2 2

a² + b² = ½
Alternative method for question 4(i)
2a = (s+c) & 2b = (s−c) or a = ½(s+c) & b = ½(s−c)
1 1
a²+b² = ( s + c ) + ( s − c ) = ½(s²+c²)
2 2

4 4
a² + b² = ½
sinx a+b
(ii) tanx = → =2
cosx a−b
a = 3b
26 M/J 19/P12/Q6
(i) 3, –3
−½

152
Unit 5: Answer Section
26 M/J 19/P12/Q6
(ii)

2 complete oscillations of a cosine curve starting with a maximum at (0,a), a൐0


Fully correct curve which must appear to level off at 0 and/or 2π.

Line starting on positive y axis and finishing below the x axis at 2π. Must be
straight.
(iii) 4
27 M/J 19/P11/Q6

(i) LHS =  −  =
1 s (1 − s )(1 − s ) (1 − s )(1 − s )
2
=
c c c² 1 − s2

(1 − s )(1 − s )
=
(1 − s )(1 + s )
1 − sinx
1 + sinx
(ii) Uses part (i) to obtain 1 − sin2 x = 1 → sin 2x = ½
1 + sin2 x 3
π
x=
12

(or) x =
12
28 M/J 19/P11/Q9
(i) −1 ⩽ f(x) ⩽ 5 or [–1, 5] (may use y or f instead of f(x))

(ii)

g(x) = 2 − 3cosx for 0 ⩽ x ⩽ p

153
Unit 5: Answer Section
28 M/J 19/P11/Q9
(iii) (greatest value of p =) π
(iv) x = 2 − 3cosx → cosx = ⅓(2 – x)
2− x
g−1(x) = cos−1 (may use ‘y =’)
3
29 M/J 19/P13/Q9
(i) q - f ( x ) - p + q

(ii) (a) 2
(b) 3
(c) 4
2
(iii) 3sin 2 2 x + 2 = 4 → sin 2 2 x = soi
3
2 2
Sin2x = (±)0.816(5). Allow sin 2 x = (± ) or 2 x = sin −1 (±)
3 3
(2x =) at least two of 0.955(3), 2.18(6), 4.09(7) , 5.32(8)
(x =) 0.478, 1.09, 2.05, 2.66.

30 MAR 19/P12/Q7
(a) ( )
3 1 − cos 2 2θ + 8cos 2θ = 0 → 3cos 2 2θ − 8cos 2θ − 3 ( = 0 )
1
cos 2θ = − soi
3
2θ = 109.(47)º or 250.(53)º
θ = 54.7º or 125.3º
(b) √ 3 = a + tan0 → a = √ 3
0 = tan(−bπ / 6) + √ 3 taken as far as tan −1 , angle units consistent
b=2
31 O/N 18/P12/Q6
(i) 9 9
In ∆ ABD, tanθ = → BD = or 9tan(90 – θ) or 9 cotθ
BD tanθ
9sin ( 90 − θ )
or ( 20 tanθ ) − 92  (Pythag) or
2
(Sine rule)
  sinθ
BD
In ∆ DBC, sinθ= → BD = 20sinθ
20
9
20sinθ =
tanθ

→ 20sin²θ = 9cosθ AG

(ii) Uses s² + c² = 1 → 20cos²θ + 9cosθ – 20 (= 0)


→ cosθ = 0.8
→ θ = 36.9º awrt

154
Unit 5: Answer Section
32 O/N 18/P11/Q5

(i) (cos θ − 4)(5cosθ − 2) − 4sin 2θ


( = 0)
sin θ (5cos θ − 2)

(
5cos 2θ − 22cos θ + 8 − 4 1 − cos 2θ ) ( = 0)
sin θ (5cos θ − 2)
9cos θ − 22cosθ + 4 = 0 www AG
2

(ii) Attempt to solve for cosθ , (formula, completing square expected)


θ = 78.6°, 281.4° (only, second solution in the range)

33 O/N 18/P13/Q7

(i) (tan θ + 1)(1 − cosθ ) + (tan θ − 1)(1 + cosθ )


soi
(1 + cosθ )(1 − cosθ )
tan θ − tan θ cos θ + 1 − cosθ + tan θ − 1 + tan θ cosθ − cosθ
1 − cos 2θ
2(tan θ − cosθ )
www
sin 2θ
(ii) sin θ
( 2 ) (tan θ − cosθ ) ( = 0 ) → ( 2 )  
− cos θ  ( = 0 ) soi
 cos θ 
( (
( 2 ) sin θ − 1 − sin 2θ ( = 0 )))
sin θ = 0.618 ( 0 )
θ = 38.2°
34 M/J 18/P12/Q4

(i) a + ½b = 5 (ii) a + b or their a + their b (3)

a – b = 11 a – b or their a – their b (11).

→ a = 7 and b = – 4 → k < 3 , k > 11

35 M/J 18/P12/Q10
(i) 2cos x = –3sinx → tanx = – ⅔
→ x = 146.3º or 326.3ºawrt

(ii) No labels required on either axis. Assume that the diagram is 0º


to 360º unless labelled otherwise. Ignore any part of the diagram
outside this range.

(iii) x < 146.3º, x > 326.3º

155
Unit 5: Answer Section
36 M/J 18/P11/Q4
(i) (sin θ + cos θ)(1 – sinθcosθ) ≡ sin³θ + cos³θ.
LHS = sinθ + cosθ – sin²θcosθ –sinθcos²θ
= sinθ(1 – cos²θ) + cosθ(1 – sin²θ) or (s + c – c(1 – c2) – s(1 – s2))
Uses sin²θ + cos²θ = 1 → sin³θ + cos³θ (RHS)
Or
(
LHS = ( sin θ + cos θ ) sin 2θ + cos 2θ − sin θ cos θ )
= sin θ + sinθcos θ − sin θ cos θ + cos θ sin θ + cos3θ − sin θ cos 2θ = sin 3θ + cos 3θ
3 2 2 2

(ii) (sin θ + cos θ)(1 – sinθcosθ) = 3cos³θ → sin³θ = 2cos³θ

→ tan³θ = 2 → θ = 51.6º or 231.6º (only)


37 M/J 18/P13/Q7

sin θ 2
−1
(a)(i) tan θ − 1
2
cos θ 2
=
tan 2θ + 1 sin θ 2
+1
cos θ 2
sin θ 2 − cos θ 2
=
sin θ 2 + cosθ 2
( )
= sin θ 2 − cos θ 2 = sin θ 2 − 1 − sin θ 2 = 2sin 2θ − 1
sec θ − 2
2
ALT 1
sec2θ
2
1− = 1 − 2cos 2θ
sec 2θ
( )
1 − 2 1 − sin 2θ = 2sin 2θ − 1
1 5
(a)(ii) 2sin 2θ − 1 = → sin θ = ( ± ) or ( ± ) 0.7906
4 8
θ = −52.2
38 O/N 17/P12/Q5

(i) EITHER:
sin 2 2 x
Uses tan 2 2 x = (ii) cos 2x = −½ , −1
cos 2 2 x
Uses sin²2x = (1 – cos²2x) 2x = 120°, 240° or 2x = 180°1
→ 2cos²2x + 3cos2x +1 = 0 x = 60° or 120°
OR: or x = 90º
tan 2 2 x = sec2 2 x − 1
1
sec 2 2 x =
cos²2 x
Multiply through by cos²2x and rearrange
→ 2cos²2x + 3cos2x +1 = 0

39 O/N 17/P12/Q6

(a)(i) 4 = a + ½b
3=a+b
→ a = 5, b = −2
((a)(ii) ff(x) = a + bsin(a + bsin
nx)

ff(0) = 5 – 2sin5 = 6.92


2

156
Unit 5: Answer Section
39 O/N 17/P12/Q6

(b) EITHE ER:


10 = c + d and −4 = c – d
10 = c – d and −4 = c + d
c = 3, d = 7, -7 or ±7
OR:

40 O/N 17/P11/Q7

(a) a = −2, b=3


(b)(i) s + s − sc + 2c + 2 sc − 2c 2 = s + sc → s 2 − 2c 2 + 2c = 0
2

1 − cos 2θ − 2cos 2θ + 2 cos θ = 0


3cos 2θ − 2cos θ − 1 = 0
(b)(ii) 1
cosθ = 1 or −
3
θ = 0° or 109.5° or −109.5°

41 O/N 17/P13/Q5

(i) cos θ + 4 + 5sin 2θ + 5sin θ − 5sin θ − 5 ( = 0 )


( )
5 1 − cos 2θ + cos θ − 1 ( = 0 )
5cos 2θ − cos θ − 4 = 0
(ii) cosθ = 1 and −0.8

θ = [ 0°, 360°] , [143.1°] , [ 216.9°]


42 M/J 17/P12/Q3

(i) 1 s
2
LHS =  − 
c c

=
(1 − s )2
1 − s²
(1 − s )(1 − s ) 1 − sin θ
= =
(1 − s )(1 + s ) 1 + sin θ

(ii) Uses part (i) → 2 – 2s = 1 + s


→s=⅓
θ = 19.5º or 160.5°

157
Unit 5: Answer Section
43 M/J 17/P12/Q10

1  1 
(i) 3tan  x  = −2 → tan  x  = −⅔
2  2 
½x = −0.6 (‒ 0.588) → x = ‒1.2
y+2 1 
(ii) = tan  x 
3 2 
 x+2
→ f−1(x) = 2tan −1  
 3 
−5,1
(iii)

44 M/J 17/P11/Q3

(i) 1 + cosθ sinθ 2


+ ≡ .
sinθ 1 + cosθ sinθ
(1 + c ) + s ²
2
1 + 2c + c ² + s ²
=
s (1 + c ) s (1 + c )
2 + 2c 2 (1 + c ) 2
= = →
s (1 + c ) s (1 + c ) s

(ii) 2 3 2
= →t=
s c 3

→ θ = 33.7° or 213.7°

45 M/J 17/P11/Q5
y = 2cosx

(i)

158
Unit 5: Answer Section
45 M/J 17/P11/Q5
π
(ii) P( , 1) Q(π, −2)
3
2
 2π 
→ PQ² =   + 3² → PQ = 3.7
 3 
46 M/J 17/P13/Q5

(i) 2sin θ + cosθ 2sin θ


=
sin θ + cosθ cosθ
2sin θ cos θ + cos 2θ = 2sin 2θ + 2sin θ cos θ ⇒ c 2 = 2 s 2

(ii) tan 2θ = 1 / 2 or cos 2θ = 2 / 3 or sin 2θ = 1 / 3

θ = 35.3° or 144.7°

47 O/N 16/P12/Q2
(ii) x = (tan ( their k )) ÷ 2
−1

(i) 2sin2x = 6cos2x (71.6º or −108.4º) ÷2


tan2x = k x = 35.8º, −54.2º
→ tan2x = 3 or k =3
x = 0.624c, – 0.946c
x = 0.198πc, – 0.301πc
48 O/N 16/P11/Q6

( )
2
(i) cos 4 x = 1 − sin 2 x = 1 − 2sin 2 x + sin 4 x
(ii) (
8sin 4 x + 1 − 2sin 2 x + sin 4 x = 2 1 − sin 2 x )
9sin 4 x = 1
x = 35.3o (or any correct solution)
Any correct second solution from 144.7˚, 215.3˚,
324.7˚
The remaining 2 solutions
49 O/N 16/P13/Q3
6
4sin 2 x = 6cos 2 x ⇒ tan 2 x =
4
or 4sin 2 x = 6 1 − sin 2 x ( )
[tan x = (±)1.225 or sin x = (±)0.7746 or cos x = (±)0.6325]
x= 50.8 (Allow 0.886 (rad))
Another angle correct
x = 50.8°, 129.2°, 230.8°, 309.2°
[ 0.886, 2.25/6, 4.03, 5.40 (rad) ]

50 M/J 16/P12/Q5

(i) π  AC  π  π  2x
tan   = or cos   = sin  =
3 2x  3  6  AB
→ AC = 2√3x or AB = 4x

AM = 13 x 2 , 13 x,3.61x
ˆ )= x
(ii) tan ( MAC
Their AC
1 1
θ= π – tan−1 AG
6 2√ 3

159
Unit 5: Answer Section
51 M/J 16/P12/Q7
(i) 1 + cosθ 1 − cosθ 4
− ≡
1 − cosθ 1 + cosθ sinθ tanθ

LHS =
(
1 + 2 c + c ² − 1 − 2c + c 2 )
(1 − c )(1 + c )
4c
=
1 − c2
4c
=

4
= AG
ts
(ii)  1 + cosθ 1 − cosθ 
sinθ  −  =3.
 1 − cosθ 1 + cosθ 
4 4
→s× = 3 (→ t = )
ts 3
θ = 53.1° and 233.1°

52 M/J 16/P11/Q2
3sin²θ = 4cosθ – 1
Uses s² + c² = 1
→ 3c² + 4c – 4 (= 0)
2
(→ c = or – 2)
3
→ θ = 48.2° or 311.8°
0.841, 5.44 rads, A1 only
(0.268π, 1.73π)
53 M/J 16/P13/Q8
(i) 3sin 2 x − cos 2 x + cos x = 0
Use s 2 = 1 − c 2 and simplify to 3-term quad
cos x = −3 / 4 and 1
x = 2.42 (allow 0.77 π ) or 0 (extra in range
max 1)
(ii) 2x = 2π −their 2.42 or 360 – 138.6
x = 1.21 (0.385π), 1.93 (0.614/5π), 0, π (3.14)
(extra max 1)
54 O/N 15/P12/Q4
2 2
 1 1  1 c
(i)  −  = − 
 sin x tan x   s s 
(1 − c) 2 (1 − c) 2
=
s2 1 − c2
(1 − c)(1 − c) (1 − c) 2
= or
(1 − c)(1 + c) (1 − c)(1 + c)
1 − cos x

1 + cos x
2
 1 1  2
(ii)  −  =
 sin x tan x  5
1 − cos x 2 3
= → cos x
1 + cos x 5 7
→ x = 1.13 or 5.16

160
55 O/N 15/P11/Q3 Unit 5: Answer Section

4 x 2 + x 2 = 1 / 2 soi
Solve as quadratic in x 2
x2 =1 4
x = ±1 2
56 O/N 15/P11/Q4

(i) 4 cos 2 θ + 15 sin θ = 0


4(1 − s 2 ) + 15s = 0 → 4 sin 2 θ − 15 sin θ − 4 = 0
(ii) sin θ = −1 / 4
θ = 194 .5 or 345 .5
57 O/N 15/P13/Q7
(a) 1 + 3 sin 2 θ + 4 cosθ = 0
1 + 3(1 − cos 2 θ ) + 4 cosθ + 0
3 cos 2 θ − 4 cosθ − 4 = 0
cosθ = −2 / 3
θ = 131.8 or 228.2

(b) c = b / a cao
d =a−b
58 M/J 15/P12/Q5
sin θ − cosθ
(i) .
sin θ + cosθ
Divides top and bottom by cos θ
→ t −1
t +1
sin θ − cosθ 1
(ii) = tan θ
sin θ + cosθ 6
→ t −1 = t
t +1 6
2
→ t − 5t + 6 = 0
→ t = 2 or t = 3
→ θ = 63.4º or 71.6º

59 M/J 15/P12/Q6
h = 60(1 − cos kt )
(i) Max h when cos = −1 → 120
(ii) h = 0 and t = 30, or h =120 and t = 15
→ cos30k = 1 or cos15k = –1
→ 30k = 2π or 15k = π
2π π
→ k = =
30 15
(iii) 90 = 60(1 – cos kt)
→ coskt = −6030 = −0.5

→ kt = 2π or → kt = 4π
3 3

→ Either t = 10 or 20 or both
→ t = 10 minutes

161
60 M/J 15/P11/Q1 Unit 5: Answer Section
θ is obtuse , sin θ = k

(i) cos θ = −√(1 – k


sin θ
(ii) tan θ = used
cos θ
k
→ tan θ = −
(1 − k 2 )
(iii) sin (θ + π) = −k

61 M/J 15/P11/Q8

(ii)
1 
f : x⟼5 + 3cos  x  for 0 ø x ø 2π.
2 
1 
(i) 5 + 3cos  x  = 7 2
2 
1  2
cos  x  =
2  3 (iii) No turning point on graph or 1:1
1
x = 0.84 x = 1.68 only, aef
2 (iv) 1 
y = 5 + 3cos  x 
(in given range) 2 
Order; −5, ÷3, cos−1, ×2
 x −5
62 M/J 15/P13/Q4 x = 2cos−1  3 

(i) tan θ = 1 / 3
θ = 18.4° only
(ii) tan 2 x = ( ± )1 / 3 Must be sq. root soi
( x) = 15
(x) = any correct second value (75, 105, 165)
(x) = cao
63 O/N 14/P12/Q5
1 + sinxtanx = 5cosx (ii) Soln of quadratic → (c = −⅓ or ½)
(i) Replaces t by s/c → x = 60° or 109.5°
s2
1+ = 5c
c
Replace s² by 1 − c²
→ 6c² − c − 1 (= 0)

64 O/N 14/P12/Q11

1  (ii) Range is 2 < f(x) < 10


(i) f : x ↦ 6 – 4cos  x 
2  (iii)
1  1
1  1  (iv) cos x  = (6 − y )
6 – 4cos  x  = 4 → 4cos  x  = 2 2  4
2  2 
1 1 
1 1
x= π x= π
2 x = cos −1  (6 − y )
2 3 3 2 4 
6− x
f–1(x) = 2cos–1  
 4 

162
Unit 5: Answer Section
65 O/N 14/P11/Q2
tan–1(3) = 1.249 or 71.565º
sin 1.25 or sin71.6 or 0.949 soi
(x =) 1.95 cao, accept 1 + 3 oe
10
66 O/N 14/P11/Q3
13 sin 2 θ + 2 cos θ + cos 2 θ = 4 + 2 cos θ
13 sin 2 θ + 1 − sin 2 θ = 4 → sin 2 θ = 14
or 13 − 13 cos 2 θ + cos 2 θ = 4 → cos 2 θ = 3
4
30º, 150º

67 O/N 14/P13/Q5
1 3
(i) (s 2
− c 2 )( s 2 + c 2 ) OR s 2 (1 − c 2 ) − c 2 (1 − s 2 ) (ii) 2sin2θ – 1 =
2
⇒ sinθ = (± )
2
or (± )0.866
sin2θ – cos2θ θ = 60°
2sin2θ – 1 www AG θ = 120°
θ = 240°, 300°
68 M/J 14/P12/Q3

reflex angle θ is such that cosθ = k,


(i) (a) sin θ = − √(1 − k²)
− 1− k2
(b) Uses t=s/c →
k
(ii) θ is in 4th quadrant.
2θ lies between 540° and 720°
sin2θ is negative in both these quadrants.

69 M/J 14/P12/Q5
1 cos θ
(i) − ≡ tan θ .
cos θ 1 + sin θ
1 + s − c2 s2 + s s
LHS = = =
c(1 + s ) c(1 + s ) c
= tanθ

(ii) → tanθ + 2 = 0 ie tanθ = −2


→ θ = 116.6° or 296.6°
70 M/J 14/P11/Q1

a = 1, b=2

71 M/J 14/P11/Q1
sin 2 θ − (1 − cosθ )
(i) LHS ≡ cao
(1 − cosθ )sin θ (ii) tan θ = (± )
1
1 − cos 2 θ − 1 + cosθ 2
LHS ≡
(1 − cosθ )sin θ 26.6°, 153.4°
cosθ (1 − cosθ )
LHS ≡
(1 − cosθ )sin θ
1
LHS ≡
tan θ

163
72 M/J 14/P13/Q4 Unit 5: Answer Section

tan x + 1 ≡ sin x + cos x


sin x tan x + cos x
s
  + 1
(i) LHS c = s2 + c 2
 s2  s +c

 c
+ c 
 
= RHS
(ii) s + c = 3s − 2c
→ tanx = 3 Allow cos2 = 4 , sin2 = 9
2 13 13
→ x = 0.983 and 4.12 or 4.13

73 O/N 13/P12/Q1

(i) sinx = √(1 − p²)


sin x 1 − p2
(ii) tan x = =
cos x p
p
(iii) tan(90 − x ) =
1 − p2
74 O/N 13/P12/Q8
y
(iii)
f : x a 3 cos x − 2 for 0 Y x Y 2π.
(i) 3cosx − 2 = 0 → cos x = ⅔ 0 2π x
→ x = 0.841 or 5.44
(ii) range is −5 Y f(x) Y 1

(iv) max value of k = π or 180º.  x+2


(iv) g −1(x) = cos −1  
 3 
75 O/N 13/P11/Q4

(i) ( )
4 1 − cos 2 x + 8 cos x − 7 = 0
1 θ = 60° (or 300°)
(ii)
4c − 8c + 3 = 0 → (2 cos x − 1)(2 cos x − 3) = 0
2
2
x = 60° or 300° θ = 120 ° only

76 O/N 13/P13/Q7
π 5π  13π π 
π (b) 2θ + =  or or 
(a) x2 – 1 = sin 3 6  6 6
3
x = ± 1.366 π  11π 
2θ = =  or 
2  6 
π 11π
θ= ,
4 12

77 M/J 13/P12/Q5
a = sin θ − 3 cosθ , b = 3 sin θ + cosθ (ii) 2s − 6c = 3s + c → s = −7c
(i) a ² + b² = → tanθ = −7
→ 98.1°
( s 2 + 9c 2 − 6sc) + (9s 2 + c 2 + 6sc) and 278.1°
10c² + 10s² = 10

164
Unit 5: Answer Section
78 M/J 13/P11/Q5 1 1
(ii) s 2 − (1 − s 2 ) = or 1 − c 2 − c 2 =
sin θ (sin θ − cos θ ) + cos θ (sin θ + cos θ ) 3 3
(i)
(sin θ + cos θ )(sin θ − cos θ ) or 3(s2 – c2) = c2 + s2
sin θ − sin θ cosθ + cosθ sin θ = cos 2 θ
2 2 1
sin θ = (±) or cosθ = (±)
sin 2 θ − cos 2 θ 3 3
1 or tan θ = (± ) 2
sin 2 θ − cos 2 θ
θ = 54.7°, 125.3°, 234.7°, 305.3°

79 M/J 13/P13/Q5
(i) +1
(ii) (a) sin2x = −½ → 4 solutions
0
π 2π (b) sin2x+ cosx+1=0 → 3 solutions.
–1

–2

80 O/N 12/P12/Q6

(ii) Soln of quadratic


2 cos x = 3 tan x
→ y = 15 º
(i) Replaces tan x by sinx ÷ cos x
2y can also be 180 − 30
→ 2c² =3s → 2 s² +3s − 2 = 0
→ y = 75 º.

81 O/N 12/P11/Q7

 
(i) 21  sin ! 3 sin ! (ii)  3

2 sin !  1sin !  2 0 $%&' 3! 720  $%&'150 870
! 30° or 150° ! 290°
82 O/N 12/P13/Q3
7 cos x +5=2(1–cos2 x)
2 cos x + 1( cos x + 3) =0
cos = –0.5
x=120°, 240°

83 O/N 12/P13/Q6 1 π
(ii) cos x + =1
1 π 2 6
(i) cos  x +  (= 1) cos x = 0.9528
2 6
x= –
π x = ± 0.31
3

84 M/J 12/P12/Q5
1 1 2
tan x + ≡ (ii) = 3 tan x + 1
tan x sin x cos x sin x cos x
sin x cos x 1
(i) LHS = + Uses (i) 2(tan x + ) = 3 tan x + 1
cos x sin x tan x
sin 2 x + cos 2 x 1 → tan 2 x + tan x − 2 = 0
= = → tanx = 1 or −2
sin x cos x sin x cos x
→ x = 45º or 116.6º

165
Unit 5: Answer Section

85 M/J 12/P11/Q1
tan 2x = 2
2x = 63.4 or 243.4
x = 31.7 or 121.7 (allow 122)
86 M/J 12/P13/Q1
tan 2 θ − sin 2 θ = tan 2 θ sin 2 θ
s2
(i) − s2
c2
s 2 − s 2c 2 s 2 (1 − c 2 )
→ =
c2 c2
→ t 2s2

(ii) RHS > 0 → tan² θ > sin ²θ QED


tan θ > sin θ if θ acute.
87 M/J 12/P13/Q4
sin 2 x + 3 cos 2 x = 0
(i) → tan 2x = −3
2x = 180 – 71.6 or 360 – 71.6
x = 54.2º or 144.2º
Also 234.2º and 324.2º
(ii) 12 answers.
88 O/N 11/P12/Q5
(i) (ii) Evidence of sin 30 = cos 60 = 0.5
Other root is 150º

(iii) 0 ≤ x < 30 and 150 < x ≤ 180


(x < 30 or x > 150 ok)

89 O/N 11/P11/Q3
(i) Correct cosine curve for at least 1 (ii) 4
oscillation
Exactly 2 complete oscillations in [0,2π ] (iii) 20
1
Line y = correct
2
90 O/N 11/P13/Q5
(i) 3cos2x + 8cosx + 4 = 0
(3cosx + 2)(cosx + 2) = 0
2
cosx = −
3
2
(ii) cos(θ + 70) = − , θ = 61.8
3
θ + 70 = 131.8 (or 228.2)
θ = 158.2

166
Unit 5: Answer Section
91 M/J 11/P12/Q5
cos θ cos 2 θ
(i) ≡
tan θ (1 − sin θ ) sin θ (1 − sin θ )
1 − sin 2 θ
=
sin θ (1 − sin θ )
1 + sin θ 1
= = +1
sin θ sin θ

cos θ 1
(ii) =4 → +1 = 4
tan θ (1 − sin θ ) sin θ
→ sinθ = ⅓ → θ = 19.5º, 160.5º

92 M/J 11/P12/Q9
(b) 3 − 4cos²x = 1 → cos²x = ½
(i) (a) f(x) = 3 − 4cos²x. 1
One limit is −1 → cosx = ±
Other limit is 3 2
→ x = ¼ π or ¾ π
(ii) (a)
(b) f has an inverse since it is 1:1 or
increasing or no turning points.

93 M/J 11/P11/Q5
2 sin 2 θ sin 2 θ (ii) (2 sin 2 θ − 1)(sin 2 θ + 1) = 0
5 (i) =1
1 − sin 2 θ (± )1
2 sin 4 θ + sin 2 θ − 1 = 0 sin θ =
2
θ = 45°, 135°
θ = 225°, 315°

94 M/J 11/P13/Q8
2
 1 1  1 − cos θ
(i)  −  ≡
 sin θ tan θ  1 + cos θ
 1

cos θ 
2
(1 − cosθ ) 2
  =
 sin θ sin θ  sin 2 θ
=
(1 − cosθ )(1 − cosθ ) = 1 − cosθ
1 − cos 2 θ 1 + cos θ

2
 1 1  2
(ii)  −  =
 sin θ tan θ  5
1 − cos θ 2
=
1 + cos θ 5
3
cos θ =
7
θ = 64.6° or 295.4°

167
UNIT 6.1 Binomial Expansion Unit 6.1: Binomial Expansion

1 M/J 22/P12/Q1
The coefficient of x4 in the expansion of 3 + x5 is equal to the coefficient of x2 in the expansion of
@ A
a 6
2x + .
x
Find the value of the positive constant a. [4]

2 M/J 22/P11/Q4 P Q5
4 k2 2
The coefficient of x in the expansion of 2x + is a. The coefficient of x2 in the expansion of
x
4
2kx − 1 is b.

(a) Find a and b in terms of the constant k. [3]


(b) Given that a + b = 216, find the possible values of k. [3]

3 M/J 22/P13/Q1
@ A
1 4
The coefficient of x3 in the expansion of p + x is 144.
p
Find the possible values of the constant p. [4]

4 O/N 21/P12/Q8
(a) It is given that in the expansion of 4 + 2x 2 − ax5 , the coefficient of x2 is −15.

Find the possible values of a. [4]


(b) It is given instead that in the expansion of 4 + 2x 2 − ax5 , the coefficient of x2 is k. It is also
given that there is only one value of a which leads to this value of k.
Find the values of k and a. [4]
5 O/N 21/P11/Q1
@ A
(a) Expand 1 −
1 2
. [1]
2x
(b) Find the first four terms in the expansion, in ascending powers of x, of 1 + 2x6. [2]
@ A
(c) Hence find the coefficient of x in the expansion of 1 − 1 + 2x6.
1 2
[2]
2x
6 O/N 21/P13/Q2
(a) Find the first three terms, in ascending powers of x, in the expansion of 1 + ax6 . [1]
2 6
(b) Given that the coefficient of x in the expansion of 1 − 3x 1 + ax is −3, find the possible values
of the constant a. [4]

7 M/J 21/P12/Q4 @ A
10 3 1
The coefficient of x in the expansion of 4x + is p. The coefficient of in the expansion of
@ A5 x x
k
2x + 2 is q.
x
Given that p = 6q, find the possible values of k. [5]

8 M/J 21/P11/Q3
(a) Find the first three terms in the expansion of 3 − 2x5 in ascending powers of x. [3]
(b) Hence find the coefficient of x2 in the expansion of 4 + x2 3 − 2x5 . [3]

168
Unit 6.1: Binomial Expansion
9 M/J 21/P13/Q7
(a) Write down the first four terms of the expansion, in ascending powers of x, of a − x6 . [2]
@ A
2
(b) Given that the coefficient of x2 in the expansion of 1 + a − x6 is −20, find in exact form
ax
the possible values of the constant a. [5]

10 O/N 20/P12/Q1
The coefficient of x3 in the expansion of 1 + kx 1 − 2x5 is 20.
Find the value of the constant k. [4]

11 O/N 20/P11/Q5
0 a 16
In the expansion of 2x2 + , the coefficients of x6 and x3 are equal.
x
(a) Find the value of the non-zero constant a. 0 [4]
6 3 a 16
2
(b) Find the coefficient of x in the expansion of 1 − x  2x + . [1]
x

12 O/N 20/P13/Q5
In the expansion of a + bx7 , where a and b are non-zero constants, the coefficients of x, x2 and x4
are the first, second and third terms respectively of a geometric progression.
a
Find the value of . [5]
b
13 M/J 20/P12/Q1 @ A
2 6
(a) Find the coefficient of x2 in the expansion of x − . [2]
x @ A6
2
(b) Find the coefficient of x2 in the expansion of 2 + 3x2  x − . [3]
x
14 M/J 20/P11/Q2
@ A @ A
1 1 5 2 8
The coefficient of in the expansion of kx + + 1− is 74.
x x x
[5]
Find the value of the positive constant k.

15 M/J 20/P13/Q4
(a) Expand 1 + a5 in ascending powers of a up to and including the term in a3 . [1]

(b) Hence expand 1 + x + x2 5 in ascending powers of x up to and including the term in x3 ,
simplifying your answer. [3]

16 MAR 20/P12/Q6
@ A
1 a 5
The coefficient of in the expansion of 2x + 2 is 720.
x x
(a) Find the possible values of the constant a. [3]
1
(b) Hence find the coefficient of 7 in the expansion. [2]
x
17 O/N 19/P12/Q1
@ A
x 6
The coefficient of x2 in the expansion of 4 + ax 1 + is 3. Find the value of the constant a. [4]
2
18 O/N 19/P11/Q1
@ A
1 6 [3]
Find the term independent of x in the expansion of 2x + 2
4x

169
19 O/N 19/P13/Q1 Unit 6.1: Binomial Expansion

(i) Expand 1 + y6 in ascending powers of y as far as the term in y2 . [1]


 6
(ii) In the expansion of 1 + px − 2x2  the coefficient of x2 is 48. Find the value of the positive
constant p. [3]

20 M/J 19/P12/Q1
@ A5
2
Find the coefficient of x in the expansion of − 3x . [3]
x

21 M/J 19/P11/Q1 @ A
k 6
The term independent of x in the expansion of 2x + , where k is a constant, is 540.
x
(i) Find the value of k. [3]
(ii) For this value of k, find the coefficient of x2 in the expansion. [2]

22 M/J 19/P13/Q2 @ A
1 5
(i) In the binomial expansion of 2x − , the first three terms are 32x5 − 40x3 + 20x. Find the
2x
remaining three terms of the expansion. [3]
  @ A5
1
(ii) Hence find the coefficient of x in the expansion of 1 + 4x2 2x − . [2]
2x
23 MAR 19/P12/Q1
The coefficient of x3 in the expansion of 1 − px5 is −2160. Find the value of the constant p. [3]

24 O/N 18/P12/Q1
@ A
Find the coefficient of 2 in the expansion of 3x + 2 .
1 2 7
[4]
x 3x
25 O/N 18/P13/Q1
@ A
1 2 7
Find the coefficient of in the expansion of x − . [3]
x3 x
26 M/J 18/P12/Q1
0 x 16
The coefficient of x2 in the expansion of 2 + + a + x5 is 330. Find the value of the constant a.
2
[5]
27 M/J 18/P11/Q1
(i) Find the first three terms in the expansion, in ascending powers of x, of 1 − 2x5 . [2]
(ii) Given that the coefficient of x2 in the expansion of 1 + ax + 2x2  1 − 2x5 is 12, find the value of
the constant a. [3]
28 M/J 18/P13/Q2
@ A
1 2 5
Find the coefficient of in the expansion of x − . [3]
x x
29 O/N 17/P12/Q1
@ A
1 9
Find the term independent of x in the expansion of 2x − 2 . [4]
4x

30 O/N 17/P13/Q3
@ A6
2
(i) Find the term independent of x in the expansion of − 3x . [2]
x
(ii) Find the value of a for which there is no term independent of x in the expansion of
@ A6
2
1 + ax2  − 3x . [3]
x

170
Unit 6.1: Binomial Expansion
31 M/J 17/P12/Q1 @ A
1 5
(i) Find the coefficient of x in the expansion of 2x − . @ A [2]
x 1 5
(ii) Hence find the coefficient of x in the expansion of 1 + 3x2  2x − . [4]
x

32 M/J 17/P11/Q1
The coefficients of x2 and x3 in the expansion of 3 − 2x6 are a and b respectively. Find the value of
a
. [4]
b

33 M/J 17/P13/Q1
The coefficients of x and x2 in the expansion of 2 + ax7 are equal. Find the value of the non-zero
constant a. [3]

34 O/N 16/P12/Q4
0 x 1n
In the expansion of 3 − 2x 1 + , the coefficient of x is 7. Find the value of the constant n and
2
hence find the coefficient of x2 . [6]

35 O/N 16/P11/Q2
@ A
1 8
Find the term independent of x in the expansion of 2x + 3 . [4]
2x
36 O/N 16/P13/Q2
The coefficient of x3 in the expansion of 1 − 3x6 + 1 + ax5 is 100. Find the value of the constant a.
[4]
37 M/J 16/P12/Q4
Find the term that is independent of x in the expansion of
@ A
2 6
(i) x − , [2]
x
@ A@ A
3 2 6
(ii) 2 + 2 x − . [4]
x x

38 M/J 16/P11/Q1
@ A
3 6
Find the term independent of x in the expansion of x − [3]
2x
39 M/J 16/P13/Q1
@ A5
1
Find the coefficient of x in the expansion of + 3x2 . [3]
x
40 O/N 15/P12/Q2
In the expansion of x + 2k7 , where k is a non-zero constant, the coefficients of x4 and x5 are equal.
Find the value of k. [4]

41 O/N 15/P11/Q1
@ A
2x
In the expansion of 1 − a + x5 , where a is a non-zero constant, show that the coefficient of x2
a
is zero. [3]

42 O/N 15/P13/Q2
@ A7
x 9
Find the coefficient of x in the expansion of + . [4]
3 x2

171
Unit 6.1: Binomial Expansion
43 M/J 15/P12/Q3
2 3 6
(i) Find the coefficients of x and x in the expansion of 2 − x . [3]

(ii) Find the coefficient of x3 in the expansion of 3x + 1 2 − x6 . [2]

44 M/J 15/P11/Q3

(i) Find the first three terms, in ascending powers of x, in the expansion of
(a) 1 − x6 , [2]
(b) 1 + 2x6 . [2]

(ii) Hence find the coefficient of x2 in the expansion of  1 − x 1 + 2x6 . [3]

45 M/J 15/P13/Q3
(i) Write down the first 4 terms, in ascending powers of x, of the expansion of a − x5 . [2]

(ii) The coefficient of x3 in the expansion of 1 − ax a − x5 is −200. Find the possible values of the
constant a. [4]

46 O/N 14/P12/Q3

(i)Find the first 3 terms, in ascending powers of x, in the expansion of 1 + x5 . [2]
 5
The coefficient of x2 in the expansion of 1 + px + x2  is 95.

(ii) Use the answer to part (i) to find the value of the positive constant p. [3]

47 O/N 14/P11/Q1
In the expansion of 2 + ax7 , the coefficient of x is equal to the coefficient of x2 . Find the value of
the non-zero constant a. [3]

48 O/N 14/P13/Q1
In the expansion of 2 + ax6 , the coefficient of x2 is equal to the coefficient of x3 . Find the value of
the non-zero constant.a [4]
49 M/J 14/P12/Q2
@ A
x 4 6
Find the coefficient of x2 in the expansion of 1 + x2  − . [5]
2 x

50 M/J 14/P11/Q3
@ A
3 1 8
Find the term independent of x in the expansion of 4x + . [4]
2x
51 M/J 14/P13/Q1
@ A
2 2 5
Find the coefficient of x in the expansion of x − . [3]
x
52 O/N 13/P11/Q1
(i) Find the first three terms when 2 + 3x6 is expanded in ascending powers of x. [3]

(ii) In the expansion of 1 + ax 2 + 3x6 , the coefficient of x2 is zero. Find the value of a. [2]

172
53 O/N 13/P13/Q8 Unit 6.1: Binomial Expansion
 4
(i) Find the coefficient of x8 in the expansion of x + 3x2 . [1]
 5
(ii) Find the coefficient of x8 in the expansion of x + 3x2 . [3]
  5
(iii) Hence find the coefficient of x8 in the expansion of 1 + x + 3x2 . [4]

54 M/J 13/P12/Q2
Find the coefficient of x2 in the expansion of
@ A
1 6
(i) 2x − , [2]
2x
@ A
1 6
(ii) 1 + x2  2x − . [3]
2x
55 M/J 13/P11/Q2
(i) In the expression 1 − px6 , p is a non-zero constant. Find the first three terms when 1 − px6 is
expanded in ascending powers of x. [2]

(ii) It is given that the coefficient of x2 in the expansion of 1 − x 1 − px6 is zero. Find the value
of p. [3]

56 M/J 13/P13/Q4
(i) Find the first three terms in the expansion of 2 + ax5 in ascending powers of x. [3]

(ii) Given that the coefficient of x2 in the expansion of 1 + 2x 2 + ax5 is 240, find the possible
values of a. [3]

57 O/N 12/P12/Q1

In the expansion of x2 −  , the coefficient of x5 is −280. Find the value of the constant a.
a 7
[3]
x

58 O/N 12/P11/Q4
(i) Find the first 3 terms in the expansion of (2x − x2 )6 in ascending powers of x. [3]
6
(ii) Hence find the coefficient of x8 in the expansion of (2 + x)(2x − x2 ) . [2]

59 O/N 12/P13/Q1
Find the coefficient of x3 in the expansion of 2 − 12 x .
7
[3]

60 M/J 12/P12/Q3
The coefficient of x3 in the expansion of (a + x)5 + (2 − x)6 is 90. Find the value of the positive
constant a. [5]
61 M/J 12/P11/Q2
Find the coefficient of x6 in the expansion of 2x3 − .
7
1
[4]
x2

62 M/J 12/P13/Q3
The first three terms in the expansion of (1 − 2x)2 (1 + ax)6 , in ascending powers of x, are 1 − x + bx2 .
Find the values of the constants a and b. [6]

63 O/N 11/P12/Q1
(i) Find the first 3 terms in the expansion of (2 − y)5 in ascending powers of y. [2]
5
(ii) Use the result in part (i) to find the coefficient of x2 in the expansion of 2 − (2x − x2 ) . [3]

173
Unit 6.1: Binomial Expansion
64 O/N 11/P11/Q1
1 6
Find the term independent of x in the expansion of 2x +  . [3]
x2

65 O/N 11/P13/Q1
5
The coefficient of x2 in the expansion of k + 13 x is 30. Find the value of the constant k. [3]

66 M/J 11/P12/Q2

(i) Find the terms in x2 and x3 in the expansion of 1 − 32 x .


6
[3]

(ii) Given that there is no term in x3 in the expansion of (k + 2x) 1 − 32 x , find the value of the
6

constant k. [2]

174
Unit 6.1: Answer Section
UNIT 6.1
1 M/J 22/P12/Q1
Coefficient of x 4  15
Coefficient of x2  240a 2
‘Their 240’a2 – ‘their 15’
1
a= or 0.25
4
2 M/J 22/P11/Q4
2

term is 10   2 x 2 3
 k2  (b) 80k  24k  216  0  
4 2 4 2
(a)    10k  3k  27  0
4
x
 x 
80k 4 x 4  a  80k 4  2k 2

 3 5k 2  9   0 [⇒ k 2

3
2
9
or  ]
5
[ x 2 term is [6 ](2kx)2  1 = 24k2x2 ] b  24k 2
3
k   
2
3 M/J 22/P13/Q1

1 3
4C1 p  x
p3
4
 144
p2
1
p
6
4 O/N 21/P12/Q8

(a) Terms required for x2: −5 × 24 × ax + 10 × 23 × a 2 x 2  = −80ax + 80a 2 x 2 


2× (±their coefficient of x) + 4× (±their coefficient of x2)
x2 coefficient is 320a2 – 160a = −15
 64a 2 − 32a + 3  ( 8a − 3)( 8a − 1)
1 3
a= or a =
8 8

5 O/N 21/P11/Q1

(a) 1 1
1− + 2
x 4x
(b) 1 + 12 x + 60 x 2 + 160 x 3
(c) their (1× 12 ) + their ( −1× 60 ) + their ( 14 × 160 )

[12 − 60 + 40 =] −8
6 O/N 21/P13/Q2
(a) 1 + 6ax + 15a 2 x 2
(b) their 15a 2 ± ( 3 × their 6a )

15a 2 − 18a = −3
( 3)( a − 1)( 5a − 1) [ = 0]
1
a = 1,
5

175
Unit 6.1: Answer Section

7 M/J 21/P12/Q4
[Coefficient of x or p =] 480
2
 1  3 k 
 Term in x or q =  [10 ×] ( 2 x )  x 2 
   
[10 × 23k2 =] 80k2
p = 6q used (480 = 6 × 80k2 or 80 = 80k2)
[k2 = 1 ⇒] k = ±1

8 M/J 21/P11/Q3 (b) (4 + x)2 = 16 + 8x + x2


(a) 243 Coefficient of x2 is 16 × 1080 + 8 × (−810) + 243
−810x
11043
+1080x2

9 M/J 21/P13/Q7

(a) ( a − x )6 = a 6 − 6a5 x + 15a 4 x 2 − 20a3 x3 + …


 2 
(b)
 ax 
4 2
(
 1 +  …15a x − 20a x + … leading to  x  15a − 40a
3 3 2 4 2
) ( )
15a 4 − 40a 2 = −20 leading to 15a 4 − 40a 2 + 20 [ = 0]

( 5a 2
)(
− 10 3a 2 − 2 ) [ = 0]
2
a = ± 2, ±
3

10 O/N 20/P12/Q1
Coefficient of x³ in (1 − 2 x ) is −80
5

Coefficient of x² in (1 − 2 x ) is 40
5

Coefficient of x³ in (1 + kx )(1 − 2 x ) is 40k −80 = 20


5

5
(k =)
2
11 O/N 20/P11/Q5
2 3
   a 
(a) 6C 2 ×  2 x  ×  a 
( ) ( )
4 3
2
, 6C 3 ×  2 x  × 2 (b) 0
  ( x)     ( x ) 
   
15 × 2 4 × a 2 = 20 × 23 × a 3

15 × 24 3
a= 3
=
20 × 2 2

12 O/N 20/P13/Q5

 7C1a 6b ( x )  ,
   ( )
 7C 2 a 5 b 2 x 2  ,
 ( )
[7C 4a 3b 4 x 4 ]

7C 2 a 5 b 2 x 2( ) = 7C 4 a b 3 4
(x )
4


21a 5b 2 35a 3b 4
=
7C1a 6b ( x ) 7C 2 a 5 b 2 (x )
2
7 a 6b 21a 5b 2
a 5
=
b 9

176
Unit 6.1: Answer Section
13 M/J 20/P12/Q1

(a)
( 2 + 3x ) ( x − 2x )
6

2
 −2 
Term in x² in ( x − 2 6
x ) = 15 x ×   4

 x 
Coefficient = 60
3
 −2 
Constant term in ( x − 2x ) = 20 x 3 ×   (−160)
6
(b)
 x 

Coefficient of x² in ( 2 + 3x ) ( x − 2x ) = 120 – 480 = −360


6

14 M/J 20/P11/Q2
5 8
 1  2
 kx +  +  1 − 
 x  x
5
 1
Coefficient in  kx +  = 10 × k²
 x
(B1 for 10. B1 for k²)
8
 2
Coefficient in  1 −  = 8 × −2
 x

10k² − 16 = 74 → k = 3

15 M/J 20/P13/Q4
2 3
(a) 1 + 5a + 10 a + 10 a + ...
( ) ( ) ( ) + ... SOI
2 3
(b) 1 + 5 x + x 2 + 10 x + x 2 + 10 x + x 2

1 + 5 ( x + x ) + 10 ( x
2 2
+ 2 x3 + ...) + 10 ( x + ...) + ... SOI
3

1 + 5 x + 15 x 2 + 30 x 3 + ...

16 MAR 20/P12/Q6
2
(a)  a  3
5C2  ( )   2 
 2 x  (b)  their a 
4
 x 
  ( ) 5C4  2 ( x )   2 
( )
 x 

x  1
3
10 × 8 × a 2  4  = 720  
x   x 810 identified

a = ±3
17 O/N 19/P12/Q1
6x x2
, 15 ×
2 4
× (4 + ax) → 3a + 15 = 3
a = −4
18 O/N 19/P11/Q1

1
6C2 × ( 2 x ) ×
4

(4 x 2 ) 2
1
15 × 24 ×
42
15

177
Unit 6.1: Answer Section
19 O/N 19/P13/Q1
( ) ( )
2
2
(ii) 1 + 6 px − 2 x 2 + 15 px − 2 x 2
(i) 1 + 6 y + 15y
(15 p 2
) ( )
− 12 ( x 2 ) = 48 x 2
p=2
20 M/J 19/P12/Q1
5 2
2  2
For  − 3x  term in x is 10 or 5C3 or 5C2 ×   × (−3x)³ or
x   x
5 3 2
 2  5.4.3  3 2  5 5.4  2 
   − x  or ( −3x )  
 x  3!  2  2!  3x 2 

−1080 identified
21 M/J 19/P11/Q1 2
k
Term, in x² is ( 2 x ) ×   × 6C2
4
3 (ii)
3 k  x
(i) Ind term = ( 2 x ) ×   × 6C3
x 15 × 16 × k² = 540 (or 540 x 2 )
= 540 → k = 1½

22 M/J 19/P13/Q2
(i) −5 5
+ 3−
1 5 1
(or −5 x −1 + x −3 − x −5 ) (ii) 1 × 20 + 4 × their ( −5 ) = 0
5
x 8 x 32 x 8 32

23 MAR 19/P12/Q1

5C3 ( − )( px )  soi
3
 
( −1)10 p = −2160 then ÷ and cube root
3

p=6

24 O/N 18/P12/Q1
1
For a correctly selected term in 2
: (3x)4 or 34
x
 2 
×  2  ³ or (2/3)3
 3x 
× 7C3 or 7C4
840
→ 840 or 2
x

25 O/N 18/P13/Q1
7C5 x 2 (−2 / x)5 soi
21 × −32 soi
‒672
26 M/J 18/P12/Q1
6
 x
Coefficient of x² in  2 +  is 6C2×24×(½)2 (x2) (= 60)
 2
Coefficient of x² in ( a + x ) is 5C2×a³ (x2) (= 10a3)
5

→ 60 + 10a³ = 330
a=3

178
27 M/J 18/P11/Q1 Unit 6.1: Answer Section

(i) (1 − 2x )5 = 1 −10x + 40x² (no penalty for extra terms)


(ii) → (1 + ax + 2 x² ) ( 1 −10x +40x²)
3 terms in x² → 40 – 10a + 2
Equate with 12 → a = 3

28 M/J 18/P13/Q2
3
2 −2 
2 5C3 x   SOI
 x 
−80
‒80 Accept
x
29 O/N 17/P12/Q1
EITHER:
Term is 9C3 × 26 × (−¼)3
OR1:
9
 8 x 3 − 1   1 9  1 
9

( ) ( )
3 9 3 9
 2  =  2 8 x − 1 or −  2  1 − 8x
 4x   4x   4x 
1
Term is − 9 × 9C3 × 86
4
OR2:
9

( 2 x )9 1 − 3 
1
 8x  3
9 9  1
Term is 2 × C3 ×  − 
 8
Selected term, which must be independent of x = − 84
30 O/N 17/P13/Q3
(i) 2
3
6C3   ( −3x ) SOI also allowed if seen in an expansion
3

 x
−4320 Identified as answer
4
2
6C2   ( − ) 3x 
2
(ii) SOI clearly identified as critical term
x
15a ×16 × 9 − their 4320 ( = 0)

a=2
31 M/J 17/P12/Q1
(i) Coefficient of x = 80(x)
1 1
(ii) Coefficient of = −40  
x  x
Coefficient of x = (1 × their 80) + (3 ×their − 40) = −40(x)

32 M/J 17/P11/Q1
(3–2 x)6
Coeff of x² = 34 × (−2)² × 6C2 = a
\Coeff of x³ = 3³ × (−2)³ × 6C3 = b
a 9
=−
b 8

179
33 M/J 17/P13/Q1 Unit 6.1: Answer Section

7C1 ×26 × a ( x ) , 7C2 ×25 ×  a ( x ) 


2

7 × 26 2
a= =
21 × 25 3
34 O/N 16/P12/Q4
nx
Term in x =
2
nx 3n
(3 – 2x)(1 + + …) → 7 = −2
2 2
→ n=6
n ( n − 1)  x  2
Term in x² =  
2 2
3n ( n − 1) 2n
Coefficient of x 2 = −
8 2
21
=
4

35 O/N 16/P11/Q2
2
6 1 
8C6 ( 2 x )  3  soi
 2x 
1
28 × 64 × oe (powers and factorials evaluated)
4
448

36 O/N 16/P13/Q2
( + / − ) 20 × 33 ( x3 ) , ( )
10a 3 x 3 soi
−540 + 10a 3 = 100 oe
a=4
37 M/J 16/P12/Q4 6
 3  2
(ii)  2 + 2  x − 
6  x  x
 2
(i) x−  Term in x2 = 6C2(−2)² x2
 x = 60 (x2)
Term is 6C3 × (−2)3 = (−)160
−160 Term independent of x:
= 2 × (their−160) + 3 × (their 60)
−140

38 M/J 16/P11/Q1
6
 3 
x− 
 2x 
 −3 
Term is 6C3 × x³ ×   ³
 2x 
→ −67.5 oe

39 M/J 16/P13/Q1
3
1
( )
2
5C2   3x 2
 x
10 (×1) × 32
90 (x)

180
Unit 6.1: Answer Section
40 O/N 15/P12/Q2
(x + 2k)7
Term in x5 = 21 × 4k² = 84k²
Term in x4 = 35 × 8k³ = 280k³
3
Equate and solve → k = 0.3 or
10
41 O/N 15/P11/Q1
(a + x) 5 = a 5 + 5C1 a 4 x + 5C 2 a 3 x 2 + ... soi
 2 4 3  2
 − × (their 5a ) + (their 10a ) ( x )
 a 
0
42 O/N 15/P13/Q2
 5
  9  2 
[7C2] ×  x   ×  2   soi
 3    x  

21 ×
1 5
3 5
( )  1 
x × 81 4  soi
x 
7

43 M/J 15/P12/Q3

(i) (2 − x)6
Coeff of x² is 240
Coeff of x³ is − 20 × 8 = −160

(ii) (3 x + 1)(2 − x)6


Product needs exactly 2 terms
→ 720 – 160 = 560

44 M/J 15/P11/Q3
(1 − x) 2 (1 + 2 x) 6 .
6
(i) (a) (1 − x) = 1 – 6x + 15x²
6
(b) (1 + 2 x) = 1 + 12x + 60x²
(ii) Product of (a) and (b) with >1 term
→ 60 – 72 + 15 = 3
45 M/J 15/P13/Q3

(i) a 5 − 5a 4 x + 10a 3 x 2 − 10a 2 x 3 + .... (ii) (1 − ax )(..10a 3 x 2 − 10a 2 x 3 ..) = ( x 3 )(−10a 4 − 10a 2 )
− 10a 4 − 10a 2 = −200
a2 = 4 ignore a 2 = −5

46 O/N 14/P12/Q3 a = ±2 cao

(i) (1 + x )5 = 1 + 5x + 10x²

(ii) (1 + px + x 2 )5
(1+) 5(px + x²) + 10(px + x²)2

Coeff of x² = 5 + 10p²
= 95 → p = 3

181
Unit 6.1: Answer Section
47 O/N 14/P11/Q1
7
C1 × 26 × a (=) 7C2 × 25 × a2 soi
 7 × 26  2
a =  =
5 
oe
 21 × 2  3

48 O/N 14/P13/Q1

(15 or 16
) ( )
C 2 × 2 4 × (ax ) , 20 or 6C 3 × 2 3 × (ax )
2 3

15 × 2 4 3
a= =
20 × 2 3 2

49 M/J 14/P12/Q2

(1 + x )(
2 x
2 − x)
4 6
.
1
Term in x² = 15 × 16 × (−4)² = 15
Constant term = 20 × 18 × (−4)³ = −160
Coefficient of x² = −145
50 M/J 14/P11/Q3
 1 
[ 8 C 6 or 28] × [16 or 4 2 ]( x 6 ) ×  6 6 
 (64or 2 )( x ) 
7
51 M/J 14/P13/Q1
5
− 2 
 
 x2


x
3
Term in x is 10 × (x²)² ×  − 2 
 


x 
Coefficient = −80(x)

52 O/N 13/P11/Q1
(i) 64 + 576 x + 2160 x 2

(ii) ( )
576a x 2 + 2160 x 2 = 0 ( )
2160 15
a=− oe (eg − ) or ‒3.75
576 4
53 O/N 13/P13/Q8 (iii) k × (i)
405 soi
(i) 81 (x8) + (ii)
(ii) 10 × 33 (x8) soi leading to their answer 675 (x8)
270 (x8)

54 M/J 13/P12/Q2
6
 1 
 2x − 
 2x 
(i) Coeff of x² = 15×16×(−½)² = 60

(ii) Constant term is 20 × 8x³ ×(−1 ÷ 8x³)


×(1 + x²) needs to consider 2 terms
→ 60 − 20 = 40

182
Unit 6.1: Answer Section
55 M/J 13/P11/Q2
(i) 1 – 6px +15p2x2
(ii) 15p2 × 1 – 6p × –1
3p(5p + 2) = 0
2
p = − oe
5
56 M/J 13/P13/Q4

(i) (2 + ax) 5 = 32 + 80ax + 80a²x²

(ii) × (1 + 2x)
240 = 80a² + 160a
→ a = 1 or a = −3.

57 O/N 12/P12/Q1
7
 2 a
x − 
 x
Term in x5 is 7C3 × (x²)4 × (−a/x)³
This term isolated
Equated to −280 → a = 2.
58 O/N 12/P11/Q4
(i) 2     64  192  240

(ii)  2   coeff of  = 2 240  192


288
59 O/N 12/P13/Q1
(x) 3
7C3 × 24 × –  powers 4 and 3
2
35 seen or implied
–70
60 M/J 12/P12/Q3
Coeff of x³ in (a + x) 5 = 10 × a²
Coeff of x³ in (2 − x) 6 = −160
→ 10a² − 160 = 90
→ a=5

61 M/J 12/P11/Q2
[7C3] × [(2x3)4] × [(–1/x2)3] seen soi
35 × 24 × (–1)3 leading to their answer soi
‒560(x6) as answer

62 M/J 12/P13/Q3
(1 − 2 x) 2 (1 + ax)6
Coeff ofx in (1 + ax) 6 = 6ax
Coeff ofx² in (1 + ax) 6 = 15a²x²

Multiplies by (1 − x4 + 4x²)
2 terms inx 6a − 4 = −1
→a = ½

3 terms inx² 15a² − 24a + 4 = b


→b = −4¼

183
63 O/N 11/P12/Q1 Unit 6.1: Answer Section

(i) (2 − y ) 5 = 32 − 80 y + 80 y 2

(ii) (2 − (2 x − x 2 )) 5 “y” = “ 2 x − x 2 ”
→ 80 + 320 = 400
64 O/N 11/P11/Q1
2
 1 
6C4 × [2( x )]
4
× 2 
( )
 x 
240
65 O/N 11/P13/Q1
1 2
k3 ×  x × 10 (or correct factorials)
3

1
10 × k3 × = 30 ⟹ k = 3
9

66 M/J 11/P12/Q2
(1 − 32 x )6
2
6  ± 3x  135 x 2
(i) Term in x² C2 ×   =
 2  4
3
6  ± 3x  − 540 x 3
Term in x³ C3 ×   =
 2  8

270 x 3 135kx 3
(ii) Term in x³ = −
4 2
→ k = 1.

184
UNIT 6.2 Arithmetic and geometric progressions
1 M/J 22/P12/Q2
The second and third terms of a geometric progression are 10 and 8 respectively.
Find the sum to infinity. [4]

2 M/J 22/P12/Q4
The first, second and third terms of an arithmetic progression are k, 6k and k + 6 respectively.
(a) Find the value of the constant k. [2]
(b) Find the sum of the first 30 terms of the progression. [3]

3 M/J 22/P11/Q2
The thirteenth term of an arithmetic progression is 12 and the sum of the first 30 terms is −15.
Find the sum of the first 50 terms of the progression. [5]

4 M/J 22/P13/Q3
An arithmetic progression has first term 4 and common difference d . The sum of the first n terms of
the progression is 5863.
11726
(a) Show that n − 1d = − 8. [1]
n
(b) Given that the nth term is 139, find the values of n and d , giving the value of d as a fraction. [4]

5 O/N 21/P12/Q5

The first, third and fifth terms of an arithmetic progression are 2 cos x, −6 3 sin x and 10 cos x
respectively, where 21 π < x < π.
(a) Find the exact value of x. [3]
(b) Hence find the exact sum of the first 25 terms of the progression. [3]

6 O/N 21/P12/Q6
The second term of a geometric progression is 54 and the sum to infinity of the progression is 243.
The common ratio is greater than 12 .
Find the tenth term, giving your answer in exact form. [5]
7 O/N 21/P11/Q4
The first term of an arithmetic progression is a and the common difference is −4. The first term
of a geometric progression is 5a and the common ratio is − 14 . The sum to infinity of the geometric
progression is equal to the sum of the first eight terms of the arithmetic progression.
(a) Find the value of a. [4]
The kth term of the arithmetic progression is zero.
(b) Find the value of k. [2]

8 O/N 21/P13/Q4
The first term of an arithmetic progression is 84 and the common difference is −3.

(a) Find the smallest value of n for which the nth term is negative. [2]
It is given that the sum of the first 2k terms of this progression is equal to the sum of the first k terms.
(b) Find the value of k. [3]
9 M/J 21/P12/Q8
3
The first, second and third terms of an arithmetic progression are a, 2 a and b respectively, where
a and b are positive constants. The first, second and third terms of a geometric progression are
a, 18 and b + 3 respectively.
(a) Find the values of a and b. [5]
(b) Find the sum of the first 20 terms of the arithmetic progression. [3]

185
Unit 6.2: Arithmetic & Geometric Progressions
10 M/J 21/P11/Q2
The sum of the first 20 terms of an arithmetic progression is 405 and the sum of the first 40 terms
is 1410.
Find the 60th term of the progression. [5]

11 M/J 21/P11/Q5
The fifth, sixth and seventh terms of a geometric progression are 8k, −12 and 2k respectively.
Given that k is negative, find the sum to infinity of the progression. [4]

12 M/J 21/P13/Q9
(a) A geometric progression is such that the second term is equal to 24% of the sum to infinity.
Find the possible values of the common ratio. [3]
(b) An arithmetic progression P has first term a and common difference d . An arithmetic progression
Q has first term 2 a + 1 and common difference d + 1. It is given that
5th term of P 1 Sum of first 5 terms of P 2
= and = .
12th term of Q 3 Sum of first 5 terms of Q 3
Find the value of a and the value of d . [6]

13 O/N 20/P12/Q2

The first, second and third terms of a geometric progression are 2p + 6, −2p and p + 2 respectively,
where p is positive.

Find the sum to infinity of the progression. [5]


14 O/N 20/P12/Q4

The sum, Sn , of the first n terms of an arithmetic progression is given by

Sn = n2 + 4n.
The kth term in the progression is greater than 200.
Find the smallest possible value of k. [5]

15 O/N 20/P11/Q8
A geometric progression has first term a, common ratio r and sum to infinity S. A second geometric
progression has first term a, common ratio R and sum to infinity 2S.

(a) Show that r = 2R − 1. [3]


It is now given that the 3rd term of the first progression is equal to the 2nd term of the second
progression.

(b) Express S in terms of a. [4]

16 O/N 20/P13/Q7
tan2 1

1
cos2 1 cos2 1
The first and second terms of an arithmetic progression are and , respectively, where
0 < 1 < 12 π.
(a) Show that the common difference is −
1
cos4 1
. [4]
(b) Find the exact value of the 13th term when 1 = 16 π. [3]

186
Unit 6.2: Arithmetic & Geometric Progressions
17 M/J 20/P12/Q4
The nth term of an arithmetic progression is 12 3n − 15.
Find the value of n for which the sum of the first n terms is 84. [5]
18 M/J 20/P11/Q1
The sum of the first nine terms of an arithmetic progression is 117. The sum of the next four terms
is 91.
Find the first term and the common difference of the progression. [4]

19 M/J 20/P11/Q3
Each year the selling price of a diamond necklace increases by 5% of the price the year before. The
selling price of the necklace in the year 2000 was $36 000.

(a) Write down an expression for the selling price of the necklace n years later and hence find the
selling price in 2008. [3]
(b) The company that makes the necklace only sells one each year. Find the total amount of money
obtained in the ten-year period starting in the year 2000. [2]

20 M/J 20/P13/Q8
The first term of a progression is sin2 1, where 0 < 1 < 12 π. The second term of the progression is
sin2 1 cos2 1.
(a) Given that the progression is geometric, find the sum to infinity. [3]

It is now given instead that the progression is arithmetic.

(b) (i) Find the common difference of the progression in terms of sin 1 . [3]

(ii) Find the sum of the first 16 terms when 1 = 13 π.


[3]
21 MAR 20/P12/Q8
A woman’s basic salary for her first year with a particular company is $30 000 and at the end of the
year she also gets a bonus of $600.

(a) For her first year, express her bonus as a percentage of her basic salary. [1]
At the end of each complete year, the woman’s basic salary will increase by 3% and her bonus will
increase by $100.
(b) Express the bonus she will be paid at the end of her 24th year as a percentage of the basic salary
paid during that year. [5]

22 O/N 19 /P12/Q8
(a) Over a 21-day period an athlete prepares for a marathon by increasing the distance she runs each
day by 1.2 km. On the first day she runs 13 km.
(i) Find the distance she runs on the last day of the 21-day period [1]
(ii) Find the total distance she runs in the 21-day period. [2]

(b) The first, second and third terms of a geometric progression are x, x − 3 and x − 5 respectively.
(i) Find the value of x. [2]
(ii) Find the fourth term of the progression. [2]
23 O/N 19 /P11/Q4
A runner who is training for a long-distance race plans to run increasing distances each day for 21 days.
She will run x km on day 1, and on each subsequent day she will increase the distance by 10% of the
previous day’s distance. On day 21 she will run 20 km.
(i) Find the distance she must run on day 1 in order to achieve this. Give your answer in km correct
to 1 decimal place. [3]
(ii) Find the total distance she runs over the 21 days. [2]
187
Unit 6.2: Arithmetic & Geometric Progressions
24 O/N 19 /P13/Q9
The first, second and third terms of a geometric progression are 3k, 5k − 6 and 6k − 4, respectively.
(i) Show that k satisfies the equation 7k2 − 48k + 36 = 0. [2]
(ii) Find, showing all necessary working, the exact values of the common ratio corresponding to
each of the possible values of k. [4]
(iii) One of these ratios gives a progression which is convergent. Find the sum to infinity. [2]

25 M/J 19 /P12/Q10
(a) In an arithmetic progression, the sum of the first ten terms is equal to the sum of the next five
terms. The first term is a.
(i) Show that the common difference of the progression is 13 a. [4]
(ii) Given that the tenth term is 36 more than the fourth term, find the value of a. [2]
(b) The sum to infinity of a geometric progression is 9 times the sum of the first four terms. Given
that the first term is 12, find the value of the fifth term. [4]

26 M/J 19 /P11/Q8
(a) The third and fourth terms of a geometric progression are 48 and 32 respectively. Find the sum
to infinity of the progression. [3]
(b) Two schemes are proposed for increasing the amount of household waste that is recycled each
week.
Scheme A is to increase the amount of waste recycled each month by 0.16 tonnes.
Scheme B is to increase the amount of waste recycled each month by 6% of the amount
recycled in the previous month.
The proposal is to operate the scheme for a period of 24 months. The amount recycled in the
first month is 2.5 tonnes.

For each scheme, find the total amount of waste that would be recycled over the 24-month period.
[5]
27 M/J 19 /P13/Q5
Two heavyweight boxers decide that they would be more successful if they competed in a lower
weight class. For each boxer this would require a total weight loss of 13 kg. At the end of week 1
they have each recorded a weight loss of 1 kg and they both find that in each of the following weeks
their weight loss is slightly less than the week before.

Boxer A’s weight loss in week 2 is 0.98 kg. It is given that his weekly weight loss follows an arithmetic
progression.

(i) Write down an expression for his total weight loss after x weeks. [1]
(ii) He reaches his 13 kg target during week n. Use your answer to part (i) to find the value of n. [2]
Boxer B’s weight loss in week 2 is 0.92 kg and it is given that his weekly weight loss follows a
geometric progression.

(iii) Calculate his total weight loss after 20 weeks and show that he can never reach his target. [4]

28 Mar 19 /P12/Q6
(i) The first and second terms of a geometric progression are p and 2p respectively, where p is a
positive constant. The sum of the first n terms is greater than 1000p. Show that 2n > 1001. [2]
(ii) In another case, p and 2p are the first and second terms respectively of an arithmetic progression.
The nth term is 336 and the sum of the first n terms is 7224. Write down two equations in n and
p and hence find the values of n and p. [5]

29 O/N 18 /P12/Q5
The first three terms of an arithmetic progression are 4, x and y respectively. The first three terms of
a geometric progression are x, y and 18 respectively. It is given that both x and y are positive.

(i) Find the value of x and the value of y. [4]


(ii) Find the fourth term of each progression. [3]
188
Unit 6.2: Arithmetic & Geometric Progressions
30 O/N 18 /P11/Q4
The first term of a series is 6 and the second term is 2.

(i) For the case where the series is an arithmetic progression, find the sum of the first 80 terms. [3]
(ii) For the case where the series is a geometric progression, find the sum to infinity. [2]

31 O/N 18 /P13/Q5
In an arithmetic progression the first term is a and the common difference is 3. The nth term is 94
and the sum of the first n terms is 1420. Find n and a. [6]

32 M/J 18 /P12/Q3
A company producing salt from sea water changed to a new process. The amount of salt obtained
each week increased by 2% of the amount obtained in the preceding week. It is given that in the first
week after the change the company obtained 8000 kg of salt.

(i) Find the amount of salt obtained in the 12th week after the change. [3]
(ii) Find the total amount of salt obtained in the first 12 weeks after the change. [2]
33 M/J 18 /P11/Q8
(a) A geometric progression has a second term of 12 and a sum to infinity of 54. Find the possible
values of the first term of the progression. [4]
(b) The nth term of a progression is p + qn, where p and q are constants, and Sn is the sum of the
first n terms.
(i) Find an expression, in terms of p, q and n, for Sn . [3]
(ii) Given that S4 = 40 and S6 = 72, find the values of p and q. [2]
34 M/J 18 /P13/Q3
The common ratio of a geometric progression is 0.99. Express the sum of the first 100 terms as a
percentage of the sum to infinity, giving your answer correct to 2 significant figures. [5]

35 O/N 17 /P12/Q3
(a) Each year, the value of a certain rare stamp increases by 5% of its value at the beginning of the
year. A collector bought the stamp for $10 000 at the beginning of 2005. Find its value at the
beginning of 2015 correct to the nearest $100. [2]
(b) The sum of the first n terms of an arithmetic progression is 12 n 3n + 7. Find the 1st term and the
common difference of the progression. [4]

36 O/N 17 /P11/Q3
(a) A geometric progression has first term 3a and common ratio r. A second geometric progression
has first term a and common ratio −2r. The two progressions have the same sum to infinity.
Find the value of r. [3]
(b) The first two terms of an arithmetic progression are 15 and 19 respectively. The first two terms
of a second arithmetic progression are 420 and 415 respectively. The two progressions have the
same sum of the first n terms. Find the value of n. [3]

189
Unit 6.2: Arithmetic & Geometric Progressions
37 O/N 17 /P13/Q1
An arithmetic progression has first term −12 and common difference 6. The sum of the first n terms
exceeds 3000. Calculate the least possible value of n. [4]

38 M/J 17 /P12/Q7
(a) The first two terms of an arithmetic progression are 16 and 24. Find the least number of terms of
the progression which must be taken for their sum to exceed 20 000. [4]
(b) A geometric progression has a first term of 6 and a sum to infinity of 18. A new geometric
progression is formed by squaring each of the terms of the original progression. Find the sum to
infinity of the new progression. [4]

39 M/J 17 /P11/Q4
(a) An arithmetic progression has a first term of 32, a 5th term of 22 and a last term of −28. Find the
sum of all the terms in the progression. [4]
(b) Each year a school allocates a sum of money for the library. The amount allocated each year
increases by 2.5% of the amount allocated the previous year. In 2005 the school allocated $2000.
Find the total amount allocated in the years 2005 to 2014 inclusive. [3]
40 M/J 17 /P13/Q2
The common ratio of a geometric progression is r. The first term of the progression is r 2 − 3r + 2
and the sum to infinity is S.
(i) Show that S = 2 − r. [2]
(ii) Find the set of possible values that S can take. [2]

41 O/N 16 /P12/Q8
(a) A cyclist completes a long-distance charity event across Africa. The total distance is 3050 km.
He starts the event on May 1st and cycles 200 km on that day. On each subsequent day he
reduces the distance cycled by 5 km.
(i) How far will he travel on May 15th? [2]
(ii) On what date will he finish the event? [3]

(b) A geometric progression is such that the third term is 8 times the sixth term, and the sum of the
first six terms is 31 21 . Find
(i) the first term of the progression, [4]
(ii) the sum to infinity of the progression. [1]
42 O/N 16 /P11/Q5
The sum of the 1st and 2nd terms of a geometric progression is 50 and the sum of the 2nd and 3rd
terms is 30. Find the sum to infinity. [6]

43 O/N 16 /P13/Q9
(a) Two convergent geometric progressions, P and Q, have the same sum to infinity. The first and
second terms of P are 6 and 6r respectively. The first and second terms of Q are 12 and −12r
respectively. Find the value of the common sum to infinity. [3]
(b) The first term of an arithmetic progression is cos 1 and the second term is cos 1 + sin2 1, where
0 ≤ 1 ≤ 0. The sum of the first 13 terms is 52. Find the possible values of 1. [5]
44 O/N 16 /P12/Q9
Relative to an origin O, the position vectors of the points A, B and C are given by
` a ` a ` a
−−→ 2 −−→ −2 −−→ 2
OA = −2 , OB = 3 and OC = 6 .
−1 6 5

(i) Use a scalar product to find angle AOB. [4]


−−→
(ii) Find the vector which is in the same direction as AC and of magnitude 15 units. [3]
−−→ −−→ −−→
(iii) Find the value of the constant p for which p OA + OC is perpendicular to OB. [3]

190
Unit 6.2: Arithmetic & Geometric Progressions
45 M/J 16 /P11/Q9
(a) The first term of a geometric progression in which all the terms are positive is 50. The third term
is 32. Find the sum to infinity of the progression. [3]

(b) The first three terms of an arithmetic progression are 2 sin x, 3 cos x and sin x + 2 cos x
respectively, where x is an acute angle.

(i) Show that tan x = 43 . [3]


(ii) Find the sum of the first twenty terms of the progression. [3]

46 M/J 16 /P13/Q4
The 1st, 3rd and 13th terms of an arithmetic progression are also the 1st, 2nd and 3rd terms respectively
of a geometric progression. The first term of each progression is 3. Find the common difference of
the arithmetic progression and the common ratio of the geometric progression. [5]
47 O/N 15 /P11/Q8
The first term of a progression is 4x and the second term is x2 .
(i) For the case where the progression is arithmetic with a common difference of 12, find the possible
values of x and the corresponding values of the third term. [4]
(ii) For the case where the progression is geometric with a sum to infinity of 8, find the third term.
[4]
48 O/N 15 /P13/Q6
A ball is such that when it is dropped from a height of 1 metre it bounces vertically from the ground
to a height of 0.96 metres. It continues to bounce on the ground and each time the height the ball
reaches is reduced. Two different models, A and B, describe this.
Model A : The height reached is reduced by 0.04 metres each time the ball bounces.
Model B : The height reached is reduced by 4% each time the ball bounces.

(i) Find the total distance travelled vertically (up and down) by the ball from the 1st time it hits the
ground until it hits the ground for the 21st time,
(a) using model A, [3]
(b) using model B. [3]

(ii) Show that, under model B, even if there is no limit to the number of times the ball bounces, the
total vertical distance travelled after the first time it hits the ground cannot exceed 48 metres.
[2]
49 M/J 15 /P12/Q8
(a) The first, second and last terms in an arithmetic progression are 56, 53 and −22 respectively.
Find the sum of all the terms in the progression. [4]

(b) The first, second and third terms of a geometric progression are 2k + 6, 2k and k + 2 respectively,
where k is a positive constant.
(i) Find the value of k. [3]
(ii) Find the sum to infinity of the progression. [2]

50 M/J 15 /P11/Q7
(a) The third and fourth terms of a geometric progression are 1 and 2 respectively. Find the sum to
3 9
infinity of the progression. [4]

(b) A circle is divided into 5 sectors in such a way that the angles of the sectors are in arithmetic
progression. Given that the angle of the largest sector is 4 times the angle of the smallest sector,
find the angle of the largest sector. [4]

191
Unit 6.2: Arithmetic & Geometric Progressions
51 M/J 15 /P13/Q9
(qa) The first term of an arithmetic progression is −2222 and the common difference is 17. Find the
value of the first positive term. [3]

(b) The first term of a geometric progression is ï3 and the second term is 2 cos 1, where 0 < 1 < 0.
Find the set of values of 1 for which the progression is convergent. [5]

52 M/J 14 /P12/Q8
(a) The sum, Sn , of the first n terms of an arithmetic progression is given by Sn = 32n − n2 . Find the
first term and the common difference. [3]

(b) A geometric progression in which all the terms are positive has sum to infinity 20. The sum of
the first two terms is 12.8. Find the first term of the progression. [5]

53 M/J 14 /P11/Q7
(i) A geometric progression has first term a (a ≠ 0), common ratio r and sum to infinity S. A second
geometric progression has first term a, common ratio 2r and sum to infinity 3S. Find the value
of r. [3]

(ii) An arithmetic progression has first term 7. The nth term is 84 and the (3n)th term is 245. Find
the value of n. [4]

54 M/J 14 /P13/Q4
Three geometric progressions, P, Q and R, are such that their sums to infinity are the first three terms
respectively of an arithmetic progression.

Progression P is 2, 1, 12 , 14 ,  .
Progression Q is 3, 1, 13 , 19 ,  .

(i) Find the sum to infinity of progression R. [3]

(ii) Given that the first term of R is 4, find the sum of the first three terms of R. [3]

55 M/J 14 /P12/Q6

The 1st, 2nd and 3rd terms of a geometric progression are the 1st, 9th and 21st terms respectively
of an arithmetic progression. The 1st term of each progression is 8 and the common ratio of the
geometric progression is r, where r ≠ 1. Find
(i) the value of r, [4]
(ii) the 4th term of each progression. [3]

56 M/J 14 /P11/Q5
An arithmetic progression has first term a and common difference d. It is given that the sum of the
first 200 terms is 4 times the sum of the first 100 terms.
(i) Find d in terms of a. [3]
(ii) Find the 100th term in terms of a. [2]

57 M/J 14 /P13/Q2
The first term in a progression is 36 and the second term is 32.

(i) Given that the progression is geometric, find the sum to infinity. [2]

(ii) Given instead that the progression is arithmetic, find the number of terms in the progression if
the sum of all the terms is 0. [3]

192
Unit 6.2: Arithmetic & Geometric Progressions
58 O/N 13 /P12/Q7
(a) An athlete runs the first mile of a marathon in 5 minutes. His speed reduces in such a way that
each mile takes 12 seconds longer than the preceding mile.
(i) Given that the nth mile takes 9 minutes, find the value of n. [2]
(ii) Assuming that the length of the marathon is 26 miles, find the total time, in hours and
minutes, to complete the marathon. [2]

(b) The second and third terms of a geometric progression are 48 and 32 respectively. Find the sum
to infinity of the progression. [4]

59 O/N 13 /P11/Q9
(a) In an arithmetic progression the sum of the first ten terms is 400 and the sum of the next ten
terms is 1000. Find the common difference and the first term. [5]

(b) A geometric progression has first term a, common ratio r and sum to infinity 6. A second
geometric progression has first term 2a, common ratio r2 and sum to infinity 7. Find the values
of a and r. [5]

60 O/N 13 /P13/Q5
(a) In a geometric progression, the sum to infinity is equal to eight times the first term. Find the
common ratio. [2]

(b) In an arithmetic progression, the fifth term is 197 and the sum of the first ten terms is 2040. Find
the common difference. [4]

61 M/J 13 /P12/Q10
(a) The first and last terms of an arithmetic progression are 12 and 48 respectively. The sum of the
first four terms is 57. Find the number of terms in the progression. [4]

(b) The third term of a geometric progression is four times the first term. The sum of the first six
terms is k times the first term. Find the possible values of k. [4]
62 M/J 13 /P11/Q4
The third term of a geometric progression is −108 and the sixth term is 32. Find
(i) the common ratio, [3]
(ii) the first term, [1]
(iii) the sum to infinity. [2]

63 M/J 13 /P13/Q9
2
(AAa) In an arithmetic progression, the sum, Sn , of the first n terms is given by Sn = 2n + 8n. Find the
first term and the common difference of the progression. [3]

(b) The first 2 terms of a geometric progression are 64 and 48 respectively. The first 3 terms of the
geometric progression are also the 1st term, the 9th term and the nth term respectively of an
arithmetic progression. Find the value of n. [5]

64 O/N 12 /P12/Q8
(AAa) In a geometric progression, all the terms are positive, the second term is 24 and the fourth term
is 13 21 . Find
(i) the first term, [3]
(ii) the sum to infinity of the progression. [2]

(b) A circle is divided into n sectors in such a way that the angles of the sectors are in arithmetic
progression. The smallest two angles are 3◦ and 5◦ . Find the value of n. [4]

193
Unit 6.2: Arithmetic & Geometric Progressions
65 O/N 12 /P11/Q1
The first term of an arithmetic progression is 61 and the second term is 57. The sum of the first
n terms is n. Find the value of the positive integer n. [4]

66 O/N 12 /P13/Q5
The first term of a geometric progression is 5 13 and the fourth term is 2 41 . Find
(i) the common ratio, [3]

(ii) the sum to infinity. [2]

67 M/J 12 /P12/Q7
(a) In an arithmetic progression, the sum of the first n terms, denoted by Sn , is given by

Sn = n2 + 8n.
Find the first term and the common difference. [3]

(b) In a geometric progression, the second term is 9 less than the first term. The sum of the second
and third terms is 30. Given that all the terms of the progression are positive, find the first term.
[5]
68 M/J 12 /P11/Q7

(a) The first two terms of an arithmetic progression are 1 and cos2 x respectively. Show that the sum
of the first ten terms can be expressed in the form a − b sin2 x, where a and b are constants to be
found. [3]

(b) The first two terms of a geometric progression are 1 and 31 tan2 θ respectively, where 0 < θ < 12 π .
(i) Find the set of values of θ for which the progression is convergent. [2]
(ii) Find the exact value of the sum to infinity when θ = 16 π . [2]

69 M/J 12 /P13/Q6
The first term of an arithmetic progression is 12 and the sum of the first 9 terms is 135.
(i) Find the common difference of the progression. [2]
The first term, the ninth term and the nth term of this arithmetic progression are the first term, the
second term and the third term respectively of a geometric progression.

(ii) Find the common ratio of the geometric progression and the value of n. [5]

70 O/N 11 /P12/Q10
(a) An arithmetic progression contains 25 terms and the first term is −15. The sum of all the terms
in the progression is 525. Calculate
(i) the common difference of the progression, [2]
(ii) the last term in the progression, [2]
(iii) the sum of all the positive terms in the progression. [2]

(b) A college agrees a sponsorship deal in which grants will be received each year for sports
equipment. This grant will be $4000 in 2012 and will increase by 5% each year. Calculate
(i) the value of the grant in 2022, [2]
(ii) the total amount the college will receive in the years 2012 to 2022 inclusive. [2]

194
Unit 6.2: Arithmetic & Geometric Progressions
71 O/N 11 /P11/Q6
(a) The sixth term of an arithmetic progression is 23 and the sum of the first ten terms is 200. Find
the seventh term. [4]

(b) A geometric progression has first term 1 and common ratio r. A second geometric progression
has first term 4 and common ratio 14 r. The two progressions have the same sum to infinity, S.
Find the values of r and S. [3]

72 O/N 11 /P13/Q2
The first and second terms of a progression are 4 and 8 respectively. Find the sum of the first 10 terms
given that the progression is
(i) an arithmetic progression, [2]
(ii) a geometric progression. [2]
73 M/J 11 /P12/Q10
(a) A circle is divided into 6 sectors in such a way that the angles of the sectors are in arithmetic
progression. The angle of the largest sector is 4 times the angle of the smallest sector. Given
that the radius of the circle is 5 cm, find the perimeter of the smallest sector. [6]

(b) The first, second and third terms of a geometric progression are 2k + 3, k + 6 and k, respectively.
Given that all the terms of the geometric progression are positive, calculate
(i) the value of the constant k, [3]
(ii) the sum to infinity of the progression. [2]

74 M/J 11 /P11/Q8
A television quiz show takes place every day. On day 1 the prize money is $1000. If this is not won
the prize money is increased for day 2. The prize money is increased in a similar way every day until
it is won. The television company considered the following two different models for increasing the
prize money.
Model 1: Increase the prize money by $1000 each day.
Model 2: Increase the prize money by 10% each day.

On each day that the prize money is not won the television company makes a donation to charity. The
amount donated is 5% of the value of the prize on that day. After 40 days the prize money has still
not been won. Calculate the total amount donated to charity
(i) if Model 1 is used, [4]
(ii) if Model 2 is used. [3]

75 M/J 11 /P13/Q6
(a) A geometric progression has a third term of 20 and a sum to infinity which is three times the first
term. Find the first term. [4]

(b) An arithmetic progression is such that the eighth term is three times the third term. Show that
the sum of the first eight terms is four times the sum of the first four terms. [4]

195
UNIT 6.2 Unit 6.2: Answer Section May/June 2022

Question 1 M/J 22/P12/Q2 Answer Marks Guidance

1 r = 0.8 B1 OE
a = 12.5 B1 OE

S  12.5  1  0.8  M1 a
Using with ‘their a’ and ‘their r’ but r must be  1 .
1 r

125 1 A1 1
S  , 62 or 62.5 12
2 2 2 or similar does not get A1.
1
4
5
2 M/J 22/P12/Q4
2(a) 2  6k  k  k  6 or 6k  k  k  6  6k B1 OE
or 2d = 6 leading to d = 3, ⸫ 6k – 3 = k A correct equation in k only. Can be implied by correct final
answer.

6 B1 OE
k or 0.6
10

2(b) d=3 B1 Correct value of d can be implied by a correct final answer.


Working may be seen in part (a) but must be used in (b).

30 M1 It needs to be clear that the candidate is using a correct sum


S30   2  ‘their k’  29  ‘their d’ formula. There is no requirement to check the candidates working
2
for d but it must be clearly identified.

S30 = 1323 A1 ISW if corrected to 1320.

196
9709/11 Cambridge International AS & A Level – Mark Scheme Unit 6.2: Answer Section May/June 2022
PUBLISHED
Question 3 M/J 22/P11/Q2 Answer Marks Guidance

3 a  12 d  12 B1 For correct equation.

30 B1 n
2
 2a   30  1 d   15 For correct equation in a and d. If using
2
 a  l  , must
replace l with an expression involving a and d.

a = 72, d = 5 B1 Both values correct SOI.

50 M1 Using sum formula with their a and d values obtained


S50 
2
 2  their a   49  their d   via a valid method.

S50 = 2525 A1 5

9709/13 4 M/J 22/P13/Q3 Cambridge International AS & A Level – Mark Scheme May/June 2022

4(a) n B1 Must show a useful intermediate step.


8   n  1 d   5863 leading to n 8   n  1 d   11726
2 WWW AG.
11726
leading to  n  1 d  8
n 1

4(b) 11726 *M1 OE Use of correct un formula with expression from (a) or
4   n  1 d  139 leading to  8  135 Sn formula to eliminate d.
n
11726 A1
n= = 82
143
11726 DM1 Substitute their n into a correct un or Sn formula
81d  8
82
5 A1 138
d Accept OE fraction only
3 81
If M0 DM0 scored them SC B1 B1 for correct n and d
values only.

197
9709/12 Cambridge International AS & A Level – Mark Scheme Unit 6.2: Answer Section October/November 2021
PUBLISHED
Question 5 O/N 21/P12/Q5 Answer Marks Guidance

5(a) [(3rd term – 1st term) = (5th term – 3rd term) leading to…] *M1 OE. From the given terms, obtain 2 expressions relating to
−6 3 sin x − 2cos x = 10cos x + 6 3 sin x the common difference of the arithmetic progression, attempt
to solve them simultaneously and achieve an equation just
leading to − 12 3 sin x = 12cos x 
  involving sinx and cosx.
OR
[(1st term + 5th term) = 2 × 3rd term leading to…] 12cos x = −12 3 sin x

Elimination of sinx and cosx to give an expression in tanx DM1 sinx


For use of = tanx
 1  cosx
 tanx = − 
 3

5π A1 CAO. Must be exact.


[ x =] only
6

5(b) d = 2cosx or d = 2cos(their x) B1 FT Or an equivalent expression involving sinx and cosx e.g.
−3 3sin ( their x ) − cos ( their x )  = − 3 

FT for their x from (a) only. If not ± 3 , must see


unevaluated form.

25 M1 25
S25 =
2
(
2 × ( 2cos ( their x ) ) + ( 25 − 1) × ( their d ) ) Using the correct sum formula with
2
, (25 — 1) and with

( ( )
 = 12.5 2 × − 3 + 24 − 3 
 ( ))
a replaced by either 2(cos(their x)) or ± 3 and
d replaced by either 2(cos(their x)) or ± 3 .

−325 3 A1 Must be exact.

198
9709/12 Cambridge International Unit 6.2: Answer Section October/November 2021

Question 6 O/N 21/P12/Q6 Answer Marks Guidance

6 a or their a B1 SOI
ar = 54 and = 243
1− r

54 *M1 Forming a 3-term quadratic expression in r or a using their


= 243 (1 − r ) leading to 243r 2 − 243r + 54 [ = 0] [ 9r 2 − 9r + 2 = 0] 2nd term and S∞. Allow ± sign errors.
r
OR a 2 − 243a + 13122 [ = 0]

k ( 3r − 2 )( 3r − 1) [ = 0] OR ( a − 81)( a − 162 ) [ = 0] DM1 Solving their 3-term quadratic using factorisation, formula or
completing the square. If factorising, factors must expand to
give ±their coefficient of r 2 .

 2 DM1 May be implied by final answer.


54 ÷  their  = a OR 54 ÷ ( their 81) = r
 3

512  A1 OE. Must be exact.


2 
9 8
2
Tenth term =  OR 81 ×   OR 54 ×    Special case: If B1M1DM0DM1 scored then SC B1 can be
243  3  3   awarded for the correct final answer.

7 O/N 21/P11/Q4 5

9709/11 7(a) 5a B1 Use of correct formula for sum to infinity.


1 − ( ± 14 )

8 *M1 Use of correct formula for sum of 8 terms and form


 2a + 7 ( −4 ) 
2 equation; allow 1 error.

4a = 8a − 112 leading to a = [ 28] DM1 Solve equation to reach a value of a.

a = 28 A1 Correct value.

7(b) their 28 + ( k − 1)( −4 ) = 0 M1 Use of correct method with their a.

[ k =] 8 A1

2
199
9709/13 Unit 6.2: Answer Section October/November 2021

Question 8 O/N 21/P13/Q4 Answer Marks Guidance

8(a) 84 − 3 ( n − 1) = 0 M1 OE, SOI. Allow either = 0 or < 0 (to -3).

Smallest n is 30 A1 SC B2 for answer only n = 30 WWW.

8(b)  2k  k M1 A1 M1 for forming an equation using correct formula.


  168 + ( 2k − 1)( −3)  =   168 + ( k − 1)( −3)  A1 for at least one side correct.
 2  2

k = 19 A1

9709/12 May/June 2021 9 M/J 21/P12/Q8

9(a)  3  B1 SOI
 a + b = 2 × a   b = 2a
 2 

182 = a(b + 3) OE or 2 correct statements about r from the GP, B1 SOI


18 b+3
e.g. r = and b + 3 = 18r or r 2 =
a a

324 = a(2a + 3) ⇒ 2a2 + 3a – 324[= 0] M1 Using the correct connection between AP and GP to form a
or 3-term quadratic with all terms on one side.
b2 + 3b – 648[= 0]
or
6r2 – r – 12[= 0]
or
4d2 + 3d – 162[= 0]

(a – 12)(2a + 27)[= 0] M1 Solving their 3-term quadratic by factorisation, formula or


or completing the square to obtain answers for a, b, r or d.
( b − 24 )( b + 27 ) [ = 0]
or
( 2r − 3)( 3r + 4 ) [ = 0]
or
( d − 6 )( 4d + 27 ) [ = 0]
a = 12, b = 24 A1 WWW. Condone extra ‘solution’ a = −13.5, b = −27 only.

5
200
9709/12 Cambridge International Unit 6.2: Answer Section May/June 2021

Question Answer Marks Guidance

9(b) Common difference d = 6 B1 FT a


SOI. FT their
2

20 M1 Using correct sum formula with


S20 = ( 2 × 12 + 19 × 6 )
2 their a, their calculated d and 20.

1380 A1

10 M/J 21/P11/Q2 May/June 2021


9709/11

10 10(2 a + 19d) = 405 B1

20(2a + 39d) = 1410 B1

Solving simultaneously two equations obtained from using the correct sum M1 Reach a = or d =
formulae [a = 6, d = 1.5]

Using the correct formula for 60th term with their a and d M1

60th term = 94.5 A1 189


OE, e.g.
2

201
9709/11 11 M/J 21/P11/Q5 Cambridge International Unit 6.2: Answer Section May/June 2021

Question Answer Marks Guidance

11 −12)2 = 8k × 2k M1 Forming an equation in k

k = −3 A1

Using correct formula for S∞ [r = 0.5, a = −384] M1 With −1 < r < 1

S∞ = −768 A1

Alternative method for Question 5

2k M1
r2 =
8k

r = [ ± ] 0.5 A1

Using correct formula for S∞ [r = 0.5, a = −384] M1 −1 < r < 1

S∞ = −768 4 A1

9709/13 12 M/J 21/P13/Q9 Cambridge International AS & A Level – Mark Scheme May/June 2021

12(a) 24 a M1 Form an equation using a numerical form of the


ar = × percentage and correct formula for u2 and S∞
100 1 − r

100r 2 − 100r + 24 [ = 0] A1 OE. All 3 terms on one side of an equation.

2 3 A1 Dependent on factors or formula seen from their


( 20r − 8)( 5r − 3) [ = 0] → r= , quadratic.
5 5

202
9709/13 Cambridge International Unit 6.2: Answer Section May/June 2021

Question Answer Marks Guidance

12(b) 3 × {( a + 4 d )} = {( 2 ( a + 1) + 11( d + 1) )} *M1 SOI Attempt to cross multiply with contents of


at least one { } correct

Simplifies to a + d = 13 A1

5 5 *M1 SOI Attempt to cross multiply with contents of


   
{ }
 2  × 3{( 2a + 4d )} =  2  × 2 ( 4 ( a + 1) + 4 ( d + 1) ) at least one { } correct

Simplifies to −a + 2d = 8 A1

Solve 2 linear equations simultaneously DM1 Elimination or substitution expected

d = 7, a = 6 A1 SC B1 for a=6, d=7 without complete working

9709/12 13 O/N 20/P12/Q2 Cambridge International AS & A Level – Mark Scheme October/November 2020

13 −2 p p+2 M1 OE. Using “a, b, c then b² = ac” or a =2p+6 , ar = −2p and


(−2p)² = (2p + 6) × (p + 2) or = ar² = p + 2 to form a correct relationship in terms of p only
2 p + 6 −2 p

( 2 p ² − 10 p − 12 = 0 ) p = 6 A1

a = 18 and r = −⅔ A1

(s∞ ) = their a ÷ (1 − their r) M1 Correct formula used with their values for a and r , r < 1
 5 Both a & r from the same value of p.
 = 18 ÷ 
 3

(s∞ = )10.8 A1 OE. A0 if an extra solution given

2p + 6 2p + 6
SC B2 for s∞ = or ignore any
−2 p p+2
1− 1−
2p + 6 −2 p
subsequent algebraic simplification.

5
203
9709/12 Cambridge International Unit 6.2: Answer Section October/November 2020

Question 14 O/N 20/P12/Q4 Answer Marks Guidance

14 S x and S x +1 M1 Using two values of n in the given formula

a = 5, d = 2 A1 A1

a + (n – 1) d > 200 → 5 + 2(k – 1) > 200 M1 Correct formula used with their a and d to form an equation
or inequality with 200, condone use of n

(k =) 99 A1 Condone ⩾ 99

Alternative method for question 4

n d 1  M1 Equating two correct expressions of S n and equating


( 2a + (n – 1) d ) ≡ n2 + 4n →  = 1 , a − d = 4 
2 2 2  coefficients of n and n2

d = 2, a = 5 A1 A1

a + (n – 1) d > 200 → 5 + 2(k – 1) > 200 M1 Correct formula used with their a and d to form an equation
or inequality with 200, condone use of n

(k =) 99 A1 Condone ⩾ 99

Alternative method for question 4

sumk − sumk −1 → k 2 + 4 k − ( k − 1) 2 − 4 ( k − 1) M1 A1 Using given formula with consecutive expressions


subtracted. Allow k+1 and k.

2k + 3 > 200 or = 200 M1 A1 Simplifying to a linear equation or inequality

(k =) 99 A1 Condone ⩾ 99

204
9709/11 Unit 6.2: Answer Section October/November 2020

Question 15 O/N 20/P11/Q8 Answer Marks Guidance

15(a) a a B1
S= , 2S = SOI at least one correct
1− r 1− R

2a a M1
= SOI
1− r 1− R

2 − 2R = 1 − r → r = 2R − 1 A1 AG

15(b) ar 2 = aR → ( a )( 2 R − 1) = R ( a )
2 *M1

4 R 2 − 5 R + 1 ( = 0 ) → ( 4 R − 1)( R − 1) ( = 0 ) DM1 Allow use of formula or completing square.

1 A1
R= Allow R = 1 in addition
4

2a A1
S=
3

Alternative method for question 8(b)

ar 2 = aR → ( a ) r 2 = ½ ( r + 1)( a ) *M1 Eliminating 1 variable

DM1 Allow use of formula or completing square. Must solve a


2r 2 − r − 1 ( = 0 ) → ( 2r + 1)( r − 1) ( = 0 )
quadratic.

1 A1
r=− Allow r = 1 in addition
2

2a A1
S=
3

205
Unit 6.2: Answer Section
9709/13 October/November 2020

Question 16 O/N 20/P13/Q7 Answer Marks Guidance

16(a) tan 2θ 1 B1 Allow sign error(s). Award only at form (d =)... stage
( d =) − −
cos θ cos 2θ
2

sin 2θ 1 −sec2θ M1 Allow sign error(s). Can imply B1


− − or
cos θ cos 2θ
4
cos 2θ

1 M1
2 2 −
−sin θ − cos θ cos 2θ
or
cos 4θ cos 2θ

1 A1 AG, WWW

cos 4θ

16(b) 4 16 B1 1 1
a= , d =− SOI, both required. Allow a = ,d= −
3 9 3 9
4 16

1 12 4  −16  M1 Use of correct formula with their a and their d. The first 2 steps
u13 = − = + 12   could be reversed
cos θ cos 4θ
2
3  9 

‒20 A1 WWW

206
9709/12 Cambridge International Unit 6.2: Answer Section May/June 2020

Question 17 M/J 20/P12/Q4 Answer Marks

17 1st term is−6, 2nd term is −4.5 M1


(M1 for using kth terms to find both a and d)

→ a = −6, d = 1.5 A1 A1

Sn = 84 → 3n² − 27n – 336 = 0 M1

Solution n = 16 A1

9709/11 18 M/J 20/P11/Q1 Cambridge International AS & A Level – Mark Scheme May/June 2020

18 9 B1
117 = ( 2 a + 8d )
2

Either 91 = S4 with ‘a’ as a + 4d or 117 + 91 = S13 M1M1


(M1 for overall approach. M1 for Sn)

Simultaneous Equations → a = 7, d = 1.5 A1

19 M/J 20/P11/Q3 4

19(a) −1 ⩽ f(x) ⩽ 2 B1 B1

19(b) k=1 B1

Translation by 1 unit upwards parallel to the y-axis B1

19(c) 3 1 B1
y=− cos2 x −
2 2

207
9709/13 Cambridge International Unit 6.2: Answer Section May/June 2020

Question 19 M/J 20/P13/Q8 Answer Marks

19(a) r = cos 2θ SOI M1

sin 2θ M1
S∞ =
1 − cos2θ

1 A1

19(b)(i) d = sin 2θ cos 2θ − sin 2θ M1

(
sin 2θ cos 2θ − 1 ) M1

−sin 4θ A1

19(b)(ii) 16 M1
Use of S16 = [ 2a + 15d ]
2

3 9 A1
With both a = and d = −
4 16

1 A1
S16 = −55
2

208
9709/12 Cambridge International Unit 6.2: Answer Section March 2020

Question Answer Marks Guidance

21(a) 2% B1

21(b) Bonus = 600 + 23× 100 = 2900 B1

Salary = 30000 × 1.0323 M1 Allow 30000 × 1.0324 (60984)

= 59207.60 A1 Allow answers of 3significant figure accuracy or better

their 2900 M1 SOI


their 59200

4.9(0)% A1

22 O/N 19 /P12/Q8 5

22(a)(i) 21st term = 13 + 20 × 1.2 = 37(km) B1

22(a)(ii) S21= ½×21 × (26 + 20 × 1.2) or ½ × 21 × (13 + their 37) M1 A correct sum formula used with correct values for a, d and n.

525 (km) A1

22(b)(i) x−3 x−5 M1 Any valid method to obtain an equation in one variable.
= oe (or use of a, ar and ar2)
x x−3

(a = or x =) 9 A1

209
9709/12 Unit 6.2: Answer Section October/November 2019

22 O/N 19 /P12/Q8
22(b)(ii)  x −3  x−5 x−5 M1 Any valid method to find r and the fourth term with their a & r.
r=   or   or = ⅔. Fourth term = 9 × (⅔)3
 x   x −3 x

2⅔ or 2.67 A1 OE, AWRT

22(b)(iii) a 9 M1 Correct formula and using their ‘r’ and ‘a’, with r <1, to
S∞ = =
1 − r 1 − 23 obtain a numerical answer.

27 or 27.0 A1 AWRT

23 O/N 19 /P11/Q4 2

23(i) Identifies common ratio as 1.1 B1

Use of x (1.1) = 20 M1 SOI


20

  A1 Accept 2.97
20
x=  = 3.0
 (1.1)20 
 

23(ii) (1.1)21 − 1 M1 Correct formula used for M mark.


their 3.0 ×   → 192 A1 Allow 2.97 used from (i)
1.1 − 1 Accept 190 from x = 2.97…

210
Unit 6.2: Answer Section October/November 2019
9709/13

Question 24 O/N 19 /P13/Q9 Answer Marks Guidance

24(i) 5k − 6 6k − 4 M1 OR any valid relationship


= → ( 5k − 6 )2 = 3k ( 6k − 4 )
3k 5k − 6

25k 2 − 60k + 36 = 18k 2 − 12k → 7 k 2 − 48k + 36 A1 AG

24(ii) 6 B1B1 6
k= ,6 Allow 0.857(1) for
7 7

6 2 B1 Must be exact
When k = ,r= −
7 3

4 B1
When k = 6, r =
3

24(iii) a 2 6 M1 Provided 0 < |their ‒2/3| < 1


Use of S∞ = with r = their − and a = 3 × their
1− r 3 7

18  2  54 A1 FT if 0.857(1) has been used in part (ii).


÷ 1 +  = or 1.54
7  3  35

211
9709/12 25 M/J 19 /P12/Q10 Cambridge International AS/A Level – Mark Scheme Unit 6.2: Answer Section

Question Answer Marks Guidance

25(a)(i) S10 = S15 – S10 or S10 = S(11 to 15) M1 Either statement seen or implied.

5(2a + 9d) oe B1

5 A1
7.5(2a + 14d) – 5(2a + 9d) or [ (a + 10d) + (a+14d)] oe
2

a A1 Correct answer from convincing working


d= AG
3

4 a
Condone starting with d = and evaluating both
3
summations as 25a.

25(a)(ii) ( a + 9d ) = 36 + ( a + 3d ) M1 Correct use of a + ( n − 1) d twice and addition of


±36

a = 18 A1

2 Correct answer www scores 2/2

25(b)
S∞ = 9 ×S4;
a
=9
(
a 1− r4 )
or 9(a + ar+ ar2+ar3)
B1 May have 12 in place of a.

1− r 1− r

9(1 – rn) = 1 where n = 3,4 or 5 M1 Correctly deals with a and correctly eliminates
‘1 – r’

8 A1
r4 = oe
9

(5th term =) 10⅔ or 10.7 A1

4 Final answer of 10.6 suggests premature


approximation – award 3/4 www.

212
9709/11 Cambridge International AS/A Level – Mark Scheme Unit 6.2: Answer Section

Question 26 M/J 19 /P11/Q8 Answer Marks Guidance

26(a) ar² = 48, ar³ = 32, r = ⅔ or a = 108 M1 Solution of the 2 eqns to give r (or a). A1 (both)

r = ⅔ and a = 108 A1

108 A1 FT Needs correct formula and r between −1 and 1.


S∞ = = 324
1
3 3
26(b) Scheme Aa = 2.50, d = 0.16 M1 Correct use of either AP Sn formula.
Sn = 12(5 + 23×0.16)

Sn = 104 tonnes. A1

Scheme B a = 2.50, r = 1.06 B1 Correct value of r used in GP.

=
(
2.5 1.0624 − 1) M1 Correct use of either Sn formula.

1.06 − 1

Sn = 127 tonnes. A1

9709/13

Question Answer Marks Guidance

27(i) x B1 Allow ‒ or + 0.02. Allow n used


 2 + ( x − 1)( − / +0.02 )  or 1.01x − 0.01x 2 or 0.99 x + 0.01x 2 oe
2

27(ii) Equate to 13 then either simplify to a 3-term quadratic equation or M1 Expect n2 ‒ 101n + 1300 (=0) or 0.99 x + 0.01x 2 = 13 . Allow x used
find at least 1 solution (need not be correct) to an unsimplified
quadratic

16 A1 Ignore 85.8 or 86

213
28 Mar 19 /P12/Q6
9709/13 Cambridge International AS/A Level – Mark Scheme Unit 6.2: Answer Section

Question Answer Marks Guidance

27(iii) a (1 − r n ) M1
Use of with a = 1, r = 0.92, n = 20 soi
1− r

(=) 10.1 A1

Use of ( S∞ = )
a
with a = 1, r = 0.92
M1 (1) (1 − 0.92n) = 13 → 0.92n = −0.04 oe
OR
1− r 1 − 0.92

S ∞ = 12.5 so never reaches target or < 13 A1 Conclusion required – 'Shown' is insufficient


No solution so never reaches target or < 13

4
9709/12 28 Mar 19 /P12/Q6 March 2019

28(i)
Sn =
(
p 2n − 1 ) soi M1

2 −1

( )
p 2n − 1 > 1000 p → 2 n > 1001 AG A1

28(ii) p + ( n − 1) p = 336 B1 Expect np = 336

n B1 n
 2 p + ( n − 1) p  = 7224 Expect ( p + np ) = 7224
2 2

Eliminate n or p to an equation in one variable M1 Expect e.g. 168(1 + n) = 7224 or 1 + 336/p =43 etc

n = 42, p = 8 A1A1

214
9709/12 Unit 6.2: Answer Section October/November 2018

Question 29 O/N 18 /P12/Q5 Answer Marks Guidance

29(i) From the AP: x − 4 = y − x B1 y+4


Or equivalent statement e.g. y = 2x – 4 or x = .
2

y 18 B1 y2
From the GP: = Or equivalent statement e.g. y2 = 18x or x = .
x y 18

Simultaneous equations: y ² − 9 y − 36 = 0 or 2 x ² − 17 x + 8 = 0 M1 Elimination of either x or y to give a three term quadratic


(= 0)

OR

4 + 2d B1
4+d =x, 4+2d=y → = r oe
4+d
2
4 + 2d  M1 Uses ar2 = 18 to give a three term quadratic (= 0)
( 4 + d )  2
 = 18 → 2d − d − 28 = 0
 4 + d 

d=4 B1 −7
Condone inclusion of d = oe
2

29(i) OR

y 18 B1
From the GP =
x y

y2 y2 y2 B1
→x= → 4+ d = →d = –4
18 18 18

 y2  M1
4 + 2  − 4  = y → y ² − 9 y − 36 = 0
 18 

x = 8, y = 12. A1 1 
Needs both x and y. Condone  , −3  included in final
 2 
answer.
Fully correct answer www 4/4.

4
215
29 O/N 18 /P12/Q5 Unit 6.2: Answer Section
9709/12 October/November 2018

29(ii) AP 4th term =16 B1 −13


Condone inclusion of oe
2

 12 
3 M1 A valid method using their x and y from (i).
GP 4th term = 8 ×  
8
= 27 A1 Condone inclusion of –108

Note: Answers from fortuitous x = 8, y = 12 in (i) can only


score M1.
Unidentified correct answer(s) with no working seen after
valid x = 8, y = 12 to be credited with appropriate marks.

9709/11 30 O/N 18 /P11/Q4 Cambridge International AS/A Level – Mark Scheme October/November 2018

4(i) 80 80 M1A1 Correct formula (M1). Correct a, d and n (A1) .


S80 = 12 + 79 × ( −4 )  or [6 + l ] , l = −310
2  2

–12 160 A1

4(ii) 6 M1A1 Correct formula with r < 1 for M1


S∞ = =9
1
1−
3

9709/13 31 O/N 18 /P13/Q5 Cambridge International AS/A Level – Mark Scheme October/November 2018

31 a + ( n − 1) 3 = 94 B1

n n B1
 2a + ( n − 1) 3 = 1420 OR [ a + 94] = 1420
2 2

Attempt elimination of a or n M1

3n 2 − 191n + 2840 ( = 0 ) OR a 2 − 3a − 598 ( = 0) A1 3-term quadratic (not necessarily all on the same side)

216
9709/13 Cambridge International AS/A Level – Mark Scheme Unit 6.2: Answer Section

31 3n 2 − 191n + 2840 ( = 0 ) OR a 2 − 3a − 598 ( = 0) A1 3-term quadratic (not necessarily all on the same side)

n = 40 (only) A1

a=‒
‒23 (only) A1 Award 5/6 if a 2nd pair of solutions (71/3, 26) is given in
addition or if given as the only answer.

6
9709/12 32 M/J 18 /P12/Q3 Cambridge International AS/A Level – Mark Scheme May/June 2018

32(i) 102 B1 Can be awarded here for use in Sn formula.


r = 1.02 or used in a GP in some way.
100

Amount in 12th week = 8000 (their r)11 M1 8000


8000 Use of arn – 1 with a = 8000 & n = 12 or with a = and n = 13.
or (their a from ) ( their r )12 1.02
their r.

= 9950 (kg) awrt A1 Note: Final answer of either 9943 or 9940 implies M1.
Full marks can be awarded for a correct answer from a list of terms.

32(ii)
In 12 weeks, total is
(
8000 ( their r ) − 1
12
) M1 Use of Sn with a = 8000 and n = 12 or addition of 12 terms.

( ( their r ) − 1)
= 107000 (kg) awrt A1 Correct answer but no working 2/2

217
9709/11 33 M/J 18 /P11/Q8 Cambridge International AS/A Level – Mark Scheme Unit 6.2: Answer Section

Question Answer Marks Guidance

33(a) a B1 B1 CAO, OE CAO, OE


ar = 12 and = 54
1− r

Eliminates a or r → 9r ² − 9r + 2 = 0 or a ² − 54 a + 648 = 0 M1 Elimination leading to a 3-term quadratic in a or r

2 1 A1 Needs both values.


→ r= or hence to a → a = 18 or 36
3 3 4

33(b) nth term of a progression is p + qn

33(b)(i) first term =p + q. Difference = q or last term = p + qn B1 Need first term and, last term or common difference

n n M1A1 Use of Sn formula with their a and d.


Sn =
2
( 2 ( p + q ) + ( n − 1) q ) or ( 2 p + q + nq )
2 3 ok unsimplified for A1.

33(b)(ii) Hence 2 ( 2 p + q + 4q ) = 40 and 3 ( 2 p + q + 6q ) = 72 DM1 Uses their Sn formula from (i)

Solution → p = 5 and q = 2 A1 Note: answers 7, 2 instead of 5, 2 gets M1A0 – must


[Could use Sn with a and d → a = 7, d = 2 → p = 5, q = 2.] attempt to solve for M1

9709/13 34 M/J 18 /P13/Q3 Cambridge International AS/A Level – Mark Scheme May/June 2018
34
(
 a 1− rn

)  [÷]  a 
M1M1 Correct formulae used with/without r = 0.99 or n = 100.

 1− r  1 − r 
 
 

DM1 Sn
Allow numerical a (M1M1). 3rd M1 is for division (or ratio)
S∞
SOI

63 ( a ) A1 Could be shown multiplied by 100(%). Dep. on DM1


1 − 0.99100 SOI OR SOI
100 ( a )

63(%) Allow 63.4 or 0.63 but not 2 infringements (e.g. 0.634, 0.63%) A1 n = 99 used scores Max M3. Condone a = 0.99 throughout
5 S n = S∞ ( without division shown ) scores 2 / 5

218
9709/12 October/Novembe 2017 Unit 6.2: Answer Section

Question 35 O/N 17 /P12/Q3 Answer Marks Guidance

35(a) Usesr = (1.05 or 105%)9, 10 or 11 B1 Used to multiply repeatedly or in any GP formula.

New value = 10000 × 1.0510 = ($)16 300 B1

2
35(b) EITHER: (B1 Uses n = 1 to find a
n=1→5 a=5

n = 2 → 13 B1 Correct Sn for any other value of n (e.g. n = 2)

a + (a + d) = 13 →d=3 M1 A1) Correct method leading to d =

OR: n
n n 36 O/N 17 /P11/Q3   maybe be ignored
  ( 2a + ( n − 1) d ) =   ( 3n + 7 )
2
2 2

(*M1A1 Method mark awarded for equating terms in n from correct Sn


∴ dn + 2a − d = 3n + 7 → dn = 3n →d = 3
formula.

2a − ( their 3) = 7, a=5 DM1 A1) 4

36 O/N 17 /P11/Q3 9709/11 October/November 2017


36(i) 3a a M1 Attempt to equate 2 sums to infinity. At least one correct
=
1 − r 1 + 2r

3 + 6r = 1 − r DM1 Elimination of 1 variable (a) at any stage and multiplication

2 A1
r=−
7

36(ii) ½ n  2 × 15 + ( n − 1) 4 ]= ½ n[ 2 × 420 + ( n − 1)( −5 )  M1A1 Attempt to equate 2 sum to n terms, at least one correct (M1). Both correct
(A1)

n = 91 A1

3
219
9709/13 October/November 2017 Unit 6.2: Answer Section

37 O/N 17 /P13/Q1 Answer Marks Guidance

 24 + ( n − 1) 6 ~ 3000
37 ½ n − M1 Use correct formula with RHS ≈ 3000 (e.g. 3010).
Note: ~ denotes any inequality or equality

( 3) ( n 2 − 5n − 1000 ) ( ~ 0 ) A1 Rearrange into a 3-term quadratic.

n ~34.2 ( & − 29.2) A1

35. Allow n . 35 A1 4

38 M/J 17 /P12/Q7 9709/12 May/June 2017

38(a) n M1 M1 correct formula used with d from 16 + d = 24


(Sn =) 32 + ( n − 1) 8 and 20000
2

A1 A1 for correct expression linked to 20000.

→ n² + 3n – 5000 (<,=,> 0) DM1 Simplification to a three term quadratic.

→ (n = 69.2) → 70 terms needed. A1 Condone use of 20001 throughout.


Correct answer from trial and improvement gets 4/4.

Total: 4

38(b) a M1A1 Correct S∞ formula used to find r.


a = 6, = 18 → r = ⅔
1− r

4 M1 Obtain new values for a and r by any valid method.


New progression a = 36, r = oe
9

36 324 A1 (Be aware that r =−⅔ leads to 64.8 but can only score M marks)
New S∞ = → 64.8 or oe
4 5
1−
9

Total: 4

220
9709/11 Cambridge International AS/A Level – Mark Scheme Unit 6.2: Answer Section

Question 39 M/J 17 /P11/Q4 Answer Marks Guidance

39(a) a = 32, a + 4d = 22, → d = −2.5 B1

a + (n – 1)d = −28 → n = 25 B1

25 M1 A1 M1 for correct formula with n = 24 or n = 25


S25 = ( 64 − 2.5 × 24 ) = 50
2

Total: 4

39(b) a = 2000, r = 1.025 B1 r = 1 + 2.5% ok if used correctly in Sn formula

1.02510 − 1 M1 A1 M1 for correct formula with n = 9 or n = 10 and their a and r


S10 = 2000( ) = 22400 or a value which rounds to this
1.025 − 1
Total: 3 SR: correct answer only for n = 10 B3, for n = 9, B1 (£19 900)

9709/13 40 M/J 17 /P13/Q2 May/June 2017

40(i) r 2 − 3r + 2 M1
S=
1− r

S=
( r − 1)( r − 2 ) = − (1 − r )( r − 2 ) = 2 − r OR A1 AG Factors must be shown. Expressions requiring minus sign taken out must
1− r 1− r be shown
(1 − r )( 2 − r ) = 2 − r OE
1− r

Total: 2

40(ii) Single range 1 < S < 3 or (1, 3) B2 Accept 1 < 2 − r < 3 .


Correct range but with S = 2 omitted scores SR B1
1 - S - 3 scores SR B1.
[S > 1 and S < 3] scores SR B1.

Total: 2

221
Unit 6.2: Answer Section
41 O/N 16 /P12/Q8
Page 7 Mark Scheme Syllabus Paper
Cambridge International AS/A Level – October/November 2016 9709 12

41 (a) (i) 200 + (15 − 1)( + / −5 ) M1 Use of nth term with a = 200, n = 14 or 15and
d = +/– 5.
= 130 A1
[2]

n
(ii)  400 + ( n − 1)( + / −5 )  = (3050) M1 Use of Sn a=200 and d = +/– 5.
2
→ 5n 2 − 405n + 6100 (= 0) A1
→ 20 A1
[3]

(b) (i) ar² , ar5 → r=½ M1 A1 Both terms correct.

=
(
63 a 1 − ½
6
) → a = 16 M1 A1 Use of Sn = 31.5 with a numeric r.
2 ½ [4]

16
(ii) Sum to infinity = = 32 B1 for their a and r with │r│< 1.
½ [1]

42 O/N 16 /P11/Q5
Page 4 Mark Scheme Syllabus Paper
Cambridge International AS/A Level – October/November 2016 9709 11

42 a (1 + r ) = 50 or
(
a 1− r2 ) = 50 B1
1− r

ar (1 + r ) = 30 or
(
a 1 − r3 ) = 30 + a B1 Or otherwise attempt to solve
1− r for r
Eliminating a or r M1 Any correct method
r =3/5 A1
a = 125 / 4 oe A1
S = 625 / 8 oe A1 Ft through on their r and a
[6] (−1 < r < 1)

222
Unit 6.2: Answer Section
Page Mark Scheme Syllabus Paper
Cambridge International AS/A Level – October/November 2016 9709 13

6 12
43 (a) = M1
1− r 1+ r
1
r= A1
3
S =9 A1
[3]

13
(b)  2cos θ + 12sin 2θ  = 52 M1* Use of correct formula for sum of
2 
AP
2cos θ + 12(1 − cos ²θ ) = 8 → 6cos 2θ − cos θ − 2 ( = 0 ) DM1 Use s 2 = 1 − c 2 & simplify to 3-
term quad
cos θ = 2 / 3 or −1 / 2 soi A1
Accept 0.268π, 2π/3. SRA1 for
θ = 0.841 , 2.09 Dep on previous A1 A1A1 48.2˚, 120˚ Extra solutions in
[5] range –1

Page Mark Scheme Syllabus Paper


Cambridge International AS/A Level – October/November 2016 9709 12
JJJG JJJG
44 (i) −4 – 6 – 6 = −16 M1 Use of x1x2 + y1y2 + z1z2 on their OA & OB
JJJG JJJG
x12 + y12 + z12 or x22 + y22 + z22 M1 Modulus once on either their OA or OB
JJJG JJJG
3 × 7 × cos θ = − 16 M1 All linked using their OA & OB
→ θ = 139.6º or 2.44c or 0.776π A1
[4]

0
JJJG  
(ii) AC = c – a =  8  B1
6
 
Magnitude = 10
0  0 
15    
Scaling → × 8 = 12 M1 For 15 × their unit vector.
their10     A1
6  9 
[3]

2 + 2p
 
(iii) 6 − 2p B1 Single vector soi by scalar product.
 5− p 
  JJJG JJJG JJJG
→ –2(2 +2p) + 3(6 – 2p) +6(5 – p)= 0 M1 Dot product of (p OA + OC ) and OB = 0.
→ p =2¾ A1
[3]

223
Unit 6.2: Answer Section
Page Mark Scheme Syllabus Paper
Cambridge International AS/A Level – May/June 2016 9709 11

45 (a) a = 50, ar
ar²² = 32 B1 seen or implied
4 4
→r= (allow – for M mark) M1 Finding r and use of correct S∞
5 5 formula
→ S∞ = 250 A1 Only if |r| < 1
[3]
(b) (i) 2sinx, 3cosx
2sinx 3cosx, (sinx
(sinx + 2cosx
2cosx).
3c – 2s
2s = (s
(s + 2c
2c) – 3c
3c M1 Links terms up with AP, needs one
(or uses a, a + d, a + 2d
2d) expression for d.
4
4c = 3s
→ 4c 3s → t = M1 A1 Arrives at t = k. ag
3 [3]
4
SC uses t = to show
3
8 9 10
u1 = , u2 = , u3 = , B1 only
5 5 5
3 4
(ii) →c= ,s= or calculator x = 53.1º M1
5 5
→ a = 1.6, d = 0.2 M1 Correct method for both a and d.

→ S20 = 70 A1 (Uses Sn formula)


[3]

46 M/J 16 /P13/Q4
Page Mark Scheme Syllabus Paper
Cambridge International AS/A Level – May/June 2016 9709 13

46 3 + 2d 3 + 12d 3 + 12d
r= or or r 2 = B1 1 correct equation in r and d only is
3 3 + 2d 3 sufficient

( 3 + 2d )2 = 3 ( 3 + 12d ) oe M1 Eliminate r or d using valid method


OR
sub 2d = 3r – 3

( 4) d ( d − 6) = 0 DM1 Attempt to simplify and solve


OR quadratic
3r 2 = 18r − 15 → ( r − 1)( r − 5 )

d =6 A1 Ignore d = 0 or r = 1
A1 Do not allow −5 or ±5
r =5
[5]

224
Unit 6.2: Answer Section
Page 6 Mark Scheme Syllabus Paper
Cambridge International AS/A Level – October/November 2015 9709 11

47 47(i) x 2 − 4 x = 12 M1 4 x − x 2 = 12 scores M1A0


x = −2 or 6 A1
3rd term = (−2) 2 + 12 = 16 or 6 2 + 12 = 48 A1A1 SC1 for 16, 48 after x = 2, − 6
[4]
x2  x 
(ii) r2 =  =  soi M1
4x  4 
4x
=8 M1 Accept use of unsimplified
x
1− x2 4x 4
4 or 2 or
4x x x
4 1 A1
x = or r =
3 3
16 A1
3rd term = (or 0.593)
27 [4]

ALT
4x 1 4x
= 8 → r = 1 − x or = 8 → x = 2(1 − r ) M1
1− r 2 1− r
 1  2(1 − r )
x 2 = 4 x1 − x  r= M1
 2  4
4 1 A1
x= r=
3 3

Page 6 Mark Scheme Syllabus Paper


Cambridge International AS/A Level – October/November 2015 9709 13

4 (i) (a)
48 1.92 + 1.84 + 1.76 + ... oe B1 OR a=0.96, d= –.04 & ans
20
[2 × 1.92 + 19 × (−0.08)] oe M1 doubled/adjusted
2
23.2 cao A1 Corr formula used with corr d & their
[3] a, n
a = 1, n = 21 → 12.6 (25.2),
a = 0.96, n = 21 → 11.76 (23.52)
(b) 1.92 + 1.92(.96) + 1.92(.96) 2 + ... B1
20
1.92(1 − .96
M1 OR a=.96, r =.96 & ans
1 − .96
/doubled/adjusted
26.8 cao A1 Corr formula used with r =.96 & their
[3] a, n
a = .96, n = 21 → 13.82 (27.63)
a = 1, n = 21 →14.39 (28.78)
1.92 0.96
(ii) = 48 or = 24 & then M1A1 a = 1→25 (50) but must be doubled
1 − .96 1 − 0.96
[2] for M1
Double AG (1 − 0.96 n )
1.92 < 48 → 0.96 n > 0
1 − 0.96
(www)
'which is true' scores SCB1

225
Unit 6.2: Answer Section
Page Mark Scheme Syllabus Paper
Cambridge International AS/A Level – May/June 2015 9709 12
49 (a) 1st, 2nd, nth are 56, 53 and −22
a = 56, d = −3
−22 = 56 + (n – 1)(−3) M1 Uses correct un formula.
→ n = 27 A1 co
S27 = 272 (112 + 26(− 3)) M1 Needs positive integer n
→ 459 A1 Co
[4]

(b) 1st, 2nd, 3rd are 2k + 6, 2k and k + 2.

(i) Either 2k2k+6 = k2+k2


or uses a, r and eliminates M1 Correct method for equation in k.
→ 2k 2 − 10k − 12 = 0 DM1 Forms quad. or cubic equation with
no brackets or fractions.
→k=6 A1 Co
[3]

(ii) 2k or k + 2 (= 2 )
S∞ = 1−ar with r = 2k+ 6 2k 3 M1 Needs attempt at a and r and S∞
→ 54 A1 Co
[2]

Page Mark Scheme Syllabus Paper


Cambridge International AS/A Level – May/June 2015 9709 11

1 2
50 (a) ar² = , ar³ =
3 9
2
→ r= aef M1 Any valid method, seen or implied.
3 Could be answers only.
3
Substituting → a = A1 Both a and r
4
3
→ S∞ = 4
1
= 2 14 aef M1 A1 Correct formula with r < 1 , cao
3 [4]

Page Mark Scheme Syllabus Paper


Cambridge International AS/A Level – May/June 2015 9709 13

51 (a) 2222/17 (=131 or 130.7) M1 Ignore signs. Allow 2239/17→131.7 or 132


131 × 17 (=2227) M1 Ignore signs. Use 131.
−2222 + 2227 = 5 A1 5 www gets 3/3
[3]

(b) r = 2 cos θ soi oe B1


3
(−1 <) 2 cos θ < 1 or (0 <) 2 cos θ < 1 soi M1 Ft on their r. Ignore a 2nd inequality on
3 3 LHS

π / 6, 5π / 6 soi (but dep. on M1) A1A1 Allow 30º, 150º.


π / 6 < θ < 5π / 6 cao A1 Accept Y
[5]

226
Unit 6.2: Answer Section
Page Mark Scheme Syllabus Paper
Cambridge International AS/A Level – October/November 2014 9709 12

52 (a) Sn = 32n − n².


Set n to 1, a or S1 = 31 B1 co
Set n to 2 or other value S2 = 60
→ 2nd term = 29 → d = − 2 M1 A1 Correct method.
(or equates formulae – compares co
coeffs n², n) [3]
[M1 comparing, A1 d A1 a] [M1 only when coeffs compared]

a a(1 − r ) 2
(b) = 20 , , or a + ar = 12.8 B1 B1 co co
1− r 1− r

a
Elimination of or a or r M1 ‘Correct’ elimination to form equation in a
1− r or r
→ (r = 0.6) → a = 8 DM1 A1 Complete method leading to a =
[5] Condone a = 8 and 32

Page 5 Mark Scheme Syllabus Paper


Cambridge International AS/A Level – October/November 2014 9709 11

53 (i) S = a , 3S = a B1 At least 3 S = a
1−r 1−2 r 1−2 r
1 – r = 3 – 6r M1 Eliminate S
r = 52 A1
[3]

(ii) 7 + (n − 1)d = 84 and/or 7 + (3n − 1)d = 245 B1 At least one of these equations seen
[ (n − 1)d = 77 , (3n − 1)d = 238 , 2nd = 161 ] B1 Two different seen – unsimplified ok
n −1 = 77 (must be from the correct un formula) M1 Or other attempt to elim d. E.g. sub d = 161
3 n −1 238 2n
(if n is eliminated d must be found)
n = 23 (d = 77 = 3.5) A1
22 [4]

Page Mark Scheme Syllabus Paper


Cambridge International AS/A Level – October/November 2014 9709 13

2 3
54 (i) S P = , SP = M1 At least one correct
1 1
1− 1−
2 3
9 A1
S P = 4, S Q = At least one correct
2
SR = 5 cao A1
[3]
4
(ii) = their S R M1
1− r
1
r= A1
5
4 4 24
R=4+ + = 4 or 4.96 cao A1
5 25 25
[3]

227
Unit 6.2: Answer Section
Page Mark Scheme Syllabus Paper
GCE AS/A LEVEL – May/June 2014 9709 11

56 (i) 200/2(2a + 199d) = 4 × 100/2(2a + 99d) M1A1 Correct formula used (once) M1, correct
eqn A1
d = 2a cao A1
[3]
(ii) a + 99d = a + 99 × 2a M1 Sub. their part(i) into correct formula
199a cao A1 [2]

Page Mark Scheme Syllabus Paper


GCE AS/A LEVEL – May/June 2014 9709 13

57 36, 32, ...


(i) r = 8 S∞ = (their a) ÷ (1 – their r) M1 Method for r and S∞ ok. (│r│< 1)
9
S∞ = 36 ÷ 1 = 324 A1 co
9
(ii) d = −4 B1 [2] co
0 = n (72 + (n – 1)(–4)) Sn formula ok and a value for d  ≠ 8 
 

2 M1
 9
→ n = 19 A1 Condone n = 0 but no other soln
[3]

Page Mark Scheme Syllabus Paper


GCE AS/A LEVEL – October/November 2013 9709 12

58 (a) (i) a = 300, d = 12


→ 540 = 300 + (n − 1)12 → n = 21 M1 A1 Use of nth term. Ans 20 gets 0.
[2] Ignore incorrect units
(ii) S26 = 13 (600 + 25×12) = 11700 M1 Correct use of sn formula.
→ 3 hours 15 minutes. A1
[2]
(b) ar = 48 and ar² = 32 → r = ⅔ M1 Needs ar and ar² + attempt at a and r.
→ a =72. A1
S∞ = 72 ÷ ⅓ = 216. M1 Correct S∞ formula with │r│ < 1
A1
[4]

Page Mark Scheme Syllabus Paper


GCE AS/A LEVEL – October/November 2013 9709 11

10
59 (a) (2a + 9d ) = 400 oe B1 → 2a + 9d = 80
2
20
(2a + 19d ) = 1400 OR
2
10
[2(a + 10d ) + 9d ] = 1000 B1 → 2a + 19 d = 140 or 2a + 29 d = 200
2
d = 6 a = 13 M1A1A1 Solve sim. eqns both from S n
[5] formulae
a 2a B1B1
(b) =6 =7
1− r 1− r2
12(1 − r ) 1 − r 2 12
=7 or = M1 Substitute or divide
1− r2 1− r 7
5
r = or 0.714 A1
7
12 A1 Ignore any other solns for r and a
a= or 1.71(4)
7 [5]

228
Unit 6.2: Answer Section
Page Mark Scheme Syllabus Paper
GCE A LEVEL – October/November 2013 9709 13
a
60 (a) = 8a ⇒ 1(a) = 8(a) (1 − r ) B1
1− r

‫ ݎ‬ൌ oe B1

[2]
(b) a + 4d = 197 B1 Or 2a + 9d = 408
10
[2a + 9d ] = 2040 B1 Attempt to solve simultaneously
2
d = 14 M1A1
[4]

Page Mark Scheme Syllabus Paper


GCE AS/A LEVEL – May/June 2013 9709 12

61 (a) 57 = 2(24 + 3d) → d = 1.5


48 = 12 + (n − 1)1.5 → n = 25 M1 A1 Use of correct Sn formula.
M1 A1 Use of correct Tn formula.
(b) ar² = 4a r = ±2 [4]
a ( r 6 − 1) B1 (allow for r = 2)
= ka B1
r −1
→ k = 63 or k = − 21
B1 B1
[4]

Page Mark Scheme Syllabus Paper


GCE AS/A LEVEL – May/June 2013 9709 11

62 (i) ar2 = –108, ar5 = 32 B1


32  8 
r3 = =−  M1 Eliminating a
− 108  27 
 2 2 3
r =  −  or –0.666 or –0.667 A1 − from little or no working → www
3 3
 3 [3]
 108 32 
(ii) a = –243 B1 ft on their r  − 2 or 5 
 r r 
[1]
− 243 729
(iii) S ∞ = = − or –145.8 M1A1
2 5 Accept –146. For M1 r must be < 1
1+ [2]
3

Page Mark Scheme Syllabus Paper


GCE AS/A LEVEL – May/June 2013 9709 13

63 (a) S = 2n 2 + 8n
n

S1 = 10 = a B1
S2 = 24 = a + (a + d) d = 4 M1 A1 [3] correct use of Sn formula.

(b) GP a = 64 ar = 48 → r = ¾ B1

→3rd term is ar² = 36 M1 ar² numerical – for their r


AP a = 64, a + 8d = 48 → d = −2 B1
36 = 64 + (n − 1)(−2) M1 correct use of a+(n−1)d
→ n = 15. A1 [5]

229
Unit 6.2: Answer Section
Page Mark Scheme Syllabus Paper
GCE AS/A LEVEL – October/November 2012 9709 12

64 (a) (i) ar = 24, ar³ = 13½


Eliminates a (or r) → r = ¾ B1 Both needed
→ a = 32 M1 Method of Solution.
A1 co
[3]
(ii) sum to infinity = 32 ÷ ¼ = 128
M1A1 Correct formula used. on value of r
[2]

(b) a = 3, d = 2 B1 Correct value for d


n
(6 + (n − 1)2 ) ( = 360) M1 Correct Sn used. no need for 360 here.
2
→ 2n 2 + 4n – 720=0 A1 Correct quadratic
→ n = 18 A1 co
[4]

Page Mark Scheme Syllabus Paper


GCE AS/A LEVEL – October/November 2012 9709 11


65 122    14 M1 Attempt sum formula with a = 61, d = –4


 122    14 A1 Equated to n cao

2  31 0 DM1 Attempt to solve. Accept div. by n
 31 A1 cao
[4]

Page Mark Scheme Syllabus Paper


GCE AS/A LEVEL – October/November 2012 9709 13
1 1 3 M1 A1
66 (i) 2 =5
4 3
9 3 27
r3 = × =
4 16 64
 A1 [3]
r or 0.75


513 64 ଵ
(ii) S∞ = = (or 21 or 21.3) M1 A1 cao
1– 34 3 ଷ
[2]

Page 5 Mark Scheme: Teachers’ version Syllabus Paper


GCE AS/A LEVEL – May/June 2012 9709 12

67 (a) Sn = n² + 8n.

S1 = 9 → a = 9 B1 co
S2 = 20 → a + d = 11 → d = 2 M1 A1 Realises that S2 is a + (a + d). co
(or equating n² + 8n with Sn and comparing [3]
coefficients)

(b) a − ar = 9 B1 co
ar + ar 2 = 30 B1 co
Eliminates a → 3r 2 + 13r − 10 = 0 M1 Complete elimination of r or a
or → 2a 2 − 57 a + 81 = 0 Correct quadratic.
→ r=⅔ A1
→ a = 27 A1 co (condone 27 or 1.5)
[5]

230
Unit 6.2: Answer Section
Page Mark Scheme: Teachers’ version Syllabus Paper
GCE AS/A LEVEL – May/June 2012 9709 11

10
68 (a) S10 = M1 Correct formula with d = ± (cos 2 x − 1)
2[2 + 9(cos 2 x − 1)]
S10 = 5[2 − 9 sin 2 x] M1 Use of c 2 + s 2 = 1 in a correct S10
S10 = 10 − 45 sin 2 x A1 Or a = 10, b = 45
[3]

(b) (i) (0 < ) 1 tan 2 θ < 1 oe M1 Allow <


3
(0 < )θ < π A1 cao Allow <
3 [2]

1
(ii) S∞ = M1
1 π
1 − tan 2
3 6
9
S∞ = or 1.125 A1 cao
8 [2]

Page Mark Scheme: Teachers’ version Syllabus Paper


GCE AS/A LEVEL – May/June 2012 9709 13

69 (i) Uses Sn
9 M1 Uses correct formula
(24 + 8d ) = 135 → d = ¾ A1 co
2
[2]

(ii) 9th term of AP = 12 + 8×¾ = 18 B1 on “d”


GP 1st tern 12, 2nd term 18
Common ratio = r = 18 ÷ 12 = 1½ M1 Uses “ar”
3rd term of GP = ar² = 27 M1 Uses ar² or “ar” × r
nth term of AP is 12 + (n − 1)¾
12 + (n − 1)¾ = 27 → n = 21 M1A1 Links AP with GP. co
[5]

Page Mark Scheme: Teachers’ version Syllabus Paper


GCE AS/A LEVEL – October/November 2011 9709 12

70 (a) a = −15, n = 25
(i) Use of Sn → d = 3. M1 A1 Must be correct formula. co
[2]
(ii) Last term = a + 24d M1 Must be a + 24d
→ 57 A1√ √ for his d.
(or 525 = ½ × 25 × (−15 + l) → l = 57) [2]

(iii) Positive terms are 3,6, ....57


Either a = 0 or 3, n = 19 or 20 M1 Correct use of formula for Sn.
Use of S19 or S20
→ 570 A1 co
[2]
(b) r = 1.05 B1 In either part (i) or (ii).

(i) 11th term = ar10 = $6516 or $6520 B1 co


[2]
4000 × (1.0511 − 1)
(ii) S11 = M1 Correct sum formula with their r.
.05 A1 co
= $56800 or (56827) [2]

231
Unit 6.2: Answer Section
Page Mark Scheme: Teachers’ version Syllabus Paper
GCE AS/A LEVEL – October/November 2011 9709 11
71 (a) a + 5d = 23 B1 Solution of 2 linear equations
5(2a + 9d ) = 200 B1
Attempt solution, expect d = 6 a = −7 M1
29 A1 [4]

Page Mark Scheme: Teachers’ version Syllabus Paper


GCE AS/A LEVEL – October/November 2011 9709 13
72 (i) 5[8 + 9 × 4] M1 Use correct formula with a=4, d=4
220 A1
[2]

(ii)
(
4 2 −110
) M1 Use correct formula with a=4, r=2 or ½
2 −1

4092 A1 4090 without 4092 A0


[2]

Page Mark Scheme: Teachers’ version Syllabus Paper


GCE AS/A LEVEL – May/June 2011 9709 12

( a + 4a )
73 (a) a + 5d = 4a or ×6 B1 co
2
6 ( a + 4a )
(2a + 5d ) or × 6 = 360 M1 A1 Correct left-hand side. All correct.
2 2

Sim Eqns a = 24º or rads A1 Either answer.
15
Arc length = 5θ M1 Correct use of arc length with θ in rads.
Perimeter = 12.1. A1 co
[6]

k +6 k
(b) (i) = M1 A1 Correct eqn for k.
2k + 3 k + 6
→ k 2 − 9k − 36 = 0 → k = 12 A1 Co condone inclusion of k = −3.
(NB stating a, ar, ar² as f(k) gets M1) [3]

(ii) r = ⅔, a = 27 Correct formula for S∞ must have


→ S∞ = 27 ÷ ⅓ = 81. M1 A1 −1 ≤ r ≤ 1. co.
[2]

Page Mark Scheme: Teachers’ version Syllabus Paper


GCE AS/A LEVEL – May/June 2011 9709 11

74 (i) 1000, 2000, 3000... or 50, 100, 150... M1 Recognise series, correct a/d (or 3 terms )
40 40
or M1 Correct use of formula
2(1000 + 40000) 2(2000 + 39000)
× 5% of attempt at valid sum M1 Can be awarded in either (i) or (ii)
41000 A1 cao
[4]
(ii) 1000, 1000 × 1.1, 1000 × 1.12 + ... or with a = 50 M1 Recognise series, correct a/r ( or 3 terms)
1000(1.140 − 1) M1 Correct use of formula. Allow e.g. r = 0.1
1.1 − 1
A1 Or answers rounding to this
22100
[3]

232
Unit 6.2: Answer Section
Page Mark Scheme: Teachers’ version Syllabus Paper
GCE AS/A LEVEL – May/June 2011 9709 13

75 (a) ar2 = 20 B1 co
a
= 3a B1 co
1− r
Soln of equations → (r = ⅔) a = 45 M1 A1 Complete method to find a. co
[4]

(b) a + 7d = 3(a +2d) M1 Use of a + (n – 1)d


→ 2a = d A1 co
S8 = 4(2a + 7d) = 32d or 64a M1 correct use of Sn formula once.
S4 = 2(2a + 3d) = 8d or 16a A1 ag
[4]

233
Differentiation Unit 7: Differentiation
UNIT 7
1 M/J 22/P13/Q10/b
The function f is defined by f x = 4x + 2−2 for x > − 12 .
A point is moving along the curve y = f x in such a way that, as it passes through the point A, its
y-coordinate is decreasing at the rate of k units per second and its x-coordinate is increasing at the
rate of k units per second.
(a) Find the coordinates of A. [6]

2 O/N 21/P12/Q9

The volume V m3 of a large circular mound of iron ore of radius r m is modelled by the equation
V= 3 r − 1 3 − 1 for r ≥ 2. Iron ore is added to the mound at a constant rate of 1.5 m3 per second.
2 2

(a) Find the rate at which the radius of the mound is increasing at the instant when the radius is 5.5 m.
[3]
(b) Find the volume of the mound at the instant when the radius is increasing at 0.1 m per second.
O/N 21/P12/Q10 [3]
3
k
The function f is defined by f x = x2 + + 2 for x > 0.
x
(a) Given that the curve with equation y = f x has a stationary point when x = 2, find k. [3]
(b) Determine the nature of the stationary point. [2]
(c) Given that this is the only stationary point of the curve, find the range of f. [2]

4 M/J 21/P11/Q11 
The equation of a curve is y = 2 3x + 4 − x.
(a) Find the equation of the normal to the curve at the point 4, 4, giving your answer in the form
y = mx + c. [5]
(c) Determine the nature of the stationary point. [2]
(d) Find the exact area of the region bounded by the curve, the x-axis and the lines x = 0 and x = 4.
[4]
5 M/J 21/P13/Q2
The function f is defined by f x = 13 2x − 1 2 − 2x for 21 < x < a. It is given that f is a decreasing
function.

Find the maximum possible value of the constant a. [4]


6 O/N 20/P12/Q7/a
−1 −3
The point 4, 7 lies on the curve y = f x and it is given that f ′ x = 6x 2 − 4x 2 .
(a) A point moves along the curve in such a way that the x-coordinate is increasing at a constant rate
of 0.12 units per second.
Find the rate of increase of the y-coordinate when x = 4. [3]
7 O/N 20/P11/Q3
Air is being pumped into a balloon in the shape of a sphere so that its volume is increasing at a constant
rate of 50 cm3 s−1 .
Find the rate at which the radius of the balloon is increasing when the radius is 10 cm. [3]

8 O/N 20/P11/Q6

The equation of a curve is y = 2 + 25 − x2 .
Find the coordinates of the point on the curve at which the gradient is 43 . [5]

9 O/N 20/P13/Q8
The equation of a curve is y = 2x + 1 + for x > − 12 .
1
2x + 1
dy d2 y
(a) Find and 2 . [3]
dx dx
(b) Find the coordinates of the stationary point and determine the nature of the stationary point. [5]
234
Unit 7: Differentiation
10 M/J 20/P12/Q3
A weather balloon in the shape of a sphere is being inflated by a pump. The volume of the balloon is
increasing at a constant rate of 600 cm3 per second. The balloon was empty at the start of pumping.

(a) Find the radius of the balloon after 30 seconds. [2]


(b) Find the rate of increase of the radius after 30 seconds. [3]

11 M/J 20/P12/Q10
The equation of a curve is y = 54x − 2x − 73 .

dy d2 y
(a) Find and 2 . [4]
dx dx
(b) Find the coordinates of each of the stationary points on the curve. [3]
(c) Determine the nature of each of the stationary points. [2]
12 M/J 20/P11/Q9

The equation of a curve is y = 3 − 2x3 + 24x.


dy d2 y
(a) Find expressions for and 2 . [4]
dx dx
(b) Find the coordinates of each of the stationary points on the curve. [3]

13 M/J 20/P13/Q6
A point P is moving along a curve in such a way that the x-coordinate of P is increasing at a constant
1
rate of 2 units per minute. The equation of the curve is y = 5x − 1 2 .

(a) Find the rate at which the y-coordinate is increasing when x = 1. [4]
(b) Find the value of x when the y-coordinate is increasing at 58 units per minute. [3]

14 MAR 20/P12/Q1
1
The function f is defined by f x = + x2 for x < −1.
3x + 2
Determine whether f is an increasing function, a decreasing function or neither. [3]
15 MAR 20/P12/Q4
A curve has equation y = x2 − 2x − 3. A point is moving along the curve in such a way that at P the
y-coordinate is increasing at 4 units per second and the x-coordinate is increasing at 6 units per second.
Find the x-coordinate of P. [4]

16 O/N 19/P12/Q5

15 cm h cm

The diagram shows a solid cone which has a slant height of 15 cm and a vertical height of h cm.

(i) Show that the volume, V cm3, of the cone is given by V = 13 0 225h − h3 . [2]

[The volume of a cone of radius r and vertical height h is 13 0r2 h.]


(ii) Given that h can vary, find the value of h for which V has a stationary value. Determine, showing
all necessary working, the nature of this stationary value. [5]
235
17 O/N 19/P11/Q2 Unit 7: Differentiation

An increasing function, f, is defined for x > n, where n is an integer. It is given that f ′ x = x2 − 6x + 8.


Find the least possible value of n. [3]

18 O/N 19/P11/Q3
The line y = ax + b is a tangent to the curve y = 2x3 − 5x2 − 3x + c at the point 2, 6. Find the values
of the constants a, b and c. [5]

19 O/N 19/P13/Q3
The equation of a curve is y = x3 + x2 − 8x + 7. The curve has no stationary points in the interval
a < x < b. Find the least possible value of a and the greatest possible value of b. [4]

20 O/N 19/P13/Q5

x cm 4x cm

2x cm
The dimensions of a cuboid are x cm, 2x cm and 4x cm, as shown in the diagram.
(i) Show that the surface area S cm2 and the volume V cm3 are connected by the relation
2
S = 7V 3 . [3]
(ii) When the volume of the cuboid is 1000 cm3 the surface area is increasing at 2 cm2 s−1 . Find the
rate of increase of the volume at this instant. [4]

21 O/N 19/P13/Q8
A function f is defined for x > 12 and is such that f ′ x = 3 2x − 1 21 − 6.
(i) Find the set of values of x for which f is decreasing. [4]
(ii) It is now given that f 1 = −3. Find f x. [4]

22 M/J 19/P12/Q3
dy 4
A curve is such that = x3 − 2 . The point P 2, 9 lies on the curve.
dx x
(i) A point moves on the curve in such a way that the x-coordinate is decreasing at a constant rate
of 0.05 units per second. Find the rate of change of the y-coordinate when the point is at P. [2]

(ii) Find the equation of the curve. [3]

23 M/J 19/P12/Q9
The curve C1 has equation y = x2 − 4x + 7. The curve C2 has equation y2 = 4x + k, where k is a
constant. The tangent to C1 at the point where x = 3 is also the tangent to C2 at the point P. Find the
value of k and the coordinates of P. [8]

24 M/J 19/P13/Q8
dy
A curve is such that = 3x2 + ax + b. The curve has stationary points at −1, 2 and 3, k. Find
dx
the values of the constants a, b and k. [8]

236
25 MAR 19/P12/Q4 Unit 7: Differentiation

A curve has equation y = 2x − 1−1 + 2x.


dy d2 y
(i) Find and 2 . [3]
dx dx
(ii) Find the x-coordinates of the stationary points and, showing all necessary working, determine
the nature of each stationary point. [4]

26 MAR 19/P12/Q10 y

1
y = 4x 2

x
O

1
The diagram shows the curve with equation y = 4x 2 .

(i) The straight line with equation y = x + 3 intersects the curve at points A and B. Find the length
of AB. [6]
(ii) The tangent to the curve at a point T is parallel to AB. Find the coordinates of T . [3]
(iii) Find the coordinates of the point of intersection of the normal to the curve at T with the line AB.
[3]
y
27 O/N 18/P12/ Q3
y = 5x

Q R

P
y = x 9 − x2 

x
O

The diagram shows part of the curve y = x 9 − x2  and the line y = 5x, intersecting at the origin O and
the point R. Point P lies on the line y = 5x between O and R and the x-coordinate of P is t. Point Q
lies on the curve and PQ is parallel to the y-axis.

(i) Express the length of PQ in terms of t, simplifying your answer. [2]


(ii) Given that t can vary, find the maximum value of the length of PQ. [3]

237
Unit 7: Differentiation
28 O/N 18/P11/ Q6
dy
A curve has a stationary point at 3, 9 21  and has an equation for which = ax2 + a2 x, where a is a
dx
non-zero constant.
(i) Find the value of a. [2]
(ii) Find the equation of the curve. [4]

29 O/N 18/P11/ Q10


 
A curve has equation y = 12 4x − 3−1 . The point A on the curve has coordinates 1, 12 .

(i) (a) Find and simplify the equation of the normal through A. [5]
(b) Find the x-coordinate of the point where this normal meets the curve again. [3]
(ii) A point is moving along the curve in such a way that as it passes through A its x-coordinate is
decreasing at the rate of 0.3 units per second. Find the rate of change of its y-coordinate at A.
[2]
30 O/N 18/P13/ Q2
The function f is defined by f x = x3 + 2x2 − 4x + 7 for x ≥ −2. Determine, showing all necessary
working, whether f is an increasing function, a decreasing function or neither. [4]
31 M/J 18/P11/ Q2
5
A point is moving along the curve y = 2x + in such a way that the x-coordinate is increasing at a
x
constant rate of 0.02 units per second. Find the rate of change of the y-coordinate when x = 1. [4]

32 M/J 18/P13/ Q8
(i) The tangent to the curve y = x3 − 9x2 + 24x − 12 at a point A is parallel to the line y = 2 − 3x.
Find the equation of the tangent at A. [6]
(ii) The function f is defined by f x = x3 − 9x2 + 24x − 12 for x > k, where k is a constant. Find the
smallest value of k for f to be an increasing function. [2]

33 O/N 17/P12/ Q8
dy
A curve is such that = −x2 + 5x − 4.
dx

(i) Find the x-coordinate of each of the stationary points of the curve. [2]
2
d y
(ii) Obtain an expression for and hence or otherwise find the nature of each of the stationary
dx2
points. [3]
(iii) Given that the curve passes through the point 6, 2, find the equation of the curve. [4]

34 O/N 17/P11/ Q1
3 1
A curve has equation y = 2x 2 − 3x − 4x 2 + 4. Find the equation of the tangent to the curve at the point
4, 0. [4]

35 O/N 17/P11/ Q4

Machines in a factory make cardboard cones of base radius r cm and vertical height h cm. The volume,
V cm3 , of such a cone is given by V = 13 0r 2 h. The machines produce cones for which h + r = 18.

(i) Show that V = 60r 2 − 13 0r3 . [1]


(ii) Given that r can vary, find the non-zero value of r for which V has a stationary value and show
that the stationary value is a maximum. [4]
(iii) Find the maximum volume of a cone that can be made by these machines. [1]

238
Unit 7: Differentiation
36 O/N 17/P13/ Q4
3
The function f is such that f x = 2x − 1 2 − 6x for 21 < x < k, where k is a constant. Find the largest
value of k for which f is a decreasing function. [5]

37 O/N 17/P13/ Q10


A curve has equation y = f x and it is given that f ′ x = ax2 + bx, where a and b are positive constants.

(i) Find, in terms of a and b, the non-zero value of x for which the curve has a stationary point and
determine, showing all necessary working, the nature of the stationary point. [3]
(ii) It is now given that the curve has a stationary point at −2, −3 and that the gradient of the curve
at x = 1 is 9. Find f x. [6]

38 M/J 17/P12/ Q5
12
A curve has equation y = 3 + .
2−x
(i) Find the equation of the tangent to the curve at the point where the curve crosses the x-axis. [5]
(ii) A point moves along the curve in such a way that the x-coordinate is increasing at a constant rate
of 0.04 units per second. Find the rate of change of the y-coordinate when x = 4. [2]

39 M/J 17/P12/ Q9

The equation of a curve is y = 8 x − 2x.

(i) Find the coordinates of the stationary point of the curve. [3]
d2 y
(ii) Find an expression for 2 and hence, or otherwise, determine the nature of the stationary point.
dx
[2]
(iii) Find the values of x at which the line y = 6 meets the curve. [3]
(iv) State the set of values of k for which the line y = k does not meet the curve. [1]

40 M/J 17/P11/ Q6

The horizontal base of a solid prism is an equilateral triangle of side x cm. The sides of the prism are
vertical. The height of the prism is h cm and the volume of the prism is 2000 cm3 .

(i) Express h in terms of x and show that the total surface area of the prism, A cm2 , is given by
ï3 2 24 000 −1
A= x + x . [3]
2 ï3
(ii) Given that x can vary, find the value of x for which A has a stationary value. [3]
(iii) Determine, showing all necessary working, the nature of this stationary value. [2]

41 O/N 16/P12/ Q7
3
The equation of a curve is y = 2 + .
2x − 1
dy
(i) Obtain an expression for . [2]
dx
(ii) Explain why the curve has no stationary points. [1]

At the point P on the curve, x = 2.

(iii) Show that the normal to the curve at P passes through the origin. [4]

(iv) A point moves along the curve in such a way that its x-coordinate is decreasing at a constant
rate of 0.06 units per second. Find the rate of change of the y-coordinate as the point passes
through P. [2]

239
42 O/N 16/P11/ Q11 Unit 7: Differentiation
1 9
The point P 3, 5 lies on the curve y = − .
x−1 x−5
(i) Find the x-coordinate of the point where the normal to the curve at P intersects the x-axis. [5]

(ii) Find the x-coordinate of each of the stationary points on the curve and determine the nature of
each stationary point, justifying your answers. [6]
43 O/N 16/P13/ Q4
The function f is such that f x = x3 − 3x2 − 9x + 2 for x > n, where n is an integer. It is given that f is
an increasing function. Find the least possible value of n. [4]

44 M/J 16/P11/ Q5
xm

ym

A farmer divides a rectangular piece of land into 8 equal-sized rectangular sheep pens as shown in the
diagram. Each sheep pen measures x m by y m and is fully enclosed by metal fencing. The farmer
uses 480 m of fencing.

(i) Show that the total area of land used for the sheep pens, A m2, is given by
A = 384x − 9.6x2 . [3]

(ii) Given that x and y can vary, find the dimensions of each sheep pen for which the value of A is a
maximum. (There is no need to verify that the value of A is a maximum.) [3]

45 M/J 16/P13/ Q5
A curve has equation y = 8x + 2x − 1−1 . Find the values of x at which the curve has a stationary
point and determine the nature of each stationary point, justifying your answers. [7]

46 M/J 16/P13/ Q7
3
The point P x, y is moving along the curve y = x2 − 10 3
x 2 + 5x in such a way that the rate of change
of y is constant. Find the values of x at the points at which the rate of change of x is equal to half the
rate of change of y. [7]

47 O/N 15/P12/ Q3 D C
m
40 c
F A B
B 30Å 30Å
A
30Å 30Å
h cm

E E
Fig. 1 Fig. 2

Fig. 1 shows an open tank in the shape of a triangular prism. The vertical ends ABE and DCF are
identical isosceles triangles. Angle ABE = angle BAE = 30Å. The length of AD is 40 cm. The tank is
fixed in position with the open top ABCD horizontal. Water is poured into the tank at a constant rate
of 200 cm3 s−1 . The depth of water, t seconds after filling starts, is h cm (see Fig. 2).

240
Unit 7: Differentiation
47 O/N 15/P12/ Q3
(i) Show that, when the depth of water in the tank is h cm, the volume, V cm3 , of water in the tank
is given by V = 40ï3h2 . [3]

(ii) Find the rate at which h is increasing when h = 5. [3]

48 O/N 15/P12/ Q9
12
The curve y = f x has a stationary point at 2, 10 and it is given that f ′′ x = .
x3
(i) Find f x. [6]

(ii) Find the coordinates of the other stationary point. [2]

(iii) Find the nature of each of the stationary points. [2]


49 O/N 15/P11/ Q2

The function f is such that f ′ x = 3x2 − 7 and f 3 = 5. Find f x. [3]

50 O/N 15/P11/ Q5
8
A curve has equation y = + 2x.
x
dy d2 y
(i) Find and 2 . [3]
dx dx

(ii) Find the coordinates of the stationary points and state, with a reason, the nature of each stationary
point. [5]

51 O/N 15/P11/ Q6
A curve has equation y = x2 − x + 3 and a line has equation y = 3x + a, where a is a constant.
(i) Show that the x-coordinates of the points of intersection of the line and the curve are given by
the equation x2 − 4x + 3 − a = 0. [1]
(ii) For the case where the line intersects the curve at two points, it is given that the x-coordinate of
one of the points of intersection is −1. Find the x-coordinate of the other point of intersection.
[2]
(iii) For the case where the line is a tangent to the curve at a point P, find the value of a and the
coordinates of P. [4]

52 O/N 15/P13/ Q3
(i) Express 3x2 − 6x + 2 in the form a x + b2 + c, where a, b and c are constants. [3]

(ii) The function f, where f x = x3 − 3x2 + 7x − 8, is defined for x ∈ >. Find f ′ x and state, with a
reason, whether f is an increasing function, a decreasing function or neither. [3]

53 O/N 15/P13/ Q9/i,iii


yd −1
A curve passes through the point A 4, 6 and is such that = 1 + 2x 2 . A point P is moving along
dx
the curve in such a way that the x-coordinate of P is increasing at a constant rate of 3 units per minute.

(i) Find the rate at which the y-coordinate of P is increasing when P is at A. [3]
(ii) The tangent to the curve at A crosses the x-axis at B and the normal to the curve at A crosses the
x-axis at C. Find the area of triangle ABC. [5]

241
Unit 7: Differentiation
54 M/J 15/P12/ Q4
Variables u, x and y are such that u = 2x y − x and x + 3y = 12. Express u in terms of x and hence
find the stationary value of u. [5]

55 M/J 15/P11/ Q2
y

y = 2x2

x
X −2, 0 O P p, 0

The diagram shows the curve y = 2x2 and the points X −2, 0 and P p, 0. The point Q lies on the
curve and PQ is parallel to the y-axis.

(i) Express the area, A, of triangle XPQ in terms of p. [2]

The point P moves along the x-axis at a constant rate of 0.02 units per second and Q moves along the
curve so that PQ remains parallel to the y-axis.

(ii) Find the rate at which A is increasing when p = 2. [3]

56 M/J 15/P11/ Q9
The equation of a curve is y = x3 + px2 , where p is a positive constant.

(i) Show that the origin is a stationary point on the curve and find the coordinates of the other
stationary point in terms of p. [4]
(ii) Find the nature of each of the stationary points. [3]
Another curve has equation y = x3 + px2 + px.
(iii) Find the set of values of p for which this curve has no stationary points. [3]

57 M/J 15/P13/ Q8
1 1
The function f is defined by f x = + for x > −1.
′ x+1 x + 12
(i) Find f x. [3]
(ii) State, with a reason, whether f is an increasing function, a decreasing function or neither. [1]
1 1
The function g is defined by g x = + for x < −1.
x+1 x + 12
(iii) Find the coordinates of the stationary point on the curve y = g x. [4]
58 O/N 14/P12/ Q4
12
A curve has equation y = .
dy 3 − 2x
(i) Find . [2]
dx
A point moves along this curve. As the point passes through A, the x-coordinate is increasing at a
rate of 0.15 units per second and the y-coordinate is increasing at a rate of 0.4 units per second.

(ii) Find the possible x-coordinates of A. [4]

242
Unit 7: Differentiation
59 O/N 14/P11/ Q9
2
The function f is defined for x > 0 and is such that f  x = 2x − . The curve y = fx passes through
x2
the point P 2, 6.

(i) Find the equation of the normal to the curve at P. [3]

(ii) Find the x-coordinate of the stationary point and state with a reason whether this point is a
maximum or a minimum. [4]
60 O/N 14/P13/ Q8
A curve y = fx has a stationary point at 3, 7 and is such that f x = 36x−3 .

(i) State, with a reason, whether this stationary point is a maximum or a minimum. [1]

(ii) Find f x and fx. [7]


61 O/N 14/P13/ Q10/b
1
(a) A point P travels along the curve y = 7x2 + 1 3 in such a way that the x-coordinate of P at time
t minutes is increasing at a constant rate of 8 units per minute. Find the rate of increase of the
y-coordinate of P at the instant when P is at the point 3, 4. [5]
62 O/N 14/P12/ Q8
d2 y
The equation of a curve is such that = 2x − 1. Given that the curve has a minimum point at
dx2
3, −10, find the coordinates of the maximum point. [8]

63 O/N 14/P11/ Q4
4
A curve has equation y = . Find the equation of the tangent to the curve at the point where
3x + 12
the line x = −1 intersects the curve. [5]

64 O/N 14/P11/ Q12(ii,iii)


dy 1 −1  
A curve is such that = x 2 − x 2 . The curve passes through the point 4, 32 .
dx
d2 y
(i) Find 2 . [2]
dx
(ii) Find the coordinates of the stationary point and determine its nature. [5]

65 M/J 14/P13/ Q5
tan x + 1
(i) Prove the identity  sin x + cos x. [3]
sin x tan x + cos x
tan x + 1
(ii) Hence solve the equation = 3 sin x − 2 cos x for 0 ≤ x ≤ 20. [3]
sin x tan x + cos x
66 M/J 14/P13/ Q6
15
5 A function f is such that f x = for 0 ≤ x ≤ 6.
2x + 3
(i) Find an expression for f ′ x and use your result to explain why f has an inverse. [3]
(ii) Find an expression for f −1 x, and state the domain and range of f −1 . [4]

67 M/J 14/P13/ Q9
The base of a cuboid has sides of length x cm and 3x cm. The volume of the cuboid is 288 cm3.

(i) Show that the total surface area of the cuboid, A cm2, is given by
768
A = 6x2 + . 3
x

243
68 O/N 13/P12/Q3(i) Unit 7: Differentiation

The equation of a curve is y = 


2
5x − 6
.

(i) Find the gradient of the curve at the point where x = 2. [3]

69 O/N 13/P12/Q6 y

S (0, 12)

R Q (x, y)

T (16, 0)
x
O P

In the diagram, S is the point 0, 12 and T is the point 16, 0. The point Q lies on ST , between S
and T , and has coordinates x, y. The points P and R lie on the x-axis and y-axis respectively and
OPQR is a rectangle.
(i) Show that the area, A, of the rectangle OPQR is given by A = 12x − 34 x2 . [3]
(ii) Given that x can vary, find the stationary value of A and determine its nature. [4]

70 O/N 13/P11/Q8
x metres

r metres

The inside lane of a school running track consists of two straight sections each of length x metres,
and two semicircular sections each of radius r metres, as shown in the diagram. The straight sections
are perpendicular to the diameters of the semicircular sections. The perimeter of the inside lane is
400 metres.
(i) Show that the area, A m2 , of the region enclosed by the inside lane is given by A = 400r − 0r2 .
[4]
(ii) Given that x and r can vary, show that, when A has a stationary value, there are no straight
sections in the track. Determine whether the stationary value is a maximum or a minimum. [5]
71 O/N 13/P13/Q9
k2
A curve has equation y = + x, where k is a positive constant. Find, in terms of k, the values of
x+2
x for which the curve has stationary points and determine the nature of each stationary point. [8]

72 O/N 13/P12/Q9
5
A function f is defined by f x = , for x ≥ 1.
1 − 3x
(i) Find an expression for f ′ x. [2]

(ii) Determine, with a reason, whether f is an increasing function, a decreasing function or neither.
[1]
(iii) Find an expression for f −1 x, and state the domain and range of f −1 . [5]

244
Unit 7: Differentiation
73 O/N 12/P12/Q3 X 2x m
B C

40 m
Playground
Y
xm
A D
60 m

The diagram shows a plan for a rectangular park ABCD, in which AB = 40 m and AD = 60 m. Points
X and Y lie on BC and CD respectively and AX , XY and YA are paths that surround a triangular
playground. The length of DY is x m and the length of XC is 2x m.

(i) Show that the area, A m2 , of the playground is given by

A = x2 − 30x + 1200. [2]

(ii) Given that x can vary, find the minimum area of the playground. [3]

74 O/N 12/P12/Q10
dy 4
A curve is defined for x > 0 and is such that = x + 2 . The point P (4, 8) lies on the curve.
dx x
(i) Find the equation of the curve. [4]

(ii) Show that the gradient of the curve has a minimum value when x = 2 and state this minimum
value. [4]

75 O/N 12/P11/Q3

An oil pipeline under the sea is leaking oil and a circular patch of oil has formed on the surface of the
sea. At midday the radius of the patch of oil is 50 m and is increasing at a rate of 3 metres per hour.
Find the rate at which the area of the oil is increasing at midday. [4]

76 O/N 12/P11/Q5
1
A curve has equation y = 2x + . Verify that the curve has a stationary point at x = 2 and
(x − 1)2
determine its nature. [5]

245
Unit 7: Differentiation

77 O/N 12/P11/Q11 y
1
y = (6x + 2)3

A (1, 2)

C x
O

1
The diagram shows the curve y = (6x + 2) 3 and the point A (1, 2) which lies on the curve. The tangent
to the curve at A cuts the y-axis at B and the normal to the curve at A cuts the x-axis at C.

(i) Find the equation of the tangent AB and the equation of the normal AC . [5]

(ii) Find the distance BC . [3]

(iii) Find the coordinates of the point of intersection, E, of OA and BC , and determine whether E is
the mid-point of OA. [4]

78 O/N 12/P13/Q2
1
It is given that f (x) = − x3 , for x > 0. Show that f is a decreasing function. [3]
x3

79 M/J 12/P12/Q2
√ 2
The equation of a curve is y = 4 x + √ .
x
dy
(i) Obtain an expression for . [3]
dx

(ii) A point is moving along the curve in such a way that the x-coordinate is increasing at a constant
rate of 0.12 units per second. Find the rate of change of the y-coordinate when x = 4. [2]

80 O/N 12/P11/Q4

A watermelon is assumed to be spherical in shape while it is growing. Its mass, M kg, and radius,
r cm, are related by the formula M = kr3 , where k is a constant. It is also assumed that the radius is
increasing at a constant rate of 0.1 centimetres per day. On a particular day the radius is 10 cm and
the mass is 3.2 kg. Find the value of k and the rate at which the mass is increasing on this day. [5]

81 O/N 12/P11/Q10
It is given that a curve has equation y = f (x), where f (x) = x3 − 2x2 + x.
(i) Find the set of values of x for which the gradient of the curve is less than 5. [4]
(ii) Find the values of f (x) at the two stationary points on the curve and determine the nature of each
stationary point. [5]

246
Unit 7: Differentiation
82 O/N 12/P13/Q7

10
The curve y = − 2 intersects the x-axis at A. The tangent to the curve at A intersects the y-axis
2x + 1
at C.

(i) Show that the equation of AC is 5y + 4x = 8. [5]

(ii) Find the distance AC . [2]

83 M/J 11/P11/Q2
A curve has equation y = 3x3 − 6x2 + 4x + 2. Show that the gradient of the curve is never negative.
[3]
84 M/J 11/P11/Q7 x

2y
3y
3x

4x

The diagram shows the dimensions in metres of an L-shaped garden. The perimeter of the garden is
48 m.

(i) Find an expression for y in terms of x. [1]

(ii) Given that the area of the garden is A m2 , show that A = 48x − 8x2 . [2]

(iii) Given that x can vary, find the maximum area of the garden, showing that this is a maximum
value rather than a minimum value. [4]

85 M/J 11/P13/Q8
A curve y = f (x) has a stationary point at P (3, −10). It is given that f ′ (x) = 2x2 + kx − 12, where k is
a constant.

(i) Show that k = −2 and hence find the x-coordinate of the other stationary point, Q. [4]

(ii) Find f ′′ (x) and determine the nature of each of the stationary points P and Q. [2]

(iii) Find f (x). [4]

86 M/J 11/P12/Q4
A curve has equation y = and P (2, 2) is a point on the curve.
4
3x − 4

(i) Find the equation of the tangent to the curve at P. [4]

(ii) Find the angle that this tangent makes with the x-axis. [2]

247
Unit 7: Differentiation
87 M/J 11/P11/Q2
The volume of a spherical balloon is increasing at a constant rate of 50 cm3 per second. Find the rate
of increase of the radius when the radius is 10 cm. [Volume of a sphere = 43 π r3 .] [4]

88 M/J 11/P11/Q6
The variables x, y and ß can take only positive values and are such that
ß = 3x + 2y and xy = 600.
1200
(i) Show that ß = 3x + . [1]
x
(ii) Find the stationary value of ß and determine its nature. [6]

248
UNIT 7
Unit 7: Answer Section
9709/13 May/June 2022

Question M/J 22/P13/Q10/b Answer Marks Guidance

1(a) dy
dx

 2  4 x  2 
3
 4
B1 B1 Allow unsimplified forms.

dy B1 SOI
Recognise = ‒1
dx

8 M1 Must be numerical.
their  their  1
dy
 4x  23
Must be some attempt to solve their equation and 0.
dx

(0, ¼) 6 A1 A1 Accept x = 0, y = ¼. y = ¼ must be from x = 0 not x = –1.

9709/12 O/N 21/P12/Q9 October/November 2021

Question Answer Marks Guidance

2(a)  dV  9  1
2 B1 OE. Accept unsimplified.
 dr =  2  r − 2 
   

  M1 Correct use of chain rule with 1.5, their differentiated


  dV
dr dr dV 1.5 = 1.5 1.5  expression for and using r = 5.5 .
= × = = dr
dt dV dt their dV  9 1  112.5 
2

dr   5.5 −  
 2 2 

3 1 A1
0.0133 or or [ metres per second]
225 75

249
Unit 7: Answer Section October/November 2021
9709/12 O/N 21/P12/Q9
Question Answer Marks Guidance

2(b)   B1 FT dV dV
Correct statement involving or their , 1.5 and 0.1.
  dr dr
dV
or their
dV 1.5
= or 15 OR 0.1 =
1.5  = 2 × 1.5 OE 
dr dr 0.1 dV   1
2 
their  9 r − 
dr 
  2 

9  1
2
 1 10 B1 OE e.g. AWRT 2.3
  r −  = 15   r = + Can be implied by correct volume.
 2  2  2 3

[Volume =] 8.13 AWRT B1 −3 + 5 30


OE e.g. . CAO.
3

O/N 21/P12/Q10 3

3(a) k B1
[f '(x) =] 2 x −
x2

 k  M1 Setting their 2-term f '(2) = 0, at least one term correct and


f '(2) = 0  2 × 2 − 2 = 0   k = … attempting to solve as far as k = .
 2 

k = 16 A1

3(b) 2k M1 Evaluate a two term f''(2) with at least one term correct.
f''(2) = e.g. 2+ Or other valid method.
23

 2k  A1 FT WWW. FT on positive k value.


 2 + 23  > 0 ⇒ minimum or = 6 ⇒ minimum
 

3(c) When x = 2, f(x) = 14 B1 SOI

[Range is or y or f(x)] ⩾ their f(2) B1 FT Not x ⩾ their f(2)

250
9709/11 M/J 21/P11/Q11 Cambridge International AS & A Level – Mark Scheme Unit 7: Answer Section May/June 2021

Question Answer Marks Guidance

4(a) dy −0.5 B1 B1 B1 All correct with 1 error, B2 if all correct


= 3 ( 3x + 4 ) − 1
dx

1 *M1 Substituting x = 4 into a differentiated expression and


Gradient of tangent = − and Gradient of normal = 4
4 using m1 m2 =−1

Equation of line is (y – 4) = 4(x – 4) or evaluate c DM1 With (4, 4) and their gradient of normal

So y = 4x – 12 A1

4(b) 3 (3x + 4 )
−0.5
−1 = 0 M1 dy
Setting their =0
dx

Solving as far as x = M1 dy −0.5


Where contains a ( bx + c ) a, b, c any values
dx

5  5 
0.5
5 13 A1
x= , y = 2 3× + 4 − =
3  3  3 3

4(c) d2 y 9 −1.5 M1 dy dy
= − ( 3x + 4 ) Differentiating their OR checking to find +ve
dx 2
2 dx dx
5
and -ve either side of their x =
3

5 d2 y A1
At x = is negative so the point is a maximum
3 dx 2

251
9709/11 M/J 21/P11/Q11 Cambridge International AS & A Level – Mark Scheme Unit 7: Answer Section May/June 2021

Question Answer Marks Guidance

4(d) 4 1 B1 B1 B1 for each correct term (unsimplified)


Area =   2 ( 3 x + 4 ) − x dx =  ( 3x + 4 ) − x 2
0.5 1.5
  9 2

4 1 2  4 1.5 256 32 M1 Substituting limits 0 and 4 into an expression obtained


 (16 ) − ( 4 )  − ( 4 ) =
1.5
−8− by integrating y
9 2  9 9 9

8 A1 152
16 Or
9 9

9709/13 Cambridge International AS & A Level – Mark Scheme May/June 2021

Question M/J 21/P13/Q2 Answer Marks Guidance

5 B2, 1, 0 Expect ( 2 x − 1)1/ 2 − 2


f ( x ) = 
 −1  (( 2 x − 1)1/2 ) × 
1
3
3
× 2 ×  ( −2 )
2

( 2 x − 1)1/ 2 − 2  0 → 2 x − 1  4 or 2 x − 1 < 4 M1 SOI. Rearranging and then squaring, must have


power of ½ not present
Allow ‘=0’at this stage but do not allow ‘ ≥ 0’ or
‘ ˃ 0’
If ‘–2’ missed then must see ⩽ or < for the M1

Value [of a] is 2½ or a = 2½ A1 5
WWW, OE e.g. , 2.5
2
Do not allow from ‘=0’ unless some reference to
negative gradient.

252
Unit 7: Answer Section
9709/12 Cambridge International AS & A Level – Mark Scheme October/November 2020
PUBLISHED
Question O/N 20/P12/Q7/a Answer Marks Guidance

6(a)  5 *M1 Substituting 4 into f ' ( x )


f '( 4)  = 
 2

 dy dy dx   dy  5 DM1 Multiplies their f ' ( 4 ) by 0.12


 = ×  →   = × 0.12
 dt dx dt   dt  2

 dy  A1 OE
 =  0.3
 dt 
9709/11 Cambridge International AS & A Level – Mark Scheme October/November 2020
Question O/N 20/P11/Q3 Answer Marks Guidance

7 B1 dr
(Derivative =) 4πr 2 (→ 400π) SOI Award this mark for
dV
50 M1 Can be in terms of r
their derivative

1 A1 AWRT
or 0.0398 3

8 dy  1 −1/2  B1 B1
=
dx  2
(
25 − x 2 )  × [ −2 x ]

−x 4 x2 16 M1 4
= → = Set = and square both sides
( 25 − x ) 2 1/2 3
2
3 25 − x 9

( )
16 25 − x 2 = 9 x 2 → 25 x 2 = 400 → x = ( ± ) 4 A1

When x = ‒ 4, y = 5 → (‒ 4, 5) A1

253
Unit 7: Answer Section
9709/13 Cambridge International AS & A Level – Mark Scheme October/November 2020

Question O/N 20/P13/Q8 Answer Marks Guidance

9(a) dy B1 B1
= [ 2]
−2
[− 2 ( 2 x + 1) ]
dx

d2 y −3 B1
= 8 ( 2 x + 1)
dx 2 3
9(b) dy M1
Set their = 0 and attempt solution
dx

(2x + 1)2 = 1 → 2x + 1 = ( ± ) 1 or 4x2 + 4x = 0 → (4)x(x + 1) = 0 M1 Solving as far as x = ....

x=0 A1 WWW. Ignore other solution.

(0, 2) A1 One solution only. Accept x = 0, y = 2 only.

d2 y 1 B1 d2 y
2
> 0 from a solution x > − hence minimum Ignore other solution. Condone arithmetic slip in value of .
dx 2 dx 2
d2 y −3
Their 2
must be of the form k ( 2 x + 1)
5 dx

9709/12 Cambridge International AS & A Level – Mark Scheme May/June 2020

10(a) 4 M1
Volume after 30 s = 18000 πr ³ = 18000
3

r = 16.3 cm A1 2

10(b) dV B1
= 4πr ²
dr

dr dr dV 600 M1
= × =
dt dV dt 4πr ²

dr A1
= 0.181 cm per second
dt 3

254
Unit 7: Answer Section
9709/12 Cambridge International AS & A Level – Mark Scheme May/June 2020

Question Answer Marks

11(a) dy B2,1
= 54 – 6(2x – 7)²
dx
d² y B2,1 FT
= −24(2x – 7)
dx ²
(FT only for omission of ‘ ×2 ’ from the bracket) 4

11(b) dy M1
= 0 → ( 2x − 7) = 9
2

dx
x = 5, y = 243 or x = 2, y = 135 A1 A1 3
11(c) d² y B1FT
x=5 = −72 → Maximum
dx ²
(FT only for omission of ‘ ×2 ’ from the bracket)
d² y B1FT
x =2 = 72 → Minimum
dx ²
(FT only for omission of ‘ ×2 ’ from the bracket) 2

9709/11 M/J 20/P11/Q9 Cambridge International AS & A Level – Mark Scheme May/June 2020

Question Answer Marks

12(a) dy B1B1
= 3(3−2x)² × −2 + 24 = −6 ( 3 − 2 x ) + 24
2

dx
(B1 without ×−2. B1 for ×−2)

d² y B1FT
= −12 ( 3 − 2 x ) ×−2 = 24(3 – 2x) B1
dx²
(B1FT from without – 2) 4
12(b) dy M1
= 0 when 6 ( 3 − 2 x ) = 24 → 3 − 2 x = ±2
2

dx

x = ½, y = 20 or x = 2½, y = 52 A1A1
(A1 for both x values or a correct pair) 3

255
Unit 7: Answer Section
M/J 20/P11/Q9
12(c) d² y B1FT
If x = ½, = 48 Minimum
dx ²

d² y B1FT
If x = 2½, = −48 Maximum
dx² 2
9709/13 Cambridge International AS & A Level – Mark Scheme May/June 2020

Question Answer Marks

13(a) dy  1 −1/2  B1 B1
=  ( 5x − 1)  × [5]
dx  2 

dy  dy  M1
Use = 2 ×  their when x = 1
dt  dx 

5 A1
2

13(b) 5 5 M1
2 × their ( 5 x − 1)−1/2 = oe
2 8

( 5 x − 1)1/2 = 8 A1

x = 13 A1

256
9709/12 Cambridge International AS & A Level – Mark Scheme Unit 7: Answer Section March 2020

Question Answer Marks Guidance

14 f ′ ( x ) =  − ( 3 x + 2 )  × [3] + [ 2 x ]
−2 B2, 1, 0
 

< 0 hence decreasing B1 Dependent on at least B1 for f ′ ( x ) and must include < 0 or
‘(always) neg’

15 dy B1
= 2x − 2
dx

dy 4 B1 OE, SOI
=
dx 6

4 M1 dy
their ( 2 x − 2 ) = their LHS and RHS must be their expression and value
6 dx

4 A1
x= oe
3

257
Unit 7: Answer Section
9709/12 Cambridge International AS/A Level – Mark Scheme October/November 2019

Question O/N 19/P12/Q5 Answer Marks Guidance

16(i) Use of Pythagoras → r2 = 152 − h2 M1

V = ⅓π(225 − h2) × h → ⅓π(225h – h3) A1 AG


WWW e.g. sight of r = 15 – h gets A0.

16(ii)  dv  π B1
 =  (225 – 3h
3h2)
 dh  3

dv M1 Differentiates, sets their differential to 0 and attempts to solve


Their =0 at least as far as h2 ≠ 0.
dh

(ℎ =) √75, 5√
5√3 or AWRT 8.66 A1 Ignore − 75 OE and ISW for both A marks

d2h π M1 Differentiates for a second time and considers the sign of the
= (–6
(–6h (→ −ve)
h) (→ second differential or any other
other valid complete method.
dh 2 3

→ Maximum A1FT Correct conclusion from correct 2nd differential, value for h
not required, or any other valid complete method. FT for their
h, if used, as long as it is positive.

π
SC Omission of π or throughout can score B0M1A1M1A0
3

258
Unit 7: Answer Section
9709/11 Cambridge International AS/A Level – Mark Scheme October/November 2019

Question O/N 19/P11/Q2 Answer Marks Guidance

17 Attempt to solve f ′ ( x ) = 0 or f ′ ( x ) > 0 or f ′ ( x ). 0 M1 SOI

(x – 2)(x – 4) A1 2 and 4 seen

(Least possible value of n is) 4 A1 Accept n = 4 or n . 4

O/N 19/P11/Q3 3

18 dy B1
= 6 x 2 − 10 x − 3
dx
dy M1
At x = 2, = 24 − 20 − 3 = 1 → a = 1 A1
dx
6= 2+b → b = 4 B1FT Substitute x = 2, y = 6 in y = ( their a ) x + b

6 = 16 − 20 − 6 + c → c = 16 B1 5 Substitute x =2, y = 6 into equation of curve

9709/13 O/N 19/P13/Q3 Cambridge International A Level – Mark Scheme October/November 2019

Question Answer Marks Guidance

19 dy B1
= 3x2 + 2x ‒ 8
dx

Set to zero (SOI) and solve M1

(Min) a = ‒2, (Max) b = 4/3. – in terms of a and b. A1 Accept a . − 2, b - 4


3
A1
SC: A1 for a > −2, b < 4
3
or for −2 < x < 4
3

259
9709/13 O/N 19/P13/Q5 Unit 7: Answer Section October/November 2019

Question Answer Marks Guidance

20(i) S = 28x2, V = 8x3 B1B1 SOI

2 B1 AG, WWW
7V 3 = 7 × 4 x 2 = S

20(ii)  dS  14V
− 13
14 *M1 Attempt to differentiate
 = = SOI when V = 1000 A1  dS  − 13
 dV  3 30 For M mark   to be of form kV
 dV 

 dV dS dV  dS 1 DM1
 = ×  OE used with dt = 2 and 14
 dt d t dS  their 30

30 A1 OE
or 4.29
7

Alternative method for question 5(ii)


3
S2  dV  3 1 1 30 *M1 Attempt to differentiate
V= → = ×S ×
2
= SOI when S = 700 A1  dV  1
7 7  dS  2 7 7 14 For M mark   to be of form kS
2

 d S 

 dV dS dV  dS 1 DM1
 = ×  OE used with dt = 2 and
 dt d t dS  their 14
30

30 A1 OE
or 4.29
7

260
Unit 7: Answer Section
9709/13 Cambridge International A Level – Mark Scheme October/November 2019

Question Answer Marks Guidance

21(i) M1 SOI
( 2x − 1) < 2 or 3( 2 x − 1) 2 < 6
1 1
2

2x −1 < 4 A1 SOI

1 5 A1 A1 Allow 2 separate statements


<x<
2 2 4
21(ii) 3 B1 B1
f(x) = [3 ( 2 x − 1) ÷   ÷ ( 2) ] [ − 6 x]
3/ 2
(+c)
2

Subsitute x = 1, y = ‒3 into an integrated expression. M1 Dependent on c being present (c = 2)

A1
f(x) = ( 2 x − 1) 2 − 6 x + 2
3

9709/12 Cambridge International AS/A Level – Mark Scheme May/June 2019

Question Answer Marks Guidance

22(i) dy dy dx M1 Multiply numerical gradient at x = 2 by ±0.05.


= × = 7 × – 0.05
dt dx dt

−0.35 (units/s) or Decreasing at a rate of (+) 0.35 A1 Ignore notation and omission of units

22(ii) x4 4 B1 Accept unsimplified


( y) = + (+c) oe
4 x

Uses (2, 9) in an integral to find c. M1 The power of at least one term increase by 1.

x4 4 A1 A0 if candidate continues to a final equation that is


c = 3 or ( y = ) + + 3 oe a straight line.
4 x

261
Unit 7: Answer Section
9709/12 Cambridge International AS/A Level – Mark Scheme May/June 2019

Question Answer Marks Guidance

23 dy B1
For C1: = 2x – 4 → m = 2
dx

y – ‘their 4’ = ‘their m’ (x – 3) or using y = mx + c M1 dy


Use of : and (3, their 4) to find the tangent
dx
equation.

y – 4 = 2( x – 3) or y = 2 x − 2 A1 If using = mx + c , getting c = −2 is enough.

2x – 2= 4 x + k (→ 4 x ² − 12 x + 4 − k = 0 ) *M1 Forms an equation in one variable using tangent &


C2

Use of b² − 4ac = 0 on a 3 term quadratic set to 0. *DM1 Uses ‘discriminant = 0’

144 = 16(4 – k) → k = − 5 A1

4 x² − 12 x + 4 − k = 0 → 4 x² − 12 x + 9 = 0 DM1 Uses k to form a 3 term quadratic in x

3  1 A1 Condone ‘correct’ extra solution.


x= or , y = 1(or – 1).
2  2 

Alternative method for question 9

dy B1
For C1: = 2x – 4 → m = 2
dx

y – ‘their 4’ = ‘their m’ (x – 3) or using y = mx + c M1 dy


Use of : and (3, their 4) to find the tangent
dx
equation.

y – 4 = 2( x – 3) or y = 2 x − 2 A1 If using = mx + c , getting c = −2 is enough.

dy −
1 *M1 dy −
1
For C2: = A(4 x + k ) 2 Finds for C2 in the form A(4 x + k ) 2
dx dx

262
9709/13 Cambridge International AS/A Level – Mark Scheme Unit 7: Answer Section May/June 2019

Question Answer Marks Guidance

24 f ′ ( −1) = 0 → 3 − a + b = 0 f ′ ( 3) = 0 → 27 + 3a + b = 0 M1 Stationary points at x = ‒1 & x = 3 gives sim. equations in a & b

a = ‒6 A1 Solve simultaneous equation

b = ‒9 A1

Hence f ′ ( x ) = 3 x 2 − 6 x − 9 → f ( x ) = x 3 − 3 x 2 − 9 x ( + c ) B1 FT correct integration for their a,b (numerical a, b)

2 = −1 − 3 + 9 + c M1 Sub x = ‒1, y = 2 into their integrated f(x). c must be present

c = ‒3 A1 FT from their f(x)

f ( 3) = k → k = 27 − 27 − 27 − 3 M1 Sub x = 3, y = k into their integrated f(x) (Allow c omitted)

k = −30 A1 8

9709/12 Cambridge International AS/A Level – Mark Scheme March 2019

Question Answer Marks Guidance

25(i) dy / dx = −2 ( 2 x − 1) + 2 B2,1,0 Unsimplified form ok (–1 for each error in ‘–2’,


−2

‘ ( 2 x − 1) ’ and ‘2’)
−2

d 2 y / dx 2 = 8 ( 2 x − 1) B1 Unsimplified form ok
−3
3
25(ii) Set dy / dx to zero and attempt to solve – at least one correct step M1

x = 0, 1 A1 Expect ( 2 x − 1)2 = 1

When x = 0, d 2 y / dx 2 = −8 (or < 0) . Hence MAX B1

When x = 1, d 2 y / dx 2 = 8 (or > 0) . Hence MIN B1 Both final marks dependent on correct x and correct
d 2 y / dx 2 and no errors
4 May use change of sign of dy / dx but not at x = 1 / 2

263
9709/12 Cambridge International AS/A Level – Mark Scheme Unit 7: Answer Section March 2019
PUBLISHED
Question MAR 19/P12/Q10 Answer Marks Guidance

26(i) 4 x1/ 2 = x + 3 → M1 Eliminate y from the 2 equations and then:


( x1/ 2 ) 2 − 4 x1/2 + 3 ( = 0 ) OR 16 x = x 2 + 6 x + 9 Either treat as quad in x1/ 2 OR square both sides and
RHS is 3-term

x1/ 2 = 1or 3 x 2 − 10 x + 9 ( = 0 ) A1 If in 1st method x1/ 2 becomes x, allow only M1 unless


subsequently squared

x = 1 or 9 A1

y = 4 or12 A1ft Ft from their x values


If the 2 solutions are found by trial substitution B1 for
the first coordinate and B3 for the second coordinate

AB 2 = ( 9 − 1) + (12 − 4 ) M1
2 2

AB = 128 or 8 2 oe or 11.3 A1 6

26(ii) dy/dx = 2 x −1/ 2 B1

2 x −1/ 2 = 1 M1 Set their derivative = their gradient of AB and attempt


to solve

(4, 8) A1 Alternative method without calculus:


MAB = 1, tangent is y = mx + c where m = 1 and meets
y = 4x1/2 when 4x1/2 = x + c. This is a quadratic with
b2 = 4ac, so 16 – 4 × 1 × = 0 so c = 4 B1 Solving
3 4x1/2 = x + 4 gives x = 4 and y = 8 M1A1

26(iii) Equation of normal is y − 8 = −1( x − 4 ) M1 Equation through their T and with gradient ‒1/their
gradient of AB. Expect y = − x + 12 ,

Eliminate y (or x) → − x + 12 = x + 3 or y − 3 = 12 − y M1 May use their equation of AB

(4½, 7½) A1

264
Unit 7: Answer Section
9709/12 Cambridge International AS/A Level – Mark Scheme October/November 2018

Question O/N 18/P12/ Q3 Answer Marks Guidance

27(i) P is (t, 5t) Q is (t, t(9 – t²)) → 4t – t³ B1 B1 B1 for both y coordinates which can be implied by
subsequent working. B1 for PQ allow 4t – t ³ or t ³ – 4t .
Note: 4x – x3 from equating line and curve 0/2 even if x then
[2] replaced by t.

27(ii) d ( PQ ) B1FT B1FT for differentiation of their PQ, which MUST be a cubic
= 4 – 3t² d
dt expression, but can be f ( x ) from (i) but not the equation
dx
of the curve.

2 M1 Setting their differential of PQ to 0 and attempt to solve for t


=0→t=+ or x.
√3

16 16 3 A1 Allow 3.08 awrt. If answer comes from wrong method in (i)


→ Maximum PQ = or award A0.
3√ 3 9 3 Correct answer from correct expression by T&I scores 3/3.

9709/11 O/N 18/P11/ Q6 October/November 2018

28(i) 0 = 9a + 3a 2 M1 dy
Sub = 0 and x = 3
dx

a = −3 only A1 2
28(ii) dy 9x2 M1A1FT Attempt integration. 3
+ ½ a 2 x 2 scores M1. Ft on
3 ax
1
= −3x 2 + 9 x → y = − x3 + ( +c )
dx 2 their a.

9½ = −27 + 40½ + c DM1 Sub x = 3, y = 9½ . Dependent on c present

c = −4 A1 9 x2
Expect y = − x3 + −4
4 2

28(iii) d2 y M1 2ax + a 2 scores M1


= −6 x + 9
dx 2

d2 y A1 Requires at least one of ‒9 or < 0. Other methods possible.


At x = 3, = −9 < 0 MAX www
dx 2 2

265
Unit 7: Answer Section
9709/11 Cambridge International AS/A Level – Mark Scheme October/November 2018
PUBLISHED
Question O/N 18/P11/ Q10 Answer Marks Guidance

29(i)(a) dy  B1B1 Can gain this in part (b)(ii)


= −½ ( 4 x − 3)  × [ 4]
−2

dx  

When x = 1, m = −2 B1FT dy
Ft from their
dx

Normal is y − ½ = ½ ( x − 1) M1 Line with gradient ‒1/m and through A

y = ½ x soi A1 Can score in part (b)

29(i)(b) M1A1 x/2 seen on RHS of equation can score previous A1


1 x
(
= → 2 x ( 4 x − 3) = 2 → ( 2 ) 4 x 2 − 3 x − 1
2 ( 4 x − 3) 2
) ( = 0)
x = −1 / 4 A1 Ignore x = 1 seen in addition

29(ii) dy M1A1 Allow +0.3 or ‒0.3 for M1


Use of chain rule: = ( their − 2 ) × ( ± ) 0.3 = 0.6
dt 2

9709/13 Cambridge International AS/A Level – Mark Scheme October/November 2018

Question O/N 18/P13/ Q2 Answer Marks Guidance

30 f ′ ( x ) = 3x 2 + 4 x − 4 B1

Factors or crit. values or sub any 2 values ( x ≠ −2) into f ′ ( x ) soi M1 Expect ( x + 2 )( 3x − 2 ) or ‒2, ⅔ or any 2 subs
(excluding x = ‒2).

For −2 < x < 2 3 , f ′ ( x ) < 0; for x > 2 3 , f ′ ( x ) >0 soi Allow - , . M1 Or at least 1 specific value (≠ −2) in each interval giving opp
signs
Or f′( 2 3 )=0 and f′′( 2 3 ) ≠ 0 (i.e. gradient changes sign at x = 2 3 )

266
O/N 18/P13/ Q2 Unit 7: Answer Section

30 Neither www A1 Must have ‘Neither’

ALT 1 At least 3 values of f(x) M1 e.g. f(0) = 7, f(1) = 6, f(2) = 15

At least 3 correct values of f(x) A1

At least 3 correct values of f(x) spanning x = ⅔ A1

Shows a decreasing and then increasing pattern. Neither www A1 Or similar wording. Must have ‘Neither’

ALT 2 f ′ ( x ) = 3 x 2 + 4 x − 4 = 3 ( x + 2 3 ) − 16 B1B1 Do not condone sign errors


2
3

16 M1
f ′( x) . −
3

f ′ ( x ) < 0 for some values and > 0 for other values. Neither www A1 Or similar wording. Must have ‘Neither’

9709/11 31 M/J 18/P11/ Q2 Cambridge International AS/A Level – Mark Scheme May/June 2018

31 5 dy 5 M1 A1 Reasonable attempt at differentiation CAO (−3)


y = 2x + → =2− = −3 (may be implied) when x = 1.
x dx x²

dy dy dx M1 A1 dy
= × → −0.06 Ignore notation, but needs to multiply by 0.02.
dt dx dt 4 dx

9709/13 O/N 17/P12/ Q8 May/June 2018

Question Answer Marks Guidance

32(i) dy M1A1 Attempt to differentiate. All correct for A mark


= 3 x 2 − 18 x + 24
dx

3 x 2 − 18 x + 24 = −3 M1 dy
Equate their to ‒3
dx
x=3 A1

y=6 A1

y − 6 = −3 ( x − 3) A1FT 6 FT on their A. Expect y = −3x + 15

267
Unit 7: Answer Section
9709/13 O/N 17/P12/ Q8 May/June 2018

32(ii) ( 3)( x − 2 )( x − 4 ) SOI or x = 2, 4 Allow ( 3)( x + 2 )( x + 4 ) M1 Attempt to factorise or solve. Ignore a RHS, e.g. = 0 or > 0, etc.

Smallest value of k is 4 A1 2 Allow k . 4 . Allow k = 4. Must be in terms of k

9709/12 Cambridge International AS/A Level – Mark Scheme October/November


2017

Question Answer Marks Guidance

33(i) dy M1 dy
=0 Sets to 0 and attempts to solve leading to two values for x.
dx dx

x = 1, x = 4 A1 Both values needed

2
33(ii) d² y B1
= −2 x + 5
dx ²

d² y M1 Evidence of any valid method for both points.


Using both of their x values in their
dx ²

x = 1 → (3) → Minimum, x = 4 →(-3) → Maximum A1

33(iii) x3 5 x 2 B2, 1, 0 +c not needed. –1 each error or omission.


y=− + − 4 x (+c)
3 2

Uses x =6, y = 2 in an integrand to find c → c = 8 M1 A1 Statement of the final equation not required.

268
Unit 7: Answer Section
9709/11 O/N 17/P11/ Q1,4 Cambridge International AS/A Level – Mark Scheme October/November
PUBLISHED 2017
Question Answer Marks Guidance

34 dy B2,1,0
= 3 x1/ 2 − 3 − 2 x −1/ 2
dx

dy M1
at x = 4, =6‒3‒1=2
dx

Equation of tangent is y = 2 ( x − 4 ) OE A1FT Equation through (4, 0) with their gradient

35(i) 1 1 B1 AG
V = π r 2 (18 − r ) = 6π r 2 − π r 3
3 3

35(ii) dV M1 Differentiate and set = 0


= 12π r − π r 2 = 0
dr

π r (12 − r ) = 0 → r = 12 A1

d 2V M1
= 12π − 2π r
dr 2

Sub r = 12 → 12π − 24π = −12π → MAX A1 AG

35(iii) Sub r = 12, h = 6 → Max V = 288π or 905 B1

269
9709/13 Unit 7: Answer Section Cambridge International AS/A Level – Mark Scheme October/November 2017
Question O/N 17/P13/ Q4 Answer Marks Guidance

36  3   B2, 1, 0 Deduct 1 mark for each [...] incorrect.


f ' ( x ) =   (2 x − 1)1/ 2  × [ 2] − [ 6]
2
  

f ' ( x ) < 0 or - 0 or = 0 SOI M1

( 2 x − 1) A1 Allow with k used instead of x


1/2
< 2 or - 2 or = 2 OE

5 A1 5 5
Largest value of k is Allow k - or k = Answer must be in terms of k (not x)
2 5 2 2

37 O/N 17/P13/ Q10

37(i) −b B1
ax 2 + bx = 0 → x ( ax + b ) = 0 → x =
a

−b M1
Find f″ ( x ) and attempt sub their into their f″ ( x )
a
−b  −b  A1
When x = , f ″ ( x ) = 2a   + b = −b MAX
a  a  3
37(ii) Sub f ' ( −2 ) = 0 M1

Sub f ' (1) = 9 M1

a =3 b=6 *A1 Solve simultaneously to give both results.

f ' ( x ) = 3x 2 + 6 x → f ( x ) = x3 + 3x 2 ( +c ) *M1 Sub their a, b into f ' ( x ) and integrate ‘correctly’. Allow
ax3 bx 2
+ ( +c )
3 2

−3 = −8 + 12 + c DM1 Sub x = −2, y = −3 . Dependent on c present. Dependent also on a,


b substituted.

f ( x ) = x3 + 3x 2 − 7 A1 6

270
9709/12 M/J 17/P12/ Q5 Cambridge International AS/A Level – Mark Scheme Unit 7: Answer Section May/June 2017
PUBLISHED
Question Answer Marks Guidance
38(i) Crossesx-axis at (6, 0) B1 x = 6 is sufficient.

dy B2,1,0 −1 for each incorrect term of the three or addition of + C.


= (0 +) −12 (2 – x)−2 × (−1)
dx

Tangent y = ¾ ( x − 6 ) or 4 y = 3x − 18 M1 A1 Must use dy/dx, x= their 6 but not x = 0 (which gives m = 3), and correct
form of line equation.

Using y = mx + c gets A1 as soon as c is evaluated.

Total: 5

38(ii) Ifx = 4, dy/dx = 3

dy M1 A1FT dy
= 3 × 0.04 = 0.12 M1 for (“their m” from and x = 4) × 0.04.
dt dx
Total: 2 Be aware: use of x = 0 gives the correct answer but gets M0.

9709/12 M/J 17/P12/ Q9 Cambridge International AS/A Level – Mark Scheme May/June 2017

Question Answer Marks Guidance

39(i) dy B1 Accept unsimplified.


= 4 x −½ − 2
dx

= 0 when x =2

x = 4, y = 8 B1B1

Total: 3

39(ii) d² y −
3 B1FT FT providing –ve power of x
= −2 x 2
dx ²

271
Unit 7: Answer Section

 d² y 1 B1 d² y
 = −  → Maximum Correct and x=4 in (i) are required.
 dx ² 4 dx ²
d² y
Followed by“< 0 or negative” is sufficient” but must be correct if
dx ²
evaluated.
Total: 2

39(iii) EITHER: (M1 Eg x =u → 2u ² − 8u + 6 = 0


Recognises a quadratic in x

1 and 3 as solutions to this equation A1

→ x = 9, x = 1. A1)

OR: (M1 x needs to be isolated before squaring both sides.


Rearranges then squares

→ x 2 − 10 x + 9 = 0 oe A1

→ x = 9, x = 1. A1) Both correct by trial and improvement gets 3/3

Total: 3

39(iv) k>8 B1 1

9709/11 O/N 16/P12/ Q7 Cambridge International AS/A Level – Mark Scheme May/June 2017

Question Answer Marks Guidance

40(i) 1 3 8000 M1 Use of (area of triangle, with attempt at ht) × h =2000, h = f ( x )
Volume =   x ² h = 2000 → h =
2 2 √ 3x ²

1 3 M1 Uses 3 rectangles and at least one triangle


A = 3 xh + ( 2 ) ×   × x 2 ×
2 2

√ 3 2 24 000 −1 A1 AG
Sub for h → A = x + x
2 3

Total: 3

272
Unit 7: Answer Section
9709/11 O/N 16/P12/ Q7 Cambridge International AS/A Level – Mark Scheme May/June 2017

Question Answer Marks Guidance

40(ii) dA 3 24000 −2 B1 CAO, allow decimal equivalent


= 2x − x
dx 2 3

= 0 when x³ = 8000 → x = 20 M1 A1 dA
Sets their to 0 and attempt to solve for x
Total: 3 dx

40(iii) d² A 3 48000 −3 M1 d² A
= 2+ x >0 Any valid method, ignore value of providing it is positive
dx ² 2 3 dx ²

→ Minimum A1 FT FT on their x providing it is positive

Page 6 Mark Scheme Syllabus Paper


Cambridge International AS/A Level – October/November 2016 9709 12

dy −3
41(i) = ×2 B1 B1for a single correct term (unsimplified)
dx ( 2 x − 1)2 without ×2.
B1
[2]

dy
(ii) e.g. Solve for = 0 is impossible. B1 Satisfactory explanation.
dx [1]

dy −6
(iii) If x = 2, = and y = 3 M1* Attempt at both needed.
dx 9
9
Perpendicular has m = M1* Use of m1m2 = −1 numerically.
6
3
→ y − 3 = ( x − 2) DM1 Line equation using (2, their 3) and their m.
2
Shows when x=0 then y=0 AG A1
[4]

dx
(iv) = −0.06
dt
dy dy dx 2
= × → − × −0.06 = 0.04 M1 A1
dt dx dt 3 [2]
273
Unit 7: Answer Section
Page Mark Scheme Syllabus Paper
Cambridge International AS/A Level – October/November 2016 9709 11

dy
= − ( x − 1) + 9 ( x − 5)
−2 −2
42 (i) M1A1 May be seen in part (ii)
dx
1 9
mtangent = − + = 2 B1
4 4
Equation of normal is y − 5 = −½ ( x − 3) M1 Through (3, 5) and with
m = −1 / mtangent
x = 13 A1
[5]

(ii) dy
( x − 5) = 9 ( x − 1)
2 2
B1 Set = 0 and simplify
dx
x − 5 = ( ± ) 3 ( x − 1) or (8) ( x 2 − x − 2 ) = 0 M1 Simplify further and attempt
solution
x = −1 or 2 A1
d2 y
= 2 ( x − 1) − 18 ( x − 5 )
−3 −3
B1 If change of sign used, x values
dx 2 close to the roots must be used
and all must be correct
d2 y 1
When x = −1, 2
=− <0 MAX B1
dx 6
d2 y 8
When x = 2, 2 = >0 MIN B1
dx 3 [6]

Page Mark Scheme Syllabus Paper


Cambridge International AS/A Level – October/November 2016 9709 13

43 f ′ ( x ) = 3 x 2 − 6 x − 9 soi B1
Attempt to solve f ′ ( x ) = 0 or f ( x ) > 0 or f ( x ) . 0 soi
' '
M1
( 3)( x − 3)( x + 1) or 3,−1 seen or 3 only seen A1 With or without
equality/inequality signs
Least possible value of n is 3. Accept n = 3. Accept n . 3 A1 Must be in terms of n
[4]

Page Mark Scheme Syllabus Paper


Cambridge International AS/A Level – May/June 2016 9709 11

44 (i) A = 2 y × 4 x ( = 8 xy ) B1
10 y + 12 x = 480 B1
B1 answer given
→ A = 384x – 9.6x² [3]
dA
(ii) = 384 − 19.2x B1
dx
= 0 when x = 20 M1 Sets to 0 and attempt to solve oe
Might see completion of square

→ x = 20, y = 24. A1 Needs both x and y

[3] Trial and improvement B3.


b −384
Uses x = − = = 20 , M1, A1
2a −19.2
y = 24 , A1
From graph: B1 for x = 20, M1, A1 for
y = 24

274
Page Mark Scheme Syllabus Paper
Cambridge International AS/A Level – May/June 2016 9709 13

dy −2
45 = [8] + [−2] [ ( 2 x − 1) ] B2,1,0
dx
2
= 0 → 4 ( 2 x − 1) = 1 oe eg 16 x 2 − 16 x + 3 = 0 M1 Set to zero, simplify and attempt to
solve soi
1 3
x= and A1 Needs both x values. Ignore
4 4 y values
d2 y −3
ft to k ( 2 x − 1)
−3
2
= 8 ( 2 x − 1) B1 * where k > 0
dx
1 d2 y
When x = , ( = −64 ) and/or < 0 MAX DB1 Alt. methods for last 3 marks
4 dx 2 (values either side of 1/4 & 3/4)
3 d2y must indicate which x-values and
When x = , ( = 64 ) and/or > 0 MIN DB1
cannot use x = 1/2. (M1A1A1)
4 dx 2 [7]

Page Mark Scheme Syllabus Paper


Cambridge International AS/A Level – May/June 2016 9709 13

46 dy
= 2 x − 5 x1/ 2 + 5 B1
dx
dy
= 2 B1
dx
2 x − 5 x1/2 + 5 = 2 M1 Equate their dy/dx to their 2 or ½.
2 x − 5 x1/ 2 + 3 ( = 0 ) or equivalent 3-term
quadratic A1
Attempt to solve for x1/2 e.g.
( 2x 1/ 2
)( )
− 3 x1/2 − 1 = 0 DM1 Dep. on 3-term quadratic
x1/2 = 3 / 2 and 1 A1 ALT
x = 9 / 4 and 1 A1 5 x½ = 2 x + 3 → 25 x = ( 2 x + 3)
2

[7]
4 x 2 − 13 x + 9 ( = 0 )
x = 9 / 4 and 1

Page 4 Mark Scheme Syllabus Paper


Cambridge International AS/A Level – October/November 2015 9709 12

x
47 (i) tan 60 = → x = h tan60 B1 Any correct unsimplified length
h
A=h×x M1 Correct method for area
V = 40 (3h ) 2 A1 ag
[3]

275
Page Mark Scheme Syllabus Paper
Cambridge International AS/A Level – October/November 2015 9709 12

12
48 f ʹʹ(x) =
x3

6
(i) f ʹ(x) = − ( + c) B1 Correct integration
x2
3
= 0 when x = 2 → c = M1 A1 Uses x = 2, f ʹ(x = 0)
2
6 3x
f(x) = + (+A) B1 B1 For each integral
x 2
= 10 when x = 2 → A = 4 A1
[6]

Page Mark Scheme Syllabus Paper


2 Cambridge International AS/A Level – October/November 2015 9709 11

49 f ( x) = x 3 − 7 x (+c) B1
5 = 27 − 21 + c M1 Sub x = 3, y = 5. Dep. on c present
c = −1 → f ( x) = x 3 − 7 x − 1 A1
[3]

50 0(i) dy 8
= − 2 + 2 cao B1B1
dx x
d 2 y 16
= cao B1
dx 2 x 3
[3]
8
(ii) − + 2 = 0 → 2x 2 − 8 = 0 M1 Set = 0 and rearrange to quadratic form
x2
x = ±2 A1
y = ±8 A1 If A0A0 scored, SCA1 for just (2, 8)

d2 y Ft for " correct" conclusion if 


> 0 when x = 2 hence MINIMUM B1  2 
dx 2 d y 
 2 incorrect or 
d2 y B1  dx 
< 0 when x = −2 hence MAXIMUM
dx 2 any valid method inc. a good sketch
[5]  

51 (i) x 2 − x + 3 = 3x + a → x 2 − 4 x + (3 − a) = 0 B1 AG
[1]

(ii) 5 + (3 − a ) = 0 → a = 8 B1 Sub x = −1 into (i)


2
x − 4x − 5 = 0 → x = 5 B1 OR B2 for x = 5 www
[2]

(iii) 16 − 4(3 − a ) = 0 (applying b 2 − 4ac = 0 ) M1 OR dy dx = 2 x − 1 → 2 x − 1 = 3


a = −1 A1 x=2
( x − 2) 2 = 0 → x = 2 A1 y = 22 − 2 + 3 → y = 5
y =5 A1 5 = 6 + a → a = −1
[4]

276
Page Mark Scheme Unit 7: Answer Section Syllabus Paper
Cambridge International AS/A Level – October/November 2015 9709 13

5
52 (i) [3] [( x − 1) 2 ] [− 1] B1B1B1
[3]
(ii) f ' ( x) = 3x 2 − 6 x + 7 B1 Ft their (i) + 5
= 3( x − 1) 2 + 4 B1
> 0 hence increasing DB1 Dep B1√ unless other valid reason
[3]

(i) dy
53 At x = 4, =2 B1
dx
dy dy dx
= × = 2×3= 6 M1A1 Use of Chain rule
dt dx dt [3]
1
(ii) ( y ) = x + 4 x 2 (+c) B1
1
Sub x = 4, y = 6 → 6 = 4 + (4 × 42 ) +c M1 Must include c
1
c = −6 → ( y = x + 4x 2 −6 A1
[3]
(iii) Eqn of tangent is y − 6 = 2( x − 4) or M1 Correct eqn thru (4, 6) & with m =
(6 − 0) /( 4 − x) = 2 A1 their 2
B = (1, 0) (Allow   1) M1 [Expect eqn of normal:   ½ 
Gradient of normal = −1/2 A1 8]
C = (16, 0) (Allow   16) A1
1 [5] Or AB = 45 , AC = 180 →
Area of triangle = × 15 × 6 = 45
2 Area = 45.0

Page M/J 15/P12/ Q4 Mark Scheme Syllabus Paper


Cambridge International AS/A Level – May/June 2015 9709 12

54 u = 2 x( y − x) and x + 3 y = 12 ,
 12 − x 
u = 2 x − x M1 A1 Expresses u in terms of x
 3 
2
= 8 x − 83x
du 16 x M1 Differentiate candidate’s quadratic,
= 8−
dx 3 sets to 0 + attempt to find x, or
A1 other valid method
= 0 when x = 1 12
→ ( y = 3 12 ) A1 Complete method that leads to u
→u=6 [5] Co

Page M/J 15/P11/ Q2 Mark Scheme Syllabus Paper


Cambridge International AS/A Level – May/June 2015 9709 11

55 y = 2x² , X(−2, 0) and P(p, 0)


(i) 1 M1 A1 Attempt at base and height in terms
A= × (2 + p) × 2p² (= 2p² + p³) [2] bh
2 of p and use of
dA 2
(ii) = 4p + 3p² B1 cao
dp
dA d A dp
= × = 0.02 × 20 = 0.4 M1 A1 any correct method, cao
dt dp d t
dA dp dp
or = 4p + 3 p2
dt dt dt [3]
277
Page M/J 15/P11/ Q2 Mark Scheme Syllabus Paper
Cambridge International AS/A Level – May/June 2015 9709 11

56 y = x 3 + px 2 M/J 15/P11/ Q9
dy
(i) = 3x² + 2px B1 cao
dx
2p
Sets to 0 → x = 0 or − M1 Sets differential to 0
3
 2 p 4 p3 
→ (0, 0) or  − ,  A1 A1 cao cao, first A1 for any correct
 3 27  [4] turning point or any correct pair of
x values. 2nd A1 for 2 complete
TPs

d2 y
(ii) = 6x + 2p M1 Other methods include; clear
dx 2 demonstration of sign change of
gradient, clear reference to the
shape of the curve
At (0, 0) → 2p +ve Minimum A1 www
 2 p 4 p3 
At  − ,  → −2p –ve Maximum A1
 3 27  [3]
(iii) y = x 3 + px 2 + px → 3x² + 2px + p (= 0) B1
2
Uses b − 4 ac M1 Any correct use of discriminant
→ 4p² − 12p< 0
→ 0 <p< 3 aef A1 cao (condone ø)
[3]

Page M/J 15/P13/ Q8 Mark Scheme Syllabus Paper


Cambridge International AS/A Level – May/June 2015 9709 13

57 (i) − ( x + 1) −2 − 2( x + 1) −3 M1A1 M1 for recognisable attempt at differentn.


A1 2
− x − 4 x − 3 from Q rule. (A2,1,0)
[3] Allow 4
( x + 1)

(ii) f ' ( x) < 0 hence decreasing B1 Dep. on their (i) < 0 for x > 1
[1]

(iii) 2 dy
−1
2
− 2
3
= 0 or − x − 4 x4 − 3 = 0 M1* Set to 0
( x + 1) dx
( x + 1) ( x + 1)
− ( x + 1) − 2 M1 OR mult by ( x + 1)3 or ( x + 1)5 (i.e.×mult)
3
= 0 → − x − 1 − 2 = 0 or
( x + 1) Dep* × multn → −( x + 1)3 − 2( x + 1) 2 = 0
− x2 − 4x − 3 = 0

x = −3, y = −1 / 4 A1A1 (−3, −1/4) www scores 4/4


[4]

278
Page Mark Scheme Syllabus Paper
Cambridge International AS/A Level – October/November 2014 9709 12

12
58 y=
3 − 2x
(i) Differential = −12(3 – 2x)−2 × −2 B1 B1 co co (even if 1st B mark lost)
[2]
dy dy dx
(ii) = ÷ = 0.4 ÷ 0.15 M1 Chain rule used correctly (AEF)
dx dt dt
24 8 dy 8 3
→ = M1 Equates their with their or
2 3
(3 − 2 x ) dx 3 8
→ x = 0 or 3 A1 A1 co co
[4]

Page O/N 14/P11/ Q9 Mark Scheme Syllabus Paper


Cambridge International AS/A Level – October/November 2014 9709 11

59 (i) f ′(2) = 4 − 12 = 72 → gradient of normal = − 72 B1M1


y − 6 = − 72 ( x − 2) AEF A1 Ft from their f ′(2 )
[3]

(ii) 2 x − 22 = 0 ⇒ 2 x 3 − 2 = 0 M1 Put f ′(x ) = 0 and attempt to solve


x
x =1 A1 Not necessary for last A mark as
x > 0 given
f ′′ ( x ) = 2 + 43 or any valid method M1
x
f ′′(1) = 6 OR > 0 hence minimum A1 Dependent on everything correct
[4]

Page Mark Scheme Syllabus Paper


Cambridge International AS/A Level – October/November 2014 9709 13

60 (i) Minimum since f ″(3) (= 4/3) > 0 www B1


[1]
(ii) f ′(x ) = −18 x (+ c ) −2 B1
0 = −2 + c M1 Sub f ᇱ 3  0. (dep c present)
(
c = 2 → f ′(x ) = −18 x −2 + 2 ) A1 c = 2 sufficient at this stage
f (x ) = 18 x −1 + 2 x (+ k ) B1 B1 Allow cx at this stage
7=6+6+k M1 Sub f(3) = 3 (k present & numeric
k = −5 → ( f ( x ) = 18 x −1 + 2 x − 5 ) cao A1
(or no) c)
[7]

Page Mark Scheme Syllabus Paper


Cambridge International AS/A Level – October/November 2014 9709 13
2
dy  1 − 
61 (b) =  ( 7 x 2 + 1) 3  × [14 x ] B1B1
dx  3 
dy 1 2
 7
= × (64) 3 × 42

When x = 3 , =  M1
dx 3  8
dy dy dx 7 DM1
= × = ×8 Use chain rule
dt dx dt 8
7 A1
[5]

279
Page Mark Scheme Syllabus Paper
GCE AS/A LEVEL – May/June 2014 9709 12

d2 y
62 = 2x − 1
dx 2
dy
→ ∫ = x² − x +c B1 Correct integration (ignore +c)
dx
= 0 when x = 3 → c = − 6 M1 A1 Uses a constant of integration. co
x 2 − x − 6 = 0 when x = − 2 (or 3) A1 Puts dy/dx to 0
→ ∫ y = 1 3 x 3 − 1 2 x 2 − 6 x (+k) B1 B1 first 2 terms, for cx.
= −10 when x =3 M1 Correct method for k
→ k = 3½
→ y = 10 65 A1 Co –r 10.8
[8]

Page Mark Scheme Syllabus Paper


GCE AS/A LEVEL – May/June 2014 9709 11

63
dy
dx
[
= − 2 × 4(3 x + 1)−3 × [3] ] B1B1 [–2 × 4u–3] × [3] is B0B1 unless resolved
dy
When x = −1, =3 B1
dx
When x = −1, y = 1 soi B1
1
y – 1 = 3(x + 1) (→ y = 3x + 4) B1 Ft on their ‘3’ only (not − ). Dep on diffn
[5] 3

Page Mark Scheme Syllabus Paper


GCE AS/A LEVEL – May/June 2014 9709 11

1 − 12 1 − 32
64 (i) x + x oe B1B1
2 2 [2]
(ii) 1 − 12 x −1
x2 − x =0→ =0 M1 Equate to zero and attempt to solve
x
x=1 A1
2 2
When x = 1, y = − 2 − = −2 M1A1 Sub. their ‘1’ into their ‘y’
3 3
d2 y
When x = 1, (= 1) > 0 Hence minimum B1 Everything correct on final line. Also dep on
dx 2 [5] correct (ii). Accept other valid methods

280
Page 5 Mark Scheme Syllabus Paper
M/J 14/P13/ Q5 GCE AS/A LEVEL – May/June 2014 9709 13

65 f(x) = 15
2x + 3
(i) f '(x) = −15 ×2
 2 x + 3
2 B1 B1 Without the “×2”. For “×2”
  (indep of 1st B1).
( )² always +ve → f '(x) < 0
(No turning points) – therefore an inverse B1 providing ( )² in f '(x). 1–1 insuff.
[3]

(ii) y = 15 → 2x + 3 = 15
y M1 Order of ops – allow sign error
2x + 3
15 − 3
→x= y → 15 − 3x A1 co as function of x. Allow y = …
2 2x
(Range) 0=Y f −1(x) Y 6.
Allow 0 Y yY 6, [0,6] B1 For range / domain ignore letters
(Domain) 1 Y x Y 5. Allow [1, 5] B1 unless range / domain not identified
[4]
M/J 14/P13/ Q6
dy 12
66 = P (2, 14) Normal 3y + x = 44
dx 4x + a
(i) m of normal = − 1 B1 co
3
dy 12
=3= → a=8 M1 A1 Use of m1m₂ = −1. AG.
dx 4x + a
[3]
1
1
(ii) ∫ y = 12(4x + a) ÷ 2 ÷ 4 (+c) B1 B1 Correct without “÷4”. for “÷4”.
2
Uses (2, 14) M1 Uses in an integral only. Dep ‘c’.
c = −10 A1 co All 4 marks can be given in (i)
[4]

Page Mark Scheme Syllabus Paper


M/J 14/P13/ Q9 GCE AS/A LEVEL – May/June 2014 9709 13

67 (i) 3x²y = 288 y is the height B1 co


A = 2(3x² + xy + 3xy) M1 Considers at least 5 faces (y ≠ x)

Sub for y → A = 6x2 + 768 A1 co answer given


x
[3]

(ii) dA =12x − 768


B1 co
dx x2
= 0 when x = 4 → A = 288. Allow (4 , 288) M1 A1 Sets differential to 0 + solution. co
d2 A =12 + 1536
M1 Any valid method
dx2 x3
(= 36) > 0 Minimum A1 co www dep on correct f″ and x = 4
[5]

281
Page Mark Scheme Syllabus Paper
GCE AS/A LEVEL – October/November 2013 9709 12

2
68 y=
5x − 6
dy −3
(i) = 2 × −½ × (5 x − 6) 2 × 5 B1 B1 B1 without ‘×5’. B1 For ‘×5’
dx B1 Use of ‘uv’ or ‘u/v’ ok.
→ − 85 [3]

Page Mark Scheme Syllabus Paper


GCE AS/A LEVEL – October/November 2013 9709 12

69
y 12
(i) Sim triangles = (or trig) M1
16 − x 16 Trig, similarity or eqn of line
→ y = 12 − ¾x A1 (could also come from eqn of line)
A = xy = 12x − ¾x². A1 ag – check working.
[3]
dA 6x
(ii) = 12 − B1
dx 4
= 0 when x = 8. → A = 48. M1 A1
Sets to 0 + solution.
This is a Maximum. B1
From −ve quadratic or 2nd differential. [4] Can be deduced without any working.
Allow even if ‘48’ incorrect.

Page 5 Mark Scheme Syllabus Paper


GCE AS/A LEVEL – October/November 2013 9709 11

70 (i) A = 2 xr + πr 2 B1
2 x + 2πr = 400 (⇒ x = 200 − πr ) B1
A = 400 r − πr 2 M1A1 Subst & simplify to AG (www)
[4]
dA
(ii) = 400 − 2πr B1 Differentiate
dr
=0 M1 Set to zero and attempt to find r
200
r= oe A1
π
x = 0 ⇒ no straight sections AG A1
d2 A
= −2π ( < 0 ) Max B1 Dep on − 2π , or use of other valid
dr 2 [5] reason

282
Unit 7: Answer Section

Page Mark Scheme Syllabus Paper


O/N 13/P13/Q9 GCE A LEVEL – October/November 2013 9709 13

dy
= −k 2 (x + 2) + 1 = 0
−2
71 M1A1 Attempt differentiation & set to zero
dx
x+2=±k DM1 Attempt to solve
x = –2 ± k A1 cao
d2 y
= 2k 2 (x + 2 )
−3
M1 Attempt to differentiate again
dx 2
d2 y
M1 Sub their x value with k in it into
dx 2
d2 y  2 
When x = –2 = k, =   which is (> 0) min A1 Only 1 of bracketed items needed for each
dx 2  k 
d2 y  2  d2 y
When x = –2 – k, =  which is (< 0) A1 but and x need to be correct.
dx 2  − k  dx 2
max
[8]

Page Mark Scheme Syllabus Paper


O/N 13/P12/Q9 GCE AS/A LEVEL – May/June 2013 9709 12

5
72 f(x) = , x≥1
1 − 3x
−5
(i) f ′(x) = × −3 B1 B1 B1 without × −3. B1 for ×−3, even if first B
(1 − 3 x) 2 [2] mark is incorrect

(ii) 15 > 0 and (1 − 3x)² >0, f′(x) > 0 B1 providing ( )² in denominator.


→ increasing [1]
5 5
(iii) y = → 3x = 1 − M1 Attempt to make x the subject.
1 − 3x y A1 Must be in terms of x.
x−5 5
→ f −1(x) = or ⅓–
3x 3x
Range is ≥ 1 B1 must be ≥
Domain is − 2.5 ≤ x < 0 B1 B1 condone <
[5]

283
Unit 7: Answer Section

Page Mark Scheme Syllabus Paper


GCE AS/A LEVEL – October/November 2012 9709 12
73 (i) A = 2400 − 20(60 − 2x) −x(40 − x) − 30x
→ A = x² −30x + 1200. M1 Needs attempts at all areas
(could be trapezium − triangle) A1 co answer given
[2]
dA
(ii) = 2x − 30 or (x − 15)² + 975 B1 co - either method okay
dx
= 0 when x = 15 or Min at x = 15
→ A = 975 . M1 Sets differential to 0 + solution. co
A1 co.
[3]

74 dy 4
= x + 2 and P (4, 8)
dx x
x2 4
(i) y= − + (c) B1 B1 co.co (ignore +c at this stage)
2 x
Uses (4, 8) → c=1 M1 A1 Uses the point after integration for c
[4]
d2y 8
(ii) 2
=1 − 3 B1 Co
dx x
= 0 when x = 2 B1 Sets to 0 + solution or verifies and states
a conclusion (stationary or min)

→ gradient of 3 B1 Allow for x = 2 into dy/dx.


8 24
d/dx( 1 − 3 ) = 4 → +ve → Min. B1 Any valid method - 3rd differential +ve
x x
[4] 2nd diff goes −0+, or 1st goes >3,3,>3

Page Mark Scheme Syllabus Paper


GCE AS/A LEVEL – October/November 2012 9709 11


75    →   2 B1

   M1
 used
  
 B1
3 soi

300 (or 942) A1
[4]
 
76 2  2  1  B2,1,0 –1 each error in 2, –2,   1


Sub x = 2 → 22 0 ⇒ stat value at x = 2 B1 AG

మ  
6  1 (and sub x = 2) M1 Reasonable attempt to diff form   1
 మ
మ మ
(At x = 2, 6) > 0 ⟹ Minimum A1 Correct మ and ‘minimum’ is required
 మ 
[5] Or other valid method for last 2 marks

284
Unit 7: Answer Section

Page Mark Scheme Syllabus Paper


GCE AS/A LEVEL – October/November 2012 9709 11

  మ
77 (i) 6   6  2 య B1B1 Independent
 

Equation of tangent is   2 G  1 M1 


Where m = numerical
 
Equation of normal is   2    1 M1 Including use of G G 1


Both eqns correct with G cao A1 SC 1/3 Blatant tangent/normal reversal

[5]
(ii) B = (0, 1½); C = (2, 0) B1 Both cao

BC = H2  1భ మ
= 2½ M1A1 ft from their B and C
[3]
 
(iii) BC:   1    0 or M1 or 

 1

 
  
    2 M1

 
Intersection (E) :    1 2 A1 cao
 

 ;  B1 Dependent on correct x values or y values
 
Mid-point of OA = (½, 1) →E not mid-point [4] for both E and the mid-point of OA

Page Mark Scheme Syllabus Paper


GCE AS/A LEVEL – October/November 2012 9709 13

78 f′(x) = – 3x–4 – 3x2 B1 B1


 0 ⇒ decreasing function B1 Dependent upon minus signs & even
[3] powers

Page 4 Mark Scheme: Teachers’ version Syllabus Paper


GCE AS/A LEVEL – May/June 2012 9709 12

2
79 y=4 x +
x
dy
(i) = 4.½.x –½ + 2.(−½)x −1.5 M1 Reducing “their” power by 1 once.
dx Allow unsimplified
2 1
or y = − 1.5 A1 A1
x x [3]
dy dy dx
(ii) = × used M1 Must be used correctly
dt dx dt
A1 co – fraction or decimal.
→ = ⅞ × 0.12 = 0.105
[2]

285
Unit 7: Answer Section
Page4 Mark Scheme: Teachers’ version Syllabus Paper
O/N 12/P11/Q4 GCE AS/A LEVEL – May/June 2012 9709 11

3.2 2
80 1000k = 3.2 ⇒ k = or or 0.0032 oe M1A1
1000 625
 dM  2
  = 3kr B1
 dr 
dM dM dr
= × used e.g. 3 × k × 10 2 × 0.1 M1 Must eventually make dM/dt subject
dt dr dt cao. Non-calculus methods (e.g. →
0.096 A1
0.09696) can score only 1st 2 marks
[5]

81 (i) 3 x 2 − 4 x + 1(< )5 M1 Attempt differentiate & put 5 on RHS


(3x + 2)(x − 2) < 0 M1 Attempt to factorise or solve
2  2   2  2
− < x < 2 or − , 2 or  − , 2  . A2 SC Allow A1 for − and 2 seen
3  3   3  [4] 3
Allow <

(ii) 3 x 2 − 4 x + 1 = 0 ⇒ (3 x − 1)(x − 1) = 0 M1 Derivative = 0 & any attempt to solve


1
x = or 1 A1 Both
3
4
y= or 0 A1 Both
27
1
f″(x) = 6x – 4 → f″   = –2 (< 0);
 3
f″(1) = 2 (>0) M1 Or other valid method
1 4 
max at  ,  ; min at (1, 0) cao A1 Allow just x values or just y values
 3 27  [5] given for identification

Page Mark Scheme: Teachers’ version Syllabus Paper


GCE AS/A LEVEL – May/June 2012 9709 13

10
82 y= −2
2x + 1
dy − 10
(i) = ×2
dx (2 x + 1) 2 B1 B1 Without the “×2”. For the “×2”.
At A, y = 0, → x = 2
4 B1 For x = 2
m at x = 2, is −
5
Eqn of tangent is y = − 54 ( x − 2)
M1 Must be using differential as m
→ 5y + 4x = 8 A1 co – answer given.
[5]
(ii) C (0, 1.6)
d= (1.6 2
)
+ 2 2 = 2.56 M1 A1 Correct method – needs . co
[2]

286
Unit 7: Answer Section
Page Mark Scheme: Teachers’ version Syllabus Paper
GCE AS/A LEVEL – October/November 2011 9709 11

δy
83 = 9 x 2 − 12 x + 4 M1A1
δx
(3x − 2)2 ≥ 0 A1 [3]
1
84 (i) y = oe B1 [1]
6(48 − 8 x )
(ii) A = 4 xy + 2 xy or 3 xy + 3 xy = 6 xy M1

A = x(48 − 8 x ) = 48 x − 8 x 2 A1 [2] AG

δA
(iii) = 48 − 16 x B1
δx
Attempt to solve derivative = 0
A = 72 cao M1A1
Expect x = 3

δ 2A
= −16 (< 0) ⇒ Maximum B1 [4] www Accept other complete methods
δx 2

Page Mark Scheme: Teachers’ version Syllabus Paper


GCE AS/A LEVEL – October/November 2011 9709 13

85 (i) f´(3) = 0 ⇒ 18 + 3k – 12 = 0 M1
k = –2 A1 AG
(x – 3)(x + 2) = 0 M1
x = –2, (Allow also = 3) A1
[4]
(ii) f´´(x) = 4x ‒ 2
f´´(3) > 0 hence min at P B1
f´´(‒2) <0 hence max at Q
B1 3 min, –2 max independent of f´´(x)
[2]
2
(iii) f(x) = x3 – x2 – 12x (+ c) B2,1,0 Accept anywhere in question
3
Sub (3, –10) → –10 = 18 – 9 – 36 + c M1 Dependent on c present
c = 17 A1 Condone y =, or equation =
[4]

Page Mark Scheme: Teachers’ version Syllabus Paper


M/J 11/P12/Q4 GCE AS/A LEVEL – May/June 2011 9709 12
4
86 y=
3x − 4
dy
(i) = −4(3 x − 4) −2 × 3 B1 B1 Correct without ×3. For ×3.
dx
If x = 2, m = −3
Eqn of tangent y − 2 = −3( x − 2) M1 A1 Correct line eqn. co (for normal M0A0)
[4]

(ii) tanθ = ±(−3) M1 Correct link with (± his gradient)


→ θ = ±108.4º (or ±71.6º) A1√ co (accept acute or obtuse) or −71.6º or
[2] radians
or scalar product, tanθ = y-step ÷ x-step
or use of tan (A − B) M1A1 for each

287
Unit 7: Answer Section
Page Mark Scheme: Teachers’ version Syllabus Paper
GCE AS/A LEVEL – May/June 2011 9709 11

87  dv = 4πr 2 M1
 dr 
= 4π × 10 2 A1 SOI at any point
dv
dr M1 Correct link between differentials with
= dt OE used
dt dv dr
dr finally as subject
dt
50 1 50
2
= or 0.0398 A1 Allow .
4π × 10 8π 400π
[4]
0
Non-calculus methods
4

88 (i)  600  (z − 3x ) = 600 OE B1


z = 3 x + 2  or x
 x  2 [1]
→ AG

dz 1200 dz 1800
(ii) = 3− 2 or = 2− 2 B1
dx x dy y
= 0 → x = 20 or = 0 → y = 30 M1A1 Set to 0 & attempt to solve. Allow ±20
120 Ft from their x provided positive
z = 60 + = 120 A1√ Or other valid method
20
d 2 z 2400 d2z k
= 3 B1√ Dep. on 2
= 3 (k > 0) or other
dx 2 x dx x
> 0 ⇒ minimum B1 valid method.
[6]

288
UNIT 8 Integration Unit 8: Integration

1 M/J 22/P12/Q6 y = 2x + 2
y
1
y = 5x 2

x
O

1
The diagram shows the curve with equation y = 5x 2 and the line with equation y = 2x + 2.
Find the exact area of the shaded region which is bounded by the line and the curve. [5]

2 M/J 22/P12/Q9
1
The equation of a curve is y = 3x + 1 − 4 3x + 1 2 for x > − 13 .
dy d2 y
(a) Find and 2 . [3]
dx dx
(b) Find the coordinates of the stationary point of the curve and determine its nature. [4]

3 M/J 22/P12/Q10/e
Functions f and g are defined as follows:
2x + 1
f x = for x ≠ 12 ,
2x − 1
g x = x2 + 4 for x ∈ >.
2 2x + 1
(a) Show that 1 + can be expressed as . Hence find the area of the triangle enclosed
2x − 1 2x − 1
by the tangent to the curve y = f x at the point where x = 1 and the x- and y-axes. [6]

4 M/J 22/P11/Q7 y
y = 12 x + 1

B 1
y = 3x − 2 2

x
O

1
The diagram shows the curve with equation y = 3x − 2 2 and the line y = 12 x + 1. The curve and the
line intersect at points A and B.

(a) Find the coordinates of A and B. [4]


(b) Hence find the area of the region enclosed between the curve and the line. [5]

289
Unit 8: Integration
5 M/J 22/P11/Q10
d2 y 4  
The equation of a curve is such that 2
= 6x2 − 3 . The curve has a stationary point at −1, 92 .
dx x
 
(a) Determine the nature of the stationary point at −1, 92 . [1]
(b) Find the equation of the curve. [5]
(c) Show that the curve has no other stationary points. [3]
(d) A point A is moving along the curve and the y-coordinate of A is increasing at a rate of 5 units
per second.
Find the rate of increase of the x-coordinate of A at the point where x = 1. [3]

6 M/J 22/P13/Q8 y
1 1
y = x 2 + 4x− 2

A 1, 5
B 16, 5

x
O

−1
The diagram shows the curve with equation y = x 2 + 4x 2 . The line y = 5 intersects the curve at the
1

points A 1, 5 and B 16, 5.

(a) Find the equation of the tangent to the curve at the point A. [4]
(b) Calculate the area of the shaded region. [4]

7 M/J 22/P13/Q10/a
The function f is defined by f x = 4x + 2−2 for x > − 12 .


(a) Find Ó f x dx. [4]
1
A point is moving along the curve y = f x in such a way that, as it passes through the point A, its
y-coordinate is decreasing at the rate of k units per second and its x-coordinate is increasing at the
rate of k units per second.

8 O/N 21/P12/Q4
dy 8  2

A curve is such that = . The curve passes through the point 2, 5 3 .
dx 3x + 22
Find the equation of the curve.
[4]

290
Unit 8: Integration
9 O/N 21/P12/Q11
y
1
y = 21 x + 7
10 − 1
x − 2 3
A 3, 65 

x
O 5
2

7 − 1
The diagram shows the line x = 52 , part of the curve y = 12 x + 10 1
and the normal to the curve
x − 2 3
 
at the point A 3, 65 .

(a) Find the x-coordinate of the point where the normal to the curve meets the x-axis. [5]
(b) Find the area of the shaded region, giving your answer correct to 2 decimal places. [6]

10 O/N 21/P11/Q9

A curve has equation y = f x, and it is given that f ′ x = 2x2 − 7 −


4
9 .
x2

(a) Given that f 1 = − 13 , find f x. [4]


(b) Find the coordinates of the stationary points on the curve. [5]
(c) Find f ′′ x. [1]
(d) Hence, or otherwise, determine the nature of each of the stationary points. [2]

11 O/N 21/P11/Q10

Find Ô
1
(a) dx. [4]
3x − 2 2
3
1

y
1
y= 3
3x − 2 2

x
O 1 2

The diagram shows the curve with equation y =


1
. The shaded region is bounded by the
3x − 2 2
3

curve, the x-axis and the lines x = 1 and x = 2. The shaded region is rotated through 360Å about the
x-axis.

(b) Find the volume of revolution. [4]


The normal to the curve at the point 1, 1 crosses the y-axis at the point A.

(c) Find the y-coordinate of A. [4]

291
12 O/N 21/P13/Q8 Unit 8: Integration
y

1
A 4, 2

5 1
y= 2 − x2

1 B 4, 21 
y = x− 2
x
O

− 12 1
The diagram shows the curves with equations y = x and y = 52 − x 2 . The curves intersect at the points
   
A 14 , 2 and B 4, 21 .

(a) Find the area of the region between the two curves. [6]
− 12
(b) The normal to the curve y = x at the point 1, 1 intersects the y-axis at the point 0, p.

Find the value of p. [4]

13 O/N 21/P13/Q10
A curve has equation y = f x and it is given that
 −2
f ′ x = 12 x + k − 1 + k−2 ,

where k is a constant. The curve has a minimum point at x = 2.

(a) Find f ′′ x in terms of k and x, and hence find the set of possible values of k. [3]

It is now given that k = −3 and the minimum point is at 2, 3 12 .


(b) Find f x. [4]
(c) Find the coordinates of the other stationary point and determine its nature. [4]

14 M/J 21/P12/Q9
y

y2 = x − 2

x
0 5

The diagram shows part of the curve with equation y2 = x − 2 and the lines x = 5 and y = 1. The
shaded region enclosed by the curve and the lines is rotated through 360Å about the x-axis.

Find the volume obtained. [6]

292
15 M/J 21/P12/Q11 Unit 8: Integration

dy
The gradient of a curve is given by = 6 3x − 53 − kx2 , where k is a constant. The curve has a
dx
stationary point at 2, −3.5.

(a) Find the value of k. [2]


(b) Find the equation of the curve. [4]
d2 y
(c) Find . [2]
dx2
(d) Determine the nature of the stationary point at 2, −3.5. [2]

16 M/J 21/P11/Q1
dy 3
The equation of a curve is such that = 4 + 32x3 . It is given that the curve passes through the point
1  dx x
2
, 4 .
Find the equation of the curve. [4]

17 M/J 21/P13/Q1
8
A curve with equation y = f x is such that f ′ x = 6x2 − . It is given that the curve passes through
x2
the point 2, 7.

Find f x. [3]

18 M/J 21/P13/Q11
y

1 1
y = x 2 + k2 x− 2

x
O 9 2
4k2
4k

1 −1
The diagram shows part of the curve with equation y = x 2 + k2 x 2 , where k is a positive constant.

(a) Find the coordinates of the minimum point of the curve, giving your answer in terms of k. [4]
The tangent at the point on the curve where x = 4k2 intersects the y-axis at P.

(b) Find the y-coordinate of P in terms of k. [4]


The shaded region is bounded by the curve, the x-axis and the lines x = 94 k2 and x = 4k2 .

(c) Find the area of the shaded region in terms of k. [3]

19 O/N 20/P12/Q7/b
− 12 −3
The point 4, 7 lies on the curve y = f x and it is given that f ′ x = 6x − 4x 2 .

(a) A point moves along the curve in such a way that the x-coordinate is increasing at a constant rate
of 0.12 units per second.
(b) Find the equation of the curve. [4]
293
Unit 8: Integration

20 O/N 20/P12/Q7/b y

x
O M

2
The diagram shows part of the curve y = − x and its minimum point M , which lies on the
3 − 2x2
x-axis.

dy d2 y
(a) Find expressions for , and Ó y dx. [6]
dx dx2
(b) Find, by calculation, the x-coordinate of M . [2]
(c) Find the area of the shaded region bounded by the curve and the coordinate axes. [2]

21 O/N 20/P11/Q2
dy 1
The equation of a curve is such that = + x. It is given that the curve passes through the
dx x − 32
point 2, 7.

Find the equation of the curve. [4]

y
22 O/N 20/P11/Q12
B

A 4, 0
O x
1
y = 4x 2 − 2x
y=3−x

1
The diagram shows a curve with equation y = 4x 2 − 2x for x ≥ 0, and a straight line with equation
y = 3 − x. The curve crosses the x-axis at A 4, 0 and crosses the straight line at B and C.

(a) Find, by calculation, the x-coordinates of B and C. [4]


(b) Show that B is a stationary point on the curve. [2]
(c) Find the area of the shaded region. [6]

23 O/N 20/P13/Q2
The function f is defined by f x = for x > −2.
2
x + 22

(a) Find Ó f x dx. [3]
1

= f x. It is given that the point −1, −1 lies on the


dy
(b) The equation of a curve is such that
dx
curve.
Find the equation of the curve. [2]

294
Unit 8: Integration
24 O/N 20/P13/Q10
−1
A curve has equation y = x + x 2 + 2 where x > 0 and k is a positive constant.
1 12 1
k k

(a) It is given that when x = 14 , the gradient of the curve is 3.

Find the value of k. [4]


2
k @ A
−1
(b) It is given instead that Ô x + x 2 + 2 dx = 13
1 12 1
12
.
1 k2 k k
4 [5]
Find the value of k.
y
25 M/J 20/P12/Q8

1, 6

3, 2
1, 2 6
y=
x

x
O
6
The diagram shows part of the curve y = . The points 1, 6 and 3, 2 lie on the curve. The shaded
x
region is bounded by the curve and the lines y = 2 and x = 1.

(a) Find the volume generated when the shaded region is rotated through 360Å about the y-axis. [5]
(b) The tangent to the curve at a point X is parallel to the line y + 2x = 0. Show that X lies on the
line y = 2x. [3]

26 M/J 20/P11/Q11
y

2y + x = 8

C
B 8
y=
x+2
x
O

8
The diagram shows part of the curve y = and the line 2y + x = 8, intersecting at points A and B.
x+2
The point C lies on the curve and the tangent to the curve at C is parallel to AB.

(a) Find, by calculation, the coordinates of A, B and C. [6]


(b) Find the volume generated when the shaded region, bounded by the curve and the line, is rotated
through 360Å about the x-axis. [6]

295
Unit 8: Integration

27 M/J 20/P13/Q2
dy 1 −1
The equation of a curve is such that = 3x 2 − 3x 2 . It is given that the point 4, 7 lies on the curve.
dx
Find the equation of the curve.
[4]

28 M/J 20/P13/Q11
y

A
y = x3 − 2bx2 + b2 x

x
O a b

The diagram shows part of the curve with equation y = x3 − 2bx2 + b2 x and the line OA, where A is
the maximum point on the curve. The x-coordinate of A is a and the curve has a minimum point at
b, 0, where a and b are positive constants.

(a) Show that b = 3a. [4]


(b) Show that the area of the shaded region between the line and the curve is ka4 , where k is a fraction
to be found. [7]

29 MAR 20/P12/Q3 y

y = x2 + 1

x
O

The diagram shows part of the curve with equation y = x2 + 1. The shaded region enclosed by the
curve, the y-axis and the line y = 5 is rotated through 360Å about the y-axis.

Find the volume obtained. [4]

30 MAR 20/P12/Q10
dy 1
The gradient of a curve at the point x, y is given by = 2 x + 3 2 − x. The curve has a stationary
dx
point at a, 14, where a is a positive constant.
(a) Find the value of a. [3]
(b) Determine the nature of the stationary point. [3]
(c) Find the equation of the curve. [4]

296
Unit 8: Integration

31 O/N 19/P12/Q3
dy k
A curve is such that =  , where k is a constant. The points P 1, −1 and Q 4, 4 lie on the
dx x
curve. Find the equation of the curve. [4]

32 O/N 19/P12/Q10 y
4
y= 1−
2x + 12
B
x
O A

4
The diagram shows part of the curve y = 1 − . The curve intersects the x-axis at A. The
2x + 12
normal to the curve at A intersects the y-axis at B.

dy
(i) Obtain expressions for and Ó y dx. [4]
dx
(ii) Find the coordinates of B. [4]
(iii) Find, showing all necessary working, the area of the shaded region. [4]

33 O/N 19/P11/Q9

dy 1
A curve for which = 5x − 1 2 − 2 passes through the point 2, 3.
dx

(i) Find the equation of the curve. [4]


2
d y
(ii) Find 2 . [2]
dx
(iii) Find the coordinates of the stationary point on the curve and, showing all necessary working,
determine the nature of this stationary point. [4]

297
Unit 8: Integration
y
34 O/N 19/P11/Q11

y = x2 + 4x + 3

x
−2 O
y = −1
−1

The diagram shows a shaded region bounded by the y-axis, the line y = −1 and the part of the curve
y = x2 + 4x + 3 for which x ≥ −2.

(i) Express y = x2 + 4x + 3 in the form y = x + a2 + b, where a and b are constants. Hence, for
x ≥ −2, express x in terms of y. [4]

(ii) Hence, showing all necessary working, find the volume obtained when the shaded region is
rotated through 360Å about the y-axis. [6]

35 O/N 19/P13/Q11
y

A 2, 3

y = x − 1−2 + 2

x
O 1 3

The diagram shows part of the curve y = x − 1−2 + 2, and the lines x = 1 and x = 3. The point A on
the curve has coordinates 2, 3. The normal to the curve at A crosses the line x = 1 at B.

(i) Show that the normal AB has equation y = 12 x + 2. [3]


(ii) Find, showing all necessary working, the volume of revolution obtained when the shaded region
is rotated through 360Å about the x-axis. [8]

298
y Unit 8: Integration
36 M/J 19/P12/Q11

 9
M y= 4x + 1 + 
4x + 1

x
O

 9
The diagram shows part of the curve y = 4x + 1 +  and the minimum point M .
4x + 1

dy
(i) Find expressions for and Ó y dx. [6]
dx
(ii) Find the coordinates of M . [3]

The shaded region is bounded by the curve, the y-axis and the line through M parallel to the x-axis.

(iii) Find, showing all necessary working, the area of the shaded region. [3]

37 M/J 19/P11/Q10
d2 y
A curve for which = 2x − 5 has a stationary point at 3, 6.
dx2

(i) Find the equation of the curve. [6]


(ii) Find the x-coordinate of the other stationary point on the curve. [1]
(iii) Determine the nature of each of the stationary points. [2]

38 M/J 19/P11/Q11 y

3
y= 
1 + 4x

P 2, 1

x
O Q
3
The diagram shows part of the curve y =  and a point P 2, 1 lying on the curve. The
1 + 4x
normal to the curve at P intersects the x-axis at Q.

(i) Show that the x-coordinate of Q is 16


9
. [5]
(ii) Find, showing all necessary working, the area of the shaded region. [6]

299
y Unit 8: Integration

39 M/J 19/P13/Q10
A

1
y = 3x + 4 2

x
O 4

1
The diagram shows part of the curve with equation y = 3x + 4 2 and the tangent to the curve at the
point A. The x-coordinate of A is 4.

(i) Find the equation of the tangent to the curve at A. [5]


(ii) Find, showing all necessary working, the area of the shaded region.
region. [5]
(iii) A point is moving along the curve. At the point P the y-coordinate is increasing at half the rate
at which the x-coordinate is increasing. Find the x-coordinate of P. [3]

40 MAR 19/P12/Q2
A curve with equation y = f x passes through the points 0, 2 and 3, −1. It is given that
f ′ x = kx2 − 2x, where k is a constant. Find the value of k. [5]
y 
41 MAR 19/P12/Q9 y= x3 + x2 
P

x
O 3


The diagram shows part of the curve with equation y = x3 + x2 . The shaded region is bounded by
the curve, the x-axis and the line x = 3.

(i) Find, showing all necessary working, the volume obtained when the shaded region is rotated
through 360Å about the x-axis. [4]
(ii) P is the point on the curve with x-coordinate 3. Find the y-coordinate of the point where the
normal to the curve at P crosses the y-axis. [6]

300
Unit 8: Integration

42 O/N 18/P12/Q2
4@ A
Showing all necessary working, find Ô x +  dx.
 2
[4]
x
1
43 O/N 18/P12/Q11 y

M 
y = 3 4x + 1 − 2x

x
O

The diagram shows part of the curve y = 3 4x + 1 − 2x. The curve crosses the y-axis at A and the

stationary point on the curve is M .

and Ó y dx.
dy
(i) Obtain expressions for [5]
dx
(ii) Find the coordinates of M . [3]
(iii) Find, showing all necessary working, the area of the shaded region. [4]

44 O/N 18/P11/Q7
y
y=x

A 2, 2

y = k x3 − 7x2 + 12x
O x

The diagram shows part of the curve with equation y = k x3 − 7x2 + 12x for some constant k. The
curve intersects the line y = x at the origin O and at the point A 2, 2.

(i) Find the value of k. [1]


(ii) Verify that the curve meets the line y = x again when x = 5. [2]

301
Unit 8: Integration
45 O/N 18/P13/Q8
dy
11 and has an equation for which
A curve passes through 0, 11 = ax2 + bx − 4, where a and b are
dx
constants.

(i) Find the equation of the curve in terms of a and b. [3]


3. Find the values of a and b.
(ii) It is now given that the curve has a stationary point at 2, 3 [5]
y
46 O/N 18/P13/Q10 2
x= 3

1
y = 2 3x − 1 − 3 x=3

x
O 1 2 3

− 13
The diagram shows part of the curve y = 2 3x − 1 and the lines x = 23 and x = 3. The curve and the
line x = 23 intersect at the point A.

(i) Find, showing all necessary working, the volume obtained when
when the shaded region is rotated
360Å about the x-axis.
through 360Å [5]
(ii) Find the equation of the normal to the curve at A, giving your answer in the form y = mx + c.
[5]
47 M/J 18/P12/Q9
dy 
A curve is such that = 4x + 1 and 2, 5 is a point on the curve.
dx

(i) Find the equation of the curve. [4]


(ii) A point P moves along the curve in such a way that the y-coordinate is increasing at a constant
rate of 0.06 units per second. Find the rate of change of the x-coordinate when P passes through
2, 5. [2]
d2 y dy
(iii) Show that × is constant. [2]
dx2 dx

302
Unit 8: Integration
48 M/J 18/P12/Q11
y
x 6
y= +
2 x

P Q y=4

x
O

x 6
The diagram shows part of the curve y = + . The line y = 4 intersects the curve at the points P
2 x
and Q.

(i) Show that the tangents to the curve at P and Q meet at a point on the line y = x. [6]
(ii) Find, showing all necessary working, the volume obtained when the shaded region is rotated
through 360Å about the x-axis. Give your answer in terms of 0 . [6]

49 M/J 18/P11/Q3
dy 12
A curve is such that = . The point 1, 1 lies on the curve. Find the coordinates of the
dx 2x + 12
point at which the curve intersects the x-axis. [6]

50 M/J 18/P11/Q10
The curve with equation y = x3 − 2x2 + 5x passes through the origin.
(i) Show that the curve has no stationary points. [3]
(ii) Denoting the gradient of the curve by m, find the stationary value of m and determine its nature.
[5]
(iii) Showing all necessary working, find the area of the region enclosed by the curve, the x-axis and
the line x = 6. [4]
51 M/J 18/P13/Q4
A curve with equation y = f x passes through the point A 3, 1 and crosses the y-axis at B. It is given
−1
that f ′ x = 3x − 1 3 . Find the y-coordinate of B. [6]

303
Unit 8: Integration
y
52 M/J 18/P13/Q11

y = x + 12 + x + 1−1

x=1
A

x
O 1

The diagram shows part of the curve y = x + 12 + x + 1−1 and the line x = 1. The point A is the
minimum point on the curve.
(i) Show that the x-coordinate of A satisfies the equation 2 x + 13 = 1 and find the exact value of
d2 y
at A. [5]
dx2
(ii) Find, showing all necessary working, the volume obtained when the shaded region is rotated
through 360Å about the x-axis. [6]

53 O/N 17/P12/Q10 y

y= 5x − 1

P 2, 3

Q
x
O

The diagram shows part of the curve y = 5x − 1 and the normal to the curve at the point P 2, 3.
This normal meets the x-axis at Q.

(i) Find the equation of the normal at P. [4]


(ii) Find, showing all necessary working, the area of the shaded region. [7]

y
54 O/N 17/P11/Q10
1
y= 2 x4 − 1

x
O 1
− 12

The diagram shows part of the curve y = 12 x4 − 1, defined for x ≥ 0.


(i) Find, showing all necessary working, the area of the shaded region. [3]
(ii) Find, showing all necessary working, the volume obtained when the shaded region is rotated
through 360Å about the x-axis. [4]
(iii) Find, showing all necessary working, the volume obtained when the shaded region is rotated
through 360Å about the y-axis. [5]

304
Unit 8: Integration
55 O/N 17/P13/Q8 y

y = 3 − 2x

B 
y=4−3 x
x
O


The diagram shows parts of the graphs of y = 3 − 2x and y = 4 − 3 x intersecting at points A and B.

(i) Find by calculation the x-coordinates of A and B. [3]


(ii) Find, showing all necessary working, the area of the shaded region. [5]

56 O/N 17/P12/Q6 y

A 1, 4

x+y=5

4
y=
x B 4, 1

x
O

4
The diagram shows the straight line x + y = 5 intersecting the curve y = at the points A 1, 4 and
x
B 4, 1. Find, showing all necessary working, the volume obtained when the shaded region is rotated
through 360Å about the x-axis. [7]

57 M/J 17/P11/Q7
dy
A curve for which = 7 − x2 − 6x passes through the point 3, −10.
dx

(i) Find the equation of the curve. [3]


2 2
(ii) Express 7 − x − 6x in the form a − x + b , where a and b are constants. [2]
(iii) Find the set of values of x for which the gradient of the curve is positive. [3]

305
Unit 8: Integration
58 M/J 17/P11/Q10 y
4
y=
5 − 3x

x
O 1

4
The diagram shows part of the curve y = .
5 − 3x

(i) Find the equation of the normal to the curve at the point where x = 1 in the form y = mx + c,
where m and c are constants. [5]
The shaded region is bounded by the curve, the coordinate axes and the line x = 1.

(ii) Find, showing all necessary working, the volume obtained when this shaded region is rotated
through 360Å about the x-axis. [5]

59 M/J 17/P13/Q10
y
(a)

y=h

y = x2 − 1 Fig. 1

x
O

Fig. 1 shows part of the curve y = x2 − 1 and the line y = h, where h is a constant.
(i) The shaded region is rotated through
 360Å
 about the y-axis. Show that the volume of
revolution, V , is given by V = 0 12 h2 + h . [3]
(ii) Find, showing all necessary working, the area of the shaded region when h = 3. [4]

(b)

h Fig. 2

Fig. 2 shows a cross-section of a bowl containing water.  When  the height of the water level is
h cm, the volume, V cm3 , of water is given by V = 0 12 h2 + h . Water is poured into the bowl
at a constant rate of 2 cm3 s−1 . Find the rate, in cm s−1 , at which the height of the water level is
increasing when the height of the water level is 3 cm. [4]

306
Unit 8: Integration
60 M/J 17/P13/Q11

The function f is defined for x ≥ 0. It is given that f has a minimum value when x = 2 and that
−1
f ′′ x = 4x + 1 2 .

(i) Find f ′ x. [3]


(iii) Find f x, and hence find the minimum value of f. [5]

61 O/N 16/P11/Q7 y

y = 2x − 12
B

x
O A
y2 = 1 − 2x

The diagram shows parts of the curves y = 2x − 12 and y2 = 1 − 2x, intersecting at points A and B.

(i) State the coordinates of A. [1]

(ii) Find, showing all necessary working, the area of the shaded region. [6]

62 O/N 16/P11/Q10
1 −1
A curve has equation y = f x and it is given that f ′ x = 3x 2 − 2x 2 . The point A is the only point on
the curve at which the gradient is −1.

(i) Find the x-coordinate of A. [3]

(ii) Given that the curve also passes through the point 4, 10, find the y-coordinate of A, giving your
answer as a fraction. [6]

63 O/N 16/P13/Q10
dy 2 − 12 −3
A curve is such that = x + ax 2 , where a is a positive constant. The point A a2 , 3 lies on the
dx a
curve. Find, in terms of a,
(i) the equation of the tangent to the curve at A, simplifying your answer, [3]
(ii) the equation of the curve. [4]

It is now given that B 16, 8 also lies on the curve.

(iii) Find the value of a and, using this value, find the distance AB. [5]

307
O/N 16/P13/Q11 Unit 8: Integration
64
A curve has equation y = kx − 3−1 + kx − 3, where k is a non-zero constant.

(i) Find the x-coordinates of the stationary points in terms of k, and determine the nature of each
stationary point, justifying your answers. [7]

(ii)
y

2
x
O

y = x − 3−1 + x − 3

The diagram shows part of the curve for the case when k = 1. Showing all necessary working,
find the volume obtained when the region between the curve, the x-axis, the y-axis and the line
x = 2, shown shaded in the diagram, is rotated through 360Å about the x-axis. [5]

65 M/J 16/P12/Q2
dy 8
A curve is such that = . Given that the curve passes through 2, 7, find the equation of
dx 5 − 2x2
the curve. [4]

66 M/J 16/P12/Q10
y

8
y= + 2x
x

x
O

8
The diagram shows the part of the curve y = + 2x for x > 0, and the minimum point M .
x

dy d2 y
(i) Find expressions for , and Ó y2 dx. [5]
dx dx2

(ii) Find the coordinates of M and determine the coordinates and nature of the stationary point on
the part of the curve for which x < 0. [5]

(iii) Find the volume obtained when the region bounded by the curve, the x-axis and the lines x = 1
and x = 2 is rotated through 360Å about the x-axis. [2]

308
Unit 8: Integration
67 M/J 16/P11/Q3
y

2 12
x= −2
y2

x
O
12
The diagram shows part of the curve x = − 2. The shaded region is bounded by the curve, the
y2
y-axis and the lines y = 1 and y = 2. Showing all necessary working, find the volume, in terms of 0,
when this shaded region is rotated through 360Å about the y-axis. [5]

68 M/J 16/P11/Q4
dy −1
A curve is such that = 2 − 8 3x + 4 2 .
dx
(i) A point P moves along the curve in such a way that the x-coordinate is increasing at a constant
rate of 0.3 units per second. Find the rate of change of the y-coordinate as P crosses the y-axis.
[2]

The curve intersects the y-axis where y = 43 .

(ii) Find the equation of the curve. [4]

69 M/J 16/P13/Q2 y

P 2, 3

1
y = x3 + 1 2

x
O 2

1
The diagram shows part of the curve y = x3 + 1 2 and the point P 2, 3 lying on the curve. Find,
showing all necessary working, the volume obtained when the shaded region is rotated through 360Å
about the x-axis. [4]

70 M/J 16/P13/Q3
dy k
A curve is such that = 6x2 + 3 and passes through the point P 1, 9. The gradient of the curve
dx x
at P is 2.

(i) Find the value of the constant k. [1]

(ii) Find the equation of the curve. [4]

309
Unit 8: Integration

71 O/N 15/P12/Q10 y


y= 9 − 2x2 

P 2, 1

B
x
A O

The diagram shows part of the curve y = 9 − 2x2 . The point P 2, 1 lies on the curve and the
normal to the curve at P intersects the x-axis at A and the y-axis at B.

(i) Show that B is the mid-point of AP. [6]

The shaded region is bounded by the curve, the y-axis and the line y = 1.

(ii) Find, showing all necessary working, the exact volume obtained when the shaded region is
rotated through 360Å about the y-axis. [5]

72 O/N 15/P11/Q11
y
P 6, 5
1
y = 1 + 4x 2

Q 8, 0
x
O
1
The diagram shows part of the curve y = 1 + 4x 2 and a point P 6, 5 lying on the curve. The line
PQ intersects the x-axis at Q 8, 0.

(i) Show that PQ is a normal to the curve. [5]

(ii) Find, showing all necessary working, the exact volume of revolution obtained when the shaded
region is rotated through 360Å about the x-axis. [7]

[In part (ii) you may find it useful to apply the fact that the volume, V , of a cone of base radius r
and vertical height h, is given by V = 13 0r2 h.]

310
Unit 8: Integration
73 O/N 15/P13/Q9(ii)
dy −1
A curve passes through the point A 4, 6 and is such that = 1 + 2x 2 . A point P is moving along
dx
the curve in such a way that the x-coordinate of P is increasing at a constant rate of 3 units per minute.
(i) Find the equation of the curve. [3]

74 O/N 15/P13/Q10
The function f is defined by f x = 2x + x + 1−2 for x > −1.
(i) Find f ′ x and f ′′ x and hence verify that the function f has a minimum value at x = 0. [4]
y

A − 12 , 3 y = 2x + x + 1−2

B 1, 2 41 

x
O
The points A − 12 , 3 and B 1, 2 41  lie on the curve y = 2x + x + 1−2 , as shown in the diagram.
(ii) Find the distance AB. [2]
(iii) Find, showing all necessary working, the area of the shaded region. [6]

75 M/J 15/P12/Q1
The function f is such that f ′ x = 5 − 2x2 and 3, 5 is a point on the curve y = f x. Find f x. [3]

76 M/J 15/P12/Q10
4
The equation of a curve is y = .
2x − 1
(i) Find, showing all necessary working, the volume obtained when the region bounded by the
curve, the x-axis and the lines x = 1 and x = 2 is rotated through 360Å about the x-axis. [4]

(ii) Given that the line 2y = x + c is a normal to the curve, find the possible values of the constant c.
[6]

311
Unit 8: Integration

y
77 M/J 15/P11/Q10
B

x=4
A Q
0, 4

8
y= 
3x + 4
P

x
O

8
The diagram shows part of the curve y =  . The curve intersects the y-axis at A 0, 4. The
3x + 4
normal to the curve at A intersects the line x = 4 at the point B.

(i) Find the coordinates of B. [5]

(ii) Show, with all necessary working, that the areas of the regions marked P and Q are equal. [6]

78 M/J 15/P13/Q2
dy 1
A curve is such that = 2x + 1 2 and the point 4, 7 lies on the curve. Find the equation of the
dx
curve. [4]

79 M/J 15/P13/Q10

A 2, 9

y = 9 + 6x − 3x2

x
O B C
3, 0

Points A 2, 9 and B 3, 0 lie on the curve y = 9 + 6x − 3x2 , as shown in the diagram. The tangent at
A intersects the x-axis at C. Showing all necessary working,
(i) find the equation of the tangent AC and hence find the x-coordinate of C, [4]
(ii) find the area of the shaded region ABC. [5]

312
y
Unit 8: Integration
80 O/N 14 /P12 /Q1
2, 5

y = x2 + 1

0, 1

x
O
The diagram shows part of the curve y = x2 + 1. Find the volume obtained when the shaded region is
rotated through 360 about the y-axis. [4]

81 O/N 14 /P12 /Q10


d2 y 24
A curve is such that = 3 − 4. The curve has a stationary point at P where x = 2.
dx2 x
(i) State, with a reason, the nature of this stationary point. [1]
dy
(ii) Find an expression for . [4]
dx
(iii) Given that the curve passes through the point 1, 13, find the coordinates of the stationary
point P. [4]

82 O/N 14 /P11 /Q9(ii)


2
The function f is defined for x > 0 and is such that f  x = 2x − . The curve y = fx passes through
x2
the point P 2, 6.
(ii) Find the equation of the curve. [4]

83 O/N 14 /P11 /Q11

1
The diagram shows parts of the curves y = 4x + 1 2 and y = 12 x2 + 1 intersecting at points P 0, 1
and Q 2, 3. The angle between the tangents to the two curves at Q is .

(i) Find , giving your answer in degrees correct to 3 significant figures. [6]
(ii) Find by integration the area of the shaded region. [6]

313
Unit 8: Integration

y
84 O/N 14 /P13 /Q9

y = 3x
y=x+2

x
O

The diagram shows parts of the graphs of y = x + 2 and y = 3x intersecting at points A and B.

(i) Write down an equation satisfied by the x-coordinates of A and B. Solve this equation and hence
find the coordinates of A and B. [4]

(ii) Find by integration the area of the shaded region. [6]

85 M/J/14 /P12 /Q9 y

y = 8 − ï 4 − x
P 3, 7

x
O

The diagram shows part of the curve y = 8 − 4 − x and the tangent to the curve at P 3, 7.

dy
(i) Find expressions for and Ó y dx. [5]
dx

(ii) Find the equation of the tangent to the curve at P in the form y = mx + c. [2]

(iii) Find, showing all necessary working, the area of the shaded region. [4]

314
Unit 8: Integration
86 M/J/14 /P13 /Q10 y

y = 2x + 1

y = −x2 + 12x − 20

x
O

The diagram shows the curve y = −x2 + 12x − 20 and the line y = 2x + 1. Find, showing all necessary
working, the area of the shaded region. [8]

87 O/N/13 /P12 /Q3(ii)

The equation of a curve is y = 


2
5x − 6
.

(i) Find Ô  dx and hence evaluate Ô 


2 2
5x − 6 5x − 6
dx. [4]
2

88 O/N/13 /P12 /Q9


y
C y = 8x + 2x
A
B

x
O

The diagram shows part of the curve y = + 2x and three points A, B and C on the curve with
8
x
x-coordinates 1, 2 and 5 respectively.

(i) A point P moves along the curve in such a way that its x-coordinate increases at a constant rate
of 0.04 units per second. Find the rate at which the y-coordinate of P is changing as P passes
through A. [4]

(ii) Find the volume obtained when the shaded region is rotated through 360Å about the x-axis. [6]

315
89 O/N/13 /P11 /Q2 Unit 8: Integration

1 6
A curve has equation y = f x. It is given that f ′ x =  + 2 and that f 3 = 1. Find f x. [5]
x + 6 x

90 O/N/13 /P11 /Q10 y

y = (3 – 2x)3

(12 , 8)

x
O
1 
The diagram shows the curve y = 3 − 2x3 and the tangent to the curve at the point 2
,8 .
(i) Find the equation of this tangent, giving your answer in the form y = mx + c. [5]
(ii) Find the area of the shaded region. [6]

91 O/N/13 /P13 /Q2

− 32
A curve has equation y = f x. It is given that f ′ x = x + 1 and that f 4 = 5. Find f x. [4]

92 O/N/13 /P13 /Q11 y

y = Ö(x 4 + 4x + 4)

x
–1 O

The diagram shows the curve y = x4 + 4x + 4 .

(i) Find the equation of the tangent to the curve at the point 0, 2. [4]

(ii) Show that the x-coordinates of the points of intersection of the line y = x + 2 and the curve are
given by the equation x + 22 = x4 + 4x + 4. Hence find these x-coordinates. [4]

(iii) The region shaded in the diagram is rotated through 360Å about the x-axis. Find the volume of
revolution. [4]

93 M/J/13 /P12 /Q1

dy 6
A curve is such that = 2 and 2, 9 is a point on the curve. Find the equation of the curve. [3]
dx x

316
Unit 8: Integration
94 M/J/13 /P12 /Q8

The volume of a solid circular cylinder of radius r cm is 2500 cm3 .

(i) Show that the total surface area, S cm2 , of the cylinder is given by
5000
S = 20r2 + . 2
r
(ii) Given that r can vary, find the stationary value of S. [4]
(iii) Determine the nature of this stationary value. [2]

95 M/J/13 /P12 /Q11 y


y = Ö(1 + 4x)

x
A O C

The diagram shows the curve y = 1 + 4x , which intersects the x-axis at A and the y-axis at B. The
normal to the curve at B meets the x-axis at C. Find
(i) the equation of BC, [5]
(ii) the area of the shaded region. [5]

96 M/J/13 /P11 /Q9


1 − 12
A curve has equation y = f x and is such that f ′ x = 3x 2 + 3x − 10.
1
(i) By using the substitution u = x 2 , or otherwise, find the values of x for which the curve y = f x
has stationary points. [4]
(ii) Find f ′′ x and hence, or otherwise, determine the nature of each stationary point. [3]

(iii) It is given that the curve y = f x passes through the point 4, −7. Find f x. [4]

97 M/J/13 /P11 /Q10


y

C A (1, 1) y = (x – 2)4

x
O B

The diagram shows part of the curve y = x − 24 and the point A 1, 1 on the curve. The tangent at
A cuts the x-axis at B and the normal at A cuts the y-axis at C.
(i) Find the coordinates of B and C [6]

317
Unit 8: Integration
97 M/J/13 /P11 /Q10

a
(ii) Find the distance AC, giving your answer in the form , where a and b are integers. [2]
b
(iii) Find the area of the shaded region. [4]

98 M/J/13 /P13 /Q1


dy 
A curve is such that = 2x + 5 and 2, 5 is a point on the curve. Find the equation of the curve.
dx
[4]
y
99 M/J/13 /P13 /Q11
A (1, 7)

8
y= –x
Öx
C

B (4, 0) x
O

8
The diagram shows part of the curve y =  − x and points A 1, 7 and B 4, 0 which lie on the
x
curve. The tangent to the curve at B intersects the line x = 1 at the point C.

(i) Find the coordinates of C. [4]

(ii) Find the area of the shaded region. [5]

100 O/N/12 /P12 /Q9


y

B (0, 3)

y= 9
2x + 3
C A (3, 1)

x
O

9
The diagram shows part of the curve y = , crossing the y-axis at the point B (0, 3). The point
2x + 3
A on the curve has coordinates (3, 1) and the tangent to the curve at A crosses the y-axis at C.

(i) Find the equation of the tangent to the curve at A. [4]

(ii) Determine, showing all necessary working, whether C is nearer to B or to O. [1]

(iii) Find, showing all necessary working, the exact volume obtained when the shaded region is
rotated through 360◦ about the x-axis. [4]

318
Unit 8: Integration
101 O/N/12 /P11 /Q2

dy 8
A curve is such that = − 3 − 1 and the point (2, 4) lies on the curve. Find the equation of the
dx x
curve. [4]

102 O/N/12 /P11 /Q8


y
3y = 2x – 1

y 2 = 2x – 1

O x
1 a
2

The diagram shows the curve y2 = 2x − 1 and the straight line 3y = 2x − 1. The curve and straight line
intersect at x = 12 and x = a, where a is a constant.

(i) Show that a = 5. [2]

(ii) Find, showing all necessary working, the area of the shaded region. [6]

103 O/N/12 /P13 /Q8


A curve is such that
dy 3
= 2(3x + 4) 2 − 6x − 8.
dx

d2 y
(i) Find . [2]
dx2
(ii) Verify that the curve has a stationary point when x = −1 and determine its nature. [2]

(iii) It is now given that the stationary point on the curve has coordinates (−1, 5). Find the equation
of the curve. [5]

319
Unit 8: Integration
104 O/N/12 /P13 /Q11
y

2
y = x(x – 2)

O x
b a

The diagram shows the curve with equation y = x(x − 2)2 . The minimum point on the curve has
coordinates (a, 0) and the x-coordinate of the maximum point is b, where a and b are constants.

(i) State the value of a. [1]

(ii) Find the value of b. [4]

(iii) Find the area of the shaded region. [4]

dy
(iv) The gradient, , of the curve has a minimum value m. Find the value of m. [4]
dx

105 M/J/12 /P12 /Q1 y

y= 6
2x – 3

x
O 2 3

6
The diagram shows the region enclosed by the curve y = , the x-axis and the lines x = 2 and
2x − 3
x = 3. Find, in terms of π , the volume obtained when this region is rotated through 360◦ about the
x-axis. [4]

320
Unit 8: Integration
106 M/J/12 /P12 /Q9
y
A

y = –x2 + 8x –10

x
O

The diagram shows part of the curve y = −x2 + 8x − 10 which passes through the points A and B. The
curve has a maximum point at A and the gradient of the line BA is 2.

(i) Find the coordinates of A and B. [7]

(ii) Find ã y dx and hence evaluate the area of the shaded region. [4]

107 M/J/12 /P11 /Q11


y

2
y=
Ö + 1)
(x

y=1

x
O

The diagram shows the line y = 1 and part of the curve y = √


2
(x + 1)
.

(i) Show that the equation y = √ can be written in the form x = 2 − 1.


2 4
(x + 1)
[1]
y

(ii) Find ä  − 1 dy. Hence find the area of the shaded region.
4
[5]
y2

(iii) The shaded region is rotated through 360◦ about the y-axis. Find the exact value of the volume
of revolution obtained. [5]

321
Unit 8: Integration
108 M/J/12 /P13 /Q5

y
A

1 B (6, 1) 8
x= –2
y2

x
O

8
The diagram shows part of the curve x = − 2, crossing the y-axis at the point A. The point B (6, 1)
y2
lies on the curve. The shaded region is bounded by the curve, the y-axis and the line y = 1. Find the
exact volume obtained when this shaded region is rotated through 360◦ about the y-axis. [6]

109 M/J/12 /P13 /Q9


d2 y
A curve is such that = −4x. The curve has a maximum point at (2, 12).
dx2
(i) Find the equation of the curve. [6]

A point P moves along the curve in such a way that the x-coordinate is increasing at 0.05 units
per second.

(ii) Find the rate at which the y-coordinate is changing when x = 3, stating whether the y-coordinate
is increasing or decreasing. [2]

110 O/N/11 /P12 /Q7


dy 8
A curve is such that = 5 − 2 . The line 3y + x = 17 is the normal to the curve at the point P on the
dx x
curve. Given that the x-coordinate of P is positive, find
(i) the coordinates of P, [4]
(ii) the equation of the curve. [4]

111 O/N/11 /P12 /Q8



The equation of a curve is y = (8x − x2 ). Find
dy
(i) an expression for , and the coordinates of the stationary point on the curve, [4]
dx
(ii) the volume obtained when the region bounded by the curve and the x-axis is rotated through
360◦ about the x-axis. [4]

322
Unit 8: Integration
112 O/N/11 /P11 /Q10

y
y = Ö(1 + 2x)
C

x
A O


The diagram shows the curve y = (1 + 2x) meeting the x-axis at A and the y-axis at B. The
y-coordinate of the point C on the curve is 3.

(i) Find the coordinates of B and C. [2]

(ii) Find the equation of the normal to the curve at C. [4]

(iii) Find the volume obtained when the shaded region is rotated through 360◦ about the y-axis. [5]

113 O/N/11 /P13 /Q3


y

y = 2x

O x

y = 2x 5 + 3x 3

The diagram shows the curve y = 2x5 + 3x3 and the line y = 2x intersecting at points A, O and B.

(i) Show that the x-coordinates of A and B satisfy the equation 2x4 + 3x2 − 2 = 0. [2]

(ii) Solve the equation 2x4 + 3x2 − 2 = 0 and hence find the coordinates of A and B, giving your
answers in an exact form. [3]

323
Unit 8: Integration
114 O/N/11 /P13 /Q10
y

y = Ö(x + 1)

y=x+1

–1 x
O


The diagram shows the line y = x + 1 and the curve y = (x + 1), meeting at (−1, 0) and (0, 1).

(i) Find the area of the shaded region. [5]

(ii) Find the volume obtained when the shaded region is rotated through 360◦ about the y-axis. [7]

115 M/J/11 /P12 /Q1

Find ä x3 +  dx.


1
[3]
x3

116 M/J/11 /P12 /Q11

M
y = 4 Öx – x

x
O A


The diagram shows part of the curve y = 4 x − x. The curve has a maximum point at M and meets
the x-axis at O and A.

(i) Find the coordinates of A and M . [5]

(ii) Find the volume obtained when the shaded region is rotated through 360◦ about the x-axis, giving
your answer in terms of π . [6]

324
UNIT 8 9709/12 May/June 2022

Question M/J 22/P12/Q6 Answer Marks Guidance

1 Alternative method for question 6

1 1 M1 Equating line and curve and rearranging so that terms are all on
Line meets curve when: 2 x  2  5 x 2 ⇒ 2 x — 5 x 2 2   0  same side, condone sign errors, and making a valid attempt to
or 4 x 2  8 x  4  25 x ⇒ 4 x 2  17 x  4   0  solve by factorising, using the formula or completing the square.
1 1
2
y Factors are:(2 x2 -1)( x2 -2), (4x-1)(x-4) and (2y-5)(y-10).
or x  ⇒ 2 y 2  25 y  50   0 
25

1 A1 SC: If M1 not scored, SC B1 available for correct answers, could


x ,x  4 just be seen as limits.
4

1 *M1 Intention to integrate and subtract areas. Or integrate curve and


Area =  5 x 2 dx 
 { (2 x  2)dx or area of trapezium} subtract area of trapezium.

4
10 3   2 DM1 3
4 1 15 
 x 2     x  2 x  1 or  sum of ‘their y values’ ‘their ’ Integrating ( kx 2 seen) and substituting ‘their points of
 3  1  4 2 4  intersection’ (but limits need to be found, not assumed to be 0 and
4
15
  10   10 1     1 1  1  5  15  something) or a trapezium using the correct formula (‘their ’
    8         16  8       or   10   4
 3   3 8     16 2   2  2 4 1
must be ‘their 4’ – ‘their ’ but not 0).
4

45 13 A1 OE exact answer.
or 2 or 2.8125
16 16 45 45
Condone  if corrected to . A0 for inclusion of π.
16 16
SC: If *M1 DM0 scored, SC B1 available for correct answer.

325
Unit 8: Answer Section 9709/12 May/June 2022

Question M/J 22/P12/Q9 Answer Marks Guidance

2(a) dy  1 
1
   
1 B1 B1 Correct differentiation of 3 x  1 and no other terms and correct
 3  4  (3x  1)  3   3  6(3x  1) 2 
2 1
dx  2    differentiation of 4  3x  1 2 . Accept unsimplified.

 d2 y  1 3 3 B1 dy
 2     6  3x  1 2  3 [ 9  3x  1 2 ]
 
WWW. Accept unsimplified. Do not award if is incorrect.
dx
 dx  2

2(b) dy 1 M1 dy
 0 leading to 3  6  3x  1 2  0

Setting their = 0.
dx dx

1 A1 CAO – do not ISW for a second answer.


 3x  1 2  2  3x  1  4 leading to x  1

y = −4 [coordinates (1, −4)] A1 Condone inclusion of second value from a second answer.

d2 y 3
9 A1 d2 y
 9  3  1  1 2 = or > 0 so minimum

2
Some evidence of substitution needed but . Do not award if
dx 8 dx 2
d2 y
is incorrect or wrongly evaluated. Accept correct
dx 2
consideration of gradients either side of x  1 .

326
9709/12 Cambridge International AS & A Level – Mark Scheme Unit 8: Answer Section May/June 2022

Question Answer Marks Guidance

3(a) 2 2x 1 2 2x  1 B1 AG
f  x  1   + = Do not condone equating expressions and verification.
2x 1 2x 1 2x 1 2x  1

f   x   4  2 x  1 *M1 For k  2 x  12 and no other terms or correct use of the product or
2

 2 x  1 2  2  2 x  1
  or
quotient rule then ISW.
or 2  2 x  1 +   2 x  1 2  2 x  1
1 2

(2 x  1) 2

Gradient m = −4 A1 Differentiation must have clearly taken place.

Equation of tangent is y  3  4  x  1   y  4 x  7  DM1 Using (1, 3) in the equation of a line with their gradient.

7  A1 FT SOI from their straight line or by integration from 0 to ‘their 7/4’.


Crosses axes at  ,0  and  0, 7 
4 

49 A1 OE e.g. 6.13 AWRT.


[Area =] If M0 A0 DM0, SC B2 available for correct answer.
8

9709/11 Cambridge International AS & A Level – Mark Scheme May/June 2022

Question Answer Marks Guidance

4(a) 2 M1 1 2
1
1 1  1 x  1 M0
 3x  2 2  x  1  3x  2   x  1  x2  x  1 Equating curve and line, attempt to square;
4
2 2  4

1 M1 Forming and solving a 3TQ by factorisation, formula or


 x 2  2 x  3  0   x 2  8 x  12  0   x  6  x  2    0 completing the square – see guidance.
4

(2, 2) and (6, 4) A1 A1 A1 for each point, or A1 A0 for two correct x-values.
If M0 for solving, SC B2 possible: B1 for each point or
B1 B0 for two correct x-values.

327
9709/11 Cambridge International AS & A Level – Mark Scheme Unit 8: Answer Section May/June 2022

Question Answer Marks Guidance

4(b)  6 *M1 For intention to integrate and subtract (M0 if squared).


 1
1 
Area =    3x  2  2   x  1 [dx]

 2  2 

2 3
1 
6 B1 B1 B1 for each bracket integrated correctly (in any form).
   3x  2  2   x 2  x  
 9  4  2

2 3
1   2 3  1   DM1  ( F(their 6) – F(their 2)) with their integral.
   16  2    36  6      4  2    4  2    Allow 1 sign error.
 9 4   9 4  

4 A1 AWRT 0.444.
9 4
SC1 B1 for if *M1 B1 B1 DM0 .
9
4
SC2 B1 for if *M1 B0 B0 DM0, provided limits
9
stated.

4(b) 4 A1 AWRT 0.444.


9 4
SC1 B1 for if *M1 B1 B1 DM0 .
9
4
SC2 B1 for if *M1 B0 B0 DM0, provided limits
9
stated.

328
9709/11 Cambridge International AS & A Level – Mark Scheme Unit 8: Answer Section May/June 2022
Question Answer Marks Guidance

5(a) d2 y 4 d2 y B1 d2 y
 
2
 6  1   0 minimum or  10  minimum Sub x  1 into , correct conclusion. WWW
dx 2  13 dx 2
1
dx 2

5(b) dy 2 *M1 d2 y
 2 x3  2  c  Integrating (at least one term correct).
dx x dx 2

0 = −2 + 2 + c leading to c = [0] DM1 dy


Substituting x  1,  0 (need to see) to evaluate c.
dx
DM0 if simply state c  0 or omit c.

1 4 2 A1 FT Integrated. FT their non-zero value of c if DM1


y x    their c  x  k awarded.
2 x

9 1 DM1 9
  2  k leading to k = [2] Substituting x = –1, y = to evaluate k (dep on *M1).
2 2 2

1 2 A1 4
y  x4   2 OE e.g. 2 x 1 . or
2 x 2
A0 (wrong process) if c not evaluated but correct answer
5 obtained.
5(c) dy 2 M1 dy
 2 x3  2  0 Their  0.
dx x dx

Leading to x 5  1 M1 Reaching equation of the form x5  a .

So only stationary point is when x = −1 A1 3 x  1 and stating e.g. ‘only’ or ‘no other solutions.

5(d) dy *M1 dy
At x = 1,   4 Substituting x  1 into their .
dx dx

dx dx dy 1 DM1 OE Using chain rule correctly SOI.


   5
dt dy dt 4

5 A1 OE e.g. 1.25.
4 3

329
9709/13 M/J 22/P13/Q8 Cambridge International AS & A Level – Mark Scheme Unit 8: Answer Section May/June 2022

Question Answer Marks Guidance

6(a)  dy  B1 B1 Allow unsimplified versions.


 dx   ½ x 1/2  2 x 3/2
 

dy 1 3 M1 Substitute x = 1 into a differentiated y.


At x = 1,   2 
dx 2 2

3 A1 3 13
Equation of tangent is y  5    x  1 WWW Or y   x  .
2 4 2 2

6(b) x 3/ 2 B1 OE Integrate to find area under curve, allow unsimplified


 8 x1/ 2 versions.
3/ 2

 128  2  M1 Apply limits 1 → 16 to an integrated expression.


 3  32    3  8  
   

Area under line = 15  5 = 75 B1 16


Or by 5dx .
1

Required area = 75 ‒ 66 = 9 A1

M/J 22/P13/Q10/a 4

7(a)   4 x  2 1  B1 B1 OE If more than one function of x present then B0 B0.



 1 
1
16 

4 or eg     x  0.5  
1
or
1
16 x  8

0  (1 / 24) M1 Apply limits to an integral, ∞ must be used correctly.

1/24 A1 Allow 0.0417 AWRT.

330
9709/12 Cambridge International AS & A Level – Mark Scheme October/November 2021
Unit 8: Answer Section

Question Answer Marks Guidance

8 8 *B1 For ( 3x + 2 )−1


y=− 3 [ +c ]
( 3x + 2 ) DB1 8
For −
3

8 M1  2
2 Substituting  2, 5  into their integrated expression –
5 =− 3 +c  3
3 (3 × 2 + 2) defined by power = -1, or dividing by their power. + c needed

8 A1 8 −1
y=− +6 OE e.g. y = − (3x + 2 ) + 6
3 ( 3x + 2 ) 4 3

9709/12 O/N 21/P12/Q11 Cambridge International AS & A Level – Mark Scheme October/November 2021

Question Answer Marks Guidance

9(a) dy 1 1 B1 OE. Allow unsimplified.


= + 4
dx 2
3( x − 2) 3

  *M1 Substituting x = 3 into their differentiated expression –


dy  1 1 5 defined by one of 3 original terms with correct power of x.
Attempt at evaluating their at x = 3 + =
dx  2 4
6
 3 ( 3 − 2 ) 3 

−1  6 *DM1 dy
Gradient of normal = =−  Negative reciprocal of their evaluated .

dy  5 dx
their
dx
6 DM1 Using their normal gradient and A in the equation of a
Equation of normal y − = ( their normal gradient )( x − 3) straight line.
5
Dependent on *M1 and *DM1.
 6 
 y = − 5 x + 4.8  5 y = −6 x + 24 
 
[When y = 0,] x = 4 A1 5 or (4, 0)

331
Unit 8: Answer Section
9709/12 O/N 21/P12/Q11 Cambridge International AS & A Level – Mark Scheme October/November 2021

Question Answer Marks Guidance

9(b)   M1 For intention to integrate the curve (no need for limits).
1 7 1
Area under curve =   x + −  dx Condone inclusion of π for this mark.
2 10 1 [ ]
 ( x − 2)3 
 
2 A1 For correct integral. Allow unsimplified.
1 2 7 3( x − 2) 3 Condone inclusion of π for this mark.
x + x−
4 10 2

 2
 M1 Clear substitution of 3 and 2.5 into their integrated
 9 3   6.25 3 × 0.5 3
 expression (with at least one correct term) and subtracting.
 + 2.1 −  −  + 1.75 − 
4 2  4 2 
 

0.48[24] A1 If M1A1M0 scored then SC B1 can be awarded for correct


answer.

[Area of triangle =] 0.6 B1 OE

[Total area =] 1.08 A1 Dependent on the first M1 and WWW.


6

9709/11 O/N 21/P11/Q9 Cambridge International AS & A Level – Mark Scheme October/November 2021

Question Answer Marks Guidance

10(a) f ( x ) = 23 x3 − 7 x + 4 x −1 [ +c ] B2, 1, 0 Allow terms on different lines; allow unsimplified.

− 13 = 23 − 7 + 4 + c leading to c = [2] M1 Substitute f(1) = − 1 into an integrated expression and


3
evaluate c.

f ( x ) = 23 x3 − 7 x + 4 x −1 + 2 A1 OE.

332
Unit 8: Answer Section
9709/11 O/N 21/P11/Q9 Cambridge International AS & A Level – Mark Scheme October/November 2021

Question Answer Marks Guidance

10(b) 2 x 4 − 7 x 2 − 4 [ = 0] M1 Forms 3-term quadratic in x2 with all terms on one side.


Accept use of substitution e.g. 2 y 2 − 7 y − 4 [ = 0] .

( 2x 2
)(
+ 1 x2 − 4 ) [ = 0] M1 Attempt factors or use formula or complete the square.
Allow ± sign errors. Factors must expand to give their
coefficient of x2 or e.g. y. Must be quartic equation.
Accept use of substitution e.g. ( 2 y + 1)( y − 4 ) .

x = [ ±] 2 A1 If M0 for solving quadratic, SC B1 can be awarded for


[ ±] 2 .
2 3 4  14  B1 B1 B1 B1 for correct coordinates clearly paired; B1 for each
 3 ( 2 ) − 7 ( 2 ) + 2 + 2 leading to   2, − 3  correct point; B1 B0 if additional point.
  
2 4   26 
 3 ( −2 ) − 7 ( −2 ) + −2 + 2 leading to   −2, 3 
3

   

10(c) f ′′ ( x ) = 4 x + 8 x −3 B1 OE

333
Unit 8: Answer Section
9709/11 O/N 21/P11/Q9 Cambridge International AS & A Level – Mark Scheme October/November 2021
PUBLISHED
Question Answer Marks Guidance

10(d) f ′′ ( 2 ) = 9 > 0 MINIMUM at x = their 2 B1 FT FT on their x = [ ± ] 2 provided f ′′ ( x ) is correct.


Must have correct value of f ′′ ( x ) if x = 2 .

f ′′ ( −2 ) = −9 < 0 MAXIMUM at x = their − 2 B1 FT FT on their x = [ ±] 2 provided f ′′ ( x ) is correct.


Must have correct value of f ′′ ( x ) if x = −2 .
Special case: If values not shown and B0B0 scored, SC B1
for f ′′ ( 2 ) > 0 MIN and f ′′ ( −2 ) < 0 MAX

Alternative method for question 9(d)

Evaluate f ' ( x ) for x-values either side of 2 and –2 M1 FT on their x = [ ± ] 2

MINIMUM at x = their 2, MAXIMUM at x = their 2 A1 FT FT on their x = [ ± ] 2 . Must have correct values of f ' ( x ) if
shown.
Special case: If values not shown and M0A0 scored SC B1
f ′ ( 2 ) − /0/+ MIN and f ' ( −2 ) +/0/ − MAX

Alternative method for question 9(d)

Justify maximum and minimum using correct sketch graph B1 B1 Need correct coordinates in (b) for this method.

334
Unit 8: Answer Section
9709/11 Cambridge International AS & A Level – Mark Scheme October/November 2021

Question Answer Marks Guidance

11(a)  ( 3x − 2 )− 12  B2, 1, 0 Attempt to integrate


  ÷ {3}
 −1/ 2 

− 23 [ 0 − 1] M1 M1 for applying limits 1 → ∞ to an integrated expression


(either correct power or dividing by their power).

2 A1
3 4
11(b) ( 3x − 2 )−2 *M1 A1 M1 for attempt to integrate y2 (power increases); allow 1
[ π]  y 2
dx = [ π ]  ( 3x − 2 ) dx = [ π ]
−3
error. A1 for correct result in any form.
−2 × 3

DM1 Apply limits 1 and 2 to an integrated expression and


[ π ]  −
1 1 
6  16 − 1 subtract correctly; allow 1 error.
  

5π A1 OE
32 4

11(c) dy 3 −
5 M1 M1 for attempt to differentiate (power decreases); allow 1
= − × 3 ( 3x − 2 ) 2 error.
dx 2

dy 9 *M1 Substitute x = 1 into their differentiated expression; allow 1


At x = 1, =− error.
dx 2

2 DM1 Forms equation of line or evaluates c using (1, 1) and


[Equation of normal is] y − 1 = ( x − 1) OR evaluates c −1
9 gradient .
dy
their
dx

7 A1 OE e.g. AWRT 0.778; must clearly identify y-intercept


At A, y =
9

335
9709/13 O/N 21/P13/Q8 Cambridge International AS & A Level – Mark Scheme Unit 8: Answer Section October/November 2021

Question Answer Marks Guidance

12(a) 5 1
− 
1 M1 5 
  − x − x 2  dx
2
2 
( )
OR as 2 separate integrals   − x1/2  dx −  x −1/2 dx
2 

 5 2 2 
3
 1  A1 A1 A1 If two separate integrals with no subtraction SC B1 for
 x − x  { −} 2 x 2  each correct integral.
 2 3   

 16  5 1  DM1 1
10 − − 4  −  − − 1 Substitute limits → 4 at least once, must be seen.
 3   8 12  4

9 A1 WWW. Cannot be awarded if π appears in any integral.


or 1.125
8

12(b)  dy  1 −
3 B1
 dx = − x 2
  2

1 M1 Substitute x = 1 into a differential.


When x = 1, m = −
2

[Equation of normal is] y − 1 = 2 ( x − 1) M1 1 1− p


Through (1, 1) with gradient − or =2
m 1

[When x = 0,] p = ‒1 A1 WWW

336
9709/13 Cambridge International AS & A Level – Mark Scheme October/November 2021

Question O/N 21/P13/Q10 Answer Marks Guidance

13(a) f ′′ ( x ) = − ( 12 x + k )
−3 B1

f ′′ ( 2 ) > 0  − (1 + k ) > 0
−3 M1 Allow for solving their f ′′ ( 2 ) > 0

k < −1 A1 WWW

13(b)  ( 1 x − 3)−1  B1 B1 1 
−1

 (( )  x −2
f ( x ) = 1
x − 3) − ( −2 )
−2 −2 
dx =  2  −  Allow −2  x + k  OE for 1st B1 and − (1 + k ) x OE
 2
  −1 × 12   4 2 
  for 2nd B1

3 12 = 1 - 12 + c M1 Substitute x = 2, y = 3 1 into their integral with c present.


2

−2 x A1 OE
f ( x) = − +3
( 1
2
x − 3) 4

13(c) M1 Substitute k = −3 and set to zero.


( 12 x − 3)
−2 −2
− ( −2 ) =0

A1
( 12 x − 3)
2
leading to = 4  12 x − 3 = ( ± ) 2 leading to x = 10

(10, ‒ 12 ) A1 Or when x = 10, y = ‒1 ‒ 2 1 + 3 = ‒ 1


2 2

f ′′ (10 )  = − ( 5 − 3)
−3
→  < 0 → MAXIMUM A1 WWW
 

337
9709/12 Cambridge International AS & A Level – Mark Scheme May/June 2021

Question Answer Marks Guidance

14 Curve intersectsy = 1 at (3, 1) B1 Throughout Question 9: 1 < their 3 < 5


Sight of x = 3

Volume = [ π ]  ( x − 2 ) [ dx ] M1 M1 for showing the intention to integrate ( x − 2 ) .


Condone missing π or using 2 π.

A1 Correct integral. Condone missing π or using 2 π.


[ π ]   1 2
1 2
x − 2 x  or [ π ]  ( x − 2 ) 
2  2 

 52   their 32  M1 Correct use of ‘their 3’ and 5 in an integrated expression.


= [ π ]  − 2 × 5 −  − 2 × their 3   Condone missing π or using 2 π. Condone +c.
 2   2   Can be obtained by integrating and substituting between 5 and 2
5 3 and then 3 and 2 then subtracting.
= [ π]  + as a minimum requirement for their values
2 2 

Volume of cylinder = π × 12 × ( 5 − their 3) [ = 2π ] B1 FT Or by integrating 1 to obtain x (condone y if 5 and their 3 used).

[Volume of solid = 4 π − 2π =] 2π or 6.28 A1 AWRT


14 Alternative method for Question 9

Curve intersects y = 1 at (3, 1) B1 Sight of x = 3

Volume of solid = π  ( x − 2 ) − 1[ dx ] M1 B1 M1 for showing the intention to integrate ( x − 2 )


B1 for correct integration of –1.
Condone missing π or 2 π for M1 but not for B1.

A1 Correct integral, allow as two integrals.


[ π ]   1 2
1 2
x − 3 x  or [ π ]  ( x − 3)  Condone missing π or using 2 π.
2  2 

 52   their 32  M1 Correct use of ‘their 3’ and 5 in an integrated expression.


= [ π ]  − 3 × 5  −  − 3 × their 3   Condone missing π or using 2 π. Condone +c.
 2   2   Can be obtained by integrating and substituting between 5 and 2
and then 3 and 2 then subtracting.

[Volume of solid = 4 π − 2π =] 2π or 6.28 A1 AWRT 6

338
9709/12 M/J 21/P12/Q11 Cambridge International AS & A Level – Mark Scheme May/June 2021

Question Answer Marks Guidance

15(a) dy M1 dy
= 0 so 6 ( 3 × 2 − 5) − k × 22 = 0
3
At stationary point Setting given = 0 and substituting x = 2 into it.
dx dx

3 A1 OE
[k = ]
2

15(b) 6 1 *M1 dy
[ y =] ( 3x − 5)4 − kx3 [+c] . A1FT Integrating (increase of power by 1 in at least one term) given dx
4×3 3
1 1
. Expect ( 3x − 5) − x3 .
4

2 2
FT their non zero k.

7 1 1 3 DM1 Using (2,-3.5) in an integrated expression. + c needed.


− = ( 3 × 2 − 5) − × × 23 + c [leading to −3.5 + c = −3.5 ]
4

2 2 3 2 Substitution needs to be seen, simply stating c = 0 is DM0.

1 1 A1 y = or f(x)= must be seen somewhere in solution.


y= ( 3x − 5 )4 − x3
2 2
15(b) Alternative method for Question 11(b)

81 4 541 3 x3 *M1 dy
[ y =] x − x + 675 x 2 − 750 x ( +c ) or -270 x3 − k
3 2 2
A1 FT From dx = 162 x − 810 x − kx − 1350 x − 750 . FT their k
2 2 3

7 81 541 3 DM1 Using (2, -3.5) in an integrated expression. + c needed


− = × 24 − × 2 + 675 × 22 − 750 × 2 + c
2 2 2

81 4 541 3 625 A1 y = or f(x)= must be seen somewhere in solution.


y= x − x + 675 x 2 − 750 x +
2 2 2

339
9709/12 M/J 21/P12/Q11 Cambridge International AS & A Level – Mark Scheme May/June 2021

Question Answer Marks Guidance

15(c) B2,1,0 FT FT their k.


[3 ×] 18 ( 3x − 5)2  [ −2kx ] Square brackets indicate each required component.
B2 for fully correct, B1 for one error or one missing component,
B0 for 2 or more errors.

Alternative method for Question 11(c)

486 x 2 − 1623 x + 1350 or −1620 x − 2kx B2,1,0 FT FT their k.


B2 for fully correct, B1 for one error, B0 for 2 or more errors.

15(d)  d2 y  M1 OE. Substituting x = 2 into their second differential or other valid


[At x = 2]  2 =  54 ( 3 × 2 − 5 ) − 4k or 48
2
method.
 dx 

[> 0] Minimum A1 WWW

9709/11 M/J 21/P11/Q1 Cambridge International AS & A Level – Mark Scheme May/June 2021

Question Answer Marks Guidance

16 1 B1 B1 OE. Accept unsimplified.


[ y =] − 3
+ 8 x 4 [+ c]
x

1 M1 1 
4 = –8 + +c Substituting  , 4  into an integrated expression
2 2 

1 23 A1 OE. Accept −x−3 ; must be 8; y = must be seen in


y=− 3
+ 8x4 +
x 2 working.

340
9709/11 M/J 21/P13/Q1 Cambridge International AS & A Level – Mark Scheme May/June 2021

Question Answer Marks Guidance

17 1 B1 B1 OE. Accept unsimplified.


[ y =] − + 8 x 4 [+ c]
x3

1 M1 1 
4 = –8 + +c Substituting  , 4  into an integrated expression
2 2 

1 23 A1 OE. Accept −x−3 ; must be 8; y = must be seen in


y=− 3
+ 8x4 +
x 2 working.

9709/13 Cambridge International AS & A Level – Mark Scheme May/June 2021

Question Answer Marks Guidance

18(a) dy 1 −1/2 1 2 −3/2 B1 B1 Allow any correct unsimplified form


= x − k x
dx 2 2
1 −1/2 1 2 −3/2 1 1 M1 OE. Set to zero and one correct algebraic step
x − k x = 0 leading to x −1/2 = k 2 x −3/2 towards the solutions.
2 2 2 2
dy
must only have 2 terms.
dx

(k 2
, 2k ) A1 4
18(b) dy  1 1  3 B1 OE
When x = 4k2, = − =
dx  4k 16k  16k

 1  5k B1 k
y =  2k + k 2 ×  = OE. Accept 2k +
 2k  2 2
5k 3 5k 3 M1 Use of line equation with their gradient and
Equation of tangent is y − =
2 16k
( )
x − 4k 2 or y = mx + c → =
2 16k
( )
4k 2 + c
( 4k 2 , their y ) ,

 5k 3k  7k 7k A1 OE
When x = 0, y =  − =  or from y = mx + c, c =
2 4  4 4 4

341
9709/13 Cambridge International AS & A Level – Mark Scheme May/June 2021

Question Answer Marks Guidance

18(c) 3 B1 Any unsimplified form


 1 − 
1
2x 2
1
  x 2 + k 2 x 2  dx = + 2k 2 x 2
  3
 

 16k 3   9k 3  M1 9 2
 + 4k 3  −  + 3k 3  Apply limits k → 4k 2 to an integration of y.
4
 3   4 
M0 if volume attempted.

49k 3 A1 OE. Accept 4.08 k 3


12

9709/12 Cambridge International AS & A Level – Mark Scheme October/November 2020

Question Answer Marks Guidance

19(a) 1

1 B1 B1 B1 for each unsimplified integral.
6x 2 4x 2
1

1
( +c )

2 2

Uses (4, 7) leading to c = (-21) M1 Uses (4, 7) to find a c value

1

1
8 A1 Need to see y or f(x) = somewhere in their solution and 12
y or f ( x ) = 12 x 2 + 8x 2 − 21 or 12 x + − 21 and 8
x

342
9709/12 Cambridge International AS & A Level – Mark Scheme October/November 2020

Question Answer Marks Guidance

20(a) dy  8  B2, 1, 0 B2 for all three elements correct, B1 for two elements
( (
) = [8] ×[ 3 − 2 x) −3  + [ −1]  = −1 correct, B0 for only one or no elements correct.
dx  (3 − 2x) ³ 

 B1 FT FT providing their bracket is to a negative power


d² y 48 
= −3 × 8 × (3 − 2 x) −4 × ( −2 ) = 
dx²  ( 3 − 2x )4 
 

 1 1  B1 B1 B1 Simplification not needed, B1 for each correct element


∫ydx = [(3− 2x)−1 ] [2 ÷ (−1 × −2)] [ − ½x²] (+c) = − x2 + c 
 3 − 2x 2  6
20(b) dy M1 Setting their 2-term differential to 0 and attempts to solve
= 0 → (3 – 2x)³ = 8 → 3 – 2x = k → x = as far as x =
dx

1 A1
2

20(c)  1  M1 Using their integral, their positive x limit from part (b) and
  ² 0 correctly.
1 2  1 
Area under curve = their  −   − − 0
3 − 2×  1  2  3 − 2× 0 
   
2

1 A1
24

343
9709/11 Cambridge International AS & A Level – Mark Scheme October/November 2020

Question O/N 20/P11/Q2 Answer Marks Guidance

21 B1 B1
( y = )  − ( x − 3)
−1
 + 1 x2  ( +c )
  2 

M1 Substitute x = 2, y = 7 into an integrated expansion (c present).


7=1+2+c
Expect c = 4

−1 1 A1 OE
y = − ( x − 3) + x 2 +4
2 4

9709/11 Cambridge International AS & A Level – Mark Scheme October/November 2020

Question O/N 20/P11/Q12 Answer Marks Guidance

22(a) 1 1 *M1
4x 2 − 2x = 3 − x → x − 4x 2 + 3( = 0) 3-term quadratic. Can be expressed as e.g. u 2 − 4u + 3 (=0)

 1  1  DM1
 x 2 − 1 x 2 − 3  ( = 0 ) or ( u − 1)( u − 3)( = 0 ) Or quadratic formula or completing square
 
  

1 A1
x 2 = 1, 3 SOI

x = 1, 9 A1

Alternative method for question 12(a)

 1
2 *M1
1
 4 x 2  = ( 3 + x )
2
Isolate x 2
 

16 x = 9 + 6 x + x 2 → x 2 − 10 x + 9 ( = 0 ) A1 3-term quadratic

DM1 Or formula or completing square on a quadratic obtained by a


( x − 1)( x − 9 ) ( = 0 ) correct method

x = 1, 9 A1

344
9709/11 Cambridge International AS & A Level – Mark Scheme October/November 2020
PUBLISHED
Question O/N 20/P11/Q12 Answer Marks Guidance

22(b) dy *B1
= 2 x1/2 − 2
dx

dy DB1
or 2 x1/2 − 2 = 0 when x =1 hence B is a stationary point
dx

22(c) M1 9

1 1 2
Area of correct triangle =
2
(9 ‒ 3) × 6 or  ( 3 − x )( dx ) = 3x − 2 x
3
 → −18

 3  B1 B1
1
 4x 2 2
 (4 x 2 − 2 x) ( dx ) =  −x 
3
 
 2 

M1
( 72 − 81) −  
64
− 16  Apply limits 4 → their 9 to an integrated expression
 3 

−14 13 A1 OE

Shaded region = 18 − 14 13 = 3 23 A1 OE

345
9709/13 Cambridge International AS & A Level – Mark Scheme October/November 2020

Question O/N 20/P13/Q2 Answer Marks Guidance

23(a) −2 B1 Integrate f(x). Accept −2 ( x + 2 )−1 . Can be unsimplified.


x+2

 2 2 M1 A1 Apply limit(s) to an integrated expansion. CAO for A1


0−−  =
 3 3

23(b) −1 = − 2 + c M1 Substitute x = −1, y = −1 into their integrated expression (c


present)

−2 A1 Accept y = −2 ( x + 2 )−1 + 1 . ‒2 must be resolved.


y= +1
x+2

9709/13 Cambridge International AS & A Level – Mark Scheme October/November 2020

Question O/N 20/P13/Q10 Answer Marks Guidance

24(a) dy  x −1/2   x −3/2  B2, 1, 0 ([0]) implies that more than 2 terms counts as an error
= −  + ( [ 0 ])
dx  2k   2 

dy 1 1 M1
Sub = 3 when x = Expect 3 = −4
dx 4 k

1 A1
k= (or 0.143)
7 4

346
9709/13 Cambridge International AS & A Level – Mark Scheme October/November 2020

Question Answer Marks Guidance


24(b) 1 1  2 x 3/2   x B2, 1, 0 OE
 x1/2 + x −1/2 + 2 =   +  2 x  +  2 
1/2
k k  3k  k 

 2k 2   k2 1 M1 k2
 + 2k + 1 −  + k +  Apply limits → k 2 to an integrated expression.
 3   12 4 4
7 2 3
Expect k +k+
12 4

7 2 3 13 M1 13
k +k + = Equate to and simplify to quadratic.
12 4 12 12
OE, expect 7 k 2 + 12k − 4 ( = 0 )

2 A1 Dependent on ( 7k − 2 )( k + 2 ) ( = 0 ) or formula or completing


k= only (or 0.286)
7 square.

9709/12 M/J 20/P12/Q8 Cambridge International AS & A Level – Mark Scheme May/June 2020

Question Answer Marks

25(a) 36 *M1
Volume = π ∫ x²dy = π  dy

 −36  A1
= π 
 y 

Uses limits 2 to 6 correctly → (12π) DM1

Vol of cylinder = π.1².4 or  12.dy = [y] from 2 to 6 M1

Vol = 12π − 4π = 8π A1

347
9709/11 Cambridge International AS & A Level – Mark Scheme October/November 2020

Question M/J 20/P12/Q8 Answer Marks Guidance

25(b) dy −6 B1
=
dx x²

−6 M1
= −2 → x = 3

6 A1
y= = 2 3 Lies on y = 2x
3

9709/11 Cambridge International AS & A Level – Mark Scheme May/June 2020

Question 26 M/J 20/P11/Q11 Answer Marks

26(a) 8 M1
Simultaneous equations = 4 − ½x
x+2

x = 0 or x = 6 → A (0, 4) and B (6, 1) B1A1

−8 1 B1
At C = − → C (2, 2)
( x + 2) ² 2
M1A1
(B1 for the differentiation. M1 for equating and solving) 6
26(b)  x³  M1
Volume under line = π  ( − 12 x + 4 ) ²dx = π  − 2 x² + 16 x  = (42π) A2,1
12 
(M1 for volume formula. A2,1 for integration)

 8 
2
 −64  A1
Volume under curve = π    dx = π 
 x+2 
 = (24π)
 x + 2

Subtracts and uses 0 to 6 → 18π M1A1 6

348
9709/13 Cambridge International AS & A Level – Mark Scheme May/June 2020

Question Answer Marks

27 3x 2
3
3x 2
1
B1 B1
( y) = 3
− 1
( +c )
2 2

7 = 16 ‒ 12 + c M1
(M1 for subsituting x = 4, y = 7 into their integrated expansion)
3
y = 2x 2 − 6x 2 + 3
1
A1
4

9709/13 Cambridge International AS & A Level – Mark Scheme May/June 2020

Question Answer Marks

28(a) dy B1
= 3x 2 − 4bx + b 2
dx

3x 2 − 4bx + b 2 = 0 → ( 3 x − b )( x − b ) ( = 0 ) M1

b A1
x= or b
3

b A1
a= → b = 3a AG
3

Alternative method for question 11(a)

dy B1
= 3x 2 − 4bx + b 2
dx

dy M1
Sub b = 3a & obtain = 0 when x = a and when x = 3a
dx

d2 y A1
= 6 x − 12 a
dx 2

< 0 Max at x = a and > 0 Min at x = 3a. Hence ܾ = 3ܽ AG A1

349
9709/13 Cambridge International AS & A Level – Mark Scheme May/June 2020
Question M/J 20/P13/Q11 Answer Marks

28(b) (
Area under curve =  x 3 − 6ax 2 + 9a 2 x dx ) M1

x4 9a 2 x 2 B2,1,0
− 2ax 3 +
4 2

a4 9a 4  11a 4  M1
− 2a 4 + = 
4 2  4 
(M1 for applying limits 0 → a)

3 3 3
When x = a, y = a − 6a + 9a = 4a
3 B1

1 M1
Area under line = a × their 4a3
2

11a 4 3 A1
Shaded area = − 2a 4 = a 4
4 4 7

9709/12 MAR 20/P12/Q3 Cambridge International AS & A Level – Mark Scheme March 2020

Question Answer Marks Guidance

29 ( π )  ( y − 1) dy *M1 SOI
Attempt to integrate x2 or ( y − 1)

 y2  A1
( π)  − y
2 

 25   1  DM1 Apply limits 1 → 5 to an integrated expression


( π )  − 5  −  − 1 
 2   2 

8π or AWRT 25.1 A1

350
9709/12 Cambridge International AS & A Level – Mark Scheme March 2020

Question Answer Marks Guidance

30(a) 1 M1 SOI.
2 ( a + 3) 2 − a = 0 dy
Set = 0 when x = a. Can be implied by an answer in terms of a
dx

4 ( a + 3) = a 2 → a 2 − 4a − 12 = 0 M1 Take a to RHS and square. Form 3-term quadratic

( a − 6 )( a + 2 ) → a = 6 A1 Must show factors, or formula or completing square. Ignore a = ‒2


SC If a is never used maximum of M1A1 for x = 6 ,with visible
solution

30(b) d2 y −
1
B1
( )
2

2
= x + 3 −1
dx

d2 y 1 2 M1A1 A mark only if completely correct


Sub their a → 2
= − 1 = − (or < 0) → MAX 2
dx 3 3 If the second differential is not − correct conclusion must be
3
drawn to award the M1

30(c) 3
2 ( x + 3) 2 B1B1
1
( y =) 3
− x2 ( +c )
2
2

4 3 M1 Substitute into an integrated expression. c must be present.


Sub x = their a and y = 14 → 14 = ( 9 ) 2 − 18 + c Expect c = ‒4
3

4 3
1 A1 Allow f ( x ) = ….
y= ( x + 3) 2 − x 2 − 4
3 2

351
9709/12 Cambridge International AS/A Level – Mark Scheme October/November 2019

Question Answer Marks Guidance

31 1 
− +1  B1 OE
kx 2  k x
(y =) = (+c)
1  1 
− +1
2  2 

Substitutes both points into an integrated expression with a ‘+c’ and M1 Expect to see –1 = 2k + c and 4 = 4k + c
solve as far as a value for one variable.
k = 2½ and c = −6 A1 WWW

y = 5 x −6 A1 OE
4 From correct values of both k & c and correct integral.

9709/12 Cambridge International AS/A Level – Mark Scheme October/November 2019

Question Answer Marks Guidance

32(i) dy B2,1,0 OE. Full marks for 3 correct components. Withhold one mark
= [ 0] + (2 x + 1)−3  × [+ 16] for each error or omission.
dx

∫ydx = [ x ] + (2 x + 1)−1  × [ +2] (+c) B2,1,0 OE. Full marks for 3 correct components. Withhold one mark
for each error or omission.

32(ii) At A, x = ½. B1 Ignore extra answer x = −1.5

dy *M1 dy
= 2 → Gradient of normal ( = −½ ) With their positive value of x at A and their , uses
dx dx
m₁m₂ = −1

Equation of normal: DM1 Use of their x at A and their normal gradient.


y − 0 = −½ ( x − ½ ) or y − 0 = −½ (0 – ½) or 0 = −½×½ + c

B (0, ¼) A1

352
9709/12 Cambridge International AS/A Level – Mark Scheme October/November 2019

Question O/N 19/P12/Q10 Answer Marks Guidance

32(iii) 1 *M1 ∫ y dx SOI with 0 and their positive x coordinate of A.


2
4
∫1 − ( 2 x + 1) ( dx )
0
2

[½ + 1] – [0 + 2] = (−½) DM1 Substitutes both 0 and their ½ into their ∫ydx and subtracts.

 1 B1
Area of triangle above x-axis = ½ × ½ × ¼  = 
 16 

9 A1 OE (including AWRT 0.563)


Total area of shaded region =
16

Alternative method for question 10(iii)


0
1 1 *M1  −
1

∫ 1
− ( dy ) ∫ x dy SOI. Where x is of the form k 1 − y ) 2 + c  with 0 and
 
2
−3
(1 − y ) 2  
their negative y intercept of curve.

DM1 Substitutes both 0 and their –3 into their ∫xdy and subtracts.
[ −2] −  −4 +
32() 3
= (½)
 2 

 1 B1
Area of triangle above x-axis = ½ × ½ × ¼  = 
 16 

9 A1 OE (including AWRT 0.563)


Total area of shaded region =
16

353
9709/11 Cambridge International AS/A Level – Mark Scheme October/November 2019
PUBLISHED
Question Answer Marks Guidance

33(i) y = [( 5 x − 1) ÷ 32 ÷5 ] [ − 2 x] B1
1/ 2

B1

27 M1 Substitute x = 2, y = 3
3= −4+c
( 2) × 5
3 /

 3
 A1
18 17
c=7− = 
→ y=
2 ( 5 x − 1) 2
− 2x +
17 
5 5  15 5 

 

33(ii) d 2 y / dx 2 = ½ ( 5 x − 1)
−1/2
 [×5] B1
  B1

33(iii) ( 5 x − 1)1/ 2 − 2 = 0 → 5x − 1 = 4 M1A1 dy


Set = 0 and attempt solution (M1)
x =1 dx

16 17 37 A1  37 
y= −2+ = Or 2.47 or  1, 
25 5 15  15 

d2 y 5 1 5 A1 OE
= × = (> 0) hence minimum
dx x 2 2 4

9709/11 Cambridge International AS/A Level – Mark Scheme October/November 2019


PUBLISHED
Question Answer Marks Guidance

34(i) ( y = ) ( x + 2 )2 − 1 B1 2nd B1 dependent on 2 in bracket


DB1

x + 2 = ( ± )( y + 1) M1
1/ 2

x = −2 + ( y + 1) A1
1/ 2

354
9709/11 Cambridge International AS/A Level – Mark Scheme October/November 2019

Question Answer Marks Guidance

34(ii) x 2 = 4 + ( y + 1) − / + 4 ( y + 1) 2
1
*M1A1 SOI. Attempt to find x 2 .The last term can be ‒ or + at this
stage

 3 
A2,1,0
 y 2
4 ( y + 1) 2

( π ) ∫ x 2 ( dy ) = ( π )  5 y + − 3 
 2 
 2 

 9 64  1  DM1 Apply y limits


( π ) 15 + − −  −5 +  
 2 3  2 

8π A1
or 8.38
3

9709/13 O/N 19/P13/Q11 Cambridge International A Level – Mark Scheme October/November 2019

Question Answer Marks Guidance

35(i) dy B1
= −2 ( x − 1)
−3

dx

1 M1 m must come from differentiation


When x = 2, m = ‒2 → gradient of normal = −
m

A1 1
Equation of normal is y − 3 = ½ ( x − 2 ) → y = ½ x + 2 AG Through (2, 3) with gradient − . Simplify to AG
m

355
9709/13 Cambridge International A Level – Mark Scheme October/November 2019

Question O/N 19/P13/Q11 Answer Marks Guidance

35(ii) ( π) ∫ y12 ( dx ) , ( π ) ∫ y2 2 ( dx ) *M1 Attempt to integrate y 2 for at least one of the functions

( π ) ∫ ( 12 x + 2 )
2
or ( 1
4
x2 + 2 x + 4 ) A1A1 A1 for
( 12 x + 2 )
2
depends on an attempt to integrate this

( π ) ∫ ( ( x − 1)−4 + 4 ( x − 1)−2 + 4 )
form later

( π )  23 ( 12 x + 2 )
3
or 1
x3 + x 2 + 4 x  A1A1 Must have at least 2 terms correct for each integral
12 
 ( x − 1)−3 4 ( x − 1)
−1

(π)  + + 4x
 −3 −1 

 125 2 1    −1  −1  DM1 Apply limits to at least 1 integrated expansion


(π) 18 − or +4+8− + 1 + 4    − 2 + 12 −  − 4 + 8 
 12 3  12    24  3 

Attempt to add 2 volume integrals (or 1 volume integral + frustum) DM1

{
π 7
12
7
+6
7
24 }
7 111 A1 2 1  −1  −1 
13 π or π or 13.9π or 43.6 + 4+8− +1+ 4 − 2 + 12 −  − 4 + 8 
8 8 3  12  24  3 

356
9709/12 Cambridge International AS/A Level – Mark Scheme May/June 2019
PUBLISHED
Question M/J 19/P12/Q11 Answer Marks Guidance

36(i) dy  1 − 
1
 9 − 
3 B1B1B1 B1 B1 for each, without × 4. B1 for ×4 twice.
=  ( 4 x + 1) 2  [× 4]  − 2 ( 4 x + 1) 2  [× 4]
dx  2   
  SC If no other marks awarded award B1 for both
 2 18 8 x − 16  powers of (4x +1) correct.
 4x + 1 − or 3 
( )
3


4x + 1 ( 4 x + 1) 2 
 3   1  B1B1B1 B1 B1 for each, without ÷ 4. B1 for ÷4 twice.
 ( 4 x + 1) 2   9 ( 4 x + 1) 2  + C not required.
∫ydx =   [÷ 4] +   [÷ 4] (+C)
3 1
   
 2   2 

(
 4x + 1
)  SC If no other marks awarded , B1 for both powers
3

 6
+
9
2
( ) 
4 x + 1 ( +C )  of (4x +1) correct.
  6
 
36(ii) dy 2 18 M1 dy
=0 → − 3
=0 Sets their to 0 (and attempts to solve
dx 4x + 1 dx
( 4 x + 1) 2

4x + 1 = 9 or (4x + 1)2 = 81 A1 Must be from correct differential.

x = 2, y = 6 or M is (2, 6) only. A1 Both values required.


3 Must be from correct differential.

36(iii) Realises area is ∫y dx and attempt to use their 2 and sight of 0. *M1 Needs to use their integral and to see ‘their 2’
substituted.

1 DM1 Uses both 0 and ‘their 2’ and subtracts. Condone


Uses limits 0 to 2 correctly → [4.5 + 13.5] – [ + 4.5] ( = 13⅓ ) wrong way round.
6

(Area =) 1⅓ or 1.33 A1 Must be from a correct differential and integral.

3 13⅓ or 1⅓ with little or no working scores


M1DM0A0.

357
9709/11 Cambridge International AS/A Level – Mark Scheme May/June 2019

Question M/J 19/P11/Q10 Answer Marks Guidance

37(i) dy B1
integrating → = x² − 5x (+c)
dx

= 0 when x = 3 M1 Uses the point to find c after ∫ = 0.

c=6 A1

x³ 5x² B1 FT Integration again FT if a numerical constant term is


integrating again → y = − + 6x ( +d ) present.
3 2

use of (3, 6) M1 Uses the point to find d after ∫ = 0.

d = 1½ A1

37(ii) dy B1
= x² − 5x + 6 = 0 → x = 2
dx

37(iii) d²y B1 www


x = 3, =1 and/or +ve Minimum.
dx ²
d²y
x = 2, = −1 and/or −ve Maximum
dx ²

dy B1 www
May use shape of ‘ + x 3 ’ curve or change in sign of
dx SC: x = 3 , minimum, x = 2 , maximum, B1

358
9709/11 Cambridge International AS/A Level – Mark Scheme May/June 2019

Question M/J 19/P11/Q11 Answer Marks Guidance

38(i) 3 B1
3 × −½ × (1 + 4x )

2

dy 3 B1 Must have ‘× 4’
= 3 × −½ × (1 + 4x ) 2 × 4

dx

2 9 M1 Use of m1.m₂ = − 1
If x = 2, m = − , Perpendicular gradient =
9 2

9 M1 Correct use of line eqn (could use y=0 here)


Equation of normal is y − 1 = ( x − 2)
2

16 A1 AG
Put y = 0 or on the line before →
9

38(ii) 2
3 3 1 + 4x B1 B1 Correct without ‘÷4’. For 2nd B1, ÷4’.
Area under the curve = ∫
0
1 + 4x
dx =
1
÷4

Use of limits 0 to 2 → 4½ − 1½ M1 Use of correct limits in an integral.

3 A1

2 1
2
9  M1 Any correct method.
Area of the triangle = ½ × 1 ×
9
=
9
or attempt to find ∫  2 x − 8  dx
16/9

1 8 A1
Shaded area = 3 − =2
9 9

359
9709/13 Cambridge International AS/A Level – Mark Scheme May/June 2019

Question Answer Marks Guidance

39(i) 1 − 
1 B1 oe
 2 ( 3x + 4 ) 2 
 

dy  1 − 
1 B1 Must have ‘ ×3 ’
=  ( 3x + 4 ) 2  × 3
dx  2 

dy 3 B1
At x = 4, = soi
dx 8

3 M1 If y ≠ 4 is used then clear evidence of substitution of x = 4 is needed


Line through (4, their4) with gradient their
8

3 3 5 A1 oe
Equation of tangent is y − 4 = ( x − 4 ) or y = x +
8 8 2 5

39(ii) 1 5 B1 4
3 5 3 5 
Area under line =  4 +  × 4 = 13 OR ∫ 8 x + 2 = 16 x
2
+ x = [3 + 10] = 13
2 2
0
2 

1  ( 3x + 4 )3/2  B1B1 Allow if seen as part of the difference of 2 integrals


Area under curve: ∫ ( 3 x + 4 ) 2 =  [ ÷3] First B1 for integral without [ ÷3]
 3 / 2 
Second B1 must have [ ÷3]

128 16 112 4 M1 Apply limits 0 → 4 to an integrated expression


− = = 12
9 9 9 9
4 5 A1
Area = 13 ‒ 12 = (or 0.556)
9 9
39(iii) dy 1 B1
=
dx 2
3 1
1 M1 3 1
( 3 x + 4 )− 2 = Allow M1 for ( 3 x + 4 )− 2 = 2 .
2 2 2

1
5 A1
( 3x + 4 ) 2 = 3 → 3x + 4 = 9 → x = oe
3
3

360
9709/12 Cambridge International AS/A Level – Mark Scheme March 2019

Question Answer Marks Guidance

40 y= 1
3 kx
3
− x 2 ( +c ) M1A1 Attempt integration for M mark

Sub (0, 2) DM1 Dep on c present. Expect c = 2

Sub (3, ‒1) → −1 = 9k − 9 + their c DM1

k = 2/3 A1

40(i) (
V = ( π ) ∫ x 3 + x 2 ( dx ) ) M1 Attempt ∫ y 2 dx

 x 4 x3 
3 A1
(π )  + 
4 3 0

DM1 May be implied by a correct answer


(π )  ( −0 )
81
+9
4 

117π A1 Accept 91.9


oe If additional areas rotated about x-axis, maximum of
4
M1A0DM1A0

40(ii) B2,1,0 Omission of 3 x 2 + 2 x is one error


dy 1 3
( ) ( )
−1/2
= x + x2 × 3x 2 + 2 x
dx 2

(At x = 3,) y = 6 B1

1 1 11 DB1ft Ft on their dy / dx providing differentiation attempted


At x = 3, m = × × 33 = soi
2 6 4

4 DM1 Equation through (3, their 6) and with gradient ‒1/their


Equation of normal is y − 6 = − ( x − 3) m
11

1 A1
When x = 0, y = 7 oe
11 6

361
9709/12 Cambridge International AS/A Level – Mark Scheme March 2019

Question Answer Marks Guidance

41(i) (
V = ( π ) ∫ x 3 + x 2 ( dx )) M1 Attempt ∫ y 2 dx

 x 4 x3 
3 A1
(π )  + 
4 3 0

DM1 May be implied by a correct answer


(π )  ( −0 )
81
+9
4 

117π A1 Accept 91.9


oe If additional areas rotated about x-axis, maximum of
4
M1A0DM1A0

41(ii) B2,1,0 Omission of 3 x 2 + 2 x is one error


dy 1 3
( ) ( )
−1/2
= x + x2 × 3x 2 + 2 x
dx 2

(At x = 3,) y = 6 B1

1 1 11 DB1ft Ft on their dy / dx providing differentiation attempted


At x = 3, m = × × 33 = soi
2 6 4

4 DM1 Equation through (3, their 6) and with gradient ‒1/their


Equation of normal is y − 6 = − ( x − 3) m
11

1 A1
When x = 0, y = 7 oe
11

362
9709/12 Cambridge International AS/A Level – Mark Scheme October/November 2018

Question Answer Marks Guidance

42 3 1 B1 B1 B1 for each term correct – allow unsimplified. C not


x2 x2 required.
Integrate → +2 (+C)
3 1
2 2

 3 1 
4 M1 Evidence of 4 and 1 used correctly in their integrand ie at
 x2 x2  40 14 least one power increased by 1.
 3 +2 1  → −
  3 3
 2 2 1

26 A1 Allow 8.67 awrt. No integrand implies use of integration


= oe function on calculator 0/4. Beware a correct answer from
3
4 wrong working.

9709/12 Cambridge International AS/A Level – Mark Scheme October/November 2018

Question Answer Marks Guidance

43(i) dy 3 1
  6  B2,1,0 Looking for 3 components
=  × ( 4 x + 1) 2  [×4] [− 2] 

− 2
dx 2   4x + 1 

 3
3 2 x² B1 B1 B1 33
2 x2
∫ ydx = 3 ( 4 x + 1) 2 ÷  [ ÷ 4 ] [ − ] (+ C) B1 for 3 ( 4 x + 1) 2 ÷
B1 for ‘÷4’. B1 for ‘− ’.
 2 2 2 2
 3
 Ignore omission of + C. If included isw any attempt at
 = ( 4 x + 1) 2 − x 2  evaluating.
 2 
 
 

43(ii) dy 6 M1 dy
At M, =0→ =2 Sets their 2 term to 0 and attempts to solve
dx 4x + 1 dx
(as far as x = k)

x = 2, y = 5 A1 A1

3
363
9709/12 Cambridge International AS/A Level – Mark Scheme October/November 2018

Question Answer Marks Guidance

43(iii) 1 3

2 M1 Uses their integral and their ‘2’ and 0 correctly
Area under the curve =  ( 4 x + 1) 2 − x ² 
2 0

(13.5 – 4) – 0.5 or 9.5 – 0.5 = 9 A1 No working implies use of integration function on calculator
M0A0.

Area under the chord = trapezium = ½ × 2 × (3 + 5) = 8 M1 Either using the area of a trapezium with their 2, 3 and 5 or
 x2 
2 ∫ ( their x + 3) dx using their ‘2’ and 0 correctly.
Or  + 3 x  = 8
2 0

(Shaded area = 9 – 8) = 1 A1 Dependent on both method marks,

OR Area between the chord and the curve is:


2 M1 Subtracts their line from given curve and uses their ‘2’ and 0
∫ 3 4 x + 1 − 2 x − ( x + 3) dx correctly.
0
2


= 3 4 x + 1 − 3 x − 3dx
0

1 
2 A1 All integration correct and limits 2 and 0.
3
x2
= 3  ( 4 x + 1) 2 − − x
6 2 0

 27   1  M1 Evidence of substituting their ‘2’ and 0 into their integral.


= 3  − 2 − 2  −  
 6   6 

1 1  1  A1 No working implies use of a calculator M0A0.


= 3 −  = 3  = 1
 2 6  3

[4]

364
9709/12 Cambridge International AS/A Level – Mark Scheme October/November 2018

Question O/N 18/P12/Q2 Answer Marks Guidance

43 3 1 B1 B1 B1 for each term correct – allow unsimplified. C not


x2 x2 required.
Integrate → +2 (+C)
3 1
2 2

 3 1 
4 M1 Evidence of 4 and 1 used correctly in their integrand ie at
 x2 x2  40 14 least one power increased by 1.
 3 +2 1  → −
  3 3
 2 2 1

26 A1 Allow 8.67 awrt. No integrand implies use of integration


= oe function on calculator 0/4. Beware a correct answer from
3 4
wrong working.
9709/12 O/N 18/P12/Q11 Cambridge International AS/A Level – Mark Scheme October/November 2018

Question Answer Marks Guidance

44(i) dy 3 1
  6  B2,1,0 Looking for 3 components
=  × ( 4 x + 1) 2  [×4] [− 2] 

− 2
dx 2   4x + 1 

 3
3 2 x² B1 B1 B1 33
2 x2
∫ ydx = 3 ( 4 x + 1) 2 ÷  [ ÷ 4 ] [ − ] (+ C) B1 for 3 ( 4 x + 1) 2 ÷
B1 for ‘÷4’. B1 for ‘− ’.
 2 2 2 2
 3
 Ignore omission of + C. If included isw any attempt at
= ( 4 x + 1) 2
−x 2 evaluating.
 2 
 
 

44(ii) dy 6 M1 dy
At M, =0→ =2 Sets their 2 term to 0 and attempts to solve
dx 4x + 1 dx
(as far as x = k)

x = 2, y = 5 A1 A1

365
9709/12 Cambridge International AS/A Level – Mark Scheme October/November 2018

Question Answer Marks Guidance

43(iii) 1 3

2 M1 Uses their integral and their ‘2’ and 0 correctly
Area under the curve =  ( 4 x + 1) 2 − x ² 
2 0

(13.5 – 4) – 0.5 or 9.5 – 0.5 = 9 A1 No working implies use of integration function on calculator
M0A0.

Area under the chord = trapezium = ½ × 2 × (3 + 5) = 8 M1 Either using the area of a trapezium with their 2, 3 and 5 or
 x2 
2 ∫ ( their x + 3) dx using their ‘2’ and 0 correctly.
Or  + 3 x  = 8
2 0

(Shaded area = 9 – 8) = 1 A1 Dependent on both method marks,

OR Area between the chord and the curve is:


2 M1 Subtracts their line from given curve and uses their ‘2’ and 0
∫ 3 4 x + 1 − 2 x − ( x + 3) dx correctly.
0
2


= 3 4 x + 1 − 3 x − 3dx
0

1 
2 A1 All integration correct and limits 2 and 0.
3
x2
= 3  ( 4 x + 1) 2 − − x
6 2 0

 27   1  M1 Evidence of substituting their ‘2’ and 0 into their integral.


= 3  − 2 − 2  −  
 6   6 

1 1  1  A1 No working implies use of a calculator M0A0.


= 3 −  = 3  = 1
 2 6  3

[4]

366
9709/11 Cambridge International AS/A Level – Mark Scheme October/November 2018

Question Answer Marks Guidance

44(i) 2 =k(8 ‒ 28 + 24) → k = 1/2 B1

44(ii) Whenx = 5, y = [½](125 ‒ 175 + 60) = 5 ( )


M1 Or solve [½ ] x 3 − 7 x 2 + 12 x = x ⇒ x = 5 [ x = 0, 2]

Which lies on y = x, oe A1

44(iii) 1 M1 1 7
∫[ 2 (x
3
− 7 x 2 + 12 x) − x]dx . Expect ∫ x3 − x 2 + 5 x
2 2

1 4 7 3 5 2 B2,1,0FT Ft on their k
x − x + x
8 6 2

2 ‒ 28/3 +10 DM1 Apply limits 0 → 2

8/3 A1

1 4 7 3 B2,1,0FT Integrate to find area under curve, Ft on their k


OR x − x + 3x 2
8 6

2 ‒ 28/3 +12 M1 Apply limits 0 → 2. Dep on integration attempted


2 M1

Area ∆ = ½ × 2 × 2 or xdx = ½ x 2  = 2
0

8/3 A1

367
9709/13 Cambridge International AS/A Level – Mark Scheme October/November 2018

Question O/N 18/P13/Q8 Answer Marks Guidance

45(i) y = ⅓ ax 3 + ½ bx 2 − 4 x ( +c ) B1

11 = 0 + 0 + 0 + c M1 Sub x = 0, y = 11 into an integrated expression. c must be present

y= 1
3 ax
3
+ 1 2 bx 2 − 4 x + 11 A1

45(ii) 4a + 2b − 4 = 0 M1 Sub x = 2, dy / dx = 0

1
3 (8a ) + 2b − 8 + 11 = 3 M1 Sub x = 2, y = 3 into an integrated expression. Allow if 11
missing

Solve simultaneous equations DM1 Dep. on both M marks

a = 3, b = −4 A1A1 Allow if no working seen for simultaneous equations

9709/13 Cambridge International AS/A Level – Mark Scheme October/November 2018

Question Answer Marks Guidance

46(i)  ( 3 x − 1)1/3  M1A1A1 Recognisable integration of y2 (M1) Independent A1, A1


V = 4 (π ) ∫ ( 3 x − 1) dx = 4 ( π )   [ ÷3]
−2/3
for [ ] [ ]
 1 / 3 

4 (π ) [ 2 − 1] DM1 Expect 4 (π )( 3x − 1) 13

4 π or 12.6 A1 Apply limits ⅔ → 3. Some working must be shown.

368
9709/13 Cambridge International AS/A Level – Mark Scheme October/November 2018

Question O/N 18/P13/Q10 Answer Marks Guidance

46(i)  ( 3 x − 1)1/3  M1A1A1 Recognisable integration of y2 (M1) Independent A1, A1


V = 4 (π ) ∫ ( 3 x − 1) dx = 4 ( π )   [ ÷3]
−2/3
for [ ] [ ]
 1 / 3 

4 (π ) [ 2 − 1] DM1 Expect 4 (π )( 3x − 1) 13

4 π or 12.6 A1 Apply limits ⅔ → 3. Some working must be shown.

5
46(ii) dy / dx = (−2 / 3) ( 3x − 1) B1 Expect −2 ( 3x − 1)−4/3
−4/3
×3

When x = 2 / 3, y = 2 soi dy / dx = −2 B1B1 2nd B1 dep. on correct expression for dy//dx

Equation of normal is y − 2 = ½ ( x − 2 3 ) M1 Line through (⅔, their 2) and with grad ‒1/m. Dep on m from
diffn

1 5 A1
y= x+
2 3

369
9709/12 Cambridge International AS/A Level – Mark Scheme May/June 2018
PUBLISHED
Question 47 M/J 18/P12/Q9 Answer Marks Guidance

47(i)  3
 B1 B1 B1 without ÷ 4. B1 for ÷ 4 oe. Unsimplified OK
2 3
 ( 4 x + 1) 2 
y= ( 4 x + 1) 2 ÷ 4 (+ C)  = 
3  6 
 

Uses x = 2, y = 5 M1 Uses (2, 5) in an integral (indicated by an increase in power by 1).

→ c = ½ oe isw A1 No isw if candidate now goes on to produce a straight line equation

47(ii) dy dy dx
= ÷
dx dt dt

dx M1 Ignore notation. Must be 0.06÷3 for M1.


= 0.06 ÷ 3
dt

= 0.02 oe A1 Correct answer with no working scores 2/2

47(iii) d2 y B1
= ½ ( 4 x + 1) × 4
−½
2
dx

d 2 y dy 2 B1FT Must either show the algebraic product and state that it results in a
× = × 4 x + 1 (= 2) constant or evaluate it as ‘= 2’. Must not evaluate at x =2.
dx 2 dx 4x + 1
d2 y
ft to apply only if 2 is of the form k ( 4 x + 1)
−½

dx

370
9709/12 Cambridge International AS/A Level – Mark Scheme May/June 2018

Question Answer Marks Guidance

48(i) x 6 B1 B1 Inspection or guesswork OK


y= + = 4 → x = 2 or 6
2 x
dy 1 6 B1 Unsimplified OK
= −
dx 2 x 2
*
When x = 2, m = ─1 → x + y = 6 M1 Correct method for either tangent
1 1
When x = 6, m = → y = x + 2
3 3
Attempt to solve simultaneous equations DM1 Could solve BOTH equations separately with y = x and get x = 3
both times.

(3,3) A1 6 Statement about y = x not required.


*
48(ii)  x² 36  M1 Integrate using π ∫ y ²dx (doesn’t need π or dx). Allow incorrect
V = (π) ∫  + 6 +  (dx)
 4 x²    x 6 
2

squaring. Not awarded for π ∫ 4 −  +   dx.


  2 x 
Integration indicated by increase in any power by 1.

x³ 36 A2,1 3 things wanted —1 each error, allow + C. (Doesn’t need π)


Integration → + 6 x─
12 x

1 160 DM1 Evidence of their values 6 and 2 from (i) substituted into their
Using limits ‘their 2’ to ‘their 6’ (53 π, π , 168 awrt)
3 3  16 
integrand and then subtracted. 48 ─  −  is enough.
 3

Vol for line: integration or cylinder (→ 64π) M1   x 6 


2
2
Use of πr²h or integration of 4 (could be from 4 −  +   )
  2 x 

2  32  A1
Subtracts → 10 π oe  e.g. π , 33.5 awrt 
3  3 

371
9709/12 Cambridge International AS/A Level – Mark Scheme May/June 2018

Question M/J 18/P12/Q11 Answer Marks Guidance

48(ii) OR

 x 6
2 M1 *M1 Integrate using π ∫ y ²dx (doesn’t need π or dx)
2
V = (π) ∫ 4 −  +  (dx)
2 x Integration indicated by increase in any power by 1.

 x² 36 
= (π) ∫ 16 −  + 6 +  (dx)
 4 x² 

  x3 36   A2,1  x3 36 
= (π) 16 x −  + 6 x −   (dx) Or 10 x − + 
  12 x    12 x 

= (π) ( 48 - 37⅓) DM1 Evidence of their values 6 and 2 from (i) substituted

2  32  A1
= 10 π oe  eg π , 33.5 awrt 
3  3 

9709/11 M/J 18/P11/Q3 Cambridge International AS/A Level – Mark Scheme May/June 2018
PUBLISHED
Question Answer Marks Guidance

49 dy 12 −12 B1 B1 Correct without “ ÷ 2”. For “ ÷ 2”. Ignore “c”.


= → y= ÷ 2 ( + c)
dx ( 2 x + 1) 2
2x + 1

−6 M1 A1 Finding “c” following integration. CAO


Uses (1, 1) → c = 3 ( → y = + 3)
2x + 1

1 1 DM1 A1 1
Sets y to 0 and attempts to solve for x → x = → (( , 0)) Sets y to 0. x = is sufficient for A1.
2 2 2

372
9709/11 Cambridge International AS/A Level – Mark Scheme May/June 2018

Question M/J 18/P11/Q10 Answer Marks Guidance

50 y = x³ − 2x² + 5x

50(i) dy B1 CAO
= 3x² − 4x + 5
dx

Using b ² − 4 ac → 16 – 60 → negative → some explanation M1 A1 Uses discriminant on equation (set to 0).


or completed square and explanation 3 CAO

50(ii) m = 3x² − 4x + 5 B1FT FT providing differentiation is equivalent


dm dm
= 6x – 4 (= 0) (must identify as )
dx dx

2 11 dy 11 M1 A1 Sets to 0 and solves. A1 for correct m.


→ x = , m= or =
3 3 dx 3
2
 2  11 11 Alt1: B1 for completing square, M1A1 for ans
Alt1: m = 3  x −  + , m =
 3 3 3
11
Alt2: 3 x 2 − 4 x + 5 − m = 0, b 2 − 4ac = 0, m = Alt2: B1 for coefficients, M1A1 for ans
3

d²m M1 A1 M1 correct method. A1 (no errors anywhere)


= 6 +ve → Minimum value or refer to sketch of curve or
dx ²
2 5
check values of m either side of x = ,
3

50(iii) x4 2 x³ 5x² B2,1 Loses a mark for each incorrect term


Integrate → − +
4 3 2

Uses limits 0 to 6 → 270 (may not see use of lower limit) M1 A1 Use of limits on an integral. CAO
Answer only 0/4

373
9709/13 Cambridge International AS/A Level – Mark Scheme May/June 2018

Question Answer Marks Guidance

51  2  B1B1
 ( 3 x − 1) 3 
f ( x) = 
2  [ ÷3] ( +c )
 
 3 

2 M1 Sub y = 1, x = 3 Dep. on attempt to integrate and c present


83
1= +c
2

1 2 A1
c = ‒1 → y = ( 3x − 1) 3 − 1 SOI
2

1 2
1 DM1A1 Dep. on previous M1
When x = 0, y = ( −1) 3 − 1 =−
2 2 6

9709/13 Cambridge International AS/A Level – Mark Scheme May/June 2018

Question Answer Marks Guidance

52(i) dy B1
= 2 ( x + 1) − ( x + 1)
−2

dx

Set = 0 and obtain 2 ( x + 1) = 1 convincingly www B1


3
AG

d2 y B1
= 2 + 2 ( x + 1) www
−3
2
dx

1 M1 Requires exact method – otherwise scores M0


 1 3
Sub, e.g., ( x + 1)
−3
= 2 OE or x =   − 1
2

d2 y A1 and exact answer – otherwise scores A0


=6 CAO www
dx 2

374
9709/13 Cambridge International AS/A Level – Mark Scheme May/June 2018

Question Answer Marks Guidance

52(ii) y 2 = ( x + 1) + ( x + 1) + 2 ( x + 1) SOI B1 OR y 2 = ( x 4 + 4 x3 + 6 x 2 + 4 x + 1) + ( 2 x + 2 ) + ( x + 1)−2


4 −2

 ( x + 1)5   ( x + 1)−1   2 ( x + 1)2  B1B1B1 Attempt to integrate y 2 . Last term might appear as ( x 2 + 2 x)
(π ) ∫ y 2 dx = (π )  + + 
 5   −1   2 
 x5   1 
OR (π )  + x 4 + 2 x 3 + 2 x 2 + x  +  x 2 + 2 x  +  − 
5   x + 1

 32 1 1  M1 Substitute limits 0 →1 into an attempted integration of y 2 .


(π )  − + 4 −  − 1 + 1 
5 2 5  Do not condone omission of value when x = 0

9.7π or 30.5 A1 Note: omission of 2 ( x + 1) in first line → 6.7π scores 3/6

Ignore initially an extra volume, e.g. (π) ∫ ( 4½ ) . Only take into


2

account for the final answer

375
9709/12 Cambridge International AS/A Level – Mark Scheme October/November 2017

Question Answer Marks Guidance

53(i) dy 1 1
5 B1 B1 B1 Without × 5 B1 × 5 of an attempt at differentiation
= × ( 5 x − 1) 2 × 5

(= )
dx 2 6

6 M1 Uses m1m2 = −1 with their numeric value from their dy/dx


m of normal = −
5
6 A1 Unsimplified. Can use y = mx + c to get c = 5.4 ISW
Equation of normal y − 3 = − ( x − 2 ) OE
5
−6 27
or 5y + 6x = 27 or y = x+
5 5

53(ii) EITHER: (B1 Correct expression without ÷5


3
( 5 x − 1)
()
2
For the curve ∫ 5 x − 1dx = ÷5 B1 For dividing an attempt at integration of y by 5
3
2

M1 A1
Limits from
1
5
to 2 used → 3.6 or
18
5
OE Using
1
5
(
and 2 to evaluate an integrand may be ∫ y 2 )
Normal crosses x-axis when y = 0, → x= (4½) M1 Uses their equation of normal, NOT tangent

15 A1 This can be obtained by integration


Area of triangle = 3.75 or OE
4

147 A1)
Total area=3.6 + 3.75 = 7.35, OE
20

376
9709/11 Cambridge International AS/A Level – Mark Scheme October/November 2017

Question Answer Marks Guidance

54(i)  x5  *B1
( )
Area = ∫ ½ x 4 − 1 dx = ½  − x 
5 

1  2 DM1A1 Apply limits 0→1


½  − 1 −0 = (−)
5  5 3
54(ii) Vol = π ∫ y 2 dx = (
¼ (π ) ∫ x8 − 2 x 4 + 1 dx ) M1 (If middle term missed out can only gain the M marks)

 x9 2 x5  *A1
¼ (π )  − + x
9 5 

1 2  DM1
¼ (π ) [ − + 1  − 0
9 5 

8π A1
or 0.559
45 4

54(iii) Vol = π ∫ x 2 dy = (π ) ∫ ( 2 y + 1) M1 Condone use of x if integral is correct


1/ 2
dy

 ( 2 y + 1)3/2  *A1A1  ( 2 y + 1)3/2 


(π )   [ ÷2] Expect (π )  
 3 / 2   3 

DM1
(π ) 
1 
− 0
3 

π A1 1
or 1.05 Apply − → 0
3 2

377
9709/13 Cambridge International AS/A Level – Mark Scheme October/November
PUBLISHED 2017
Question Answer Marks Guidance

55(i) EITHER: (M1 Form 3-term quad & attempt to solve for √x.
4 ‒ 3√x = 3 ‒ 2x → 2x ‒ 3√x + 1 (=0) or e.g. 2k2‒3k + 1 (=0)

x = ½, 1 A1 Or k = ½ or 1 (where k = √x).

x = ¼, 1 A1)

OR1: (M1
2
(3 x) = (1 + 2 x )
2

4 x2 − 5x + 1 ( =0) A1

x = ¼, 1 A1)

OR2: (M1 Eliminate x


2
3− y  4− y 
2
=
 3 
2
(
 → 2 y − 7 y + 5( = 0) )
5 A1
y= ,1
2

x = ¼, 1 A1)

378
9709/13 Cambridge International AS/A Level – Mark Scheme October/November
2017
Question Answer Marks Guidance

55(ii) EITHER: (B1


Area under line = ∫ ( 3 − 2 x ) dx = 3 x − x 2

  3 1  M1 Apply their limits (e.g. ¼ → 1) after integn.


= ( 3 − 1) −  −  
  4 16  

( )
Area under curve = ∫ 4 − 3 x1/ 2 dx = 4 x − 2 x3/ 2 B1

( 4 − 2 ) − (1 − ¼ )  M1 Apply their limits (e.g. ¼ → 1) after integration.

21 5 1 A1)
Required area = ‒ = (or 0.0625)
16 4 16

OR: (*M1 Subtract functions and then attempt integration


 1
 1
+/‒ ∫ ( 3 − 2 x ) 
− 4 − 3x 2  = + / − ∫ (−1 − 2 x + 3x 2 )

 

 3x 3/2  A2, 1, 0 FT FT on their subtraction. Deduct 1 mark for each term incorrect
+/‒  − x − x 2 + 
 3/ 2 

  1 1 1  1 DM1 A1) Apply their limits ¼ → 1


+/‒  −1 − 1 + 2 −  − + +   = (or 0.0625)
  4 16 8   16

379
9709/12 Cambridge International AS/A Level – Mark Scheme May/June 2017

Question O/N 17/P12/Q6 Answer Marks Guidance

56 16 M1* Use of volume formula at least once, condone omission of π and limits and
∫ ( 5 − x ) dx − π ∫
2
Vol = π dx dx .

DM1 Subtracting volumes somewhere must be after squaring.

(5 – x) ³ B1 B1 B1 Without ÷ (−1). B1 for ÷( −1)


∫ ( 5 − x ) ²dx = ÷ −1
3

(or 25x – 10x²/2 +⅓x³) (B2,1,0) −1 for each incorrect term

16 16 B1
∫ dx = −
x² x
Use of limits 1 and 4 in an integrated expression and subtracted. DM1 Must have used“y²” at least once. Need to see values substituted.

→ 9π or 28.3 Total: A1 7

9709/11 M/J 17/P11/Q7 Cambridge International AS/A Level – Mark Scheme May/June 2017

57 dy
= 7 − x² − 6 x
dx
57(i) x³ 6 x² B1 CAO
y = 7x − − (+ c)
3 2

Uses (3, − 10) → c = 5 M1 A1 Uses the given point to find c

Total: 3

57(ii) 7 − x ² − 6 x = 16 − ( x + 3) ² B1 B1 B1 a = 16, B1 b = 3.

Total: 2

57(iii) 16 − ( x + 3) ² > 0 → (x + 3)² < 16, and solve M1 or factors (x + 7)(x – 1)

End-points x = 1 or −7 A1

→ −7 < x < 1 A1 needs <, not ⩽. (SR x < 1 only, or x > −7 only B1 i.e. 1/3)

380
9709/11 Cambridge International AS/A Level – Mark Scheme May/June 2017

Question Answer Marks Guidance

58(i) dy −4 B1 B1 B1 without ×(−3) B1 For ×(−3)


= × (−3)
dx ( 5 − 3x ) ²

Gradient of tangent = 3, Gradient of normal – ⅓ *M1 Use of m1m2 = −1 after calculus

1 DM1 Correct form of equation, with (1, their y), not (1,0)
→ eqn: y − 2 = − ( x − 1)
3

1 7 A1 This mark needs to have come from y = 2, y must be subject


→ y=− x+
3 3

Total: 5

58(ii) 1
16 M1 Use of V = π ∫ y ²dx with an attempt at integration
Vol = π ∫0 ( 5 − 3x ) ² dx

 −16  A1 A1 A1 without( ÷ −3), A1 for (÷ −3)


π ÷ − 3
 ( 5 − 3x ) 

 16 16  8π M1 A1 Use of both correct limits M1


= ( π −  )= (if limits switched must show – to +)
 6 15  5

Total: 5

381
9709/13 Cambridge International AS/A Level – Mark Scheme May/June 2017
PUBLISHED
Question Answer Marks Guidance

59(a)(i) Attempt to integrate V = (π ) ∫ ( y + 1) dy M1 Use of h in integral e.g. ∫ ( h + 1) = ½ h 2 + h is M0. Use of ∫ y 2 dx is M0

 y2  A1
= (π )  + y 
2 

 h2  A1 AG. Must be from clear use of limits 0→ h somewhere.


= π  + h
2  Total: 3

59(ii) ∫ ( y + 1)
1/2
dy ALT ( )
6 ‒ ∫ x 2 − 1 dx M1 Correct variable and attempt to integrate

⅔ ( y + 1) *A1 Result of integration must be shown


3/2
oe ALT 6 ‒ ( ⅓ x3 − x ) CAO

8  1  DM1 Calculation seen with limits 0→3 for y. For ALT, limits are 1→2 and
⅔ [8 − 1] ALT 6 − [ − 1 −  − 1 ]
rectangle.
3  3 

14/3 ALT 6 ‒ 4/3 = 14/3 A1 16/3 from ⅔ × 8 gets DM1A0 provided work is correct up to applying
Total: 4 limits.
59(b) Clear attempt to differentiate wrt h M1 dV
Expect = π ( h + 1) . Allow h + 1. Allow h.
dh

Derivative = 4π SOI *A1

2 DM1
. Can be in terms of h
their derivative

2 1 A1
or or 0.159
4π 2π

Total: 4

382
9709/13 M/J 17/P13/Q11 Cambridge International AS/A Level – Mark Scheme May/June 2017

Question Answer Marks Guidance

60(i) f ' ( x ) = [( 4 x + 1) ÷ ½ ] [ ÷ 4] ( +c ) B1 B1 Expect ½ ( 4 x + 1)1/2 ( +c )


1/2

3 3 B1 FT 3
f '( 2) = 0 ⇒ Expect ½ ( 4 x + 1) . FT on their f ′ ( x ) = k ( 4 x + 1) + c . (i.e. c = −3k )
1/2 1/2
+ c = 0 ⇒ c = − (Sufficient) −
2 2 2

Total: 3

60(ii) f ″ ( 0 ) = 1 SOI B1

f ' ( 0 ) = 1 / 2 − 1½ = −1 SOI B1 FT Substitute x = 0 into their f ′(x) but must not involve c otherwise B0B0

f(0) = – 3 B1 FT FT for 3 terms in AP. FT for 3rd B1 dep on 1st B1. Award marks for the AP
method only.

Total: 3

60(iii) f ( x ) = ½ ( 4 x + 1)
3/ 2
÷ 3 / 2 ÷ 4 ]−[1½ x  ( + k ) B1 FT Expect (1/12) ( 4 x + 1)3/2 − 1½ x ( + k ) . FT from their f ′(x) but c numerical.
  B1 FT

−3 = 1 / 12 − 0 + k ⇒ k = −37 / 12 CAO M1A1 Sub x = 0, y = their f ( 0 ) into their f(x). Dep on cx & k present (c numerical)

27 37 23 A1
Minimum value = f(2) = −3− =− or − 3.83
12 12 6

Total: 5

383
Page 5 O/N 16/P11/Q7 Mark Scheme Syllabus Paper
Cambridge International AS/A Level – October/November 2016 9709 11

61 (i) A = (½, 0) B1 Accept x = 0 at y = 0


[1]

1  (1 − 2 x )3/2 
(ii) ∫ (1 − 2 x ) dx = 
2
 ( −2 ) 
 ÷ B1B1 May be seen in a single
 3 / 2  expression
 ( 2 x − 1)3  1
∫ ( 2 x − 1) dx =   [ ÷2 ]
2

 3 
B1B1 ∫
May use x dy , may expand
a

[0 − (−1 / 3)] − [0 − (−1 / 6)] M1 ( 2 x − 1)


2

1/6 A1 Correct use of their limits


[6]
O/N 16/P11/Q10
2
62 (i) 3z − = −1 ⇒ 3z 2 + z − 2 = 0 M1 Express as 3-term quad. Accept
z
x1/ 2 for z
x1/2 ( or z ) = 2 / 3 or − 1 A1 (OR
x = 4 / 9 only A1 3x − 1 = − x , 9 x 2 − 13x + 4 = 0
[3] M1, A1,A1 x = 4 / 9 )

3x3/2 2 x1/2
(ii) f ( x) = − ( +c ) B1B1
3 / 2 1/ 2
Sub x = 4, y = 10 10 = 16 − 8 + c ⇒ c=2 M1A1 c must be present
3/ 2 1/ 2
4 4 4
When x = , y = 2   − 4  +2 M1 Substituting x value from part
9 9 9 (i)
−2 / 27 A1
[6]

Page 6 63 O/N 16/P13/Q10 Mark Scheme Syllabus Paper


Cambridge International AS/A Level – October/November 2016 9709 13

dy 2 1  3  2 1
63 (i) at x = a 2 , = + or 2a −2 + a −2  = 2 or 3a −2  B1 2
+ 2 or 2a −2 + a −2 seen
dx a 2 a 2  a  a a
anywhere in (i)
y −3=
3
a 2 ( ) a
3
a
3
x − a 2 or y = 2 x + c → 3 = 2 a 2 + c M1 Through ( a 2 ,3) & with their
grad as f(a)
3
y= x or 3a −2 x cao A1
a2 [3]

2 x½ ax −½
(ii) ( y) = + (+ c) B1B1
a ½ −½
sub x = a 2 , y = 3 into ∫ dy / dx M1 c must be present. Expect
½ 3=4‒2+c
4x
c=1 (y= − 2ax −½ + 1 ) A1
a [4]

4 1
(iii) sub x = 16, y = 8 → 8 = × 4 − 2a × + 1 *M1 Sub into their y
a 4
a 2 + 14a − 32 ( = 0 ) A1
a=2 A1 Allow ‒16 in addition
A = (4, 3), B = (16, 8) AB 2 = 122 + 52 → AB = 13 DM1A1
[5]
384
Page O/N 16/P13/Q10 Mark Scheme Syllabus Paper
Cambridge International AS/A Level – October/November 2016 9709 13

dy
= −k ( kx − 3) + k = 0 Must contain ( kx − 3) + other
−2 −2
64 (i) Attempt diffn. and equate to 0 *M1
dx
term(s)
( kx − 3) = 1 or k 3 x 2 − 6k 2 x + 8k ( = 0 )
2
DM1 Simplify to a quadratic
2 4
x= or *A1*A1 Legitimately obtained
k k
d2 y
= 2k 2 ( kx − 3)
−3
B1 Ft must contain Ak 2 ( kx − 3)
−3
dx 2
2 d2 y where A>0
When x = ,
k dx 2
(
= −2 k 2 < 0 ) MAX All previous DB1 Convincing alt. methods (values
either side) must show which
4 d2 y
When x = ,
k dx 2
= 2k 2 > 0( ) MIN working correct
DB1
values used & cannot use
x =3/ k

[7]

2
V = (π ) ∫ ( x − 3) + ( x − 3)  dx
−1
(ii) *M1 Attempt to expand y² and then
 
integrate
= (π ) ∫[( x − 3) + ( x − 3) + 2 ]dx
−2 2
A1
 ( x − 3 )3 
= (π ) − ( x − 3) + ( +2 x ) Condone missing 2x
−1
A1 Or
 3   x3 
 − ( x − 3 ) −1
+ − 3x 2 + 9 x + 2 x 
 3 
 1 1 
= (π ) 1 − + 4 −  − 9 + 0   DM1 Apply limits 0→2
 3 3 
= 40π / 3 oe or 41.9 A1 2 missing → 28π / 3 scores
[5] M1A0A1M1A0

Page Mark Scheme Syllabus Paper


Cambridge International AS/A Level – May/June 2016 9709 12

8
65 f ′( x) =
( 5 − 2 x )2
−1
8(5 − 2x )
f(x) = ÷ −2 (+c) B1 Correct without (÷ by −2)
−1 B1 An attempt at integration (÷ by−2)

Uses x = 2, y = 7, M1 Substitution of correct values into


an integral to find c
c=3 A1
[4]

385
Page 7 M/J 16/P12/Q10 Mark Scheme Syllabus Paper
Cambridge International AS/A Level – May/June 2016 9709 12

8
66 y= + 2 x.
x
dy
(i) = −8x−2 + 2 B1 unsimplified ok
dx
d²y
= 16x−3 B1 unsimplified ok
dx ²
4 x³
∫y²dx = –64x −1 oe+ 32x oe + oe (+c) 3 × B1 B1 for each term – unsimplified ok
3 [5]

dy
(ii) sets to 0 → x = ±2 M1 Sets to 0 and attempts to solve
dx
→M(2, 8) A1 Any pair of correct values A1
Other turning point is (−2, −8) A1 Second pair of values A1

d²y d²y
If x = −2, <0 M1 Using their if kx−3 and x< 0
dx ² dx ²
∴Maximum A1
[5]
(iii) Vol = π × [ part (i) ] from 1 to 2 M1 Evidence of using limits 1&2 in
their integral of y² (ignore π)
220π
,73.3π, 230 A1
3 [2]

Page 4 M/J 16/P11/Q3 Mark Scheme Syllabus Paper


Cambridge International AS/A Level – May/June 2016 9709 11

12
67 x= − 2.

Vol = (π) × ∫ x² dy
 −144 48  M1 Ignore omission of π at this stage
→ + 4y + 
 3y³ y 3 × A1 Attempt at integration
Un-simplified

Limits 1 to 2 used A1 only from correct integration


→ 22π
M/J 16/P11/Q4 [5]

dy −½
68 (i) = 2 − 8 ( 3x + 4 )
dx
dy
(x = 0, → = −2 )
dx
dy dy dx dy
= × → −0.6 M1A1 Ignore notation. Must be × 0.3
dt dx dt [2] dx

 
 8 3x + 4 
(ii) y = {2 x} − ÷ 3 ( +c ) B1 B1 No need for +c.
 1 
 2 
4
x = 0, y = → c = 12. M1 A1 Uses x, y values after ∫ with c
3 [4]

386
Page 4 Mark Scheme Syllabus Paper
Cambridge International AS/A Level – May/June 2016 9709 13
M/J 16/P13/Q2

69 (π ) ∫ ( x3 + 1) dx M1 Attempt to resolve y 2 and attempt


to integrate
 x4 
(π )  + x A1
4 
6π or 18.8 DM1A1 Applying limits 0 and 2.
[4] (Limits reversed: Allow M mark
and allow A mark if final answer is
M/J 16/P13/Q3 6π)

70 (i) 6 + k = 2 → k = −4 B1
[1]

6 x3 4 −2 k −2
(ii) ( y) = − x (+c) B1B1 x
ft on their k. Accept +
3 −2 −2
9=2+2+c c must be present M1 Sub (1,9) with numerical k. Dep on
attempt ∫
( y ) = 2 x3 + 2 x −2 + 5 A1 Equation needs to be seen
[4] Sub (2, 3) →c = –13½ scores M1A0

Page O/N 15/P12/Q10 Mark Scheme Syllabus Paper


Cambridge International AS/A Level – October/November 2015 9709 12

71 y = (9 − 2 x 2 ) P (2, 1)

dy 1 B1 Without “ × −4x”
(i) = × −4 x B1 Allow even if B0 above.
dx 2 (9 − 2 x 2 )
At P , x = 2, m = −4 Normal grad = ¼ M1 For m1m2 = −1 calculus needed
Eqn AP y − 1 = 1 4 ( x − 2) M1 Normal, not tangent
→ A (−2, 0) or B (0, ½) A1
Midpoint AP also (0, ½) A1 Full justification.
[6]

 9 y2 
(ii) ∫ ∫
x 2 dy =  − dy
2 2 
M1 Attempt to integrate x²

9 y y3
= − A1 Correct integration
2 6
Upper limit = 3 B1 Evaluates upper limit
Uses limits 1 to 3 DM1 Uses both limits correctly
→ volume = 4⅔ π A1
[5]

387
Page O/N 15/P11/Q11 Mark Scheme Syllabus Paper
Cambridge International AS/A Level – October/November 2015 9709 11

dy  1 
7 (i)
72 =  (1 + 4 x) −1 2  × [4] B1B1
dx  2 
dy 2
At x = 6, = B1
dx 5
1
Gradient of normal at P = − B1 OR eqn of norm
2
5
y − 5 = their − ( x − 6)
5 2
Gradient of PQ = − hence PQ is a normal, When y = 0, x = 8 hence result
2
or m1 m 2 = −1 B1
[5]
(ii) ∫
Vol for curve = (π ) (1 + 4 x) and attempt to M1
integrate ‫ ݕ‬ଶ
[ ]
= (π ) x + 2 x 2 ignore ‘+ c’ A1
= (π )[6 + 72 − 0] DM1 Apply limits 0 → 6 (allow reversed if
= 78(π ) A1 corrected later)
8
 5  
3
1 M1   − x + 20  
Vol for line = × (π ) × 5 2 × 2 2
3 OR (π )    
 5 
50 A1  3× − 
= (π ) 2
3   6
2 A1
Total Vol = 78π + 50π 3 = 94 π (or 284π 3 )
3 [7]

Page 7 Mark Scheme Syllabus Paper


Cambridge International AS/A Level – October/November 2015 9709 13

73
73 (i) ( y ) = x + 4 x 2 (+c) B1
1
Sub x = 4, y = 6 → 6 = 4 + (4 × 42 ) +c M1 Must include c
1
c = −6 → ( y = x + 4 x 2 − 6 A1
[3]

388
Page Mark Scheme Syllabus Paper
Cambridge International AS/A Level – October/November 2015 9709 13

74 (i) f ' ( x) = 2 − 2( x + 1) −3 B1
f" ( x) = 6( x + 1) −4 B1
f'0 = 0 hence stationary at x = 0 B1 AG
f" 0 = 6 > 0 hence minimum B1 www. Dependent on correct f ″(x)
−4
[4] except − 6( x + 1) → < 0 MAX
(ii) AB 2 = (3 / 2) 2 + (3 / 4) 2 M1 scores SC1
AB = 1.68 or 45 / 4 oe A1
[2]
Area under curve = ∫ f ( x ) = x 2 − ( x + 1) −1 B1 Ignore +c even if evaluated
(iii)
 1 1  Do not penalise reversed limits
= 1 −  −  − 2  = 9 / 4
 2 4 
(Apply limits − 1 2 → 1) M1A1 Allow reversed subtn if final ans
1 9 3 positive
Area trap. = (3 + ) ×
2 4 2 M1
= 63 / 16 or 3.94 A1
Shaded area 63 / 16 − 9 / 4 + 27 / 16 or 1.69 A1
[6]
ALT eqn AB is y = − 1 2 x + 11/ 4 B1
Area = ∫ − 1 2 x + 11 / 4 − ∫ 2 x + ( x + 1) −2 M1 Attempt integration of at least one
 1 11 
[
=  − x 2 + x  − x 2 − ( x + 1) −1 ] A1A1 Ignore +c even if evaluated
 4 4 
Dep. on integration having taken
place
Apply limits − 1 2 → 1 to both integrals M1 Allow reversed subtn if final ans
27/16 or 1.69
A1 positive

Page Mark Scheme Syllabus Paper


Cambridge International AS/A Level – May/June 2015 9709 12

75 f ′( x) = 5 − 2 x 2 and (3, 5)
3
f(x) = 5x − 23x (+c) B1 For integral
Uses (3, 5) M1 Uses the point in an integral
→c=8 A1 co
[3]

76 y= 4 . B1 Correct without the ÷2


2 x−1
(i) 16 −16
∫ (2 x−1)2 dx = 2 x − 1 ÷2 B1 For the ÷2 even if first B1 is lost
M1 Use of limits in a changed
−8 
Vol = π  with limits 1 and 2 expression.
 x −1 
2
A1 co
16π [4]

3
(ii) m= 1
2 m of tangent = −2
M1 Use of m1m2 = −1
dy −4
= ×2
dx (2 x −1)2 B1 Correct without the ×2
B1 For the ×2 even if first B1 is lost
dy
Equating their dx to −2
DM1
→ x = 32 or − 12
A1 co
(y = 2 or – 2)
→ c = 52 or − 72
A1 co
[6]
389
Page 9 M/J 15/P11/Q10 Mark Scheme Syllabus Paper
Cambridge International AS/A Level – May/June 2015 9709 11

8
77 y=
3x + 4

dy −4
(i) = 3
× 3 aef B1 Without the “×3”
dx (3 x + 4) 2 B1 For “×3” even if 1st B mark lost.

3 2
→ m(x=0) = − Perpendicular m(x=0) = M1 Use of m1m2 = −1 after attempting
2 3
dy
to find (x=0)
dx
2
Eqn of normal y − 4 = ( x − 0) M1 Unsimplified line equation
3
 20  cao
Meets x = 4 at B  4,  A1
 3  [5]

8 8 (3 x + 4))
(ii) ∫ (3 x + 4)
dx =
1
÷3 B1 B1 Without “÷3”. For “÷3”

2
32
Limits from 0 to 4 → Area P = M1 A1 Correct use of correct limits. cao
3
Area Q = Trapezium – P
Area of Trapezium =
M1 Correct method for area of
1 20  64
4 + × 4 = trapezium
2 3  3

32 A1 All correct.
→ Areas of P and Q are both [6]
3

Page Mark Scheme Syllabus Paper


Cambridge International AS/A Level – May/June 2015 9709 13

 3

 (2 x + 1) 2 
78   [÷2] (+c) B1B1
3
 2 
7=9+c M1 Attempt subst x = 4, y = 7. c must be there.
3 Dep. on attempt at integration.
(2 x + 1) 2
y= −2 or unsimplified A1 c = –2 sufficient
3 [4]
79 (i) dy
= 6 − 6x B1
dx
At x = 2 , gradient = −6 soi B1
y − 9 = −6( x − 2) oe Expect y = −6 x + 21 M1 Line through (2, 9) and with gradient their
−6
When y = 0, x = 3 12 cao A1
[4]

(ii) Area under curve: ∫ 9 + 6 x − 3x 2 dx = 9 x + 3x 2 − x 3 B2,1,0 Allow unsimplified terms


(27 + 27 − 27) − (18 + 12 − 8) M1 Apply limits 2,3. Expect 5
Area under tangent: 1 × 3 × 9 ( = 27 ) OR ∫ 2 2 (−6 x + 21) dx (→ 27 ). Ft on their
7

2 2 4 B1
4
− 6 x + 21 and/or their 7/2.
Area required 27 − 5 = 7 A1
4 4
[5]
390
Page Mark Scheme Syllabus Paper
Cambridge International AS/A Level – October/November 2014 9709 12

80 Vol = ( π) ∫x²dy = (π) ∫ (y − 1) dy M1 Use of ∫x ² – not ∫y² – ignore π


A1 co
1 ( y − 1) 2
Integral is y 2 − y or B1 Sight of an integral sign with 1 and 5
2 2
Limits for y are 1 to 5

→ 8π or 25.1(AWRT) A1 co
[4] (no π max 3/4)

d 2 y 24
81 = −4
dx 2 x 3
(i) (If x = 2) it’s negative → Max B1 www
[1]

 dy 
(ii)  =  −12x−2 – 4x + (A) B2,1,0 oe one per term
 dx 
= 0 when x = 2 M1 Attempt at the constant A after ∫n
→ A = 11 A1 co
[4]
(iii) (y =) 12x−1 − 2x² + Ax + (c) B2,1,0 oe Doesn’t need +c, but does need a term A
to give “Ax”.
y = 13 when x = 1 → c = −8 M1 Attempt at c after ∫n
(If x = 2) y = 12 A1 co
[4]

Page Mark Scheme Syllabus Paper


Cambridge International AS/A Level – October/November 2014 9709 11

82 (i) f ( x ) = x 2 + 2 ( + c ) B1B1
x
6 = 4 +1+ c ⇒ c = 1 M1A1 Sub (2, 6) – dependent on c being present
[4]

83 (i) f ′(2) = 4 − 12 = 72 → gradient of normal = − 72 B1M1


y − 6 = − 72 ( x − 2) AEF A1 Ft from their f ′(2 )
[3]

(ii) f ( x ) = x 2 + 2x ( + c ) B1B1
6 = 4 +1+ c ⇒ c = 1 M1A1 Sub (2, 6) – dependent on c being present
[4]

(iii) 2 x − 22 = 0 ⇒ 2 x 3 − 2 = 0 M1 Put f ′(x ) = 0 and attempt to solve


x
x =1 A1 Not necessary for last A mark as
x > 0 given
f ′′ ( x ) = 2 + 43 or any valid method M1
x
f ′′(1) = 6 OR > 0 hence minimum A1 Dependent on everything correct
[4]

391
84 O/N 14 /P13 /Q9
Page Mark Scheme Syllabus Paper
Cambridge International AS/A Level – October/November 2014 9709 13

84 (i) x − 3 x + 2 or k 2 − 3k + 2 or 3 x ( ) 2
= (x + 2 )
2
M1 OR attempt to eliminate x eg sub
y2
x=
9
x = 1 or 2 or k = 1 or 2 or x 2 − 5 x + 4 (= 0 )
2
A1 y − 9 y + 18 = 0
x = 1 or 4 A1 y = 3 or 6
A1 x = 1 or 4
y = 3 or 6
[4]

1
 ∫ ( x + 2 ) dx or attempt at trapezium 

(ii) 3 x dx –   M1DM1 Attempt to integrate. Subtract at
2 some stage
3  1 2  1 
2x −  x + 2 x  or ( y 2 + y1 )(x 2 − x1 ) A1A1 Where (x1 , y1 ), ( x 2 , y 2 ) is their
2  2  2  (1, 3), (4, 6)

 1  1 
(16 − 2) –  (8 + 8) −  + 2  or their × 9 × 3 DM1 Apply their 1→4 limits correctly
 2  2  to curve
1 A1 For A mark allow reverse subtn→
2 1 1
− → but not reversed limits
[6] 2 2
OR
2
 ( y − 2 ) dy or attempt at trap  − y dy
∫  ∫ 9 M1DM1
3
1 2 1  y
2 y − 2 y or ( x1 + x 2 )( y 2 − y )
1  − A1A1
 2  27
  1  1 
(18 − 12) −  4 2 − 6  or 2 × 5 × 3 − [8 − 1] DM1 Apply their 3→6 limits correctly
    to curve
1 A1
2

Page Mark Scheme Syllabus Paper


GCE AS/A LEVEL – May/June 2014 9709 12

85 y =8− 4− x M/J/14 /P12 /Q9


dy 1 1
= − (4 − x ) 2 × −1

(i) B1 B1 Without (−1). For (×−1).
dx 2
3

∫y dx = 8x −
(4 − x )2 ÷ −1
3
3 × B1 B1 for "8x" and +c". B1 for all except
2 ÷(−1). B1 for ÷(−1).
[5] (n.b. these 5 marks can be gained in(ii) or
(ii) Eqn y − 7 = 1 2 (x − 3) (iii))
→ y = ½x + 5½ M1A1 M1 unsimplified. A1 as y=mx+c
[2]
(iii) Area under curve = ∫ from 0 to 3 (58/3) M1 Use of limits – needs use of “0”
Area under line = ½(5½ + 7)×3 M1 Correct method
 11x 
Or  1 4 x 2 + from 0 to 3 M1 A1 M1 Subtraction. A1 co
 2  [4]
58 75 7
→ − =
3 4 12

392
M/J/14 /P13 /Q10
Page Mark Scheme Syllabus Paper
GCE AS/A LEVEL – May/June 2014 9709 13

86 pts of intersection 2 x + 1= −x² + 12x − 20 M1A1 Attempt at soln of sim eqns. co


→ x = 3, 7
1
Area of trapezium = (4)(7 + 15) = 44 M1A1 Either method ok. co
2
(or ∫ (2x+1) dx from 3 to 7 = 44)

Area under curve = − 1 x³ + 6x² − 20x B2,1 −1 each term incorrect


3
Uses 3 to 7 → (54 2 ) DM1 Correct use of limits (Dep 1st M1)
3

Shaded area = 10 2 A1 co
3
[8]
OR

+ 10x − 21) = − x + 5x 2 − 21x 


7 3 
Functions subtracted before integration
∫  − x2

3  3 

M1 subtraction, A1A1A1 for integrated terms, Subtraction reversed allow A3A0.


DM1 correct use of limits, A1 Limits reversed allow DM1A0

Page Mark Scheme Syllabus Paper


GCE AS/A LEVEL – October/November 2013 9709 12

2
87 y=
5x − 6
2 5x − 6
(i) integral = 1
÷5 B1 B1 B1 without ‘÷5’. B1 for ‘÷ 5’
2
Uses 2 to 3 → 2.4 − 1.6 = 0.8 M1 A1 Use of limits in an integral.
[4]
8
88 y= + 2x
x
dy −8
(i) = 2 +2 M1 Attempt at differentiation.
dx x A1 algebraic – unsimplified.
(− 6 at A)
dy dy dy
= × M1 Ignore notation – needs product of 0.04
dt dx dt
A1 dy
→ − 0.24 and ‘his’ .
[4] dx

64
∫y ∫x
2
(ii) = 2
+ 4 x 2 + 32 M1 Use of integral of y² (ignore π)

− 64 4 x 3
=( + + 32 x ) A3,2,1 3 terms → −1 each error.
x 3
Limits 2 to 5 used correctly DM1 Uses correct limits correctly.
→ 271.2π or 852 A1
(allow 271π or 851 to 852) [6] (omission of π loses last mark )

393
89 O/N/13 /P11 /Q2
Page Mark Scheme Syllabus Paper
GCE AS/A LEVEL – October/November 2013 9709 11

89 Attempt integration M1
1 6
f (x ) = 2(x + 6) 2 − (+ c ) A1A1 Accept unsimplified terms
x
6
2(3) − + c = 1 M1 Sub.x = 3, y = 1. c must be present
3
c = –3 A1
[5]
90 O/N/13 /P11 /Q2

90 (i)
dy
dx
[ ]
= 3(3 − 2 x ) × [− 2]
2
B1B1 OR − 54 + 72 x − 24 x 2 B2,1,0

1 dy
At x = , = −24 M1
2 dx
 1
y − 8 = −24 x −  DM1
 2
y = −24 x + 20 A1
[5]
 (3 − 2 x )4   1 
(ii) Area under curve =   × −  B1B1 OR 27 x − 27 x 2 + 12 x 3 − 2 x 4 B2,1,0
 4   2 
 81  M1
− 2 − −  Limits 0→ ½ applied to integral with
 8 intention of subtraction shown
Area under tangent = ∫ (− 24x + 20) M1 or area trap =½(20 + 8) × ½

= − 12 x 2 + 20 x or 7 (from trap) A1 Could be implied


9 A1 Dep on both M marks
or 1.125
8 [6]

Page \ Mark Scheme Syllabus Paper


GCE A LEVEL – October/November 2013 9709 13

1 1
− −
91 f (x) = 2 x 2 + x (+c) M1A1 Attempt integ x 2 or + x needed for M
1
5 = –2 × + 4 + c M1 Sub (4, 5). c must be present
2
c=2 A1
[4]

dy  1 − 
[ ]
1
29 92 (i) =  (x 4 + 4 x + 4) 2  × 4 x 3 + 4 B1B1
dx  2 
dy 1 1
At x = 0, = × × 4 = (1) M1 Sub x = 0 and attempt eqn of line following
dx 2 2 differentiation.
Equation is y – 2 = x A1
[4]

(ii) x + 2 = x 4 + 4 x + 4 ⇒ (x + 2)2 B1 AG www


= x4 + 4x + 4
x2 – x4 = 0 oe B1
x = 0, ± 1 B2,1,0
[4]

394
Page \ Mark Scheme Syllabus Paper
GCE A LEVEL – October/November 2013 9709 13

92  x5 
(iii) (π )  + 2 x 2 + 4 x  M1A1 Attempt to integrate y2
5 

(π ) 0 −  − 1 + 2 − 4  DM1


  5 
11π
(6.91) oe A1 Apply limits − 1 → 0
5
[4]

Page Mark Scheme Syllabus Paper


GCE AS/A LEVEL – May/June 2013 9709 12

dy 6 M/J/13 /P12 /Q1,8,11


93 = 2
dx x
B1 Integration only – unsimplified
y = −6 x −1 + c M1 Uses (2, 9) in an integral
Uses (2, 9) → c = 12 A1
y = −6 x −1 + 12 [3]

250
94 (i) πr 2 h = 250π → h =
r2
→ S = 2πrh + 2πr 2
M1 Makes h the subject. πr 2 h must be right
500π
→ S = 2πr 2 + M1 Ans given – check all formulae..
r [2]
dS 500π
(ii) = 4πr − 2 B1 B1
dr r B1 for each term
= 0 when r³ = 125 → r=5
M1 Sets differential to 0 + attempt at soln
→ S= 150π
A1
[4]
d2S 1000π
(iii) = 4π +
dr 2
r3 M1 Any valid method.
This is positive → Minimum A1 2nd differential must be correct – no need for
[2] numerical answer or correct r

95 y = 1 + 4x
dy 1 −1
(i) = 2 (1 + 4 x) 2 × 4 B1 B1 B1 Without “×4”. B1 for “×4” even if first B
dx mark lost.
= 2 at B (0, 1)
Gradient of normal = −½ M1 Use of m1m2=−1
Equation y − 1 = −½ x M1 A1 Correct method for eqn.
[5]
(ii) At A x = −¼ B1
3
(1 + 4 x) 2 B1 B1 B1 Without the “÷4”. For “÷4” even if first B
∫ 1 + 4 x dx = 3
2
÷4
mark lost.
B1
1
Limits −¼ to 0 → 6
Area BOC = ½ × 2 × 1 = 1 B1 For 1 + his “1/6”.
7 [5]
→ Shaded area =
6

395
Page 6 Mark Scheme Syllabus Paper
GCE AS/A LEVEL – May/June 2013 9709 11

3 Or 3x − 10 x + 3 = 0
96 (i) 3u + − 10 = 0 B1
u Or (3 x − 1)( x − 3) or apply formula
etc.
3u 2 − 10u + 3 = 0 ⇒ (3u − 1)(u − 3) = 0 M1
1
x = or 3 A1
3
1
x = or 9 A1
9 [4]
1 3
3 − 3 −
(ii) f ′′(x) = x 2 − x 2 B1 Allow anywhere
2 2
1
At x =
9
3 3
f ′′(x) = (3) − (27) (= –36) < 0 → Max M1 Valid method. Allow innac subs, even
2 2 1
At x = 9 3,
3
3 1 3 1 4
f ′′(x) = × − × (= ) > 0 → Min A1 Fully correct. No working, no marks.
2 3 2 27 9 [3]
3 1
(iii) f(x) = 2 x 2 + 6 x 2 − 10 x (+ c) B2 B1 for 2/3 terms correct. Allow in (i)
–7 = 16 + 12 – 40 + c M1 Sub (4, –7). c must be present.
c=5 A1
[4]

dy
97 (i) = 4( x − 2) 3 B1 Or 4 x 3 − 24 x 2 + 48x − 32
dx
Grad of tangent = –4 M1 Sub x = 1 into their derivative
Eq. of tangent is y – 1 = –4(x – 1) M1 Line thru (1, 1) and with m from deriv
5
→ B ( , 0) A1
4
1
Grad of normal = M1 Use of m1m2 = –1
4
1 3
Eq. of normal is y – 1 = ( x − 1) → C(0, ) A1
4 4 [6]
2
2 1 2
(ii) AC = 1 +   M1
4
17
17 A1 Allow
[2] 16
4

(iii) ∫ (x − 2) dx =
4 ( x − 2)
5
B1 Or
x5
− 2 x 4 + 8 x 3 − 16 x 2 + 16 x
5 5
 1  1
M1 Apply limits 1 → 2 for curve
0 − ( − 5 )  = 5
  5
1 5 1 4 1
∆ = × 1 × (their − 1) = M1 Or ∫ (− 4x + 5) dx =
2 4 8 1 8
1 1 3 A1
− = or 0.075
5 8 40 [4]

396
98 M/J/13 /P13 /Q1
Page Mark Scheme Syllabus Paper
GCE AS/A LEVEL – May/June 2013 9709 13

98 dy
= 2x + 5
dx
3
B1 B1 Everything without “÷2”.
(2 x + 5) 2
3
÷ 2 ( +c) B1 B1 “÷2”
2

Uses (2, 5) → c = −4 M1 A1 [4] Uses point in an integral.


99 8 M/J/13 /P13 /Q11
y= −x
x

(i) dy −
3 B1 needs both
= −4 x 2 − 1
dx M1 Subs x = 4 into dy/dx
3
= − 2
when x = 4. M1 Must be using differential +
Eqn of BC y − 0 = − 32 ( x − 4) A1 correct form of line at B(4,0).
→ C (1, 4½) [4]

(ii) 8
area under curve = ∫( x
− x)
1
8x 2
= 1
− ½x² B1 B1 (both unsimplified)
2
Limits 1 to 4 → 8½
M1 Using correct limits.
Area under tangent = ½ × 4½ × 3 = 6¾
M1 Or could use calculus)
Shaded area = 1¾
A1 [5]

Page Mark Scheme Syllabus Paper


GCE AS/A LEVEL – October/November 2012 9709 12

100 9
y= A (3, 1) B (0, 3)
2x + 3
dy −9
(i) = × 2 B1 B1 Correct without the ×2. For × 2,
dx (2 x + 3) 2 independent of first part.
→ m = − 92 M1 Correct form of tan - numerical dy/dx
→ y − 1 = − ( x − 3) 2 A1 For his m following use of dy/dx.
9
[4] (normal →max 2/4, no calculus 0/4)

(ii) Meets the y-axis when x = 0, y = 1⅔ B1 Sets x to 0 in his tangent.


This is nearer to B than to O. [1] The 1⅔ and part (i) must be correct.

81 − 81
(iii) Integral of 2
= ÷2 B1 B1 Correct without the ÷ 2. For ÷ 2,
(2 x + 3) 2x + 3
− 9 − 81 M1 Use of limits with integral of y² only
Uses limits 0 to 3 → − = 9π
2 6 A1 [4] no π – max ¾. Use of area - 0/4,

397
Page Mark Scheme Syllabus Paper
GCE AS/A LEVEL – October/November 2012 9709 11


101     M1A1 Attempt integration. cao
మ
Sub (2, 4) ⟶  5 DM1A1 Dependent on c present
[4]

102 (i)   3 ⇒   3 0⇒ 3 or 0 M1 OR form equation in x and attempt


solution

 or 5 ⇒ a 5 AG A1 OR sub x=5 each eq (M1) → y = 3(twice)

[2] (A1)
(5,3) subst only once scores 0/2

 మ  మ 
(ii) 0 య 1 2 2 , 4   5 B1B1B1 Or ∆ = ½(5 – ½)3

  
    
4  05, 4  6  75 M1 Apply limits ½ and 5 for, at least, curve
   
Subtract areas at some stage M1 Dependent on some integration

oe A1 cao 9/4 with no working scores 0/6, but

[6] 9 − 27/4 = 9/4 scores 1/6 (M1 subtraction)

Page Mark Scheme Syllabus Paper


GCE AS/A LEVEL – October/November 2012 9709 13

1
103 (i)f ″x=9 × 3x + 42  – 6 B2,1,0 1 each error
[2]
(ii) f′(–1) = 2 + 6 – 8 = 0 hence stat value at B1 AG
x = –1
f″(–1) = 9 – 6 = 3 > 0 hence minimum B1
[2]
4 5 4
(iii) y =  oe × (3x + 4)2  –3x2 + 8x + (c) B1 B1 B1 allow unsimplified
15 15
4 4
Sub (–1, 5) → –3 + 8 + c = 5 → c = – M1 A1 Dependent on c present cao
15 15
[5]

104 (i) a = 2 B1
[1]
(ii) y = x3 – 4x2 + 4x B1
dy
= 3x2 – 8x + 4 B2, 1, 0 –1 for eeoo
dx
x – 23x – 2 = 0
2
b= B1 cao
3
[4]
x4 4x3
(iii) area = ! y dx = " – + 2x2 # B2, 1, 0 –1 for eeoo
4 3
32
4–
3
+8 M1 Apply limits 0→2 – ft their a from (i)
4
A1 cao
3
[4]
d2 y 4 d2 y
(iv) 2 = 6x – 8 = 0 ⇒ x = M1 A1 Attempt and set = 0
dx 3 dx2
4 dy 4
When x = , (or m ) = – DM1 A1 cao
3 dx 3
[4]

398
Page Mark Scheme: Teachers’ version Syllabus Paper
GCE AS/A LEVEL – May/June 2012 9709 12

6
105 y=
2x − 3
− 36
Integral of y² = ÷2 B1 B1 co allow 2nd B1 independent of 1st.
(2 x − 3)
Use of limits 2, 3 M1 Used as 2 to 3 or 3 to 2 in integral of y².
→ 12π or 37.7 A1 Co
[4] (uses area 0/4. No π Max 3/4 )
106 y = − x 2 + 8 x − 10
dy
(i) = −2x + 8 B1 co
dx
= 0 when x = 4, A is (4, 6) M1A1 Sets to 0 and attempt to solve for x. co.
Equation of AB is y − 6 = 2( x − 4) M1 Correct form of equation.
Sim eqns with eqn of curve M1 Eliminates x or y completely
→ x 2 − 6 x + 8 = 0 or y 2 − 8 y + 12 = 0 A1 Method for quadratic eqn = 0.
→ B (2, 2) A1 co (Must not be guessed from diagram)
[7]
3
x
∫− x + 4 x 2 − 10 x
2
(ii) + 8 x − 10 dx = − B2,1 3 terms, loses 1 for each error
3
Uses his x limits 2 to 4 M1 Uses x limits correctly – allow ±
→ 9⅓ A1 co – allow ± (2 must have been
[4] correctly found, not guessed)

Page Mark Scheme: Teachers’ version Syllabus Paper


GCE AS/A LEVEL – May/June 2012 9709 11

4 B1 AG At least 1 step of working needed


107 (i) x = −1 [1]
y2

 4   4  B1B1
(ii) ∫  2 − 1 dy = − − y 
y   y 
4
Upper limit = 2 B1 For − , –y
y
 4  
 − 2 − 2  − (− 4 − 1) M1 Apply limits 1 and their 2 ‘correctly’
  
SC B2 for 2(x + 1) 2 dx − 3 → 1
−1
1 A1 ∫
[5]

 16 8 
(iii) (π ) x 2 dy = (π )  4 − 2 + 1 dy
∫ ∫ B1B1
y y 

 
(π ) − 163 + 8 + y  B1
 3y y 

(π ) − 16 + 4 + 2  −  − 16 + 8 + 1 M1 Apply limits 1 and their 2 ‘correctly’


 24   3 

A1
3 [5]

399
Page Mark Scheme: Teachers’ version Syllabus Paper
GCE AS/A LEVEL – May/June 2012 9709 13

8
108 x = − 2 ; at x = 0, y = 2 B1 co
y2
64 32
→ x2 = 4 − 2 + 4
y y
64 y −3 32 y −1
Integral of x² = − + 4y B1B1B1 All co.
−3 −1
Uses limits 1 to 2
M1 Uses 1 to 2 or 2 to 1.
→ 6⅔π
A1 co.
[6]

d2 y
109 = − 4x
dx 2
dy
(i) = − 2x 2 + c
dx B1 For −2x²
dy
= 0 when x = 2, → c = 8
dx B1 c=8
2 x3
y=− + 8 x (+C) B1 B1 For each term – on “c”– ignore (+C)
3
4
Subs (2, 12) → C = M1 A1 Uses (2, 12) to find C.
3 [6]

dy dy dx
(iii) = × M1
dt dx dt
= −10 × 0.05 Must use. Enough to see product of
→ decreasing at 0.5 units per second A1 gradient and rate. bod over notation.
[2]

Page 6 Mark Scheme: Teachers’ version Syllabus Paper


GCE AS/A LEVEL – October/November 2011 9709 12

dy 8
110 = 5 − 2 , Normal 3 y + x = 17
dx x

(i) Gradient of line = −⅓ B1 co


dy M1 Use of m1m2 = − 1
= 3 → x = 2, y = 5 DM1 DM1 solution. A1 co.
dx A1
[4]
(ii) y = 5 x + 8 x −1 (+ c ) B1 B1 co.co. doesn’t need +c.
Uses (2, 5) → c = −9 M1 A1 Use of +c following integration. co.
[4]

400
Page Mark Scheme: Teachers’ version Syllabus Paper
GCE AS/A LEVEL – October/November 2011 9709 12

111 y = 8x − x 2

dy 1 −1 B1 B1 for everything but ×(8-2x)


(i) = 2 (8 x − x 2 ) 2 × (8 − 2 x)
dx B1 B1 for × (8−2x), even if B0
= 0 when x = 4. M1 Sets to 0 + attempt at solution.
→ (4, 4) A1 Co – A0 if fortuitous because of B0
[4] earlier.
(ii) y = 0 when x = 0 or 8 B1

Vol = π (8 x − x 2 )dx Anywhere

 x3  B2,1
= π 4 x 2 − 
 3 −1 for each error (not including π)
256π B1

3 [4]
co

Page Mark Scheme: Teachers’ version Syllabus Paper


GCE AS/A LEVEL – October/November 2011 9709 11

If B0B0 then SCB1 for both y = 1 &


112 (i) B = (0,1) C = (4,3) B1, B1 [2]
x=4
1
δy 1 1 − 1
(ii) = × 2(1 + 2 x ) 2

M1A1
2
required & at least one of ×2
δx 2 2
for M1
Grad. of normal = −3 B1
y − 3 = −3( x − 4 ) or y = −3 x + 15 oe B1√ [4] Ft only from their C

1
1 ∫ x δy , square 2 (y )
2 2
− 1 & attempt
(iii) y 2 = 1 + 2 x ⇒ x =
( 2
2 y −1 ) SOI B1
int n

(π ) × 1 × ∫ (y 4 − 2 y 2 + 1)δy M1
4
1  y5 2y3  Apply limits 0 → their 1 (from their
(π ) ×  − + y A1
4 5 3  B)

π
(π ) × 1  1 − 2 + 1 ∫ y δx → 4
2
cao SCB1 for (scores
DM1
4 5 3  1/5)
2
π A1 [5]
15

401
Page Mark Scheme: Teachers’ version Syllabus Paper
GCE AS/A LEVEL – October/November 2011 9709 13

113 (i) 2x5 + 3x2 = 2x ⇒ 2x5 + 3x2 – 2x = 0 M1 First line essential


[x(2x]4 + 3x2 – 2) = 0
2x4 + 3x2 – 2 = 0 A1 AG Factorising needed for A1
[2]
M1 Reasonable attempt at solving a
(ii) (x2 + 2)(2x2 – 1) = 0 quadratic in x2
A1
x=± 1 only A1 For a correct pair of solutions, either 2
2
[3] x’s or 1 x and 1 y
  SC (±0.707, ±1.41) AWRT B1
 1 2   −1 
 ,
 
 2, 2   2, −2 
 2

1 M1 Dealing with line as a triangle or integral


∫ ( x + 1) 2 − ( x + 1) ∫( y )
− 1 − ( y − 1)
2
114 (i) or with correct limits.

2 3
1 1 1
( x + 1) 2 − x2 − x or y 2 − y 2 M1A1 Attempt at integral of curve.
3 2 3 2

2  1  1 1 DM1 Applying limits –1→ 0 or 0 → 1 to


−  0 − + 1 or −
3  2  3 2 curve
1 A1 π included loses last mark.
6 [5]

(ii) V1 = (π ) ∫ ( y 2 − 1)2 =(π ) ∫ y 4 − 2 y 2 + 1 M1 Attempt at ∫x


2
dy for curve
 y5 2 y 2 
(π )  − + y A1
5 3 
1 2 
(π )  − + 1 DM1 Apply limits 0 → 1
5 3 
8
V1 = or 0.533(π) (AWRT) A1
15(π )
y3 1
or (π)  – y2 + y M1 Or   1ଶ  1
3 3
1
V2 = π A1
∫ x dy
2
3 Vol of cone or attempt to for
8 1 1 line
Volume = π π = π (or 0.628) A1
15 −3 5 [7]
OR (y4 – 2y2 + 1) – (y2 –2y + 1) ∫ x dy
2
M1 Attempt to
(π ) ∫ y 4 − 3 y 2 + 2 y M1 Attempt to ∫ (x12 – x22)
(π )  y ↑ 5 / 5 − y ↑ 3 + y ↑ 2  A1,A1,A1

1 
(π )  − 1 + 1 DM1 Apply limits 0→1 dependent on first M1
5 
1
π A1
5

402
Page Mark Scheme: Teachers’ version Syllabus Paper
GCE AS/A LEVEL – May/June 2011 9709 12

 3 1  x4 x −2
115 ∫  x +  =
dx + +c 3 × B1 Allow unsimplified, 1 mark for each term,
 x3  4 −2 [3] including “c”

Page Mark Scheme: Teachers’ version Syllabus Paper


GCE AS/A LEVEL – May/June 2011 9709 12

116 y = 4 x − x.
(i) At A, 4 x − x = 0 → A(16, 0) B1 co – independent of working.
dy −1
= 2x 2 − 1 B1 B1 B1 for each part.
dx
= 0 when x = 4 → (4, 4) M1 A1 Sets to 0 and solves his eqn. co
[5]


(ii) Vol = π y 2 dx =
3
π ∫ (16 x + x − 8 x ) dx
2 2
M1 Use of correct formula + attempt at
5 integration
x3 x 2
π [8 x 2 + −8 5 ] A3,2,1 One mark for each term – unsimplified
3 2 DM1 Correct use of his limits.
Limits 0 to 16 → 136.5π. (or 137π) A1 co – (429 ok)
[6]

403

You might also like